Download as doc, pdf, or txt
Download as doc, pdf, or txt
You are on page 1of 215

Chapter 1.................................................................................................................................................................

2
CHAPTER 2............................................................................................................................................................5
Conceptual Problems:.....................................................................................................................................5
Technical Problems:........................................................................................................................................7
Additional Problems........................................................................................................................................9
CHAPTER 3..........................................................................................................................................................16
Conceptual Problems:...................................................................................................................................16
Technical Problems:......................................................................................................................................17
Additional Problems......................................................................................................................................23
CHAPTER 4..........................................................................................................................................................28
Conceptual Problems:...................................................................................................................................28
Technical Problems:......................................................................................................................................71
Additional Problems......................................................................................................................................74
CHAPTER 5..........................................................................................................................................................78
Conceptual Problems:...................................................................................................................................78
Technical Problems:......................................................................................................................................79
Additional Problems......................................................................................................................................81
CHAPTER 6..........................................................................................................................................................84
Conceptual Problems:...................................................................................................................................84
Technical Problems:......................................................................................................................................86
Additional Problems:.....................................................................................................................................87
CHAPTER 7..........................................................................................................................................................93
Conceptual Problems:...................................................................................................................................94
Technical Problems:......................................................................................................................................97
Additional Problems:.....................................................................................................................................98
CHAPTER 8........................................................................................................................................................105
Conceptual Problems:.................................................................................................................................105
Technical Problems:....................................................................................................................................108
Additional Problems:...................................................................................................................................110
CHAPTER 9........................................................................................................................................................114
Conceptual Problems:.................................................................................................................................142
Technical Problems:....................................................................................................................................142
Additional Problems:...................................................................................................................................145
CHAPTER 10......................................................................................................................................................154
Conceptual Problems:.................................................................................................................................154
Technical Problems:....................................................................................................................................156
Additional Problems:...................................................................................................................................160
CHAPTER 11......................................................................................................................................................168
Conceptual Problems:.................................................................................................................................168
Technical Problems:....................................................................................................................................170
Additional Problems:...................................................................................................................................172
CHAPTER 12......................................................................................................................................................183
Conceptual Problems:.................................................................................................................................183
Technical Problems:....................................................................................................................................185
Additional Problems:...................................................................................................................................188
CHAPTER 13......................................................................................................................................................193
Conceptual Problems:.................................................................................................................................193
Technical Problems:....................................................................................................................................196
Additional Problems:...................................................................................................................................198

1
CHAPTER 14......................................................................................................................................................217
Conceptual Problems:.................................................................................................................................217
Technical Problems:....................................................................................................................................220
Additional Problems:...................................................................................................................................222
CHAPTER 15......................................................................................................................................................228
Conceptual Problems:.................................................................................................................................228
Technical Problems:....................................................................................................................................229
Additional Problems:...................................................................................................................................231
CHAPTER 16......................................................................................................................................................234
Conceptual Problems:.................................................................................................................................251
Technical Problems:....................................................................................................................................253
Additional Problems:...................................................................................................................................255
CHAPTER 16......................................................................................................................................................251
Conceptual Problems:.................................................................................................................................251
Technical Problems:....................................................................................................................................254
Additional Problems:...................................................................................................................................255
CHAPTER 18......................................................................................................................................................262
Conceptual Problems:.................................................................................................................................262
Technical Problems:....................................................................................................................................287
Additional Problems:...................................................................................................................................289
CHAPTER 19......................................................................................................................................................315
Conceptual Problems:.................................................................................................................................316
Technical Problems:....................................................................................................................................318
Additional Problems:...................................................................................................................................321
CHAPTER 20......................................................................................................................................................314
Conceptual Problems:.................................................................................................................................314
Technical Problems:....................................................................................................................................316
Additional Problems:...................................................................................................................................319
CHAPTER 21......................................................................................................................................................324
Conceptual Problems...................................................................................................................................324
Technical Problems:....................................................................................................................................336
Additional Problems:...................................................................................................................................338

Chapter 1

Problems

1. "A country's rate of economic growth is determined solely by the amount of resources available to the
country." Comment on this statement.

Increases in the availability of resources, that is, labor and capital used in the production of goods and services
account for only part of a nation's economic growth. The efficiency with which these factors of production are
used also affects economic growth. Increases in the efficiency of production result from increases in the
education and skill levels of the labor force and from newer and more efficient technology. In addition, the
factors of production are not fully employed all the time. During an expansion or recovery, the use of the
factors of production increases, which leads to an increase in production and output.

2. In the 1960s, increases in the rate of unemployment were generally associated with decreases in the

2
inflation rate and vice versa. But in the 1970s and 1980s unemployment and inflation often moved in the same
direction. How can you explain this?

A shift in aggregate demand causes the unemployment rate and the inflation rate to move in opposite
directions, whereas a shift in aggregate supply causes unemployment and inflation to move in the same
direction. Most disturbances in the 1960s came from the demand side, while many of the disturbances in the
1970s and 1980s came from the supply side.
3. "Since the long-run AS-curve is vertical, we can conclude that the total real output of a nation cannot grow
in the long run." Comment on this statement.

The AD-AS framework is a static framework that assumes that the level of potential GDP is fixed. However,
the potential GDP of a nation grows over time as the amount of available resources or the efficiency with
which these resources are used increases. Figure 1-4 in the text clearly indicates that the long-run (vertical)
AS-curve moves to the right by a small percentage each year.

4. "Long-run growth can best be studied by focussing on the reasons for business cycles." Comment on this
statement.

Growth theory focuses primarily on the accumulation of inputs and improvements in technology that allow for
an increased standard of living over time. Since growth theory tries to explain the average growth rate of an
economy over many years, it ignores the short-run fluctuations (recessions and booms) that occur over the
course of business cycles.

5. "In the very short run real output is fixed and therefore any increase in aggregate demand will simply
increase the price level but not affect how many goods and services are produced in an economy." Comment
on this statement.

The short-run AS-curve is completely horizontal, based on the assumption that prices are constant. An increase
in aggregate demand will therefore increase the level of output but will not affect the price level. It follows
therefore that in the short run the level of output is solely determined by aggregate demand.

6. "There is always a clear tradeoff between unemployment and inflation." Comment.

The short-run Phillips curve describes an empirical relationship between wage and price inflation and the rate
of unemployment. This curve shows that the higher the rate of unemployment, the lower the rate of inflation
and vice versa (at least in the short run). However, in the long run there is no clear-cut tradeoff between
inflation and unemployment. The economic events of the 1970s and 1980s showed us that unemployment and
inflation can increase or decrease simultaneously.

7. The index of leading economic indicators is supposed to signal future developments in the economy and is
calculated from variables in different sectors of the economy. Pick one leading economic indicator from each
of the following sectors and briefly state the rationale for including this indicator in the calculation of the
index:
(1) the labor sector, (2) business activity, (3) the financial sector, and (4) residential construction.

The answer to this question is student specific. Here is a sample answer.

3
The labor sector:
Employment figures as well as the unemployment rate should be used to look for changes in the labor sector.
The unemployment rate is more often discussed in the media, but the employment figures have smaller cyclical
variations and are thus a better indicator of labor market conditions.

Business activity:
New orders, changes in inventories, capacity utilization, and industrial production are often used to interpret
business activity. When using inventory changes it is important to distinguish between desired inventory
changes, which may reflect changes in business expectations, and undesired inventory changes that reflect
changes in the demand for the product.
The financial sector:
Stock market activity, changes in money supply, or credit conditions can be used to show trends in financial
markets. While the relationship between changes in stock values and the economy is not as close as it used to
be, a continued increase in stock prices generally reflects increased optimism about future economic
conditions. It also means an increase in wealth for stockholders and easier access to funds for firms wishing to
make new investments.
Residential construction:
A change in new building permits or housing starts may be the first sign of changing economic conditions. The
housing sector is very sensitive to interest rate changes and tends to be indicative of other sectors of the
economy, which often react in a similar way although to a lesser degree and with a time lag.

8. Briefly discuss the usefulness of each of the following as leading economic indicators:
(i) inventory changes, (ii) the GDP-deflator, (iii) the unemployment rate, (iv) the Dow Jones Industrial
Average.

Desired inventory changes reflect changes in business expectations and can be used as a leading economic
indicator. But undesired inventory changes reflect changes in demand for the product and are therefore a
lagging economic indicator. Only if one can separate out desired from undesired inventory changes, will it be
possible to spot more clearly signs of an upcoming boom or recession.
The GDP-deflator is the most complete price index, since it measures price changes of all final goods and
services currently produced in a country. But the GDP-deflator is a lagging indicator, showing what has
happened over the last quarter. In addition, initial GDP data tend to be fairly unreliable and have to be
frequently revised.
The unemployment rate has fairly small cyclical variations and is used as a measure for variations in the
demand for labor. Changes in the unemployment rate also correspond well with changes in GDP. But the
unemployment rate is a concurrent indicator.
Changes in stock values generally reflect changes in expectations of financial investors about dividends. If
stock values go up people may feel wealthier and thus consumer spending may increase. Firms may have an
easier time issuing new stock and thus they may invest more. In this case, we can expect an economic
upswing. However, changes in stock values often may simply be due to speculative behavior and may not be
related to real economic activity.

9. In each of the three pairs below, which variable would you choose (and why) as a leading economic
indicator:
(i) labor productivity or the unemployment rate
(ii) the CPI or the PPI
(iii) stock market changes or housing starts

4
Labor productivity generally shows long run trends in the labor market and will determine wages and therefore
living standards. It is a leading indicator, but cannot be easily used for forecasting.
The unemployment rate has fairly small cyclical variations and is used as a measure for variations in the
demand for labor. It is a very accurate measure of economic performance--at least as far as the labor sector is
concerned. However, the unemployment rate is a concurrent and not a leading indicator.
The CPI measures the price increase of a fixed market basket of 386 goods and services purchased by an
average urban family. The CPI is easily available and is supposed to measure cost of living increases.
However, the CPI is a concurrent indicator.
The PPI measures average price changes of a market basket of over 1,000 intermediate goods up to the
retail stage. It is relatively easily available and shows future price trends with relative accuracy. The PPI is a
leading indicator and does not always correspond exactly with the CPI.
Stock market changes reflect changes in expectations of financial investors about dividends arising from a
perceived change in economic conditions. If stock values increase consumers may feel wealthier (and thus
spend more), and firms have an easier time issuing new stock (and thus invest more). But while a change in
stock values may indicate an upcoming change in economic conditions, it also could simply have been caused
by speculative behavior. Thus, it is a leading indicator that is, however, not always reliable.
The demand for housing is very interest sensitive, since mortgage interest payments are a large part
housing costs. Housing starts tend to increase sharply when interest rates drop to low levels, that is, when the
economy is at the bottom of a recession. A change in housing starts thus is often seen as a turning point in the
economy, since other sectors in the economy are expected to react similarly to changes in interest rates but
with a lag and to a lesser degree. The increase in construction work naturally also stimulates economic activity
in other sectors of the economy. Thus housing starts is seen as a fairly reliable leading economic indicator.

CHAPTER 2

NATIONAL INCOME ACCOUNTING

Solutions to Problems in the Textbook:

Conceptual Problems:

1. Government transfer payments (TR) do not arise out of any production activity and are thus not counted in
the value of GDP. If the government hired the people who currently receive transfer payments, then their
wages would be counted as part of government purchases (G), which is counted in GDP. Therefore GDP would
rise.

2.a. If the firm buys a car for an executive's use, the purchase counts as investment (I). But if the firm pays the
executive a higher salary and she then buys a car, the purchase is counted as consumption (C).

2.b. The services that a homemaker provides are not counted in GDP (regardless of their value). However, if an
individual officially hires his or her spouse to perform household duties at a certain wage rate, then the wages
earned will be counted in GDP and GDP will increase.

2.c. If you buy a German car, consumption (C) will increase but net exports (NX = X - Q) will decrease.

5
Overall GDP will increase by the value added at the foreign car dealership, since the import price is likely to
be less than the sales price. If you buy an American car, consumption and thus GDP will increase. (Note: If the
car you buy comes out of the car dealer's inventory, then the increase in C will be partially offset be a decline
in I, and GDP will again only increase by the value added.)

3. GDP is the market value of all final goods and services currently produced within the country. (The U.S.
GDP includes the value of the Hondas produced by a Japanese-owned assembly plant that is located in the
U.S., but it does not include the value of Nike shoes that are produced by an American-owned shoe factory
located in Malaysia.)
GNP is the market value of all final goods and services currently produced using assets owned by domestic
residents. (Here the value of the Hondas produced by a Japanese-owned Honda plant is not counted but the
value of the Nikes by the American-owned shoe plant is.)
Neither is necessarily a better measure of the output of a nation. The actual value of the GDP and GNP for the
U.S. is fairly close.

4. The NDP (net domestic product) is defined as GDP minus depreciation. Depreciation measures the value
of the capital that wears out during the production process and has to be replaced. Therefore NDP comes closer
to measuring the net amount of goods produced in this country. If this is what you want to measure, then NDP
should be used.

5. Increases in real GDP do not necessarily mean increases in welfare. For example, if the population of a
country increases by more than real GDP, then the population of the country is on average worse off. Also
some increases in output come from welfare reducing events. For example, increased pollution may cause
more lung cancer, and the treatment of the lung cancer will contribute to GDP. Similarly, an increase in crime
may lead to overtime work for police officers, whose increased salary will increase GDP. But the welfare of
the people in the country may not have increased in either case. On the other hand, GDP does not always
accurately measure quality improvements in goods or services (faster computers or improved health care) that
improve people's welfare.

6. The CPI (consumer price index) and the PPI (producer price index) are both measured by looking at a
certain market basket. The CPI's basket contains mostly finished goods and services that consumers tend to
buy regularly in their daily lives. The PPI’s basket contains raw materials and semi-finished goods, that is, it
measures costs to the producer of a product and its first user. The CPI is a concurrent economic indicator,
whereas the PPI is a leading economic indicator.

7. The GDP-deflator is a price index that covers the average price increase of all final goods and services
currently produced within an economy. It is defined as the ratio of current nominal GDP to current real GDP.
Nominal GDP is measured in current dollars, while real GDP is measured in so-called base-year dollars. Even
though early estimates of the GDP-deflator tend to be unreliable, the GDP-deflator can be a more useful price
index than the CPI or PPI (both of which are fixed market baskets). This is true for two reasons: first it
measures a much wider cross-section of goods and services; second, a fixed market basket cannot account for
people substituting away from goods whose relative prices have changed, while the GDP-deflator, which
includes all goods and services produced within the country, can.

6
8. If nominal GDP has suddenly doubled, it is most likely due to an increase in the average price level.
Therefore, the first thing you would want to check is by how much the GDP-deflator has changed, to calculate
by how much real output (GDP) has changed. If nominal GDP and the GDP-deflator have both doubled, then
real GDP should be the same.

9. Assume the loan you made yields you an annual nominal return of 7%. If the rate of inflation is 4%, then
your rate of return in real terms is only 3%. If, on the other hand, if inflation rate is 10%, then you will actually
get a negative real rate of return, that is, you will lose 3% of your purchasing power. One way to protect
yourself against such a loss of purchasing power is to adjust the interest rate for inflation, that is, to index the
loan. In other words, you can require that, in addition to the specified interest rate of the loan of, let’s say, 3%,
the borrower also has to pay an inflation premium equal to the percentage change in the CPI. In this case, a real
rate of return of 3% would be guaranteed.

Technical Problems:

1. The text calculates the change in real GDP in 1992 prices in the following way:

[RGDP01 - RGDP92]/RGDP92 = [3.50 - 1.50]/1.50 = 1.33 = 133%.

To calculate the change in real GDP in 2001 prices, we first have to calculate the GDP of 1992 in 2001 prices.
Thus we take the quantities consumed in 1992 and multiply them by the prices of 2001, as follows:

Beer 1 at $2.00 = $2.00


Skittles 1 at $0.75 = $0.75
_______________________________
Total $2.75

The change in real GDP can now be calculated as

[6.25 - 2.75]/2.75 = 1.27 = 127%.

We can see that the growth rate of real GDP calculated this way is roughly the same as the growth rate
calculated above.

2.a. The relationship between private domestic saving, investment, the budget deficit and net exports is shown
by the following identity:

S - I  (G + TR - TA) + NX.

Therefore, if we assume that transfer payments (TR) remain constant, then an increase in taxes (TA) has to be
offset either by an increase in government purchases (G), a decrease in net exports (NX), or a decrease in the
difference between saving (S) and investment (I).

2.b. From the equation YD  C + S it follows that an increase in disposable income (YD) will be reflected in
an increase in consumption (C), saving (S), or both.

7
2.c. From the equation YD  C + S it follows that when either consumption (C) or saving (S) increases,
disposable income (YD) must increase as well.

3.a. Since depreciation D = Ig - In = 800 - 200 = 600 ==>

NDP = GDP - D = 6,000 - 600 = 5,400

3.b. From GDP = C + I + G + NX ==> NX = GDP - C - I - G ==>

NX = 6,000 - 4,000 - 800 - 1,100 = 100.

3.c. BS = TA - G - TR ==> (TA - TR) = BS + G ==> (TA - TR) = 30 + 1,100 = 1,130

3.d. YD = Y - (TA - TR) = 5,400 - 1,130 = 4,270

3.e. S = YD - C = 4,270 - 4,000 = 270

4.a. S = YD - C = 5,100 - 3,800 = 1,300

4.b. From S - I = (G + TR - TA) + NX ==> I = S - (G + TR - TA) - NX = 1,300 - 200 - (-100) = 1,200.

4.c. From Y = C + I + G + NX ==> G = Y - C - I - NX ==>

G = 6,000 - 3,800 - 1,200 - (-100) = 1,100.

Also: YD = Y - TA + TR ==> TA - TR = Y - YD = 6,000 - 5,100 ==> TA - TR = 900

From BS = TA - TR - G ==> G = (TA - TR) - BS = 900 - (-200) ==> G = 1,100

5. According to Equation (2) in the text, the value of total output (in billions of dollars) can be calculated as:
Y = labor payments + capital payments + profits = $6 + $2 + $0 = $8

6.a. Since nominal GDP is defined as the market value of all final goods and services currently produced in
this country, we can only measure the value of the final product (bread), and therefore we get $2 million (since
1 million loaves are sold at $2 each).

6.b. An alternative way of measuring total GDP would be to calculate all the value added at each step of
production. The total value of the ingredients used by the bakeries can be calculated as:

1,200,000 pounds of flour ($1 per pound) = 1,200,000


100,000 pounds of yeast ($1 per pound) = 100,000
100,000 pounds of sugar ($1 per pound) = 100,000
100,000 pounds of salt ($1 per pound) = 100,000
__________________________________________________________
= 1,500,000

Since $2,000,000 worth of bread is sold, the total value added at the bakeries is $500,000.

8
7. If the CPI increases from 2.1 to 2.3, the rate of inflation can be calculated in the following way:

rate of inflation = (2.3 - 2.1)/2.1 = 0.095 = 9.5%

The CPI often overstates inflation, since it is calculated by using a fixed market basket of goods and services.
But the fixed weights in the CPI's market basket cannot capture the tendency of consumers to substitute away
from goods whose relative prices have increased. Therefore, the CPI will overstate the increase in consumers'
expenditures.

The real interest rate (r) is defined as the nominal interest rate (i) minus the rate of inflation (). Therefore the
nominal interest rate is the real interest rate plus the rate of inflation, or

i = r +  = 3% + 4% = 7%.

Additional Problems

1. Explain the initial effect of the following events on GDP.


a. You sell your used car to a friend.
b. Firms decrease their inventories.
c. The value of your AT&T stock holding decreases.
d. You buy a piece of land with the intention of building a new house.
e. A sports card dealer sells a Ken Griffey, Jr. rookie card for $100.
f. A German tourist drinks Canadian beer in an American restaurant.

a. GDP will not change, since a used car is not currently produced. (Only if you sell the car through a dealer
will GDP increase by the value of the services rendered.)

b. Inventory changes are part of investment, so investment will decrease. But somebody will have to buy
these inventories, so consumption will increase. If the inventories are sold at a price higher than invoice, then
GDP will increase by the value added.

c. A loss in stock values means a loss in wealth, therefore GDP is not directly affected. Only if your
dividends decrease will your income (and thus GDP) be affected.

d. When you use savings to buy land, a transfer of wealth takes place and GDP is not affected. (Only if a real
estate agent receives a commission will GDP go up by the value of the services rendered.)

e. When the card dealer sells the rookie card, inventory decreases, so investment goes down. Selling the card
to a customer increases consumption. Thus GDP increases only by the value added at the dealer for his services
rendered.

f. GDP will increase by the value added in the American restaurant. If the beer was imported from Canada
for $1.20 and sold (exported) to a German tourist for $3.00, then net exports will increase by $1.80.

2. How will the following events affect GDP and why?


a. A hurricane destroys part of Florida.

9
b. You sell your old macroeconomics textbook to another student.
c. You sell your holdings of IBM stock.
d. Your local car dealership decides to reduce its inventory by offering price reductions.
e. A retired worker gets an increase in Social Security benefits.

a. When a hurricane destroys property, wealth is affected, not income (or GDP). However, if a significant
amount of the capital stock is destroyed, then less can be produced later, leading to a decrease in GDP. On the
other hand, the rebuilding of destroyed property means that increased economic activity will take place,
leading to an increase in GDP.

b. The sale of your old textbook will not constitute an official market transaction (since you probably will
not report your income to the IRS). In addition, the textbook has already been used and is not currently
produced. Therefore GDP will not be affected.

c. The sale of existing stock holdings is a transfer of wealth and, as such, does not affect GDP. Any fees that
you may have to pay your broker for his or her services, however, constitute payment for services rendered.
GDP will increase by that amount.

d. Inventory changes are counted as part of investment. A reduction in business inventories will lower the
level of investment (I) and thus GDP. However, the sales of the cars count as consumption (C) if consumers
buy them, or investment (I) if firms buy them. Thus the net effect on GDP depends on the difference between
the cost of the cars to the dealership and the sales price of the cars, that is, the value added.

Transfer payments that do not arise from productive activity are not counted in GDP. Thus GDP will not be
affected when Social Security benefits are paid. (Only later, when these payments are spent, will consumption
increase.)

3. If nominal GDP in Germany increased by 2.8% last year, but U.S. GDP increased by 4.2%, can we
conclude that the welfare of U.S. citizens increased by more than that of German citizens? Why or why not?

A country's nominal GDP is not a good measure of the economic welfare of its people, since nominal GDP can
change solely due to inflation. Only if real GDP grows faster than population, will real income per capita
increase. But real GDP per capita still does not take into account changes in income distribution, changes in
environmental quality, or leisure, all of which influence the economic welfare of the people in a country.
Therefore we cannot say whether the welfare of the people in the U.S. has increased more than that of the
people in Germany.

4. "Real per capita GDP is a good measure of economic welfare." Comment on this statement.

Real GDP per capita is an imperfect measure of economic welfare as it does not include non-market activities
which affect well being, such as the value of household services, volunteer work, pollution, the loss of natural
wilderness areas resulting from development, and so on. In spite of these limitations, however, real GDP per
capita still does provide some measure of economic welfare.

5. Assume real GDP in 1992 was $7,000 billion, nominal GDP in 1997 was $8,316 billion, and the
GDP-deflator has increased from 100 to 110 from 1992 and 1997. What is the average annual growth rate of
real GDP from 1992 to 1997? Do you think the welfare of all people in the U.S. has increased during that time.

10
Why or why not?

RGDP = (NGDP/deflator)*100 = (8.316/110)*100 = 7.56

Growth rate of GDP = (7.56 - 7.0)/7.0 = 0.56/7 = 0.08 = 8%

Thus real GDP has grown 8% in five years, or at an average annual growth rate of 1.6%.

An increase in a country's GDP is not a good measure for an increase in the economic welfare of its people.
For example, nominal GDP can change solely due to inflation, and real GDP has to grow faster than the
population, for real income per capita, and thus living standards, to increase. But real GDP per capita still does
not take into account changes in the income distribution, changes in environmental quality, or changes in
leisure, all of which influence the economic welfare of the people in a country

6. Explain why indirect taxes are deducted from NDP to calculate national income (Y).

Indirect taxes are not paid to factors of production for the use of their services. Therefore they are not included
in national income, which measures the income of all factors of production.

7. Comment on the following statement:


"Any accumulation of inventories by firms is not included when measuring GDP."

National income accounts do include changes in inventories when measuring investment. Inventories rise
when production exceeds sales, but fall if production falls short of demand. These changes must be allowed to
affect investment. But if investment is affected, so is GDP. Otherwise total economic activity will be over- or
underestimated to the extent that inventory changes are not accounted for.

8. Assume a Hyundai dealership in Chicago bought 30 Hyundais from Korea at a cost of $13,000 per car in
September of 2000, and by December 31, 2000 they have sold 20 of these Hyundais at a price of $16,000 each.
The remaining Hyundais were sold in January 2001 at a price of $14,000 each. How exactly does this affect
the GDP in 2000 and 2001, and which categories of GDP (C, I, G, or NX) are affected?

2000: NX = - (30*13,000) = - 390,000


C = + (20*16,000) = + 320,000
I = + (10*13,000) = + 130,000
_____________________________________
GDP = + 60,000

Check: The value added in 1999 is: 20*3,000 = 60,000

2001: C = + (10*14,000) = + 140,000


I = - (10*13,000) = - 130,000
______________________________________
GDP = + 10,000

Check: The value added in 2000 is: 10*1,000 = 10,000

11
9. Comment on the following statement:
"I bought a new home last year. If I sell it today, I will raise the level of economic activity."

This statement would be true if a realtor sold your home, as the realtor would have provided a current service
for which she would be paid. Transactions in existing assets such as artwork and residential housing do not
create economic activity in an amount equal to the value of the sale. New home construction, on the other
hand, is included in the calculation of current GDP as it does represent current economic activity.

10. True or False? Why?


"The PPI measures the cost of buying a fixed bundle of consumer goods."

False. The Consumer Price Index (CPI) measures the cost of buying a market basket of consumer goods. The
Producer Price Index (PPI) measures the cost of commodities at an early stage of the distribution system.
Changes in the PPI signal a future change in the general price level, as costs are passed on to consumers.

11. True or false? Why?


"Using the consumer price index or the GDP-deflator to calculate changes in the average price level
should produce identical the inflation rates."

False. The CPI measures the cost of a representative household's consumption bundle whereas the GDP-
deflator is a more comprehensive measure of the prices of all goods and services produced in the economy.
Rates of change in these indices vary due to the differences in their construction.

12. Do the CPI and the GDP-deflator always show the same increase in the rate of inflation?

The GDP-deflator measures the average price increase of all final goods and services that are currently
produced. These goods differ from year to year depending on what is produced. The CPI measures only the
average cost increase of a specified market basket of goods and services. The CPI also includes prices of
import goods that the GDP-deflator does not include. Thus, when import prices go up, the CPI will most likely
increase more than the GDP-deflator.

13. Briefly describe the advantages and disadvantages of using the CPI, the PPI, and the GDP -deflator as
economic indicators.

The consumer price index (CPI) measures the average price increase of a fixed market basket of goods and
services purchased by an average urban wage earner. Not all goods and services are reflected in this market
basket and substitution among these goods is not possible. So the CPI is not a perfect measure for inflation.
However, the CPI is easily available on a monthly basis and is fairly reliable.
The producer price index (PPI) measures the average price increase of a fixed market basket of intermediate
goods up to the retail stage, but it does not include services or interest payments. The PPI is relatively easily
available on a monthly basis and it is used to show future price trends. One has to be careful to avoid double
counting, since the PPI deals with intermediate goods. The PPI does not necessarily correspond with the CPI,
since firms can't always shift higher producer prices onto consumers.
The GDP-deflator is probably the most useful price index for macroeconomists since it measures the average
price increase of all goods and services currently produced in this country. It does not include import goods,

12
used goods, or interest payments, and early estimates are often unreliable and have to be revised repeatedly.
However, the GDP-deflator is the most complete of the price indexes.

14. Assume last year's real GDP was $7,000 billion, this year's nominal GDP is $8,820 billion, and the GDP-
deflator for this year is 120. What was the growth rate of real GDP?

RGDP(1) = [NGDP(1)/GDP-deflator]*100 = [8,820/120]*100 = 7,350

Since RGDP(0) = 7,000 it follows that the growth rate of RGDP is

y = [7,350 - 7,000]/7,000 = 0.05 = 5%.

15. Comment on the following statement:


“A country that spends more than its total national income must have a trade deficit.”

National income is defined as Y = C + I + G + NX. The four main components of aggregate demand are
consumption (C), investment (I), government purchases (G) and net exports (NX). But if the spending on
consumption, investment, and government purchases is greater than national income, it follows that net exports
(NX = X - Q) must be negative, that is, imports (Q) must exceed exports (X), and the country must have a
trade deficit.

16. Calculate the values for government outlays (G), saving (S), and investment (I) from the following
information.
national income Y = 5,200 budget deficit BuD = 150
disposable income YD = 4,400 trade deficit TD = 110
consumption C = 4,100

From YD = C + S ==> S = YD - C = 4,400 - 4,100 = 300

From S - I = BuD - TD ==> 300 - I = 150 - 110 ==> I = 260

From Y = C + I + G + NX ==> G = Y - C - I - NX

==> G = 5,200 - 4,100 - 260 + 110 = 950

17. From the following information (all variables are in billions of dollars) calculate the value of government
spending (G), consumption (C), and investment (I).
national income Y = 6,000 tax revenues TA = 1,500
saving S = 1,000 transfer payments TR = 700
net exports NX = - 120 budget deficit BuD = 230

From YD = Y - TA + TR ==> YD = 6,000 - 1,500 + 700 ==> YD = 5,200

From YD = C + S ==> C = YD - S = 5,200 - 1,000 = 4,200

From S - I = BuD - TD ==> 1,000 - I = 230 - 120 ==> I = 890

13
From Y = C + I + G + NX ==> G = Y - C - I - NX

==> G = 6,000 - 4,200 - 890 + 120 = 1,030

Check: BuS = TA - TR - G ==> -230 = 1,500 - 700 - G ==> G = 1,030

18. From the information below, calculate the level of investment (I), consumption (C), and national income
(Y).
government spending G = 1,200 budget surplus BuS = 60
disposable income YD = 4,500 net exports NX = -110
saving S = 500

From YD = C + S ==> C = YD - S = 4,500 - 500 = 4,000

From S - I = BuD - TD ==> 500 - I = - 60 - 110 ==> I = 670

From Y = C + I + G + NX ==> Y = 4,000 + 1,200 + 670 - 110 = 5,760

19. Assume the government cuts its purchases by $60 billion. As a result, the budget deficit is reduced by $20
billion, savings increases by $10 billion, disposable personal income decreases by $15 billion and the trade
deficit is reduced by $5 billion. By how much have investment (I), consumption(C), and national income (Y)
changed?

From S - I = (G + TR - TA) + NX ==>

I = S - (G + TR - TA) - NX ==> I = S - BD - NX

==> I = 10 - (-20) - 5 = + 25

From YD = C + S ==> C = YD - S = - 15 - 10 = - 25

From Y = C + I + G + NX ==> Y = C + I + G + NX

==> Y = - 25 + 25 - 60 + 5 = - 55.

20. Will an increase in the federal budget surplus necessarily lead to a decrease in the foreign trade deficit?
Why or why not?

The equation I - S = BS - NX states that the difference between investment and private domestic saving is
equal to the difference between the budget surplus and the trade surplus. If the budget surplus increases (or the
budget deficit decreases), then U.S. interest rates are likely to decrease. This will cause an outflow of funds,
depreciating the value of the U.S. dollar and making the U.S. more competitive on world markets. Therefore
we will see an increase in the trade surplus (or a decrease in the trade deficit). However, this does not
necessarily always happen, since the other two variables in this equation, that is, investment and saving, may
also change. For example, if there is a significant increase in investment spending due to the lower interest
rates, then we may not see a change in the trade surplus.

14
21. "High budget deficits ultimately lead to foreign trade deficits." Comment on this statement.

From S + TA - TR = I + G + NX ==> S - I = - (TA - G - TR) - (-NX) = BuD - TD,


that is, the difference between saving and investment is equal to the difference between the budget deficit and
the trade deficit. In the early 1980's the size of the budget deficit increased sharply. Saving remained low and
thus interest rates increased. High U.S. interest rates attracted funds from abroad, which drove the value of the
U.S. dollar up, making U.S. less competitive on world markets. Thus the increase in the budget deficit in the
1980's was largely responsible for the increase in the trade deficit. However, an increase in the budget deficit,
does not necessarily increase the trade deficit, since the other two variables in this equation, i.e., investment
and saving may also be affected. As long as we can finance the increase in the budget deficit domestically, no
trade deficit has to result. Instead saving may increase or investment may be crowded out.

22. In the early 1980s, the U.S. changed from being one of the biggest creditor nations to being the biggest
debtor nation in the world. Explain how this happened.

In the 1980s, the U.S. changed from a creditor nation to a debtor nation, as its competitiveness in foreign
markets substantially decreased. A variety of factors contributed to this change (a decrease in productivity
growth, emphasis on short-term profits by U.S. businesses, and lack of long-term planning and research and
development). However, to a large extent the low savings rate, combined with huge federal budget deficits
were responsible. From the equation S - I = BuD - TD, we can see that if the budget deficit increases and
private domestic saving is not adequate to finance it, then either private domestic investment has to decrease or
the U.S. has to borrow from abroad, which will then lead to a trade deficit. The increased borrowing needs by
the U.S. government in the early 1980s caused U.S. interest rates to increase, which led to an inflow of funds
from abroad. This led to a sharp appreciation in the value of the U.S. dollar, making U.S. goods less
competitive on world markets. While U.S. firms sold fewer export goods on world markets, U.S. consumers
developed a taste for imported goods, and therefore a huge trade imbalance developed.

23. Assume you are a banker and you’d like a 4% real rate of return on your loans. If you expect that the
inflation rate is likely to average about 6% over the next thirty years, what would be the most likely mortgage
interest rate that you would charge your customers for a thirty year fixed rated mortgage? How would your
answer change if you expected a 4% average inflation rate over the length of the mortgage? Explain your
answers.

The Fisher equation states that the nominal interest rate is the expected real rate of interest (re) plus the
expected rate of inflation (e), or

i = re + e.

In other words, if you expected an inflation rate of 6%, you would charge your customers a 10% mortgage
interest rate since

i = 4% + 6% = 10%.

But if you expected only a 4% inflation rate, you would charge your customers a mortgage interest rare of

i = 4% + 4% = 8%.

15
Assume a government bond pays you a fixed interest rate of 5.5% per year and the average annual rate of
inflation is 4.2%. What is your real rate of return? How would this real rate of return change if inflation
increased to 6.8%?

The real interest rate is defined as the nominal interest rate minus the inflation rate, that is,

r=i-

Therefore your real rate of return is

r = 5.5% - 4.2% = 1.3%

if the inflation rate is 4.2%. But if the inflation rate increases to 6.8%, then your real rate of return will be
negative, that is,

r = 5.5% - 6.8% = - 1.3%.

CHAPTER 3

GROWTH AND ACCUMULATION

Solutions to the Problems in the Textbook

Conceptual Problems:

1. The production function provides a quantitative link between inputs and output. For example, the Cobb-
Douglas production function mentioned in the text is of the form:

Y = F(N,K) = AN1-K,

where Y represents the level of output. (1 - ) and  are weights equal to the shares of labor (N) and capital
(K) in production, while A is often used as a measure for the level of technology. It can be easily shown that
labor and capital each contribute to economic growth by an amount that is equal to their individual growth
rates multiplied by their respective share in income.

2. The Solow model predicts convergence, that is, countries with the same production function, savings rate,
and population growth will eventually reach the same level of income per capita. In other words, a poor
country may eventually catch up to a richer one by saving at the same rate and making technological
innovations. However, if these countries have different savings rates, they will reach different levels of income
per capita, even though their long-term growth rates will be the same.

3. A production function that omits the stock of natural resources cannot adequately predict the impact of a
significant change in the existing stock of natural resources on the economic performance of a country. For
example, the discovery of new oil reserves or an entirely new resource would have a significant effect on the
level of output that could not be predicted by such a production function.

16
4. Interpreting the Solow residual purely as technological progress would ignore, for example, the impact
that human capital has on the level of output. In other words, this residual not only captures the effect of
technological progress but also the effect of changes in human capital (H) on the growth rate of output. To
eliminate this problem we can explicitly include human capital in the production function, such that

Y = F(K,N,H) = ANaKbHc with a + b + c = 1.

Then the growth rate of output can be calculated as

Y/Y = A/A + a(N/N) + b(K/K) + c(H/H).

5. The savings function sy = sf(k) assumes that a constant fraction of output is saved. The investment
requirement, that is, the (n + d)k-line, represents the amount of investment needed to maintain a constant
capital-labor ratio (k). A steady-state equilibrium is reached when saving is equal to the investment
requirement, that is, when sy = (n + d)k. At this point the capital-labor ratio k = K/N is not changing, so capital
(K), labor (N), and output (Y) all must be growing at the same rate, that is, the rate of population growth n =
(N/N).

6. In the long run, the rate of population growth n = (N/N) determines the growth rate of the steady-state
output per capita. In the short run, however, the savings rate, technological progress, and the rate of
depreciation can all affect the growth rate.

7. Labor productivity is defined as Y/N, that is, the ratio of output (Y) to labor input (N). A surge in labor
productivity therefore occurs if output grows at a faster rate than labor input. In the U.S. we have experienced
such a surge in labor productivity since the mid-1990s due to the enormous growth in GDP. This surge can be
explained from the introduction of new technologies and more efficient use of existing technologies. Many
claim that the increased investment in and use of computer technology has stimulated economic growth.
Furthermore, increased global competition has forced many firms to cut costs by reorganizing production and
eliminating some jobs. Thus, with large increases in output and a slower rate of job creation we should expect
labor productivity to increase. (One should also note that a higher-skilled labor force also can contribute to an
increase in labor productivity, since the same number of workers can produce more output if workers are more
highly skilled.)

Technical Problems:

1.a. According to Equation (2), the growth of output is equal to the growth in labor times the labor share plus
the growth of capital times the capital share plus the rate of technical progress, that is,

Y/Y = (1 - )(N/N) + (K/K) + A/A, where

1 -  is the share of labor (N) and  is the share of capital (K). Thus if we assume that the rate of
technological progress (A/A) is zero, then output grows at an annual rate of 3.6 percent, since

Y/Y = (0.6)(2%) + (0.4)(6%) + 0% = 1.2% + 2.4% = + 3.6%,

1.b. The so-called "Rule of 70" suggests that the length of time it takes for output to double can be calculated

17
by dividing 70 by the growth rate of output. Since 70/3.6 = 19.44, it will take just under 20 years for output to
double at an annual growth rate of 3.6%,

1.c. Now that A/A = 2%, we can calculate economic growth as

Y/Y = (0.6)(2%) + (0.4)(6%) + 2% = 1.2% + 2.4% + 2% = + 5.6%.

Thus it will take 70/5.6 = 12.5 years for output to double at this new growth rate of 5.6%.
2.a. According to Equation (2), the growth of output is equal to the growth in labor times the labor share plus
the growth of capital times the capital share plus the growth rate of total factor productivity (TFP), that is,

Y/Y = (1 - )(N/N) + (K/K) + A/A, where

1 -  is the share of labor (N) and  is the share of capital (K). In this example  = 0.3; therefore, if output
grows at 3% and labor and capital grow at 1% each, then we can calculate the change in TFP in the following
way

3% = (0.3)(1%) + (0.7)(1%) + A/A ==> A/A = 3% - 1% = 2%,

that is, the growth rate of total factor productivity is 2%.

2.b. If both labor and the capital stock are fixed and output grows at 3%, then all this growth has to be
contributed to the growth in factor productivity, that is, A/A = 3%.

3.a. If the capital stock grows by K/K = 10%, the effect on output would be an additional growth rate of
Y/Y = (.3)(10%) = 3%.

3.b. If labor grows by N/N = 10%, the effect on output would be an additional growth rate of

Y/Y = (.7)(10%) = 7%.

3.c. If output grows at Y/Y = 7% due to an increase in labor by N/N = 10%, and this increase in labor is
entirely due to population growth, then per capita income would decrease and people’s welfare would
decrease, since

y/y = Y/Y - N/N = 7% - 10% = - 3%.

3.d. If this increase in labor is due to an influx of women into the labor force, the overall population does not
increase and income per capita would increase by y/y = 7%. Therefore people's welfare would increase.

4. Figure 3-4 shows output per head as a function of the capital-labor ratio, that is, y = f(k). The savings
function is sy = sf(k), and it intersects the straight (n + d)k-line, representing the investment requirement. At
this intersection, the economy is in a steady-state equilibrium. Now let us assume that the economy is in a
steady-state equilibrium before the earthquake hits, that is, the steady-state capital-labor ratio is currently k*.
Assume further, for simplicity, that the earthquake does not affect people's savings behavior.
If the earthquake destroys one quarter of the capital stock but less than one quarter of the labor force, then the
capital-labor ratio falls from k* to k1 and per-capita output falls from y* to y1. Now saving is greater than the

18
investment requirement, that is, sy1 > (d + n)k1, and the capital stock and the level of output per capita will
grow until the steady state at k* is reached again.
However, if the earthquake destroys one quarter of the capital stock but more than one quarter of the labor
force, then the capital-labor ratio increases from k* to k2. Saving now will be less than the investment
requirement and thus the capital-labor ratio and the level of output per capita will fall until the steady state at
k* is reached again.
If exactly one quarter of both the capital stock and the labor stock are destroyed, then the steady state is
maintained, that is, the capital-labor ratio and the output per capita do not change.
If the severity of the earthquake has an effect on peoples’ savings behavior, then the savings function sy = sf(k)
will move either up or down, depending on whether the savings rate (s) increases (if people save more, so more
can be invested in an effort to rebuild) or decreases (if people save less, since they decide that life is too short
not to live it up).
y

y = f(k)
y2
y* (n+d)k
y1
sy

0
k1 k* k2 k

5.a. An increase in the population growth rate (n) affects the investment requirement, and the (n + d)k-line gets
steeper. As the population grows, more saving must be used to equip new workers with the same amount of
capital that the existing workers already have. Therefore output per capita (y) will decrease as will the new
optimal capital-labor ratio, which is determined by the intersection of the sy-curve and the (n1 + d)k-line.
Since per-capita output will fall, we will have a negative growth rate in the short run. However, the steady-state
growth rate of output will increase in the long run, since it will be determined by the new and higher rate of
population growth.

y (n1 + d)k
y = f(k)

yo (no + d)k

y1 sy

19
0
k1 ko k
5.b. Starting from an initial steady-state equilibrium at a level of per-capita output y*, the increase in the
population growth rate (n) will cause the capital-labor ratio to decline from k* to k1. Output per capita will
also decline, a process that will continue at a diminishing rate until a new steady-state level is reached at y1.
The growth rate of output will gradually adjust to the new and higher level n1.

y*

y1

to t1 t

k*

k1

to t1 t

6.a. Assume the production function is of the form

Y = F(K, N, Z) = AKaNbZc ==>

Y/Y = A/A + a(K/K) + b(N/N) + c(Z/Z), with a + b + c = 1.

Now assume that there is no technological progress, that is, A/A = 0, and that capital and labor grow at
the same rate, that is, K/K = N/N = n. If we also assume that all natural resources available are fixed, such
that Z/Z = 0, then the rate of output growth will be

Y/Y = an + bn = (a + b)n.

In other words, output will grow at a rate less than n since a + b < 1. Therefore output per worker will fall.

20
6.b. If there is technological progress, that is, A/A > 0, then output will grow faster than before, namely

Y/Y = A/A + (a + b)n.

If A/A > c, then output will grow at a rate larger than n, in which case output per worker will increase.

6.c. If the supply of natural resources is fixed, then output can only grow at a rate that is smaller than the rate
of population growth and we should expect limits to growth as we run out of natural resources. However, if the
rate of technological progress is sufficiently large, then output can grow at a rate faster than population, even if
we have a fixed supply of natural resources.

7.a. If the production function is of the form

Y = K1/2(AN)1/2,

and A is normalized to 1, then we have

Y = K1/2N1/2 .

In this case capital's and labor's shares of income are both 50%.

7.b. This is a Cobb-Douglas production function.

7.c. A steady-state equilibrium is reached when sy = (n + d)k.

From Y = K1/2N1/2 ==> Y/N = K1/2N-1/2 ==> y = k1/2 ==>

sk1/2 = (n + d)k ==> k-1/2 = (n + d)/s = (0.07 + 0.03)/(.2) = 1/2 ==> k1/2 = 2 = y ==> k = 4 .

21
8.a. If technological progress occurs, then the level of output per capita for any given capital-labor ratio
increases. The function y = f(k) increases to y = g(k), and thus the savings function increases from sf(k) to
sg(k).

y g(k)

y2 f(k)

(n +d)k
sg(k)
y1
sf(k)

0
k1 k2 k

8.b. Since g(k) > f(k), it follows that sg(k) > sf(k) for each level of k. Therefore the intersection of the sg(k)-
curve with the (n + d)k-line is at a higher level of k. The new steady-state equilibrium will now be at a higher
level of saving and output per capita, and at a higher capital-labor ratio.

8.c. After the technological progress occurs, the level of saving and investment will increase until a new and
higher optimal capital-labor ratio is reached. The ratio of investment to capital will also increase in the
transition period, since more has to be invested to reach the higher optimal capital-labor ratio.

k2

k1

0
t1 t2 t

9. The Cobb-Douglas production function is defined as

Y = F(N,K) = AN1-K.

The marginal product of labor can then be derived as

22
MPN = (Y)/(N) = (1 - )AN-K = (1 - )AN1-K/N = = (1 - )(Y/N)

==> labor's share of income = [MPN*(N)]/Y = (1 - )(Y/N)*[(N)/(Y)] = (1 - )


Additional Problems

1. Assume labor's share of income is 80%, capital's share of income is 20%, and the level of technology is
fixed. Give a Cobb-Douglas aggregate production function with only two inputs, labor (N) and capital (K), that
will represent such an economy. If labor grows by 2.4% and capital grows by 1.2%, by how much will output
in this economy grow?

The Cobb-Douglas production function is of the general form:

Y = F(N,K) = AN1-K,

where Y represents the level of output. (1 - ) and  are weights equal to the shares of labor (N) and capital (K)
in production, while A is often used as a measure for the level of technology. In this case, this production
function has the following form:
Y = N0.8K0.2

The growth of output can be calculated by using the following formula

Y/Y = (1 - )(N/N) + (K/K) + A/A = (0.8)(2.4%) + (0.2)(1.2%) + 0%

= 1.92% + 0.24% = + 2.16%,

2. Assume a Cobb-Douglas aggregate production function in which labor's share of income is 80% and
capital's share of income is 20%. By how much would real GDP in this economy grow if labor grows at 3%,
the capital stock grows at 2% and total factor productivity increases by 2.4%?

The growth of output can be calculated as the sum of the growth in labor times the labor share plus the growth
of capital times the capital share plus the change in total factor productivity, that is,

Y/Y = (1 - )(N/N) + (K/K) + A/A, where

1 -  is the share of labor (N),  is the share of capital (K), and A/A is the change in total factor productivity.
Therefore, we can calculate total output growth as

Y/Y = (0.8)(3%) + (0.2)(2%) + 2.4% = 2.4% + 0.4% + 2.4% = 5.2%.

3. Assume a Cobb-Douglas aggregate production function in which labor's share of income is 80% and
capital's share of income is 20%. By how much will real GDP in this economy grow if labor grows at 2.5%,
the capital stock grows at 1.5%, and total factor productivity increases by 2.1%?

The growth of output can be calculated as the sum of the growth in labor times the labor share plus the growth
of capital times the capital share plus the change in total factor productivity, that is,

Y/Y = (1 - )(N/N) + (K/K) + A/A, where

23
1 -  is the share of labor (N) and  is the share of capital (K), and A/A is the change in total factor
productivity. Therefore, we can calculate total output growth as

Y/Y = (0.8)(2.5%) + (0.2)(1.5%) + 2.1% = 2.0% + 0.3% + 2.1% = 4.4%.

4. Assume that labor's share of income is three times as large as capital's share, and that capital, labor, and
output all grow at the same rate (n). Use a Cobb-Douglas aggregate production function that has constant
returns to scale to show that labor will be a larger source of growth than capital if there is no technological
progress.

With the share of labor being 1 -  = 0.75 and the share of capital being  = 0.25, we get the following
aggregate production function:

Y = AN1-K = AN3/4K1/4

We can get the growth accounting equation by taking the total derivative of this equation and then dividing
both sides of the equation by Y, that is

Y/Y = A/A + (1 - )(N/N) + (K/K) = 0 + (0.75)n + (0.25)n = n.

Therefore we see that labor contributes three times as much to growth as capital in the absence of
technological progress, that is, if A/A = 0.

5. Assume an economy in which the level of technology and the stock of capital are fixed. If there is a
sudden increase in the labor force due to immigration, how would the standard of living be affected? Would it
make a difference if the increase in the labor force were instead caused by a higher labor force participation
rate of women?

If the increase in the labor force were entirely due to an influx of foreigners, then the population would
increase and living standards would decrease, since per-capita income would decrease. However, if the
increase in the labor force were due to an increase in the labor force participation of women, then income per
capita would increase since the overall population would not increase. Therefore there would be an increase in
living standards.

6. Briefly distinguish between these two concepts: decreasing returns to scale and the law of diminishing
marginal returns.

Decreasing returns to scale occur when a proportional increase in all inputs leads to a less-than-proportional
increase in output. The law of diminishing marginal returns states that the marginal product of each unit of
input will decline as the amount of that input is increased, while all other inputs constant are held constant. In
other words, if we only increase one of several inputs, output will increase but at a decreasing rate.

7. Comment on the following statement:


"Technological advances increase output in the short run but have no impact on long-run growth in output per
capita, since this is determined entirely by the rate of population growth."

When technological advances occur, then each input becomes more productive and the total factor productivity

24
and growth of output rise. This implies that technological advances can have very profound implications for
output growth. Total output grows at a rate equal to the sum of the growth rate of the population and the rate of
technological progress.

8. "The Solow growth model predicts that a country with a low savings rate can eventually achieve the same
living standard as a country with a high savings rate as long as both countries have the same rate of population
growth and access to the same technology." Comment on this statement.

The Solow model predicts that two countries that have the same production function, savings rate, and
population growth will eventually reach the same level of income per capita. In other words, a poor country
may eventually catch up to a richer one by saving at an equivalent rate to that of the richer country and by
implementing technological innovations. However, if the two countries have different savings rates, then they
will reach different levels of income per capita, even though their growth rates ultimately will be the same.
Therefore the country with the lower savings rate will have a lower standard of living.

9. "The neoclassical growth model predicts that a nation's per-capita GDP can only grow if technological
progress occurs." Comment on this statement.

The neoclassical growth model generally assumes a production function with constant returns to scale. In the
absence of technological progress, this model reaches a steady-state equilibrium at an optimal capital-output
ratio k*. At this point output grows at the same rate as the population. This implies that output per capita
remains constant. In the short-run, however, the capital-output ratio k may be below the optimal level k*. If so,
the level of saving will exceed the level of investment needed to keep the capital-labor ratio constant. In this
case, the capital-labor ratio will increase (which implies that output per head will increase) until the optimal
level k* is reached. At k* the capital-labor ratio and output per head are both constant again.

10. "A higher population growth is always desirable since it will lead to higher living standards." Comment
on this statement with the help of a diagram derived from the neoclassical growth model.

If the rate of population growth (n) increases, the capital stock will grow at a lower rate than population. Since
the country has to feed its people, not enough will be saved and invested to keep the capital-labor ratio (k) at
its original level. Therefore, the capital-labor ratio (k) decreases until a new steady state is reached. In other
words, the investment requirement, that is, the (n+d)k-line gets, steeper as population growth (n) increases.
The (n+d)k-line will now intersect the sy-curve at a lower steady-state capital-labor ratio (k). This implies a
lower level of output per capita (y), and therefore a lower living standard. Thus the statement is wrong.

25
y
(n 1 +d)k

y1 y
y2
(n + d)k
sy

k2 k1 k

11. "If the U.S. can increase its national savings rate, it can achieve a higher long-term growth rate." Comment
on this statement with the help of a diagram derived from the neoclassical growth model.

If the savings rate increases, then the sy-curve shifts up and we get a new intersection with the (n+d)k line at a
higher steady state capital-labor ratio and a higher level of output per capita. In other words, as the level of
savings and investment increases, the labor force will be supplied with more capital until a new and higher
optimal capital-labor ratio (k) and output per capita level (y) is reached. Thus, the growth rate of real output
will increase only temporarily. But in the long run, while the level of output per capita is higher, the growth
rate of real output will again be equal to the population growth (n).

y
y2 f(k)
y1 (n + d)k

s’f(k)

sf(k)

k
k1 k2

12. "An increase in the depreciation rate reduces the level of output per capita and the capital-labor ratio."

26
Comment on this statement

An increasing depreciation rate (d) indicates that the capital stock is wearing out at a faster rate than
previously. This leads to a reduction in the optimal capital-labor ratio and in the level of output. In other words,
the (n + d)k-line becomes steeper as the depreciation rate increases. The savings function sy = sf(k) now
intersects the new (n + d')k-line at a lower capital-labor ratio (k), which implies a lower level of output per
capita (y).

13. True or false? Why?


"The neoclassical growth model predicts that a decrease in the savings rate permanently affects the growth rate
of real output."

False. As the savings rate decreases, a smaller proportion of output is saved at each level of the capital-labor
ratio (k). The savings curve shifts from sy = sf(k) to s1y = s1f(k). Now savings is less than the investment
requirement (n + d)k at the original capital-labor ratio k*, so the capital stock begins to grow more slowly.
During the transition to the new equilibrium (at a lower steady-state capital-labor ratio and a lower output per
capita), there is a temporary decline in the growth of output, and the level of output per capita decreases.
However, in the long run the growth rate of real output will not be affected.

yo y = f(k)
(n+d)k
y1
sy

s1y

0
ko k1 k

14. Comment on the following statement:


"A nation with a declining population growth will experience a fall in living standards; therefore governments
should design policies to increase population growth."

If a nation's population growth declines, then the current level of saving will be higher than the level of
investment required to maintain a constant capital-labor ratio. Therefore the capital-labor ratio will increase
until a new steady-state equilibrium is reached. The new optimal capital-labor ratio is determined by the
intersection of the saving-curve (sy) with the new (n1 + d)k-line, that is, the new investment requirement line.
The new and larger optimal capital-labor ratio (k*) implies a larger level of income per capita (y*) and
therefore a higher standard of living for the country's population. Therefore, there is no need to design policies
to increase population growth. As a matter of fact, as the preceding analysis suggests, an increase in population
growth would decrease living standards.

27
y y = f(k)
y1
(no +d)k
yo
(n1 +d)k
sy

0
ko k1 k

15. "A high level of public spending crowds out investment spending. Therefore, the government needs to cut
its expenditures to free resources for investment." Comment on this statement.

28
By decreasing the level of public spending and reducing the budget deficit, the government can increase the
level of national saving. Therefore, more funds will be available for investment. If the rate of capital
accumulation increases, then the growth rate of output can increase at least temporarily, and this will increase
the level of output per capita. However, the neoclassical growth model predicts that the long-term growth rate
will not be affected since this rate is solely determined by population growth and technological progress. In
addition, a decrease in government spending may, in the short run, lead to a recession, negatively affecting the
level of investment spending.

CHAPTER 4

GROWTH AND POLICY

Solutions to the Problems in the Textbook

Conceptual Problems:

1. Endogenous or self-sustained growth supposedly can be achieved by policies that affect a nation's savings
rate and therefore the proportion of GDP that goes towards investment. The neoclassical growth model of
Chapter 3 predicted that long-term growth can only be achieved through technological progress and that
changes in the savings rate have only transitory effects. The endogenous growth model, however, predicts that
countries with a higher savings rate can achieve higher long-term growth and that a nation's government can
affect the long-term growth rate by implementing policies that affect the savings rate.
2. A simple model with constant returns to scale to capital alone implies increasing returns to scale to all
factors taken together, which could cause a single large firm to dominate the economy. However, such a model
ignores the possibility that external returns to capital exist, in addition to the internal (private) returns. In other
words, more investment not only leads to a higher and more efficient capital stock but also to new ideas and
new ways of doing things, which can then be copied by others. Therefore, a single firm does not necessarily
reap all of the benefits of increased output.

In the neoclassical growth model, an increase in the savings rate does not increase the long-term growth rate of
output. However, because of the short-run adjustment process, there is some transitional gain that will lead to a
higher level of output per capita. In the endogenous growth model, however, the savings rate does affect the
long-term growth rate of output.

4.a. Chapter 4 suggests that the key to long-term economic growth is investment in human and physical
capital with particular emphasis on research and development.

4.b. (i) Investment tax credits may potentially affect economic growth in the long run by achieving a higher
rate of technological progress.
(ii) R&D subsidies and grants lead to technological advances that will have private and social returns.
They are very effective in stimulating long-term economic growth.

69
(iii) According to the endogenous growth model, policies designed to increase the savings rate will
increase the long-term growth rate of output. However, empirical evidence does not lend much support to that
notion.
(iv) Increased funding for primary education has large private and social returns and is therefore an
excellent means to stimulate long-term growth, even though it may take a long time until these policies have
their full effect.

5. The notion of absolute convergence states that economies with the same savings rate and rate of
population growth will reach the same steady-state equilibrium if they have access to the same technology. The
notion of conditional convergence states that economies that have access to the same technology and the same
rate of population growth but different savings rates will reach steady-state equilibria at a different level of
output but the same economic growth rate. There is empirical evidence to support the notion of conditional
convergence across countries.

6. Endogenous growth theory assumes that the steady-state growth rate of output is affected by the rate at
which the factors of production are accumulated. Therefore, an increase in the savings rate would increase the
rate at which the capital stock is accumulated and this would increase the growth rate of output. While this
notion may be important in explaining the growth rates of highly developed countries at the leading edge of
technology, it cannot explain the differences in growth rates across poorer countries. For these countries, the
notion of conditional convergence seems to hold.

7. Investing in physical capital will lead to a higher capital stock and to a higher level of output in the short
run, but often to the detriment of long-term growth unless there are significant external returns to capital.
Therefore, investing in human capital is a better strategy, since it has high returns and leads to an increase in
long-term growth.

8.a. A country that is able to choose its rate of population growth through population control policies can shift
the investment requirement down, thereby increasing the level of steady-state output. With a lower rate of
population growth it is possible to achieve a higher level of income per capita with a lower level of investment
spending. Therefore, implementing population control policies may be an effective way to escape the so-called
poverty trap.

8.b. In an endogenous growth model, a lower population growth rate (n) will increase a nation's long-term
growth rate (y/y). We can see this since, in the second optional section, the per-capita growth rate was
derived, as follows:

y/y = sa - (n + d).

9. The Asian Tigers (Hong Kong, Singapore, South Korea, and Taiwan) experienced a high rate of economic
growth between 1966 and 1990 by concentrating on improving the education of the population and increasing
the savings rate, as suggested by the endogenous growth model. However, increases in the labor forces of these
countries suggested by the neoclassical growth model, were also at work.

10. The decline in living standards experienced by Eastern European countries in transition from centrally
planned economies to free market economies cannot easily be explained by neoclassical or endogenous growth

70
theory. The decline in GDP in these countries was largely due to disorganized markets that lacked properly
assigned property rights or liability rules and an insufficiently developed banking system. In addition, the need
for large-scale replacement of outdated production technology caused further disruption.

11. In is unclear whether countries can actually experience indefinite increases in their growth potential.
However, if technological advances occur continuously and if intelligent resource management is practiced, it
is conceivable that economic growth will continue for a very, very long time.

Technical Problems:

1.a. A production function that displays both a diminishing and a constant marginal product of capital can be
displayed by drawing a curved line (as in an exogenous growth model), followed by a upward-sloping line (as
in an endogenous growth model). Such a graph is depicted below.

1.b. The first equilibrium (Point A in the graph below) is a stable low-income steady-state equilibrium. Any
deviation from that point will cause the economy to eventually adjust again at the same steady-state income
level (and capital-output ratio). The second equilibrium (Point B) is an unstable high-income steady-state
equilibrium. Any deviation from that point will lead to either a lower income steady-state equilibrium (if the
capital-labor ratio declines) or ongoing growth (if the capital-labor ratio increases).

y
y = f(k)

yB
sy

B (n + d)k
yA

0
kA kB k

1.c. A model like the one in this question can be used to explain how some countries can find themselves in a
situation with no growth and low income while others have ongoing growth and a high level of income. In the
first case, a country may have invested in physical capital, leading to some short-term growth at the expense of
long-term growth, whereas in the second case, a country may have invested heavily in human capital, reaping
significant social returns.

2.a. If population growth is endogenous, that is, if a country can influence the rate of population growth
through government policies, then the investment requirement is no longer a straight line. Instead it is curved
as depicted below.

71
y

yC y = f(k)

yB [n(y) + d)]k
C sf(k)
yA B

0
kA kB kC k
2.b. The first equilibrium (Point A) is a stable steady-state equilibrium. It is a situation of low income and high
population growth, indicating that the country is in a poverty trap. The second equilibrium (Point B) is an
unstable steady-state equilibrium. It is a situation of medium income and low population growth. The third
equilibrium (Point C) is a stable steady-state equilibrium. It is a situation of high income and low population
growth. None of these three equilibria have ongoing growth.

2.c. To escape the poverty trap (Point A), a country has several possibilities: First, it can somehow find the
means to increase the capital-labor ratio above a level consistent with Point B (perhaps by borrowing funds or
seeking direct foreign investment). Second, it can increase the savings rate such that the savings function no
longer intersects the investment requirement curve at either Point A or Point B. Third, it can decrease the rate
of population growth through specifically designed policies, such that the investment requirement shifts down
and no longer intersects with the savings function at Point A or Point B.

3.a. If we incorporate endogenous population growth into a two-sector model in Problem 2, then we get a
curved line for the investment requirement line and a production function with first a diminishing and then a
constant marginal product of capital as depicted below. (Note that the savings function has the same shape as
the production function.)

y = f(k)
yD

sf(k)

D [n+d)]k

yC
yB
yA C

72
A

0
kA kB kC kD k

3.b. There should be four intersections of the savings function and the investment requirement. The first
equilibrium (at Point A) is a stable low-income steady-state equilibrium. Any deviation from that point will
cause the economy to eventually adjust again at the same steady-state income level (and capital-output ratio).
The second equilibrium (at Point B) is an unstable low-income equilibrium. Any deviation from that point will
lead to either a lower income steady-state equilibrium at Point A (if the capital-labor ratio declines) or a higher
income steady-state equilibrium at Point C (if the capital-labor ratio increases). Point D is again an unstable
equilibrium but at a high level of income. Any deviation from that point will lead to either a lower income
steady-state equilibrium at Point C (if the capital-labor ratio declines) or ongoing growth (if the capital-labor
ratio increases).

3.c. This model is more inclusive than either of the two models discussed previously, and therefore has greater
explanatory power. But now the graphical analysis is far more complicated. It may not be worth the effort to
introduce such complications.

4.a. The production function is of the form

Y = K1/2(AN)1/2 = K1/2(4[K/N]N)1/2 = K1/2(4K)1/2 = 2K

4.b. Since a = y/k = 2, it follows that the growth rate of output is

g = sa - (n + d) = (0.1)2 - (0.02 + 0.03) = 0.15 = 15%.

4.c. The term "a" in the equation above stands for the marginal product of capital. If we assume that the level
of labor-augmenting technology (A) is proportional to the capital-labor ratio (k), we imply that the level of
technology depends on the amount of capital per worker that we have, which may not be realistic.

4.d. In this model, we have a constant marginal product of capital, and therefore we have an endogenous
growth model.

5.a. The production function is of the form

Y = K1/2N1/2 ==> Y/N = (K/N)1/2 ==> y = k1/2.

From k = sy/(n + d) = sk1/2/(n +d) ==> k1/2 = s/(n + d)

==> y* = s/(n + d) = (0.1)/(0.02 + 0.03) = 2

==> k* = sy*/(n + d) = (0.1)(2)/(0.02 + 0.03) = 4

5.b. Steady-state consumption equals steady-state income minus steady-state investment, that is,

73
c* = f(k*) - (n + d)k* .

The golden rule capital stock corresponds to the highest permanently sustainable level of consumption. Steady-
state consumption is maximized when the marginal increase in capital produces just enough extra output to
cover the increased investment requirement.

From c = k1/2 - (n + d)k ==> (c/k) = (1/2)k-1/2 - (n + d) = 0

==> k-1/2 = 2(n + d) = 2(.02 + .03) = .1==> k1/2 = 10 ==> k = 100

Since k* = 4 < 100, we have less capital at the steady state than the golden rule suggests.

5.c. From k = sy/(n + d) = sk1/2/(n + d) ==> s = k1/2(n + d) = 10(0.05) = .5

5.d. If we have more capital than the golden rule suggests, then we are saving too much and we do not have the
optimal amount of consumption.

Additional Problems

1. "Increasing returns to scale imply that the level of output increases only if the level of all inputs are
increased by the same amount." Comment on this statement.

The level of output will increase as soon as the level of one input is increased, even if the level of other inputs
remains the same. This is always true, even under the assumption of decreasing or constant returns to scale.
Increasing returns to scale imply that the level of output increases at a increasing rate if the level of one factor
input is increased while the levels of all other factor inputs remain the same. Similarly, it follows that if there
are increasing returns to scale, and the level of all inputs are doubled, then the level of output will more than
double.

2. Assume the aggregate production function is of the following form: Y = aK. At what capital-labor ratio
(k) can a steady-state equilibrium be reached?

From the production function Y = aK, it follows that y = Y/N = a(K/N) = ak. Therefore, the savings function is
sy = s(ak) and has a constant slope sa > (n + d). There will never be an intersection between the savings
function and the investment requirement and thus a steady-state equilibrium cannot be reached.

3. “The endogenous growth model predicts conditional convergence.” Comment.

This statement is false. Conditional convergence is the notion that countries with different savings rates but the
same rate of population growth and access to the same technology will achieve the same long-term growth rate
even though they may achieve a different standard of living. This is contrary to the endogenous growth model,
which predicts that there is a positive correlation between savings rates and growth rates across countries.

4. What are social returns to capital and why are they so important for economic growth?

Paul Romer suggested that private returns to capital should be separated from social returns to capital. If a

74
country invests in new technology, the result is not only an increase in the capital stock that will produce a
higher level of output, but also the development of new ways of doing things. Since new production methods
may be applied elsewhere, they create external (social) benefits. In addition, one new idea generally leads to
others and therefore knowledge may grow indefinitely. Thus it pays to devote resources to developing human
capital, particularly research and development, since this will enhance economic growth.

5. "Higher population growth is always desirable since it will lead to higher living standards. Therefore,
nations should always implement policies that will lead to an increase in population." Comment on this
statement.

In the neoclassical framework, we learned that if the rate of population growth (n) increases, the capital stock
will also grow but at a lower rate. Since a country has to feed its people, not enough can be saved and invested
to keep the capital-labor ratio (k) at its original level. Thus, the capital-labor ratio (k) decreases until a new
steady state is reached. In other words, the investment requirement, (the [n + d]k-line) gets steeper as
population growth (n) decreases, and it will now intersect the savings function at a lower steady-state capital-
labor ratio (k). This implies a lower level of output per capita (y) and therefore a lower living standard.
If we now assume that population growth is endogenous, then the [n(y) + d]k-function (the investment
requirement) is no longer straight but instead is curved. In this case, a country that has a high rate of population
growth but a low income level can find itself in a poverty trap. Such a nation needs to implement population
control policies to increase living standards.

6. "One implication of the endogenous growth model is that countries with a higher savings rate will have a
higher economic growth rate, but only over a transitional period. In the long run, only technological advances
can bring about economic growth." Comment on this statement.

Actually, it is the neoclassical growth model that predicts that countries with access to the same technology
and the same rate of population growth but a different savings rate will reach a steady-state equilibrium. In
other words, the countries will achieve a different level of income per capita but will have the same long-term
growth rate. Endogenous growth models predict that savings rates and growth rates are positively correlated
across countries.

7. "A poor country can escape the poverty trap, if it either devotes a larger share of GDP to investment or
implements a population control program." Comment on this statement.

A poor country may be hard pressed to devote a larger share of its GDP to investment if there are barely
enough resources to feed the population. Therefore, attracting direct foreign investment or borrowing foreign
funds for capital investment projects may be a more feasible solution, as long as these funds are invested
wisely. A better alternative for increasing living standards is to control population growth. However, such
population control policies are not easily implemented and their success is not always guaranteed, since there
is often resistance among the people.

8. Can a poor country ever catch up with a rich country if both have the same population growth? Explain
your answer.

Over the long run, the growth rate of output is determined by the rate of population growth and the rate of
technological progress. In the short run, a nation's growth rate can be affected by investment in machinery,

75
infrastructure, and human capital. It is impossible to predict for sure whether lower-income countries can ever
succeed in catching up with the standard of living of higher-income countries. Empirical evidence suggests
slow "conditional convergence," that is, that the positive impact of a higher level of investment spending is
only transitory, leading to a higher level of income per capita but not a higher growth rate. Countries will
converge to steady states, depending on the share of investment to GDP, the share of government spending to
GDP, and the rate of investment in human capital. However, the process of such convergence is extremely
slow.

9. Assume a developing country that has barely enough resources to feed its people and too little to devote
resources to investment to increase per-capita output. How can such a country successfully increase the level
of investment without reducing current consumption?

To increase the level of investment, a developing country can invite foreign firms to invest directly in its
economy. An example of such direct foreign investment is Volkswagen building a manufacturing plant in
Mexico. Another possibility is to borrow funds in the world capital market, either from a bank in a foreign
country or from the World Bank. A third possibility is to solicit foreign aid from an industrialized country.
However, even if countries receive funds from abroad to increase the level of their domestic investment, the
efficiency of investment will vary widely from one country to another. A country with very inefficient resource
allocation may have difficulty achieving a sufficient increase in economic growth. Pro-growth policies should
emphasize investment in machinery, the nation's infrastructure, and human capital as well as efforts to reduce
budget deficits and government regulations.

10. Comment on the following statement:


“The Asian Tigers achieved phenomenal economic growth from 1966-1990 by devoting a large share of GDP
to investment and by relying on laissez-faire economics.”

The Asian Tigers (Hong Kong, Singapore, South Korea, and Taiwan) experienced a high rate of economic
growth by increasing their savings rate so more could be invested. In addition, however, they increased their
labor forces and concentrated on improving the education of their people. While Hong Kong relied largely on
laissez-faire economics, there was significant government involvement in Singapore, where many investment
projects were directed by the government.

11. Assume the government wants to increase the long-term growth rate of the economy and considers the
policy options listed below. In your opinion, which of these policies will be successful in stimulating long-term
growth and why?
(a) expansionary monetary policy
(b) an increase in government spending
(c) tax credits to stimulate new research and development
(d) policies to improve education
(e) policies to control population growth

a. Expansionary monetary policy may affect real output in the short run and the rate of inflation in the long
run, however it does not affect the long-term growth rate. By affecting saving and investment, it may affect the
level of GDP per capital but not the growth rate of GDP.

b. An increase in government spending may have some short-run effects on real output, but it will not affect
the long-term growth rate of GDP. Only increases in spending affecting either the level of education (human

76
capital), the level research and development, or investment spending (physical capital) may potentially affect
economic growth in the long run.

c. Investment tax credits are designed to create incentives to undertake new research and development
which may, in turn, lead to a higher rate of technological progress. Such a policy may potentially affect
economic growth in the long run.

d. Policies that support education will lead to an increase in human capital and this, in turn, will affect the
long-term growth rate. However, this is likely to be a very slow process.

e. Policies designed to affect the population growth rate will affect the steady-state level of growth. For
example, in developing countries that have rapid rates of population growth, most of the available resources
are needed to feed and educate people. This leaves only limited resources for investment in new capital
equipment that would allow for an increase in output per capita and a higher standard of living.

12. Developing countries can increase productivity growth more easily than industrial countries. Does this
imply that they eventually are able to achieve the same living standards as industrial countries? Why or why
not?

Developing countries tend to have lower levels of per-capital income than industrialized countries. If they are
able to obtain new technology from industrialized nations, they can increase their productivity growth fairly
rapidly. At the start of the process of industrialization, developing countries may have low income levels, but
eventually they will begin to catch up with the more industrialized countries. As the income levels in the
poorer countries begin to grow, however, a point will eventually be reached when productivity growth will fall
off. Thus, it is impossible to predict with certainty whether a developing country will ever succeed in catching
up with the standard of living of an industrial country.
The economic growth rate of a country is largely determined by how much technological progress is
made and how much is invested in human capital. The endogenous growth theory does not predict a
convergence of income levels among countries starting from different initial levels of capital stock. The
neoclassical growth theory, on the other hand, predicts a conditional convergence, that is, that countries will
eventually converge to steady states, which are determined by the share of investment to GDP, the share of
government spending to GDP, and the rate of investment in human capital. Empirical evidence tends to support
this prediction. The process is likely to be extremely slow and is likely to be much easier for newly
industrialized countries than for less developed nations.

13. Many countries in Eastern European are still in the process of transforming their previously centrally
planned economies, where virtually all property was owned by the state and unemployment was non-existent,
to more market-based economies. What reform strategies do you think these countries should have employed
to increase living standards and what are the chances for fast results of such strategies?

Policies that can be implemented by countries undergoing the transformation from a centrally planned system
to a free market system include the following measures:

Establishing property rights and developing contract and bankruptcy laws


Privatizing government-owned firms
Removing price controls and price subsidies
Providing access to foreign markets for consumers and firms
Restoring macroeconomic stability through responsible fiscal and monetary policies

77
Education and training of business managers, bankers, lawyers, etc., to familiarize them with a more
competitive business environment
Providing a social safety net for those who experience difficulties surviving in a more competitive marketplace
Shaking up the current bureaucracy that is predisposed against economic reform

Implementing these policies is not likely to be easy or to achieve positive results immediately. As we have
seen, most of the Eastern European countries that have implemented such policies have experienced a decline
in GDP. How long it will take to turn the situation around will vary from country to country. There will also
probably be great social costs (increases in unemployment and the crime rate, a widening income gap, etc.).

CHAPTER 5

Solutions to the Problems in the Textbook

Conceptual Problems:

1. The aggregate supply curve shows the quantity of real total output that firms are willing to supply at each
price level. The aggregate demand curve shows all combinations of real total output and the price level at
which the goods and the money sectors are simultaneously in equilibrium. Along the AD-curve nominal money
supply is assumed to be constant and no fiscal policy change takes place.

2. The classical aggregate supply curve is vertical, since the classical model assumes that nominal wages
adjust very quickly to changes in the price level. This implies that the labor market is always in equilibrium
and output is always at the full-employment level. If the AD-curve shifts to the right, firms try to increase
output by hiring more workers, who they try to attract by offering higher nominal wages. But since we are
already at full employment, no more workers can be hired and firms merely bid up nominal wages. The
nominal wage increase is passed on in the form of higher product prices. In the end, the level of wages and
prices will have increased proportionally, while the real wage rate and the levels of employment and output
will remain unchanged.
If there is a decrease in demand, then firms try to lay off workers. Workers, in turn, are willing to accept
lower wages to stay employed. Lower wage costs enable firms to lower their product prices. In the end,
nominal wages and prices will decrease proportionally but the real wage rate and the level of employment and
output will remain the same.

3. There is no single theory of the aggregate supply curve, which shows the relationship between firms'
output and the price level. A number of competing explanations exist for the fact that firms have a tendency to
increase their output level as the price level increases. The Keynesian model of a horizontal aggregate supply
curve supposedly describes the very short run (over a period of a few months or less), while the classical
model of a vertical aggregate supply curve is supposed to hold true for the long run (a period of more than 10
years). The medium-run aggregate supply curve is most useful for periods of several quarters or a few years.
This upward-sloping aggregate supply curve results from the fact that wage and price adjustments are slow and
uncoordinated. Chapter 6 offers several explanations for the fact that labor markets do not adjust quickly.
These include the imperfect information market-clearing model, the existence of wage contracts or
coordination problems, and the fact that firms pay efficiency wages and price changes tend to be costly.

78
4. The Keynesian aggregate supply curve is horizontal since the price level is assumed to be fixed. It is most
appropriate for the very short run (a period of a few months or less). The classical aggregate supply curve is
vertical and output is assumed to be fixed at its potential level. It is most appropriate for the long run (a period
of more than 10 years) when prices are able to fully adjust to all shocks.

5. The aggregate supply and aggregate demand model used in macroeconomics is not very similar to the
market demand and market supply model used in microeconomics. While the workings of both models (the
distinction between shifts of the curves versus movement along the curves) are similar, these models are really
unrelated. The "P" in the microeconomic model stands for the relative price of a good (or the ratio at which
two goods are traded), whereas the "P" in the macroeconomic model stands for the average price level of all
goods and services produced in this country, measured in money terms.

Technical Problems:

1.a. As Figure 5-9 in the text shows, a decrease in income taxes will shift both the AD-curve and the AS-curve
to the right. The shift in the AD-curve tends to be fairly large and, in the short run (when prices are fixed),
leads to a significant increase in output without a change in prices. In the long run, the AS-curve will also shift
to the right--since lower income tax rates provide an incentive to work more--but only by a fairly small
amount. Therefore we see a slightly higher real GDP with a large increase in the price level in the long run.

1.b. Supply-side economics is any policy measure that will increase potential GDP by shifting the long-run
(vertical) AS-curve to the right. In the early 1980s, supply-side economists put forth the view that a cut in
income tax rates would increase the incentive to work, save and invest. This would increase aggregate supply
so much that the inflation and unemployment rates would simultaneously decrease. The resulting high
economic growth might then even lead to an increase in tax revenues, despite lower tax rates. However, these
predictions did not become reality. As seen in the answer to 2.a., the long-run effect of a tax cut on output is
not very large, although it can increase long-term output to some degree.

2.a. According to the balanced budget theorem, a simultaneous and equal increase in government purchases
and taxes will shift the AD-curve to the right. But if the AS-curve is upward sloping, then the balanced budget
multiplier will be less than one, that is, the increase in output will be less than the increase in government
expenditures. This occurs, since part of the increase in government spending will be crowded out due a higher
price level, lower real money balances, and a resulting rise in interest rates.

P ADo AD1 AS

Po
P1

79
Yo Y1 Y

2.b. In the Keynesian case, the AS-curve is horizontal and the price level remains unchanged. There is no real
balance effect and therefore income will increase more than in 3.a. However, the interest rate will still increase
and therefore the balanced budget multiplier will be less than one (but greater than zero).

P
ADo AD1

Po AS

0
Yo Y1 Y

2.c. In the classical case, the AS-curve is vertical and the output level remains unchanged. In this case, a shift
in the AD-curve leads to a price increase and real money balances decline. Therefore interest rates increase
further than in 3.b., leading to full crowding out of investment. Hence the balanced budget multiplier is zero.

P
AD1 AS
AD0
P1

P0

Y* Y

Additional Problems

1. Briefly explain why the AS-curve is upward sloping in the intermediate run?

An upward-sloping AS-curve assumes that wage and price adjustments are slow and uncoordinated. This can
be explained most easily by the existence of wage contracts and imperfect competition. Because of wage
contracts, wages cannot be changed easily and, since the contracts tend to be staggered, they cannot be
changed all at once. In an imperfectly competitive market structure, firms are reluctant to change their prices
since they cannot accurately predict the reactions of their competitors. Therefore, wages and prices will adjust
only slowly. (Chapter 6 provides more elaborate explanations for this.)

80
2. Briefly discuss in words why the AD-curve is downward sloping.

In the AD-AS framework, we assume that nominal money supply (M) is constant unless it is changed by the
Fed's monetary policy (which would result in a shift in the AD-curve). Therefore, if the price level increases,
then real money (M/P) decreases, driving interest rates (i) up and lowering the level of investment spending (I).
This means that total output demanded (Y) will decrease.
A more elaborate answer may include that lower real money balances (M/P) result in less real wealth, leading
to a lower level of consumption (C) due to the wealth effect. This means that total output demanded (Y) will
decrease. A higher domestic price level (P) also means that domestic goods will become less competitive in
world markets. This will stimulate imports while reducing exports, leading to a reduction in net exports (NX),
and a decrease in total output demanded (Y).

3. "In the classical aggregate supply curve model, the economy is always at the full-employment level of
output and the unemployment rate is always zero." Comment on this statement.

The classical aggregate supply curve model implies a vertical AS-curve at the full-employment level of
output. However, this does not mean that the unemployment rate is zero. There is always some friction in the
labor market, which means that there is always some (frictional) unemployment as workers switch jobs. The
(positive) amount of unemployment at the full-employment level of output is called the natural rate of
unemployment and is estimated to be roughly 5.5 percent for the United States; however, an exact value for
this natural rate has not been established.

4. Assume a technological advance leads to lower production costs. Show the effect of such an event on
national income, unemployment, inflation, and interest rates with the help of an AD-AS diagram, assuming
completely flexible wage rates.

A decrease in production costs shifts the AS-curve to the right. The price level decreases, leading to a higher
level of income and lower interest rates. Since wages are completely flexible, the AS-curve is vertical and we
are always at full-employment (this is the classical case). This implies that the unemployment rate stays at the
natural rate, but output goes up since workers are now more productive.

1.2. Cost of prod. == > AS  Ex.S. == > P real ms  i  I Y

Effect: Y UR  P i

ASo AS1

Po

P1

AD

81
0
YoFE Y1FE Y

5. "Monetary expansion will not change interest rates in the classical AS-curve model." Comment on this
statement.

An increase in the nominal money supply will shift the AD-curve to the right. There will be excess
demand for goods and services, which will force the price level up. In the classical AS-curve model, a new
equilibrium will be established at the same level of output but at a higher price level. Real money balances will
be reduced to their original level and interest rates will not be affected in the long run (the classical case).

6. "Expansionary fiscal policy does not affect the level of real output or real money balances in the classical
AS-curve case." Comment on this statement.

Expansionary fiscal policy will shift the AD-curve to the right, causing excess demand for goods and
services at the existing price level. This forces the price level up, reducing real money balances. Interest rates
increase, which results in a lower level of investment spending. In the classical case, the AS-curve is vertical,
so the level of output will not change. In other words, the increase in the level of prices and interest rates
continues until private spending is reduced again to the original full-employment level.

7. "In the classical AS-curve case, a reduction in government spending will lower interest rates and the real
money stock." Comment on this statement.

A decrease in government spending will shift the AD-curve to the left, causing excess supply of goods and
services at the original price level. As the price level decreases to restore equilibrium, real money balances
increase and interest rates fall. This will increase the level of investment spending until a new equilibrium is
reached at the original level of output but at lower prices and interest rates. Thus, real money balances will
rise, but interest rates fall.

8. "In the Keynesian aggregate supply curve model, the Fed, through restrictive monetary policy, can easily
lower inflation without creating unemployment." Comment on this statement.

This statement is wrong. In the Keynesian aggregate supply curve model, the AS-curve is horizontal, since
prices are assumed to be fixed. Restrictive monetary policy will shift the AD-curve to the left. This will reduce
the level of output without any change in the price level. But a lower level of output implies a higher rate of
unemployment.

9. True or false? Why?


"Monetary policy does not affect real output in the Keynesian supply curve model."

False. An increase in money supply will shift the AD-curve to the right, leading to a higher level of
income. In the Keynesian supply curve model, the price level is fixed, hence real balances will not fall as they
would in the classical supply curve model. We will reach a new equilibrium at a higher level of output, at a
lower interest rate, but at the same price level. In this case monetary policy is not neutral.

82
10. Explain why there is so much interest in finding ways to shift the AS-curve to the right.

Shifting the AS-curve to the right seems to be the only way to offset the effects of an adverse supply shock
without negative side effects. An adverse supply shock, such as an increase in oil prices, causes a simultaneous
increase in unemployment and inflation, and policy makers have only two options for demand-management
policies. Expansionary fiscal or monetary policy will help to achieve full employment faster but will raise the
price level, while restrictive fiscal or monetary policy will reduce inflationary pressure but increase
unemployment. Therefore, any policy that would shift the short-run AS-curve back to the right seems
preferable, since it might bring the economy back to the original equilibrium by simultaneously lowering
inflation and unemployment.

11. "Restrictive fiscal policy will always lower output, prices, and interest rates." Comment.

This statement is true in the intermediate run when the AS-curve is upward sloping. Restrictive fiscal policy
will shift the AD-curve to the left. In the Keynesian case, the AS-curve is horizontal and prices remain
constant, while both output and interest rates decrease. In the classical case, the AS-curve is vertical and the
decrease in the price level will increase real money balances and interest rates. Prices will fall until spending is
again consistent with the full-employment level of output. Thus in the long run, prices and interest rates will
decline, while output will remain the same. Only in the intermediate run (when the AS-curve is upward sloping
due to slowly adjusting wages and prices) will output, prices, and the interest rate all go down.

12. "The real impact of demand management policy is largely determined by the flexibility of wages and
prices." Comment on this statement.

If wages and prices are completely flexible, then the economy will always be at the full-employment level of
output, independent of the price level. In other words, we have the classical case of a vertical (long-run) AS-
curve. In this case, a shift in the AD-curve will affect only the price level but not the level of output. However,
if wages and prices are completely inflexible, then we have the (horizontal) Keynesian aggregate supply curve.
In this case, any shift of the AD-curve will have a large effect on the level of output but will not affect the price
level. Only in the intermediate run, when we have an upward-sloping AS-curve, will the level of output and the
price level both be affected by a shift in the AD-curve. More flexibility in wages and prices implies a steeper
the AS-curve. Therefore the effect of a shift in the AD-curve will be smaller on output and larger on the price
level.

"An increase in the income tax rate will lower the level of output and increase the price level." Comment on
this statement.

Supply siders argue that a decrease in income taxes will shift both the AD-curve and the AS-curve to the right
(as shown in Figure 5-10). Conversely, an increase in the income tax rate will shift the AD-curve and the AS-
curve to the left. The shift in the AD-curve will be fairly large and, in the medium run, will lead to a significant
decrease in output without a (significant) change in the price level. However, in the long run, the AS-curve will
also shift to the left, since higher income tax rates provide a disincentive to work. Since the AS-curve will shift
only by a fairly small amount, we will see a slightly lower real GDP with a large decrease in the price level.

83
CHAPTER 6

AGGREGATE SUPPLY: WAGES, PRICES, AND UNEMPLOYMENT

Solutions to the Problems in the Textbook:

Conceptual Problems:

1. The aggregate supply curve and the Phillips curve describe very similar relationships and both curves can
be used to analyze the same phenomena. The AS-curve shows a relationship between the price level and the
level of output. The Phillips curve shows a relationship between the rate of inflation and the unemployment
rate, given certain inflationary expectations. For example, a movement along the AS-curve depicts an increase
in the price level that is associated with an increase in the level of output. As output increases, the rate of
unemployment decreases (see Okun’s law). Therefore, with a larger increase in the price level (a higher level
of inflation) there will be a decrease in unemployment, creating a downward-sloping Phillips curve. This
downward sloping Phillips curve shifts whenever inflationary expectations change. If one assumes that
workers will change their wage demands whenever their inflationary expectations change, one can conclude
that a shift in the Phillips curve corresponds to a shift in the upward sloping AS-curve, since higher wages
mean higher cost of production.

2. In the short run, when wages and prices are assumed to be fixed, there can be no inflation and thus the
Phillips curve makes no sense over this very brief time frame. But in the medium run (in this chapter also often
referred to as the short run), the Phillips curve is downward sloping as inflationary expectations are assumed to
be constant. In the long run, the Phillips curve is vertical at the natural rate of unemployment, which
corresponds to the vertical long-run AS-curve at the full-employment level of output.

3. A variety of explanations are given in this chapter for the stickiness of wages in the short or intermediate
run. One is that workers have imperfect information and nobody knows the actual price level. People don’t
know whether a change in their nominal wage is the result of an increase in prices or in the real wage they
receive for the work they provide. Due to this uncertainty, labor markets will not clear immediately. Another
argument relies on coordination problems, that is, different firms within an economy cannot coordinate price
changes in response to monetary policy changes. Individual firms change their prices only reluctantly, since
they are afraid of losing market share. The efficiency wage theory argues that employers pay above market-
clearing wages to motivate their workers to work harder. Firms are also reluctant to change wages because of
the perceived menu costs involved. There are long-term relations between firms and workers and wages are
usually set in nominal terms by wage contracts, which are renegotiated only periodically. Thus real wages
fluctuate over time as the price level changes. Finally, the insider-outsider model argues that firms negotiate
only with their own employees but not with unemployed workers. Since a turnover in the labor force is costly
to firms, they are willing to offer above market-clearing wages to the currently employed rather than hiring the
unemployed who may be willing to work for lower wages.
These different views are not necessarily mutually exclusive and it is up to students to decide which
of the arguments presented here they find most plausible. The explanations differ mainly in their assumption of
how fast markets clear and whether employment variations are voluntary.

4.a. Stagflation is defined as a period of high unemployment accompanied by high inflation.

4.b. Stagflation can occur in time periods when people have high inflationary expectations. If the economy
goes into a recession, the actual rate of inflation will fall below the expected rate of inflation. However, the
actual inflation rate may still be very high while the rate of unemployment is increasing. For example, the Fed
may have let money supply grow much too fast in the past, so everyone expects a high inflation rate. If a

84
supply shock occurs, we will see an increase in the rate of unemployment while inflationary expectations and
actual inflation remain very high. This scenario occurred during the 1970s. Once we have reached such a
situation, it becomes necessary to design policies that will reduce inflationary expectations to shift the Phillips
curve back to the left.

5. Assume a disturbance occurs and the AD-curve shifts to the right. Unemployment decreases and inflation
increases, and we move along the downward sloping Phillips curve to the left. However, as soon as people
realize that actual inflation is higher than their inflationary expectations, they adjust their inflationary
expectations upward and the downward-sloping Phillips curve shifts to the right, eventually returning
unemployment back to its natural rate. In other words, the economy adjusts back at the full-employment level
of income.
If an adverse supply shock occurs (the upward-sloping AS-curve shifts to the left), unemployment and inflation
increase simultaneously. This will correspond to a shift of the downward-sloping Phillips curve to the right.
However, when people realize that actual inflation is less than expected inflation, then the downward-sloping
Phillips curve starts to shift back and the economy adjusts back to the natural rate of unemployment in the long
run.

The expectations-augmented Phillips curve predicts that inflation will rise above the expected level when
unemployment drops below its natural rate. However, if people know that this is going to happen, why don’t
they immediately adjust to it? And if people immediately adjusted to it, wouldn’t this imply that anticipated
monetary policy would be ineffective to cause any deviation from the full-employment level of output? In
reality, however, even if people have rational expectations, they may not be able to adjust immediately. One
reason is that wage contracts often set wages for an extended time period. Similarly, prices cannot always be
changed right away and the costs of changing prices may outweigh the benefits. A further argument is that
even rational people make forecasting mistakes and learn only slowly.
In other words, the location of the expectations-augmented Phillips curve is determined by the level of
expected inflation, which is set by recent historical experience. A shift in this curve caused by changing
inflationary expectations occurs only gradually. The rational expectations model, on the other hand, assumes
that the Phillips curve shifts almost instantaneously as new information about the near future becomes
available.

Technical Problems:

A reduction in the supply of money leads to excess demand for money and increased interest rates, reducing
the level of private spending (especially investment). Therefore the AD-curve shifts to the left. This causes an
excess supply of goods and services at the original price level so the price level starts to decrease. Since the
AS-curve is upward sloping, a new short-run macro-equilibrium is reached at a lower level of output (and thus
a higher level of unemployment) and a lower price level.

P AD1 AS1 AS2

AD2

P1 1
P2
2
P2 3

85
Y1Y* Y

However, the higher level of unemployment eventually puts downward pressure on wages, reducing the cost of
production and shifting the upward-sloping AS-curve to the right. Alternatively, since this equilibrium output
level is below the full-employment level, prices will continue to fall, and the upward-sloping AS-curve will
shift to the right. As long as output is below the full-employment level Y*, the upward-sloping AS-curve will
continue to shift to the right, which means that the price level will continue to decline. Eventually a new long-
run equilibrium will be reached at the full-employment level of output (Y*) and a lower price level.

2. According to the rational expectations theory, an announced change in monetary policy would
immediately change people’s perception in regard to the expected inflation rate. If people could adjust
immediately to this change in inflationary expectations, then the rate of unemployment or the output level
would remain the same. In other words, we would immediately move from point 1 to point 3 in the diagram
used to explain the previous question and the Fed would be unable to affect the unemployment rate. In reality,
however, even if people have rational expectations and can anticipate the effects of a policy change correctly,
they may not be able to immediately adjust due to wage contracts, etc. Thus, there will always be some
deviation from the full-employment output level Y*.

3.a. A favorable supply shock, such as a decline in material prices, shifts the upward-sloping AS-curve to the
right, leading to excess supply at the existing price level. A new short-run equilibrium is reached at a higher
level of output and a lower price level. But since output is now above the full-employment level Y*, there is
upward pressure on wages and prices and the upward-sloping AS-curve shifts back to the right. A new long-run
equilibrium is reached back at the original position (Y*), and the original price level (assuming that the change
in material prices did not affect the full-employment level of output). Since nominal wages (W) will have risen
but the price level (P) will not have changed, real wages (W/P) will have increased.

P
AD AS1

AS2

P1

P2

0 Y* Y1 Y

3.b. Lower material prices lower the cost of production, shifting the upward-sloping AS-curve shifts to
the right, and leading to an increase in output and a lower price level. Since unemployment is now below its
natural rate, there is a shortage of labor, providing upward pressure on wages. This will increase the cost of
production again, eventually shifting the upward-sloping AS-curve back to the original long-run equilibrium
(assuming that potential GDP has not been affected).

Additional Problems:

1. Explain the long-run effect of an increase in nominal money supply on the amount of real money balances

86
available in the economy.

In the very short run, the price level is fixed, so if nominal money supply (M) increases, a higher level of real
money balances is available, causing interest rates to fall and the level of investment spending to increase. This
leads to an increase in aggregate demand. The shift to the right of the AD-curve causes the price level (P) to
increase, leading to a reduction in real money balances (M/P). In the medium run (an upward-sloping AS-
curve), we reach a new equilibrium at a higher output level and a higher price level. Since prices have gone up
proportionally less than nominal money supply, real money balances have increased. However, to reach a new
long-run equilibrium, prices have to increase further, and as a result, the level of real money balances will
decrease further. When the new long-run equilibrium at Y* is finally reached, the price level will have fallen
proportionally to nominal money supply and the level of real money balances will be back at its original level.

2. Assume the economy is in a recession. Describe an adjustment process that will ensure that the economy
eventually will return to full employment. How can the government speed up this process?

If the economy is in a recession, there will be downward pressure on wages and prices, which will bring the
economy back to the full-employment output level. The upward-sloping AS-curve will shift to the right due to
lower production costs. However, this process may take a fairly long time. The government can shorten this
adjustment process with the help of expansionary fiscal or monetary policies to stimulate aggregate demand.
The resulting shift to the right of the AD-curve implies that the final long-run equilibrium will be at a higher
price level. In other words, the reduction in unemployment can only be achieved at the cost of higher inflation.

3. "The stickiness of wages implies that policy makers can achieve low unemployment only if they are
willing to put up with high inflation." Comment on this statement.

There are several explanations of why wages and prices adjust only slowly. One is that workers have imperfect
information, so they do not realize that lower prices mean higher real wages. Another is that firms are reluctant
to change prices and wages since they are unsure about the behavior of their competitors and want to avoid the
perceived cost of making these changes. Finally, wage contracts tend to be long-term and staggered, so it takes
time to adjust wages to price changes. Some firms may pay their workers above market-clearing wages to keep
them happy and productive. For these reasons, wages and prices tend to be rigid in the short run. Thus it takes
time for the economy to adjust back to full-employment.
If there were a stable Phillips-curve relationship, a low rate of unemployment could only be achieved by
allowing inflation to increase. However, such a stable relationship does not exist. Wages tend to be rigid in the
short run, so expansionary policies lower unemployment and increase inflation in the short run. In the long run,
however, the economy will adjust back to the natural rate of unemployment, so expansionary policies simply
lead to a higher price level.

4. "If we assume that people have rational expectations, then fiscal policy is always irrelevant. But monetary
policy can still be used to affect the rate of inflation and unemployment." Comment on this statement.

Individuals and firms with rational expectations consistently make optimal decisions based on all information
available. As long as a policy change is anticipated, people are able to assess its long-run outcome and will try
to immediately adjust. Since fiscal policy doesn't affect inflation or unemployment in the long run, it is also
ineffective in the short run if wages and prices are assumed to be flexible. An anticipated change in monetary
growth, on the other hand, will be reflected in a change in the inflation rate. If wages are flexible, workers will
adjust their wage demands immediately and no significant change in the unemployment rate will occur.
However, even if people have rational expectations, wages tend to be fairly rigid in the short run due to wage
contracts. Therefore, it will take time for the economy to adjust back to a long-run equilibrium. This implies
that both fiscal and monetary policy can affect the rate of inflation and unemployment to some degree in the

87
short run.

5. "Inflation cannot accelerate in a recession, when the rate of unemployment is above its natural rate."
Comment on this statement.

Inflation can accelerate even in a recession, that is, when the unemployment is high, if a supply shock occurs.
An oil price increase will increase the cost of production, so the upward-sloping AS-curve will shift to the left.
This will increase the inflation rate and the rate of unemployment simultaneously, as firms increase their
product prices and cut their production. If the Fed tries to accommodate the supply shock with expansionary
monetary policy in an effort to stimulate the economy, then inflation will accelerate even more, as the AD-
curve shifts to the right.

6. Comment on the following statement:


"The coordination approach to the Phillips curve focuses on the problems that the administration has in
coordinating its fiscal policies with the monetary policies of the Fed."

The coordination approach has nothing to do with fiscal or monetary policy but is simply one explanation of
why wages adjust slowly. This view asserts that firms generally are unable to coordinate wage and price
changes in response to a monetary policy change. For example, any firm that cuts workers' wages in response
to monetary contraction while other firms don't, will anger its employees who may then choose to leave. Firms
are also reluctant to change their prices since they are unsure about their competitors' behavior. Thus wages
and prices change only slowly in response to a change in aggregate demand. This implies an upward-sloping
(short-run) AS-curve.

7. Comment on the following statement:


"The unemployment rate is zero at the full-employment level of output."

With a higher price level real wages decline, increasing the quantity of labor demanded. Therefore the nominal
wage rate is bid up until the real wage rate is restored to its unique equilibrium level. Similarly, if prices fall,
real wages increase, leading to unemployment. The nominal wage rate falls to bring the real wage rate back to
its equilibrium level. So the nominal wage rate changes in proportion to the price level to maintain a real wage
rate that clears the labor market. At this wage rate, the full-employment level of output is produced. However,
at the full-employment output level the unemployment rate is not zero. Due to frictions in the labor market,
there is always a positive unemployment rate, as workers switch between jobs. This is called the natural rate of
unemployment.

8. Briefly state the reason for the slow adjustment of wages to changes in aggregate demand.

The reasons for the slow adjustment of nominal wages can be explained in several ways. One explanation is
that workers have imperfect information, that is, they do not immediately realize whether a change in their
nominal wage is the result of an increase in prices or in the real wage they receive for the work they provide.
Another explanation is that coordination problems exist, that is, different firms within an economy are unsure
about the behavior of their competitors and thus they only reluctantly change wages or prices. The efficiency
wage theory, on the other hand, argues that firms pay above market-clearing wages to motivate their workers to
work harder. Firms are also reluctant to change wages due to the perceived cost of doing so. Another argument
is that wage contracts tend to be long-term, so real wages tend to fluctuate over the length of the contract and
output adjusts only slowly to price changes. Finally, the insider-outsider model argues that firms negotiate only
with their employees but not the unemployed. Since a turnover of the labor force is costly to firms, they are
willing to offer above market-clearing wages to the currently employed rather than hiring the unemployed who
may be willing to work for less. These various explanations are not mutually exclusive, and they all imply that

88
the AS-curve is positively sloped, that is, that a change in aggregate demand will affect both output and prices
in the short run.

9. True or false? Why?


"There is no frictional unemployment at the natural rate of unemployment."

False. The natural rate of unemployment is the rate at which the labor market is in equilibrium. But there is
always some unemployment due to new entrants into the labor force, people between jobs, and the like. This
rate of unemployment is considered normal, due to frictions in the labor market, and is often called frictional
unemployment.

10. "If everyone in this economy had rational expectations, then wages would be flexible and unemployment
could not occur." Comment on this statement.

The new Keynesian models argue that even if people have rational expectations, socially undesirable outcomes
may still occur due to imperfect competition and the existence of wage contracts. Prices may not change freely,
since firms in imperfectly competitive markets are reluctant to change them, due to the menu costs involved.
Nominal wages are set by contracts over a period of time, so the economy may adjust only slowly to a decrease
in aggregate demand. Thus a rate of unemployment higher than the natural rate can exist over an extended
period of time.

11. True or false? Why?


"If nominal wages were more flexible, expansionary policies would be more effective in reducing the rate
of unemployment."

False. In Chapter 5 we learned that in the classical case (where nominal wages are completely flexible) the AS-
curve is vertical, whereas in the Keynesian case (where wages do not change, even if unemployment persists)
the AS-curve is horizontal. From this we can conclude that more flexible nominal wages imply a steeper
upward-sloping AS-curve. Any type of expansionary demand-side policy will shift the AD-curve to the right
and this will cause the level of output and prices to increase (at least in the short-run). A steeper upward-
sloping AS-curve results in a larger price increase and a smaller increase in output. But a smaller increase in
the level of output results in a smaller reduction in unemployment. In either case, the economy will settle back
at the full-employment level of output in the long run. In the long run, the rate of unemployment always goes
back to its natural level.

12. Explain the short-run and long-run effects of expansionary an increase in the level of government
spending on output, unemployment, interest rates, prices, and real money balances.

An increase in government spending increases aggregate demand, shifting the AD-curve to the right. Because
there is excess demand, the price level increases, which reduces the level of real money balances. Therefore
interest rates increase, leading to some crowding out of investment. Due to this real balance effect, the increase
in output is less than the shift in the AD-curve. Assuming an upward-sloping AS-curve, a new equilibrium is
reached at a higher price level, a higher level of output, a lower unemployment rate and a higher interest rate.
Since output is now above the full-employment level, wages and prices will continue to rise and the upward-
sloping AS-curve will start shifting to the left. This process will continue until a new long-run equilibrium is
reached at the full-employment level of income Y*, that is, until unemployment is back at its natural rate. At
this point the price level, nominal wages, and interest rates will be higher than previously and real money
balances will be lower.

13. Briefly explain why there seems to be so much interest in finding ways to shift the upward-sloping

89
aggregate supply curve to the right.

Shifting the upward-sloping AS-curve to the right seems to be the only way to offset the effects of an adverse
supply shock without any negative side effects. An adverse supply shock, such as an increase in oil prices,
causes a simultaneous increase in unemployment and inflation, and policy makers have only two options for
demand-management policies. Expansionary fiscal or monetary policy will help to achieve full employment
faster but will raise the price level, while restrictive fiscal or monetary policy will reduce inflationary pressure
but increase unemployment. Therefore, any policy that would shift the upward sloping AS-curve back to the
right seems preferable, since it might bring the economy back to the original equilibrium by simultaneously
lowering inflation and unemployment.

14. Use an AD-AS framework to show the effect of monetary restriction on the level of output, prices and the
interest rate in the medium and the long run.

A decrease in nominal money supply will increase interest rates, leading to a decrease in investment spending.
This will shift the AD-curve to the left, creating an excess supply of goods and services. Therefore price level
will decrease and real money balances will increase. A new equilibrium will be achieved at the intersection of
the new AD-curve and the upward-sloping AS-curve at an output level that is below the full-employment level.
In the long run, higher unemployment will cause downward pressure on wages. As the cost of
production decreases, the upward-sloping AS-curve will keep shifting to the right until a new long-run
equilibrium is established at the full-employment level of output, that is, where the new AD-curve intersects
the long-run vertical AS-curve at Y*. At this point, real output, the real interest rate, real money balances, and
the real wage rate will be back at their original level. Nominal money supply, the price level and the nominal
wage rate will all have decreased proportionally.

A simplified adjustment can be shown as follows:

1-->2: Ms down ==> i up ==> I down ==> Y down ==> the AD-curve shifts left ==>excess supply ==> P
down ==> real ms up ==> i down ==> I up ==> Y up

(The first line describes a policy change, that is, a shift in the AD-curve; the second line describes the price
adjustment, that is, a movement along the AD-curve.)

Short-run effect: Y down, i up, P down

2-->3: Since Y < Y* ==> downwards pressure on nominal wages ==> cost of production down ==> the
short run AS-curve shifts right ==> excess supply of goods ==> P down ==> real ms up ==> i down ==> I up
==> Y up (This process continues until Y = Y*)

Long-run effect: Y stays at Y*, i remains the same, P up.

Note: Even though only one shift of the short-run AS-curve to the new long-run equilibrium is shown here, this
shift is actually a combination of many shifts.

AD1 AS1
AD2
AS2

90
P1

P2

P3

0 Y2 Y* Y

15. Briefly discuss the importance of Okun’s law in evaluating the cost of unemployment.

Okun’s law states that a reduction in the unemployment rate of 1 percent will increase the level of output by
about 2 percent. This relationship allows us to measure the cost to society (in terms of lost production) of a
given rate of unemployment.

16. True or false? Why?


"If monetary policy accommodates an adverse supply shock, it will worsen any inflationary effects."

True. An adverse supply shock shifts the upward-sloping AS-curve to the left. There is excess demand for
goods and services at the original price level and prices start to rise, leading to lower real money balances,
higher interest rates, and lower output. If no policy is implemented, then unemployment will force the nominal
wage down to restore equilibrium at the original position. If the government views this adjustment process as
too slow, it can respond by implementing expansionary policies. Accommodating the supply shock in this way
shifts the AD-curve to the right and a new equilibrium can be reached at full-employment but at a higher price
level. It is unlikely, though, that the economy will remain there for long since workers will realize that their
purchasing power has been diminished by higher prices and will demand a wage increase. If they are
successful, the cost of production will increase and the upward-sloping AS-curve will shift to the left again. In
other words, we will enter a wage-price spiral.

AD2 AS2
AD1 AS1

P3
P2

P1

0
Y2 Y* Y

17. Assume oil prices decline. What kind of monetary policy should the Fed undertake if its goal is to
stabilize the level of output while keeping inflation low? Show with the help of an AD-AS diagram and briefly

91
explain the adjustment process.

1-->2: As oil prices decline, the cost of production decreases and the upward-sloping AS-curve shifts to the
right, causing excess supply of goods. Thus the price level decreases, real money balances increase, and the
interest rate declines.
2-->3: A decrease in money supply will increase the interest rate, decrease private spending, and shift the AD-
curve to the left. This means that prices will decrease even further and the level of output will decline. (We
assume, for simplicity, that it goes back to the full-employment level Y*, so no long-run adjustment is needed.)
Overall, the level of output has remained at its full-employment level but the level of prices and the interest
rate have decreased.

AD1 AS1
AD2 AS2

P1 1

P2 2
3

0
Y* Y2 Y

18. Comment on the following statement:


"A favorable oil shock causes lower inflation and lower unemployment."

A decrease in material prices (or any other favorable supply shock) shifts the upward-sloping AS-curve to the
right, and prices begin to decrease. The new equilibrium is at a lower price level and a higher level of output (a
lower level of unemployment). Since output is now above the full-employment level, there will be upward
pressure on nominal wages and prices, and the upward-sloping AS-curve will start shifting back to its original
position (assuming that potential output was not affected). In the long run, unemployment will be back at its
natural rate but the price level will have decreased (and thus real wages increased).

19. “Falling oil prices will lead to increased employment, higher wage rates and increased real money
balances.” Comment on this statement with the help of an AD-AS diagram and explain the short-run and long-
run adjustment processes.

A decline in material prices shifts the upward-sloping AS-curve to the right, leading to excess supply at the
existing price level. A new equilibrium is reached at a higher level of output and a lower price level. But since
output is now above the full-employment level Y*, there is upward pressure on wages and prices and the
upward-sloping AS-curve starts shifting back to the right. A new long-run equilibrium is reached back at the
original position (Y*), and the original price level (assuming that the change in material prices did not affect
the full-employment level of output). Since nominal wages (W) will have risen but the price level (P) will not
have changed, real wages (W/P) will have increased.

92
P

AD1 AS1
AS2

P1

P2

0
Y* Y2 Y

CHAPTER 7

THE ANATOMY OF INFLATION AND UNEMPLOYMENT

Solutions to the Problems in the Textbook:

Conceptual Problems:

1. The rate of unemployment is affected by the frequency, that is, the number of times that workers become
unemployed in a period, and by the duration, that is, the length of the period for which workers are
unemployed.

1.a. In depressed industries, the duration of unemployment is likely to be long but the frequency is likely to be
low. Policies to help unemployed workers from these industries find new jobs may include retraining and
education programs to enable them to find work in other industries.

1.b. Unskilled workers tend to be more frequently unemployed, but the duration of their unemployment is
usually fairly short. On-the-job training or education programs that provide skills to obtain or maintain jobs are
often the best strategy for helping these workers. However, such programs are often costly and difficult to
implement.

1.c. Unemployment in depressed geographical areas tends to be of long duration and low frequency and is
often concentrated in specific industries (very similar to the situation in 1.a.). Policies to relocate workers to
different geographical areas may not be successful since workers are often reluctant to move. Thus policy
makers generally prefer programs designed to attract new industries to an area over programs to relocate
workers.

93
1.d. Teenage unemployment is often of high frequency and short duration. Since teenagers tend to have few
skills and little or no work experience, programs to facilitate the transition into the adult work force are
needed. Programs that offer on-the-job training will provide the highest long-term benefits. These programs
tend to be fairly costly, however, which is why some politicians advocate lowering the minimum wage for
teenagers instead.

2. The natural unemployment rate is determined by two factors: the duration and frequency of
unemployment. While the duration of unemployment depends primarily on the organization and demographic
make-up of the labor force, the availability of unemployment benefits, and the desire of the unemployed to
look for better jobs, the frequency of unemployment depends largely on the rate at which new workers enter
the work force and on the variability of the demand for labor across different employers.

2.a. It is unclear whether the elimination of unions would serve to reduce the natural rate of unemployment.
The insider-outsider theory of the labor market suggests that firms bargain with unions (the insiders) and are
not much concerned with the unemployed (the outsiders). If unions were eliminated, firms would tend to hire
unemployed workers at a lower wage rate, thus reducing the natural unemployment rate. On the other hand,
unions tend to preserve stable jobs for their members. Eliminating them may lead not only to a reduction in
bargaining power for labor in wage negotiations but also to an increase in the natural rate of unemployment.
The elimination of labor unions could also serve to eliminate the wage differentials between unionized and
non-unionized workers and, in the process, redistribute some income.

2.b. Increased labor force participation of teenagers would at least initially increase the natural rate of
unemployment, since teenagers have a higher frequency of unemployment than older, more experienced
workers. However, as more and more teenagers entered the labor force and more good and stable jobs became
available to them, the natural rate of unemployment would start to decline again. But with more people in the
labor force, the supply of labor would be higher and wage rates would be driven down, contributing to wage
stagnation.

2.c. If aggregate demand fluctuated more, then firms would offer fewer stable jobs and the frequency of
unemployment would increase, increasing the natural rate. This would not only lead to a loss in output and an
increase in personal hardship, but it would also put more financial strain on the unemployment insurance
program.

2.d. An increase in unemployment benefits would make it less urgent for the unemployed to find new jobs.
They would have the option of looking longer for jobs after being laid off and would be less likely to accept
undesirable job offers. As the length of their unemployment increased, workers might begin to look less
desirable to potential employers who might believe that they lacked either the motivation or qualifications to
work hard for them. Therefore the natural rate of unemployment would increase.

2.e. Employers who perceive the minimum wage rate to be above the value of the marginal product of low
skilled workers will not hire such workers. The elimination of the minimum wage rate might induce some
firms to hire more low-skilled workers, thus decreasing the natural rate of unemployment. However, the wage
rate that these low skilled workers were offered might be well below the amount that would ensure an adequate
standard of living.

2.f. If fluctuation in the composition of aggregate demand increased, workers would have to be shifted from
industry to industry more often and this would increase the natural rate of unemployment. However, since
skills are not always transferable, resources would have to be devoted to retraining programs.

94
3. Many unemployed teenagers are new entrants into the labor force and their frequency of unemployment is
higher than that of adult workers. Teenagers' frequency of entry into and exit from the labor force indicates that
few of them work at jobs with the promise of high job security. They have little or no training and few job
skills and thus tend to hold unattractive jobs. This perpetuates the problem since the jobs they can get do not
provide the skills needed to gain better jobs in the future.
While the frequency of unemployment is lower for adults than teenagers, the duration is often higher. There are
fewer entrants and re-entrants into the work force among adults, who are most often unemployed due to
layoffs. Overall, the unemployment rate for adults is much lower than the unemployment rate for teenagers.

4.a. Employers would benefit from a lower minimum wage rate, since they would be able to expand
production by hiring labor at a lower cost. Since the nominal minimum wage rate might no longer be above the
value of the marginal product of low skilled or inexperienced workers, the labor of these workers would be
more desirable to employers. Therefore teenagers and low skilled job seekers would also benefit. They would
get jobs more easily and gain valuable work experience that they otherwise might not have gotten. Since more
people would be hired and more output would be produced at a lower price, the whole economy would benefit
from a lower inflation rate and a lower unemployment rate.

4.b. Those workers who would have been working at jobs paying the existing minimum wage rate might lose
from a decrease in the minimum wage. With a lower minimum wage rate implemented only during the summer
months, employers might lay off current workers and replace them with new entrants at a lower cost. Thus the
number of displaced workers might increase.

4.c. Obviously, those who would gain from such a policy measure would support it--teenagers, low skilled
workers, and some firms.

5. It is possible to design a restrictive fiscal and monetary policy mix to bring the economy to a long-run
equilibrium situation at the natural rate of unemployment and at a zero rate of inflation. However, this cannot
be achieved without an increase in the rate of unemployment in the short run. Therefore a choice has to be
made among adjustment paths that differ in their inflation-unemployment mix.
In considering adjustment paths, the benefits of permanently lower inflation have to be compared with the
costs of increased short-term unemployment. The costs of unemployment are loss of output and personal
hardship. If inflation can be anticipated only imperfectly, then a redistribution of income and wealth will take
place. Some output may be lost as resources are devoted to minimizing a potential loss in purchasing power
rather than to actual production. However, the cost of perfectly anticipated inflation is minimal. Thus it
probably makes little difference whether we have a zero inflation rate or an inflation rate of 3%, as long as a
specific long-run goal is established. A positive rate of inflation may actually help in wage and price
adjustments, since it allows real wages to adjust more easily to supply shocks.
Most policy makers tend to perceive the cost of inflation as lower than the cost of an increase in
unemployment resulting from tough anti-inflation policies. However, the U.S. experience of the early 1980s
indicates that tough measures to bring the economy quickly to recovery may be acceptable if inflation reaches
the double-digit range. One way to establish a clear inflation goal is for the Fed to follow a monetary growth
rule. However, such a rule may not perform well in all situations (for example, in a supply shock). Another
option is to maintain discretionary monetary policy along with an independent central bank that has a clear
mandate to function as an inflation fighter.

6. The sacrifice ratio is the percentage of output lost for each one- percent reduction in the inflation rate. It is

95
non-zero in the short and medium runs, when output is different from the full-employment level. However, in
the long run, unemployment always returns to its natural level and therefore the sacrifice ratio is zero.

7. Okun's law states that a reduction in the unemployment rate of 1 percent will increase the level of output
by 2 percent. This relationship allows us to measure the cost to society (in terms of lost production) of a given
rate of unemployment.

8. When inflation is perfectly anticipated, then its costs consist primarily of the costs of making price
changes (menu costs) and costs related to the holding of currency, which loses purchasing power. If there is
only low to moderate inflation, these costs are very low. However, when inflation rates soar, the costs can be
substantial.

9. The cost of imperfectly anticipated inflation can be serious. There is a redistribution of wealth among
individuals. If actual inflation is higher than expected, debtors profit while creditors lose as real interest rates
are lower than expected. Equity holders are also hurt, since the real value of dividends and capital gains is
reduced. If there is no tax indexation, people may move into higher tax brackets and suffer financially due to
bracket creep.

10. Indexation is designed to make it easier to live with inflation since it eliminates the cost of unanticipated
inflation. In practice, however, indexing on a broad basis makes it more complicated to calculate contracts. It
also makes it harder for the economy to adjust to shocks if changes in relative prices are needed. This is
particularly true for supply shocks. Finally, there is some concern that indexation may weaken political
motivation to fight inflation. However, as long as inflation is low to moderate, the benefits of indexation
probably outweigh the costs.

Technical Problems:

1.a. The aggregate unemployment rate can be calculated by adding the unemployment rates of different groups
weighted by their share of the labor force. The data in the problem indicate that 10% of the labor force are
teenagers. The adult work force (the other 90%) is divided into 35% females and 65% males. Thus we get

u = (0.1)(0.19) + (0.9)[(0.35)(0.06) + (0.65)(0.07)] = 0.019 + (0.9)(0.021 + 0.0455)

= 0.019 + 0.05985 = 0.07885 = 7.9%.

1.b. If the labor force participation rate of teenagers increases to 15%, then the aggregate rate of
unemployment changes to:

u1 = (0.15)(0.19) + (0.85)[(0.35)(0.06) + (0.65)(0.07)]

= 0.0285 + (0.85)(0.021 + 0.0455) = 0.0285 + 0.056525 = 0.085025 = 8.5%.

2. The unemployment figures for each group were taken from the Economic Report of the President,
February, 1997. Figures relating to the unemployment rate were taken from Table B-40 and each group's share

96
in the civilian labor force was calculated from Table B-38.

1986 1991 1996


Males 16-19
Un.Rt./Share (19.0%)/(.030) (19.8%)/(.026) (18.1%)/(.026)
Females 16-19
Un.Rt./Share (17.6%)/(.029) (17.5%)/(.024) (15.2%)/(.025)
Males 20+
Un.Rt./Share (6.1%)/(.525) (6.4%)/(.520) (4.6%)/(.512)
Females 20+
Un.Rt./Share (6.2%)/(.416) (5.7%)/(.430) (4.8%)/(.437)

If the unemployment rate for each group in the two other years was the same as in 1991, then the overall
unemployment rate in 1986 or 1996 would have been:

u86 = (19.8%)(.030) + (6.4%)(.525) + (17.5%)(.029) + (5.7%)(.416)

= 0.594% + 3.360% + 0.508% + 2.371% = 6.83%

u96 = (19.8%)(.026) + (6.4%)(.512) + (17.5%)(.025) + (5.7%)(.437)

= 0.515% + 3.277% + 0.438% + 2.491% = 6.72%

The actual unemployment rates for these years were:

u86 = 7.0% u91 = 6.8% u96 = 5.4%

The difference between the unemployment rates calculated above and the unemployment rate in 1986 shows
the effects of changes in the composition of the labor force on the rate of unemployment.

3. The data mentioned here were taken from the Economic Report of the President, February, 1997. Table B-
42 of the report shows unemployment by duration. In 1991 the average duration was 13.7 weeks, in 1995 it
was 16.6 weeks, and in 1996 it was 16.7 weeks. (These numbers reflect the average amount of time that an
unemployed worker was out of work in a given year and not the duration of a completed spell of
unemployment.) For the years in question, the overall unemployment rates were 6.8% in 1991, 5.6% in 1995,
and 5.4% in 1996. Thus the duration of unemployment moved in the opposite direction from the overall
unemployment rate.

Additional Problems:

1. Why is it possible for the number of unemployed workers to increase at the same time that the overall
unemployment rate falls?

The unemployment rate is defined as the number of people unemployed divided by the number of people in the
labor force. If the labor force grows more than the number of unemployed, the unemployment rate falls.

2. In the manufacturing sector many workers who have been laid off later return to their original jobs. Does

97
this mean that these workers failed to look for other jobs while they were unemployed?

No. The fact that these workers return to their original jobs simply means that they did not accept other
permanent job offers. Very often, laid-off workers actively look for other jobs, but may not find comparable
jobs in their geographic area. As the demand for the goods produced by their former firm increases again, they
are called back and return to their old jobs.

3. Proposals to raise the minimum wage rate are often opposed with the argument that such a move would
not only cause an increase in unemployment but would also hurt the very people it is intended to help. Is there
any validity to that argument?

The minimum wage rate provides a price floor on nominal wages. If this wage floor is above the market-
clearing equilibrium wage rate, then a surplus of labor will be created. The purpose of the minimum wage law
is to provide an adequate standard of living for less skilled workers. However, if firms believe that the
minimum wage rate is higher than the value of the marginal product of some workers, then those workers will
not be hired. As a result, the unemployment rate among less skilled workers may actually increase. Increasing
the minimum wage rate raises the cost of production (especially since higher skilled workers may also demand
higher wages) and may result in a higher inflation combined with higher unemployment. However, empirical
evidence suggests that such effects are fairly small. Whether politicians favor an increase in the minimum
wage rate depends on more than just the efficiency and equity arguments. An increase in the minimum wage
can be enacted with little cost to the government. Alternative programs (increasing the level of education or
training of low skilled workers) may have a higher rate of return but require a greater investment of resources
and may therefore be considered unfeasible if budget deficits are a problem.

4. True or false? Why?


"When the economy enters a recession, wages tend to adjust only slowly to a market-clearing level, and
the resulting unemployment is called frictional."

False. Frictional unemployment is the unemployment that exists when the economy is at its full-employment
level of output. Unemployment in excess of this "natural rate" of unemployment occurs when the economy
enters a recession and is called cyclical unemployment.

5. Starting in the early 1990s many U.S. firms downsized their operations. In your opinion, how did this
affect the duration of unemployment?

Normally after a recession workers either return to their old jobs or find similar jobs. However, since many
jobs were completely eliminated in this particular downsizing, workers who lost their jobs in the recession of
1990/91 had a much tougher time finding new and comparable jobs. Therefore, the duration of unemployment
increased.

6. Comment on the following statement:


"More generous unemployment benefits create a higher unemployment rate."

The unemployment rate tends to be higher in countries with more generous unemployment insurance
programs. Unemployment benefits significantly reduce the hardships associated with loss of a job. Firms may
also be less concerned about laying off their workers if they know that the workers will receive unemployment

98
insurance. At the same time, the benefits decrease the cost of searching for a job and enable those who have
lost jobs to at least initially reject unsatisfactory job offers. Finally, since workers have to officially be in the
labor force to receive unemployment benefits (even if they do not want a job) they get counted as unemployed,
thus increasing the measured unemployment rate.

7. "If wages are rigid, then the government can easily reduce inflation without creating higher unemployment."
Comment on this statement.

If wages are rigid, then there is a trade-off between unemployment and inflation. In other words, if policy
makers try to reduce inflation by demand management policies (restrictive monetary policy), then
unemployment will increase at least in the short run. Only in the long run, when wages become completely
flexible, will the unemployment rate return to its natural level. Another way to lower inflation without
increasing unemployment would be through supply-side economics, that is, policies designed to shift the AS-
curve to the right. According to supply-siders, a decrease in income tax rates will increase the incentive to
save, work and invest and this will shift the upward-sloping AS-curve to the right. However, this is easier said
than done, since tax cuts generally also affect aggregate demand.

8. Distinguish between frictional, search, seasonal, and cyclical unemployment.

Frictional unemployment exists, because labor markets do not work perfectly. At any given time, some
workers are always between jobs and therefore temporarily unemployed. Search unemployment exists because
some workers who are offered a job wait for a better opportunity; they can do this for some time because they
receive unemployment insurance benefits. Seasonal unemployment occurs, since workers are needed at
different times in different areas. (Ski instructors are only needed in winter; and college students tend to enter
the labor force temporarily during the summer time.) Cyclical unemployment refers to the unemployment that
occurs as the economy enters a downturn.

9. What kinds of policies would you suggest to reduce the natural rate of unemployment?

The answer to this question is student specific. Policies to reduce the natural rate of unemployment must
reduce the frequency or duration of unemployment and affect the composition or demographic make-up of the
labor force. Policies that make workers more mobile, make information regarding job vacancies more
accessible to the public, or reduce the variability of the demand for labor across industries can also be
considered. Some people may favor a reduction in the minimum wage rate or a reduction in unemployment
benefits, but such proposals are more controversial.

10. "Restrictive monetary policy will help to decrease inflation; the resulting increase in the natural
unemployment rate is a small price to pay." Comment on this statement. In your answer, explain what policies
the government should design to lower the natural rate of unemployment.

Restrictive monetary policy lowers the rate of inflation but at the cost of increasing the actual rate of
unemployment. However, in the long run the economy will adjust back to the natural rate of unemployment.
This natural unemployment rate is not affected by monetary policy, but only by employment policies or
changes in the composition of the labor force. Employment policies involve educating workers or increasing
their skills to make them more mobile, making information about job opportunities more readily available to
them, or reducing discrimination to allow certain groups to find jobs faster.

99
11. "Unemployment hysteresis generally exists when the economy is entering a deep recession and workers
fear lay-offs." Comment on this statement.

The term unemployment hysteresis is used to describe a situation in which long periods of high unemployment
lead to an increase in the natural rate of unemployment. This phenomenon arises since workers who are out of
a job may either take advantage of unemployment benefits while doing odd jobs or get discouraged and put
less effort into finding a new job. Thus they stay unemployed longer. Since firms may perceive long periods of
unemployment as a signal that these workers are less qualified or motivated, they may not want to hire them.
Therefore, the longer the spell of unemployment, the harder it is to get out of the unemployment pool.

12. Assume the unemployment rate among different groups in the labor force is as follows: adult males 5%,
adult females 4.5%, teenagers 12%. Teenagers account for 15% of the work force and, among adults, women
workers account for 40%. What is the actual nation-wide unemployment rate?

The aggregate unemployment rate can be calculated by adding the unemployment rates of the different groups
weighted by their share of the labor force. Therefore we get

u = (0.15)(0.12) + (0.85)[(0.6)(0.05) + (0.4)(0.045)] = 0.018 + (0.85)(0.03 + 0.018)

= 0.018 + 0.0408 = 0.0588 = 5.8%.

13. Demographic studies show that the proportion of teenagers and minorities in the U.S. population is likely
to increase in the near future. In your opinion, what implications, if any, would this trend have on the natural
rate of unemployment?

Teenagers and minorities tend to have a greater frequency of unemployment than the general population. If the
proportion of teenagers and minorities in the labor force increases, then we should expect an increase in the
natural rate of unemployment.

14. In Japan, a large part of a worker's annual salary comes in the form of bonuses. In your opinion, what are
the implications of this for the unemployment rate in Japan?

The fact that these bonuses can be easily reduced when a firm experiences a tough time makes wages much
more flexible. The Japanese economy can therefore adjust more easily to full employment. In bad economic
times Japanese workers are not laid off as easily as workers in the U.S., where wages are more rigid. The
implication is that the Japanese unemployment rate should, on average, be lower.

15. Given that inflation cannot be perfectly anticipated, how can lending institutions protect themselves against
the loss of purchasing power?

If inflation cannot be perfectly anticipated, then lending institutions have great uncertainty about the real value
of the nominal payments they receive on loans. For example, if a long-term mortgage has a fixed interest rate,
then the lending institution that issued the loan will lose purchasing power if the rate of inflation is higher than
expected. The homeowner, on the other hand, is better off since the real interest rate of the mortgage loan has

100
been effectively reduced.
Variable mortgage rates (which are generally dependent on short-term interest rates) protect the lending
institution against rising inflation, since they are adjusted periodically as inflation, and thus short-term interest
rates, increase. The unanticipated capital gains accrued by borrowers due to higher inflation are therefore much
smaller since the mortgage rate is adjusted soon after short-term interest rates increase. Thus the lending
institution's purchasing power is essentially protected.
16. "Perfectly anticipated inflation should not affect currency holders." Comment.

Whenever there is inflation, currency becomes more costly to hold because of the loss in purchasing power.
The higher the rate of inflation, the larger the incentive for individuals to put their funds into interest earning
accounts with yields higher than the expected rate of inflation

17. "Unanticipated inflation benefits debtors." Comment on this statement.

Assume you get a 30-year fixed-rate mortgage from your local savings and loan institution. The terms of the
contract will specify the nominal interest rate for your loan, which contains a premium for expected inflation
over the term of the contract. If the actual inflation rate is greater than was anticipated at the time the contract
was signed, then you (as the debtor) are better off. In this case, you will be able to pay back the loan with
"cheaper" dollars since money has lost some of its purchasing power. The lending institution, on the other
hand, will be hurt by the higher than anticipated interest rates. Naturally, if the actual inflation rate is less than
was anticipated, the lending institution will benefit.

18. "Wage indexation can lead to an inflation spiral." Comment on this statement.

If material prices rise and the price increases are passed on to consumers in the form of higher product prices,
then the rate of inflation will increase. If wages are indexed for inflation, then workers' nominal wages will
increase as soon as new labor contracts go into effect. But firms will again pass the higher wage costs on to
consumers in the form of higher product prices and inflation will increase even more. This process (the so-
called wage-price spiral) may continue until decisive action is taken to stop it.

19. “Broad-based indexation is a good idea, since it eliminates the hardships associated with inflation."
Comment on this statement.

Broad-based indexation eliminates the cost of unanticipated inflation, which is primarily the redistribution of
wealth. Long-term loan contracts, such as mortgages, government bonds, or wage contracts are especially
affected by imperfectly anticipated inflation since lenders and workers cannot be certain about the real value of
the nominal payments they receive. Since inflation is less predictable when it is high than when it is low,
indexation is more prevalent in countries with high inflation than in those with low inflation. The advantage of
indexation is that it keeps nominal and real rates of return (or wages) in line, while avoiding frequent costly
contract renegotiations. However, indexation is often complicated and may weaken policy makers' will to fight
inflation. Furthermore, indexation prevents real wages from falling after a supply shock, preventing an
adjustment back to full employment.

Briefly explain how and why each of the following changes would affect the natural rate of unemployment: (i)
a decrease in unemployment benefits; (ii) an increase in the minimum wage; (iii) a higher number of teenagers
in the work force.

101
A decrease in unemployment benefits would make it more urgent for the unemployed to find new jobs. They
would be more likely to accept early job offers and the length of unemployment would be reduced. Therefore
the natural rate of unemployment would decrease.
Employers who believe the minimum wage rate to be above the value of the marginal product of low-
skilled workers will not hire such workers. A higher minimum wage rate might make some firms less likely to
hire low-skilled workers, increasing the natural rate of unemployment. However, low-skilled workers who
continue to be employed would receive a higher wage rate, ensuring them more spending power and this could
create jobs elsewhere.
Increased labor force participation of teenagers would at least initially increase the natural rate of
unemployment, sine teenagers have a higher frequency of unemployment than older, more experienced
workers do. But as more teenagers entered the labor force and more stable jobs became available to them, then
the natural rate of unemployment would decrease again.

21. Comment on the following statement:


"An incumbent president can always ensure his re-election simply by using all policy measures at his disposal
to ensure low inflation and high growth in an election year."

This statement relates to the political business cycle theory, which asserts that politicians will use those
economic policies that have high voter approval to guarantee reelection. The Fair equation, which is based on
the inflation rate and per-capita growth in an election year, has been fairly successful in predicting the outcome
of presidential elections. However, there are some other factors that work against this hypothesis. External
shocks cannot be predicted but they can influence voter behavior. The president also has no control over the
actions of the Fed nor can he use fiscal policy measures that would serve his own purposes too openly. Finally,
if expectations are indeed rational, then policy measures designed to stimulate the economy before an election
should have negligible effects. Empirical evidence in support of the political business cycle theory is mixed.

"The natural rate of unemployment is a very helpful guide for policy makers, since most inflation is driven by
wage increases." Comment on this statement.

The true value of the natural rate of unemployment is not known for sure. So is policy makers assume this rate
is lower than it actually is, they will try to stimulate the economy through expansionary policies as soon as
unemployment rises above this assumed rate. But this will reduce the actual unemployment rate only
temporarily, since the economy has a tendency to go back to the true natural rate. Policy makers will have to
stimulate the economy again and again and this creates inflationary pressure. Since inflation reduces real
wages, workers will ask for wage increases and we will enter a wage-price spiral.
The assertion that most inflation is driven by wage increases also does not hold. Instead, global
competition has broken down the relationship between wage increases and increases in inflation. In the long
urn, inflation can only persist if monetary growth is excessive.

23. "Structural unemployment is in excess of the natural rate of unemployment and can therefore be easily
lowered by expansionary fiscal policy." Comment on t his statement. In your answer, list the different types of
unemployment.

Expansionary fiscal policy will result in a temporary decrease in the actual unemployment rate, but in the long
run the economy will adjust back to the natural unemployment rate. This natural rate can only be reduced
through policies geared towards improving education, skills, information about jobs, and job mobility.

102
Structural unemployment (the mismatch of job openings and available skills) is part of the natural
unemployment rate. Frictional unemployment always exists since labor markets don't work perfectly and some
people are always between jobs. Search unemployment exists when people who are offered jobs decide not to
take them in hopes of getting better jobs. Only cyclical unemployment is in excess of this natural rate and can
be reduced through expansionary fiscal policy.

CHAPTER 8

POLICY

Solutions to the Problems in the Textbook:

Conceptual Problems:

1. The first question you should ask yourself as a policy maker is whether a disturbance is transitory or
persistent. You should then ask yourself how long it would take to put a suggested policy measure into effect
and how long it will take for the policy to have the desired effect on the economy. In addition, you need to
know how reliable the estimates of your advisors are about the effects of the policy. If a disturbance is small
and probably transitory, you may be best advised to do nothing, because any measure you take is likely to have
its effect after the economy has recovered. Therefore your action might only further aggravate the problem.

2.a. The inside lag is the time it takes after an economic disturbance has occurred to recognize and implement
a policy action that will address the disturbance.

2.b. The inside lag is divided into three parts. First, there is the recognition lag, that is, the time it takes for
policy makers to realize that a disturbance has occurred and that a policy response is warranted. Second, there
is the decision lag, that is, the time it takes to decide on the most desirable policy response after a disturbance
is recognized. Finally, there is the action lag, that is, the time it takes to actually implement the policy measure.

2.c. Inside lags are shorter for monetary policy than for fiscal policy since the FOMC meets on a regular basis
to discuss and implement monetary policy. Fiscal policy, on the other hand, has to be initiated and passed by
both houses of the U.S. Congress and this can be a lengthy process. The exceptions are the so-called automatic
stabilizers; however, they only work well for small and transitory disturbances

2.d Automatic stabilizers have no inside lag; they are endogenous and function without specific government
intervention. Examples are the income tax system, the welfare system, unemployment insurance, and the
Social Security system. They all reduce the amount by which output changes in response to an economic
disturbance.
3.a. The outside lag is the time it takes for a policy action, once implemented, to have its full effect on the
economy.

3.b. Generally, the outside lag is a distributed lag with a small immediate effect and a larger overall effect over
a longer time period. The effect is spread over time, since aggregate demand responds to any policy change
only slowly and with a lag.

103
3.c. Outside lags are longer for monetary policy since monetary policy actions affect short-term interest rates
most directly, while aggregate demand depends heavily on lagged values of income, interest rates, and other
economic variables. A change in government spending, however, immediately affects aggregate demand.

4. Fiscal policy has smaller outside lags, but significant inside lags. Monetary policy, on the other hand has
smaller inside lags and longer outside lags. Therefore large open market operations should be undertaken to get
an immediate effect, but they should be partially reversed over time to avoid a large long-run effect. If the
shock is sufficiently transitory and small, policy makers may be best advised not to undertake any policy
change at all.

5.a. An econometric model is a statistical description of all or part of the economy. It consists of a set of
equations that are based on past economic behavior.

5.b. Econometric models are generally used to forecast the behavior of the economy and the effects of
alternative policy measures.

5.c. There is considerable uncertainty about how well econometric models actually represent the workings of
the economy. There is also great uncertainty about the expectations of firms and consumers and their reactions
to policy changes. Any policy is bound to fail if the information on which it was based is poor.

The answer to this question is student specific. The main difficulties of stabilization policy arise from three
sources. First, policy always works with lags. Second, the outcome of any policy depends on the way the
private sector forms expectations and how those expectations affect the public's behavior. Third, there is
considerable uncertainty about the structure of the economy and the shocks that hit it.
It can be argued that a monetary policy rule would greatly reduce uncertainty about the Fed's policy
responses. If the government behaved in a consistent way, then the private sector would also behave more
consistently and economic fluctuations could be greatly reduced. A monetary growth rule would also reduce
any political pressure the administration might exert on the Fed. It is often initially unclear whether a
disturbance is temporary or persistent and a monetary policy rule would prevent policy mistakes in cases
where the disturbance is, in fact, temporary. If active monetary policy is applied to a temporary disturbance,
then the lags involved will guarantee that the economy will actually be destabilized.
On the other hand, the workings of the economy are not completely understood and events cannot
always be predicted. Thus it is difficult to argue for a fixed policy rule. Unanticipated large disturbances
warrant an activist policy, especially if they appear to be persistent. It is also possible to construct a more
activist monetary growth rule. For example, Equation (8) suggests that the annual monetary growth rate should
be increased by two percent for every one percent that unemployment increases above its natural rate. Such a
rule is based on the quantity theory of money equation (which relates money supply growth to the growth of
nominal GDP) and on Okun's law (which relates the unemployment rate to economic growth). Obviously,
because of the long lags for monetary policy, any monetary growth rule will work much better in the long run
than in the short run.
Fiscal policy rules may make more sense than monetary policy rules, since fiscal policy has long
inside lags but shorter outside lags. In a way, built-in stabilizers, although generally not considered "rules",
already provide some stability without any inside lag. Many of the arguments against monetary policy rules are
also valid for fiscal policy rules and many economists oppose them. The frequently proposed constitutional
amendment requiring an annually balanced budget is an example of a fiscal policy rule. There are significant
problems associated with such an amendment, since it would greatly limit the government's ability to

104
undertake active fiscal stabilization policy.

7. The arguments for a constant growth rate rule for money are based on the quantity theory of money
equation, that is,

MV = PY.

From this equation we can derive

%P = %M - %Y + %V.

If the long-run trend rate of real output (Y) and the long-run trend of velocity (V) are assumed to be fairly
stable, and if wages and prices are sufficiently flexible, then a constant monetary growth rate (M) would insure
a constant rate of inflation, that is, a constant rate of change in the price level (P). Also, since monetary policy
has long outside lags, active monetary policy can actually be more destabilizing than stabilizing. In addition,
since we do not know exactly how the economy works or may react to specific policies, it is best to follow a
rule rather than undertake actions that have uncertain outcomes. However, rules are not without problems, as
they would not allow flexibility in responding to major disturbances.

8. Dynamic inconsistency occurs if, after having committed themselves to a specific policy action designed
to achieve a long-run objective, policy makers find themselves in a situation where it seems advantageous to
abandon their original policy, in order to achieve a short-run goal. Such action will impede the long-run
objective.

9. Real GDP targeting is the best option if the primary policy goal of monetary policy is to achieve full
employment. If policy makers forecast potential GDP correctly, then full employment combined with low
inflation can be achieved. However, real GDP targeting bears the greater risk that the secondary goal of
achieving a low inflation rate will be missed. If the rate at which potential GDP grows is overestimated, then
policy makers may stimulate the economy too much. In this case, they will not be successful in achieving price
stability. By targeting nominal GDP, the central bank creates a policy tradeoff between inflation and
unemployment. If the rate at which potential GDP grows is overestimated and policy makers stimulate the
economy too much, we will get less growth but also less inflation than under real GDP targeting. Which
targeting approach should be chosen depends greatly on how steep or flat the Phillips curve is perceived to be.

Technical Problems:

1. If actual GDP is expected to be $40 billion below the full-employment level and the size of the
government spending multiplier is 2, then government spending should be increased by $20 billion over its
current level. For the next period, when actual GDP is expected to be $20 billion below potential, government
spending should be cut by $10 billion from its new level, that is, to $10 billion over its original level. In period
three, when actual GDP is expected to be at its full-employment level, the level of government spending
should again be cut by $10 billion from the last period's level to bring it back to the original level of Period 0.

2.a. If there is a one-period outside lag for government spending, then nothing can be done to close the current
GDP-gap. The government should decide to spend $10 billion more for the next period and reduce spending

105
again to its original level after that.

2.b. Graph I below shows the path of GDP for Problem 1 with no outside lag and Graph II shows the path of
GDP for problem 2.a. with a one-period outside lag. In each of the graphs the path of actual GDP is shown,
first assuming that no policy action takes place and then assuming that the policies proposed in Problems 1 and
2.a. are undertaken.

Graph I
GDP GDP

potential GDP potential GDP

0 time 0 time
GDP with fiscal policy GDP without fiscal policy
Graph II

GDP GDP

potential GDP potential GDP

0 time 0 time
GDP with fiscal policy GDP without fiscal policy

3.a. Since the government multiplier for the first period is 1, the level of government spending must be
increased by G = $40 billion to close the GDP-gap of $40 billion. But since the government multiplier in the
next period for the amount spent in this period is 1.5, the effect of an increase in government spending in the
first period by $40 billion would be an increase in GDP by $60 billion in the second period.

3.b. For the second period a GDP-gap of $20 billion is expected. However, as we saw in 3.a., GDP will
increase by $60 billion in the second period if the government increases spending by $40 billion in the first
period. Therefore, the government has to reduce spending in the second period by $40 billion from its new
level (back to its original level), since the multiplier for a spending change in the same period is 1.

3.c. In this problem, fiscal policy has an outside lag. This means that the effect of an increase in government
spending is felt both in the period in which the spending increase takes place and (to an even larger degree) in
the following period. The increase in government spending needed to close the GDP-gap in the first period is
guaranteed to overshoot the desired goal in the next period. Thus the government will be forced to reverse its
increase in spending to the original level in the second period to offset the destabilizing effect. In a case like
this, the government has to be much more active in its fiscal policy than in a situation where no distributed lag
exists.

4. If there is uncertainty about the size of the multiplier, then fiscal policy becomes much more complicated.

106
If the multiplier is 1, then an increase in government spending by $40 billion will close the GDP-gap in the
first period. If the multiplier is 2.5, we will overshoot potential GDP by $60 billion. An increase in spending by
40/2.5 = $16 billion is optimal if the multiplier is 2.5. Thus a cautious government will probably increase
spending by no more than $16 billion in the first period, and then reduce the level of spending by $8 billion in
the next period ($8 billion above the original level). Such a policy action is designed to close the GDP-gap to
some degree over the first two periods while never overshooting potential GDP. In Period 3 we will again be
back at the full-employment level. The extent to which a less cautious government might exceed these
suggested spending increases depends largely on that government's level of concern about unemployment
versus inflation.

5. To follow an established rule for its policy, the Fed needs to know the source of each disturbance. If a
disturbance comes from the goods sector, it is better to have a monetary growth target; if the disturbance comes
from the money sector, it is better to have an interest rate target.

a. Assume a disturbance comes from the money sector. If an increase in money demand increases the
interest rate, the Fed should try to maintain a constant interest rate by increasing the supply of money. This will
re-establish the old equilibrium values of the interest rate and output and effectively offset the disturbance.

b. Assume a disturbance comes from the goods sector. If an increase in autonomous investment increases the
interest rate, then it is not advisable to maintain a constant interest rate. Trying to lower the interest rate again
by increasing the money supply would aggravate the disturbance. On the other hand, maintaining a constant
money supply, while not offsetting the disturbance, will at least not make things worse.
6.a. Students will have to check the Federal Reserve Bulletin in early 2000 and compare the forecasts of the
Federal Reserve Board with the actual performance of the economy in 1999.

6.b. Regardless of how detailed it is, no econometric model can accurately represent the economy, since we do
not completely understand the way the economy works. Therefore, we can never expect perfect forecasts. It is
impossible to incorporate all the relevant information on which individuals and firms base their expectations
about the future and to determine how these expectations affect actions in any given situation. Forecasts are
generally based on the information available at the time, which may be flawed or outdated. In addition, any
unexpected change, such as a supply shock, an unanticipated international change, or an unanticipated
domestic policy change, can render the initial predictions wrong.

Additional Problems:

1. Define and distinguish between inside and outside lags.

The inside lag is the time period it takes to implement a policy action after an economic disturbance is
recognized. It is divided into the recognition lag (the time it takes for policy makers to realize that a
disturbance has occurred and that a policy response is warranted), the decision lag (the time it takes to decide
on the most desirable policy response), and the action lag (the time it takes to implement the policy measure).
The outside lag is the time it takes for a policy measure, once implemented, to have an effect on the economy.

2. How do long lags in investment spending affect the usefulness of a government policy that tries to
stabilize the economy via investment tax credits?

Long lags in investment spending suggest that investment tax credits should not be used for fine-tuning but

107
may be useful in responding to a prolonged disturbance. However, the lag is much shorter for temporary
investment tax credits, which often speed up existing or planned investments. Therefore investment tax credits
can be used for short-run stabilization, even though uncertainty about their frequency and duration may create
unstable swings in investment spending.

3. "Monetary policy should be employed only sparingly since it operates with long and variable lags."
Comment on this statement.

There are lags both in recognizing that there is a need for a policy response to a disturbance and in designing a
particular policy measure. Once the program is in place, it takes additional time to affect economic activity. For
example, expansionary monetary policy lowers short-term interest rates and then, after a lag, also lowers long-
term interest rates, which, in turn, increases investment spending. Before committing themselves to increased
investment spending, firms need to determine whether the cost of capital has risen temporarily or permanently.
Ultimately, aggregate demand is affected and then a series of induced adjustments in output and spending will
take place. Therefore, while the inside lags for monetary policy actions are short, it takes time for monetary
policy to affect aggregate demand to the desired degree since there is an additional distributed lag (the dynamic
multiplier process). If monetary policy is employed, it should be done with caution, since it can be
destabilizing. But the fairly successful monetary policy of the Fed over the last two decades indicates that it is
possible to undertake active monetary policy without destabilizing the economy.

4. "Monetary stabilization policy is difficult for the Fed because of the existence of long and variable outside
lags. But if the Fed knew the exact length of these lags, active monetary stabilization policy would always be
successful." Comment on this statement.

If lags are long and variable, it becomes very difficult to predict exactly when a certain policy measure is going
to have its desired effect on the economy. It is thus difficult to successfully implement counter-cyclical
stabilization policies. If the timing of a policy measure is misjudged, the economy may actually be destabilized
rather than stabilized. If lags were long but fixed, it would be much easier to judge when to implement a policy
measure. However, this is unrealistic since there is still considerable uncertainty about the workings of the
economy and about the accuracy of the theoretical models used in forecasting.

5. Is the fact that economists often put decimal points in their forecasts an indication that they can be very
accurate in their predictions? Why or why not?

The econometric models from which economic forecasts are derived are based on the historical record of the
economy. However, not every disturbance can be predicted with accuracy. An economic forecast represents
only a best estimate of how the economy will behave based on (often incomplete) information and a set of
initial assumptions. A forecast of 1.5% real economic growth should be interpreted as growth anywhere in the
range of 1 to 2 percent.

6. True or false? Why?


"The Fed can always maintain full employment if it uses its policy instruments appropriately."

False. Monetary policy works with long and variable outside lags. Furthermore, by the time the Fed has
identified the source and duration of an economic disturbance and decided on the appropriate response, the
shock will already have done some damage. Fine tuning the economy through the use of monetary policy is

108
impossible.

7. True or false? Why?


"Announcing a policy may be just as effective as actually implementing it."

False. There may be short-run effects of policy announcements if individuals act in anticipation of the
announced change. Such announcements in and of themselves, however, do not have lasting implications for
economic activity unless the government consistently follows through with the proposed policy. Indeed,
making announcements and then not following through threatens the credibility and reputation of policy
makers and may ultimately render policy announcements destabilizing.

8. "Policy makers committed to full employment encounter major problems if they underestimate the natural
unemployment rate." Comment on this statement.

Policy makers, who assume that the natural rate of unemployment is 4% rather than its actual 5.5%, will try to
stimulate the economy through expansionary policies as soon as the rate of unemployment goes above 4%. But
this will decrease unemployment only temporarily since the economy has a tendency to go back to the true
natural rate. To maintain the 4% level, policy makers will have to stimulate the economy again and again
which will, in turn, create inflationary pressure. Since inflation reduces real wages, workers will ask for wage
increases and we will enter the so-called wage-price spiral.

9. Briefly discuss the arguments for and against fine tuning the economy.

Fine tuning the economy involves the use of policy tools to counteract every disturbance in the economy, even
small ones. Since there are lags in stabilization policies and considerable uncertainty about whether a
disturbance is temporary or permanent, a very active stabilization policy in the face of very small disturbances
may actually be destabilizing. If a decision to fine tune is made, minimal initial policy responses to economic
disturbances are advised, especially if there is uncertainty about the size of the fiscal or monetary policy
multiplier.

10. What are the relative merits of a cold turkey over a gradualist approach to fight inflation? In your answer
discuss the concept of time inconsistency and the importance of credibility.

The key question for governments desiring to reduce inflation is how cheaply (in terms of lost output) they can
achieve a desired inflation rate. A gradual strategy attempts a slow and steady return to low inflation by
reducing monetary growth slowly in an attempt to avoid a significant increase in unemployment. This
approach takes a lot longer than the cold-turkey approach that attempts to reduce inflation quickly by
immediately and sharply reducing monetary growth. The central question here is how flexible wages and
prices are and how fast expectations adjust. With a cold turkey approach, inflation and inflationary
expectations will be reduced faster. An increase in the level of unemployment leading to a decrease in the level
of output will result in the short run in either case, but the economy will eventually adjust back to the full-
employment level of output.
The term time inconsistency refers to the problem that the Fed faces when a policy may look appropriate
at the time it is announced, but when it is time to execute it, it may no longer look desirable. If the Fed
consistently tries to keep inflation under control, even though it may cause some unemployment in the short
run, the public will keep inflationary expectations low and is more likely to adjust wage demands downwards.

109
Credibility makes it easier for the Fed to conduct its policies, since the public knows what to expect and will
react predictably. For example, if the Fed were to follow a monetary growth rule, then people could more
easily anticipate the effects of a disturbance and adjust to it. But if the Fed had a reputation of changing its
monetary policy often and unpredictably, its policies might not have the desired effects, since people might not
react in the desired way.

11. "A cold-turkey approach is better than a gradualist approach for fighting inflation, since it requires a
shorter time to establish a long-run equilibrium at a desired lower inflation rate." Comment on this statement.

The gradualist approach lowers money growth over a long period of time to minimize the severity of any
resulting recession. The cold-turkey approach achieves the reduction in the inflation rate more quickly, but at
the cost of a significant increase in short-run unemployment. Normative considerations determine the relative
merits of the two approaches. The cold-turkey approach may benefit from a credibility bonus, since workers
and firms do not have to guess whether the government is really committed to lowering the inflation rate. They
may therefore reduce their inflationary expectations faster and the economy will adjust back to full-
employment much faster. However, if long-term contracts exist, wages and prices cannot adjust quickly and a
rapid return to a low-inflation equilibrium at full-employment is fairly unlikely.

12. Comment on the following statement:


“Achieving a zero inflation rate by imposing a monetary growth rule will result in more economic
stability, a higher growth rate, and a better income distribution.”

According to the equation (%P) = (%M) - (%Y) + (%V), a zero inflation rate (%P) can be easily
achieved if monetary growth (%M) is kept at a rate equal to the long-term growth rate of real income (% Y)
adjusted for velocity (%V). A monetary growth rule works well if the economy is inherently stable, if most
disturbances are small and of short duration, and if velocity is fairly stable or at least predictable. But such a
monetary growth rule would tie the central bank's hands if a large disturbance occurred. The cost of long and
high unemployment can be very high, especially if wages and prices are fairly rigid. Periods of high and long
unemployment particularly affect low-income workers with low skills and therefore the income distribution
worsens. While some economists contend that low inflation countries have a higher average economic growth
rate, there is no good evidence available for this assertion.

13. Would you support a monetary growth rule that maintains the rate of money supply at 3%, which is
slightly higher than the long-term trend of potential output? Why or why not? Explain your answer.

The answer to this question is student specific. A student who believes that the economy and velocity are very
stable and that disturbances are fairly transitory will probably support such a rule. The argument for such a rule
is that it may create more rational expectations in the private sector regarding policy actions by the Fed. Thus,
after a disturbance, the private sector would adjust more rapidly and the economy would go back to full
employment much more quickly.
A student, who believes that disturbances may be persistent, that wages and prices do not adjust rapidly,
and that long periods of unemployment are costly, would advocate a more activist approach. This is especially
true if velocity is not very stable.
Recent experience indicates that a discretionary approach promises more success in keeping the economy
close to full employment without causing unacceptable inflation.

110
14. "Credibility is extremely important in the conduct of monetary policy." Comment on this statement and
relate your answer to the concept of dynamic inconsistency.

Assume there is a supply shock and the Fed has to decide whether to keep unemployment low by
implementing expansionary monetary policy or to concentrate on keeping inflation under control. The dynamic
inconsistency approach suggests that the Fed should refrain from a policy response that may look appropriate
in the short run but will prove unproductive in the long run. If the Fed always chooses to accommodate a
supply shock, people will come to expect such a reaction and will incorporate the assumption of a higher
inflation rate in their wage demands. But if the Fed consistently tries to keep inflation under control (even
though it may cause some unemployment in the short run), people will lower their inflationary expectations.
The Fed will keep its reputation as an inflation fighter, and the economy will adjust back to full employment
fairly rapidly.

15. "Policy rules are always preferable to discretionary policy." Briefly comment on this statement.

Policy rules predetermine the actions of policy makers and thus eliminate the uncertainty of government
actions. This may reduce some of the instability that arises from mistaken expectations. Unfortunately,
however, rules do not allow enough flexibility to respond to unforeseen events and cannot accommodate the
range of policies required to reduce large and prolonged disturbances.

16. Explain why it is important for the Fed to have credibility in its effort to maintain price stability.

Credibility is very important for the Fed, since consistent government actions lead to more accurate private
sector expectations. For example, if the Fed were to follow a monetary growth rule, then people could more
easily anticipate the effects of a disturbance and could adjust to it. If the Fed consistently tries to keep inflation
under control, even though it may cause some unemployment in the short run, people keep their inflationary
expectations low and adjust their wage demands downwards. But if the Fed has a reputation of changing its
monetary policy often and unpredictably, its policies may not have the desired effects, since people may not
react in the desired way. As a result, active monetary policy may actually be destabilizing.

17. "A monetary policy rule is preferable to discretionary stabilization policy." Comment on this statement. In
your answer discuss the arguments for and against fine tuning the economy.

Policy rules make the actions of policy makers predictable and thus eliminate the uncertainty that comes from
unanticipated government actions. They also may reduce instability arising from mistaken expectations.
However, rules do not allow flexibility in responding to unforeseen large and prolonged disturbances. Only a
very strong proponent of monetarism or the rational expectations approach would propose a strict monetary
growth rule.
Fine tuning the economy involves the use of policy tools for all (even small) disturbances. But since there
are always policy lags and uncertainty about the length of disturbances, fine-tuning may actually be
destabilizing. Therefore small policy responses should be undertaken initially and revised later, as more
information about the true nature of the disturbance becomes available.

18. "The economy always adjusts back to the full-employment level of output, so policy makers should not be
concerned with undertaking active stabilization policy. Instead they should establish more credibility by
following a well-defined policy rule." Comment on this statement.

111
Since wages are flexible in the long run, the economy will always eventually adjust back to full employment.
According to the Phillips-curve analysis, the economy will be at full employment as long as expected inflation
is equal to actual inflation. Therefore, policy makers can create more rational expectations by following a
monetary growth rule and announcing its monetary growth targets in advance. In this case, money illusion will
not exist and any announced reduction in the growth rate of money supply will lead to lower inflation without
much increase in unemployment. But if there is no such rule and the Fed has a reputation of changing its
monetary policy often, policies may not have the desired effects, since people may not react in the desired way.
It should be noted that an activist monetary growth rule can be designed in which money supply growth
changes with changes in the unemployment rate. Fiscal policy rules can also be designed (such as a balanced
budget amendment, for example). Whether policy rules actually make sense depends on how fast the economy
adjusts back to full employment and how flexible wages and prices are. In the case of large disturbances, rules
limit the flexibility of policy makers to respond to a disturbance that may result in a high rate of
unemployment. Finally, there are the questions of whether policy rules should be announced in advance and
who has the authority to change them if necessary. Finally, if disturbances are perceived to be small it may be
better to rely on automatic stabilizers.

CHAPTER 9

INCOME AND SPENDING

Solutions to the Problems in the Textbook:

Conceptual Problems:

1. In the Keynesian model, the price level is assumed to be fixed, that is, the AS-curve is horizontal and the
level of output is determined solely by aggregate demand. The classical model, on the other hand, assumes that
prices always fully adjust to maintain a full-employment level of output, that is, the AS-curve is vertical. Since
the model of income determination in this chapter assumes that the price level is fixed, it is a Keynesian model.

2. An autonomous variable’s value is determined outside of a given model. In this chapter the following
components of aggregate demand have been specified as being autonomous: autonomous consumption (C*)
autonomous investment (Io), government purchases (Go), lump sum taxes (TAo), transfer payments (TRo),
and net exports (NXo).

Since it often takes a long time for policy makers to agree on a specific fiscal policy measure, it is quite
possible that economic conditions may drastically change before a fiscal policy measure is implemented. In
these circumstances a policy measure can actually be destabilizing. Maybe the economy has already begun to
move out of a recession before policy makers have agreed to implement a tax cut. If the tax cut is enacted at a
time when the economy is already beginning to experience strong growth, inflationary pressure can be created.
While such internal lags are absent with automatic stabilizers (income taxes, unemployment benefits,
welfare), these automatic stabilizers are not sufficient to replace active fiscal policy when the economy enters a
deep recession.

112
4. Income taxes, unemployment benefits, and the welfare system are often called automatic stabilizers since
they automatically reduce the amount by which output changes as a result of a change in aggregate demand.
These stabilizers are a part of the economic mechanism and therefore work without any case-by-case
government intervention. For example, when output declines and unemployment increases, there may be an
increase in the number of people who fall below the poverty line. If we had no welfare system or
unemployment benefits, then consumption would drop significantly. But since unemployed workers get
unemployment compensation and people living in poverty are eligible for welfare payments, consumption will
not decrease as much. Therefore, aggregate demand may not be reduced by as much as it would have without
these automatic stabilizers.

5. The full-employment budget surplus is the budget surplus that would exist if the economy were at the
full-employment level of output, given the current spending or tax structure. Since the size of the full-
employment budget surplus does not depend on the position in the business cycle and only changes when the
government implements a fiscal policy change, the full-employment budget surplus can be used as a measure
of fiscal policy. Other names for the full-employment budget surplus are the structural budget surplus, the
cyclically adjusted surplus, the high-employment surplus, and the standardized employment surplus. These
names may be preferable, since they do not suggest that there is a specific full-employment level of output that
we were unable to maintain.

Technical Problems:

1.a. AD = C + I = 100 + (0.8)Y + 50 = 150 + (0.8)Y

The equilibrium condition is Y = AD ==>

Y = 150 + (0.8)Y ==> (0.2)Y = 150 ==> Y = 5*150 = 750.

1.b. Since TA = TR = 0, it follows that S = YD - C = Y - C. Therefore

S = Y - [100 + (0.8)Y] = - 100 + (0.2)Y ==> S = - 100 + (0.2)750 = - 100 + 150 = 50.

1.c. If the level of output is Y = 800, then AD = 150 + (0.8)800 = 150 + 640 = 790.

Therefore the amount of involuntary inventory accumulation is UI = Y - AD = 800 - 790 = 10.

1.d. AD' = C + I' = 100 + (0.8)Y + 100 = 200 + (0.8)Y

From Y = AD' ==> Y = 200 + (0.8)Y ==> (0.2)Y = 200 ==> Y = 5*200 = 1,000

Note: This result can also be achieved by using the multiplier formula:
Y = (multiplier)(Sp) = (multiplier)(I) ==> Y = 5*50 = 250,

that is, output increases from Yo = 750 to Y1 = 1,000.

1.e. From 1.a. and 1.d. we can see that the multiplier is 5.

1.f. Sp Y = Sp
AD1 = 200 = (0.8)Y

113
ADo = 150 + (0.8)Y

200

150

0
750 1,000 Y

2.a. Since the mpc has increased from 0.8 to 0.9, the size of the multiplier is now larger and we should
therefore expect a higher equilibrium income level than in 1.a.

AD = C + I = 100 + (0.9)Y + 50 = 150 + (0.9)Y ==>

Y = AD ==> Y = 150 + (0.9)Y ==> (0.1)Y = 150 ==> Y = 10*150 = 1,500.

2.b. From Y = (multiplier)(I) = 10*50 = 500 ==> Y1 = Yo + Y = 1,500 + 500 = 2,000.

2.c. Since the size of the multiplier has doubled from 5 to 10, the change in output (Y) that results from a
change in investment (I) now has also doubled from 250 to 500.

2.d. Sp Y = Sp
AD1 = 200 = (0.9)Y

ADo = 150 + (0.9)Y

200

150

0
1,500 2,000 Y
3.a. AD = C + I + G + NX = 50 + (0.8)YD + 70 + 200 = 320 + (0.8)[Y - (0.2)Y + 100]

= 400 + (0.8)(0.8)Y = 400 + (0.64)Y

From Y = AD ==> Y = 400 + (0.64)Y ==> (0.36)Y = 400

==> Y = (1/0.36)400 = (2.78)400 = 1,111.11

The size of the multiplier is (1/0.36) = 2.78.

3.b. BS = tY - TR - G = (0.2)(1,111.11) - 100 - 200 = 222.22 - 300 = - 77.78

114
3.c. AD' = 320 + (0.8)[Y - (0.25)Y + 100] = 400 + (0.8)(0.75)Y = 400 + (0.6)Y

From Y = AD' ==> Y = 400 + (0.6)Y ==> (0.4)Y = 400 ==> Y = (2.5)400 = 1,000

The size of the multiplier is now reduced to 2.5.

3.d. BS' = (0.25)(1,000) - 100 - 200 = - 50

BS' - BS = - 50 - (-77.78) = + 27.78

The size of the multiplier and equilibrium output will both increase with an increase in the marginal
propensity to consume. Therefore income tax revenue will also go up and the budget surplus should increase.

3.e. If the income tax rate is t = 1, then all income is taxed. There is no induced spending and equilibrium
income only increases by the change in autonomous spending, that is, the size of the multiplier is 1.

From Y = C + I + G ==> Y = Co + c(Y - 1Y + TRo) + Io + Go

==> Y = Co + cTRo + Io + Go = Ao

4. In Problem 3.d. we had a situation where the following was given:

Y = 1,000, t = 0.25, G = 200 and BS = - 50.

Assume now that t = 0.3 and G = 250 ==>

AD' = 50 + (0.8)[Y - (0.3)Y + 100] + 70 + 250 = 370 + (0.8)(0.7)Y + 80 = 450 + (0.56)Y.

From Y = AD' ==> Y = 450 + (0.56)Y ==> (0.44)Y = 450

==> Y = (1/0.44)450 = 1,022.73

BS' = (0.3)(1,022.73) - 100 - 250 = 306.82 - 350 = - 43.18

BS' - BS = -43.18 - (-50) = + 6.82

The budget surplus has increased, since the increase in tax revenue is larger than the increase in
government purchases.

5.a. While an increase in government purchases by G = 10 will change intended spending by Sp = 10, a
decrease in government transfers by TR = -10 will change intended spending by a smaller amount, that is, by
only Sp = c(TR) = c(-10). The change in intended spending equals Sp = (1 - c)(10) and equilibrium
income should therefore increase by

Y = (multiplier)(1 - c)10.

5.b. If c = 0.8 and t = 0.25, then the size of the multiplier is

115
 = 1/[1 - c(1 - t)] = 1/[1 - (0.8)(1 - 0.25)] = 1/[1 - (0.6)] = 1/(0.4) = 2.5.

The change in equilibrium income is

Y = (Ao) = [G + c(TR)] = (2.5)[10 + (0.8)(-10)] = (2.5)2 = 5

5.c. BS = t(Y) - TR - G = (0.25)(5) - (-10) - 10 = 1.25

Additional Problems:

1. "An increase in the marginal propensity to save increases the impact of one additional dollar in income on
consumption." Comment on this statement. In your answer discuss the effect of such a change in the mps on
the size of the expenditure multiplier.

The fact that the marginal propensity to save (1 - c) has risen implies that the marginal propensity to consume
(c) has fallen. This means that now one extra dollar in income earned will affect consumption by less than
before the reduction in the mpc. When the mpc is high, one extra dollar in income raises consumption by more
than when the mpc is low. If the mps is larger, then the expenditure multiplier will be larger, since the
expenditure multiplier is defined as 1/(1-c).

2. Using a simple model of the expenditure sector without any government involvement, explain the paradox
of thrift that asserts that a desire to save may not lead to an increase in actual saving.

The paradox of thrift occurs because the desire to increase saving leads to a lower consumption level. But a
lower level of spending sends the economy into a recession and we get a new equilibrium at a lower level of
output. In the end, the increase in autonomous saving is exactly offset by the decrease in induced saving due to
the lower income level. In other words, the economy is in equilibrium when S = Io. Since the level of
autonomous investment (Io) has not changed, the level of saving at the new equilibrium income level must also
equal Io.
This can also be derived mathematically. Since an increase in desired saving is equivalent to a decrease in
desired consumption, that is, Co = -So, the effect on equilibrium income is

Y = [1/(1 - c)](Co) = [1/(1 - c)](-So).

Therefore the overall effect on total saving is

S = s(Y) + So = [s/(1 - c)](-So) + So = 0, since s = 1 - c.

3. "When aggregate demand falls below the current output level, an unintended inventory accumulation
occurs and the economy is no longer in an equilibrium." Comment on this statement.

If aggregate demand falls below the equilibrium output level, production exceeds desired spending. When
firms see an unwanted accumulation in their inventories, they respond by reducing production. The level of
output falls and eventually reaches a level at which total output equals desired spending. In other words, the
economy eventually reaches a new equilibrium at a lower value of output.

116
4. For a simple model of the expenditure sector without any government involvement, derive the multiplier
in terms of the marginal propensity to save (s) rather than the marginal propensity to consume (c). Does this
formula still hold when the government enters the picture and levies an income tax?

In the text, the expenditure multiplier for a model without any government involvement was derived as

 = 1/(1 - c).

But since the marginal propensity to save is s = 1 - c, the multiplier now becomes  = 1/s = 1/(1-c).

In the text, we have also seen that if the government enters the picture and levies an income tax, then the
simple expenditure multiplier changes to

 = 1/[1 - c(1 - t)] = 1/(1 - c').

By substituting s = 1 - c, this equation can be easily manipulated, to get

’ = 1/[1 - c + ct] = 1/[s + (1 - s)t] = 1/s'.

Just as s = 1 – c, we can say that s' = 1 - c', since

s' = 1 - c' = 1 - c(1 - t) = 1 - c + ct = s + (1 - s)t.

This can also be derived in another way:


S = YD - C = YD - (C* + cYD) = - C* + (1 - c)YD = - C* + sYD

If we assume for simplicity that TR = 0 and NX = 0, then

S + TA = I + G ==> - C* + sYD + TA = I* + G* ==>

s(Y - tY - TA*) + tY + TA* = C* + I* + G* ==>

[s + (1 - s)t]Y = C* + I* + G* - (1 - s)TA* = A* ==>

Y = (1/[s + (1 - s)t])A* = (1/s')A*.

5. The balanced budget theorem states that the government can stimulate the economy without increasing the
budget deficit if an increase in government purchases (G) is financed by an equivalent increase in taxes (TA).
Show that this is true for a simple model of the expenditure sector without any income taxes.

If taxes and government purchases are increased by the same amount, then the change in the budget surplus
can be calculated as

BS = TAo - G = 0, since TAo = G.

The resulting change in national income is

117
Y = C + G = c(YD) + G = c(Y - TAo) + G

= c(Y) - c(TAo) + G = c(Y) + (1 - c)(G) since TAo = G.

==> (1 - c)(Y) = (1 - c)(G) ==> Y = G

In this case, the increase in output (Y) is exactly of the same magnitude as the increase in government
purchases (G). This occurs since the decrease in the level of consumption due to the higher lump sum tax has
exactly been offset by the increase in the level of consumption caused by the increase in income.

6. Assume a model without income taxes and in which the only two components of aggregate demand are
consumption and investment. Show that, in this case, the two equilibrium conditions Y = C + I and S = I are
equivalent.

We can derive the equilibrium value of output by setting actual income equal to intended spending, that is,

Y = C + I ==> Y = C* + cY + I* ==> (1 - c)Y = C* + I* ==> Y = [1/(1 - c)](C* + I*) = [1/(1 - c)]A*.

But since S = YD - C = Y - [C* + cY] = - C* + (1 - c)Y,

we can derive the same result from


S = I* ==> S = - C* + (1 - c)Y = I*

==> (1 - c)Y = C* + I* ==> Y = [1/(1 - c)](C* + I*) = [1/(1 - c)]A* .

7. In an effort to stimulate the economy in 1976, President Ford asked Congress for a $20 billion tax cut in
combination with a $20 billion cut in government purchases. Do you consider this a good policy proposal?
Why or why not?

This is not a good policy proposal. According to the balanced budget theorem, equal decreases in government
purchases and taxes will decrease rather than increase income. Therefore the intended result would not be
achieved.

8. Assume the following model of the expenditure sector:


Sp = C + I + G + NX C = 420 + (4/5)YD YD = Y - TA + TR TA = (1/6)Y
TRo = 180 Io = 160 Go = 100 NXo = - 40
(a) Assume the government would like to increase the equilibrium level of income (Y) to the full-employment
level Y* = 2,700. By how much should government purchases (G) be changed?
(b) Assume we want to reach Y* = 2,700 by changing government transfer payments (TR) instead. By how
much should TR be changed?
(c) Assume you increase both government purchases (G) and taxes (TA) by the same lump sum of G = TAo
= + 300. Would this change in fiscal policy be sufficient to reach the full-employment level of output at Y* =
2,700? Why or why not?
(d) Briefly explain how a decrease in the marginal propensity to save would affect the size of the expenditure
multiplier.

a. Sp = C + I + G + NX = 420 + (4/5)[Y - (1/6)Y + 100] + 160 + 180 - 40

118
= 720 + (4/5)(5/6)Y + 80 = 800 + (2/3)Y

From Y = Sp ==> Y = 800 + (2/3)Y ==> (1/3)Y = 800 ==>Y = 3*800 = 2,400

==> the expenditure multiplier is  = 3

From Y = (Ao) ==> 300 = 3(Ao) ==> (Ao) = 100

Thus government purchases should be changed by G = Ao = 100.

b. Since Ao = 100 and Ao = c(TRo) ==>100 = (4/5)(TRo) ==> TRo = 125.

c. This is a model with income taxes, so the balanced budget theorem does not apply in its strictest form,
which states that an increase in government purchases and taxes by a certain amount increases national income
by that same amount, leaving the budget surplus unchanged. Here total tax revenue actually increases by more
than 100, since taxes are initially increased by a lump sum of 100, but then income taxes also change due to the
change in income. Thus income does not increase by Y = 300, as we can see below.

Y = (G) + (-c)(TAo) = 3*300 + 3*[-(4/5)300] = 900 - 720 = 180

This change in fiscal policy will increase income by only Y = 180, from Y0 = 2,400 to Y1 = 2,580, and
we will be unable to reach Y* = 2,700.

d. If the marginal propensity to save decreases, people spend a larger portion of their additional disposable
income, that is, the mpc and the slope of the [C+I+G+NX]-line increase. This will lead to an increase in the
expenditure multiplier and equilibrium income.

9. Assume a model with income taxes similar to the model in Problem 9 above. This time, however, you
have only limited information about the model, that is, you only know that the marginal propensity to consume
out of disposable income is c = 0.75, and that total autonomous spending is Ao = 900, such that Sp = Ao + c'Y
= 900 + c'Y. You also know that you can reach the full-employment level of output at Y* = 3,150 by increasing
government transfers by a lump sum of TR = 200.
(a) What is your current equilibrium level?
(b) Is it possible to determine the size of the expenditure multiplier with the information you have?
Assume you want to change the income tax rate (t) in order to reach the full-employment level of income Y* =
3,150. How would this change in the income tax rate affect the size of the expenditure multiplier?

a. Since A = c(TR) = (0.75)200 = 150,


the new [C+I+G+NX]-line is of the form Sp1 = 1,050 + c1Y.
For each model of the expenditure sector we can derive the equilibrium level of income by using the
following equation:

Y* = Ao = 1/(1-c’) ==> 3,150 = 1,050 ==> the expenditure multiplier is  = 3.

If we now change autonomous spending by A = 150, then income will have to change by

Y = (A) ==> Y = 3*150 = 450.

119
Therefore the old equilibrium level of income must have been Y = 3,150 - 450 = 2,700.

b. From our work above we can see that the size of the multiplier is  = 3.

c. The new [C+I+G+NX]-line is of the form Sp2 = 900 + c2Y. This new intended spending line intersects the
45-degree line at Y = 3,150. Thus the slope of the new intended spending line can be derived as

c2 = (3,150 - 900)/(3,150) = 5/7.


From Y = Sp2 ==> Y = 900 + (5/7)Y ==> (2/7)Y = 900 ==>
Y = (7/2)900 = (3.5)900 = 3,150.
The new value of the multiplier is 3.5

Sp Y = Sp

Sp2 = 900 +(5/7)Y

Sp1 = 900 = (2/3)Y


3,150

rise

900

run
0

2,700 3,150 Y

10. Assume you have the following model of the expenditure sector:
Sp = C + I + G + NX C = 400 + (0.8)YD Io = 200 Go = 300 + (0.1)(Y* - Y)
YD = Y - TA + TR NXo = - 40 TA = (0.25)Y TRo = 50
(a) What is the size of the output gap if potential output is at Y* = 3,000?
(b) By how much would investment (I) have to change to reach equilibrium at Y* = 3,000, and how does
this change affect the budget surplus?
(c) From the model above you can see that government purchases (G) are counter-cyclical, that is they are
increased as national income decreases. If you compare this specification of G with a constant level of G, how
is the value of the expenditure multiplier affected?
(d)Assume the equation for net exports is changes such that NXo = - 40 is now NX1 = - 40 - mY, with 0 <
m < 1. How would this affect expenditure multiplier?

a. Sp = 400 + (0.8)YD + 200 + 300 + (0.1)(3,000 - Y) - 40

= 1,160 + (0.8)(Y - (0.25)Y + 50) - (0.1)Y = 1,200 + [(0.8)(0.75) - (0.1)]Y = 1,200 + (0.5)Y

Y = Sp ==> Y = 1,200 + (0.5)Y ==> (0.5)Y = 1,200 ==>Y = 2*1,200 = 2,400

120
The output gap is Y* - Y = 3,000 - 2,400 = 600.

b. From Y = (mult.)(A) ==> 600 = 2(I) ==> I = 300

BuS = TA - TR - G = (0.25)(2,400) - 50 - [300 + (0.1)(600)] = 600 - 50 - 300 - 60 = 190

BuS* = (0.25)(3,000) - 50 - 300 = 400, so the budget surplus increases by BuS = 210.

c. If government purchases are used as a stabilization tool, the size of the multiplier should be lower than if
the level of government spending is fixed. In the model of the expenditure sector above, the slope of the
[C+I+G+NX]-line is c' = 0.5 compared to c" = 0.6, when government purchases were defined as G = 300.

With this change, net exports decrease as national income increases. This additional leakage implies that the
size of the multiplier will decrease. In the model above, the slope of the [C+I+G+NX]-line decreases from c' =
(0.5) to c" = (0.5) - m. Therefore the expenditure multiplier will decrease from 1/[1 - (0.5)] to 1/[1 - (0.5) + m].

11. Assume you have the following model of the expenditure sector:
Sp = C + I + G + NX C = Co + cYD YD = Y - TA + TR TA = TAo
TR = TRo I = Io G = Go NX = NXo
(a) If a decrease in income (Y) by 800 leads to a decrease in savings (S) by 160, what is the size of the
expenditure multiplier?
(b) If a decrease in taxes (TA) by 400 leads to an increase in income (Y) by 1,200, how large is the
marginal propensity to save?
(c) If an increase in imports by 200 (NX = - 200) leads to a decrease in consumption (C) by 800, what
is the size of the expenditure multiplier?

Recall that the expenditure multiplier for such a simple model can be calculated as:

 = 1/(1 - c)

a. (S)/(Y) = 1 - c = (-160)/(-800) = .2 ==> 1/(1 - c) = 1/(.2) = 5 ==> the multiplier is  = 5.

b. From (Y) = [-c(TAo)] ==> (Y)/(TAo) = (-c) = (-c)/(1 - c) ==>

(1,200)/(-400) = - 3 = (-c)/(1 - c) ==> -3(1 - c) = -c ==> c = 3/4

==> mps = 1 - c = 1/4 = 0.25.

c. Y = C + NX = -800 + (-200) = - 1,000

==> c = (C)/(Y) = (-800)/(-1,000) = .8 ==> multiplier =  = 1/(1 - c) = 1/(.2) = 5

12. Explain why income taxation, the Social Security system, and unemployment insurance are considered
automatic stabilizers.

Income taxes, unemployment benefits, and the Social Security system are often called automatic stabilizers
because they reduce the amount by which output changes as a result of a change in aggregate demand. These
stabilizers are a part of the structure of the economy and therefore work without any actual government

121
intervention. For example, when output declines and unemployment increases. If we had no unemployment
insurance, people out of work would not receive any disposable income and then consumption would drop
significantly. But since unemployed workers get unemployment compensation, consumption will not decrease
as much. Therefore, aggregate demand may not be reduced by as much as it would have without these
automatic stabilizers.

13. Assume a simple model of the expenditure sector with a positive income tax rate (t). Show mathematically
how an increase in lump sum taxes (TAo ) would affect the budget surplus.

From BS = TA - G - TR = tY + TAo - G – TR

==> BS = t(Y) + TAo = t(mult.)(-c)(TAo) + TAo

= t[1/(1 - c + ct)](-c)(TAo) + TAo = ([ -(ct) + 1 - c + (ct)]/[1 - c + (ct)])(TAo)

= (1 - c) /[1 - c + (ct)])(TAo) > 0, since c < 1

In other words, a lump sum tax increase would increase the budget surplus.

14. True or false? Why?


"A tax cut will increase national income and will therefore always increase the budget surplus."

False. Although a tax cut raises national income, not all of the increase in income is spent, nor is it completely
taxed away. Income tax revenues fall and the budget deficit rises. Assume the following model of the
expenditure sector:

Sp = C + I + G + NX I = Io
C = Co + cYD G = Go
YD = Y - TA +TR NX = NXo
TA = TAo + tY BS = TA - G - TR
TR = TRo

From Y = Sp ==> Y = Co + c(Y - TAo - tY + TRo) + Io + Go + NXo ==>

Y = Co - cTAo + cTRo + Io + Go + NXo + c(1 - t)Y = Ao + c'Y ==>

Y = [1/(1 - c')]Ao with c' = c (1- t)

Thus Y = [1/(1 - c')][(-c)(TAo)]

and BS = t(Y) + (TAo) = {[t(-c)]/(1 - c') + 1}(TAo) ==>

= {[-(ct) + 1 - c + (ct)]/[1 - c + (ct)]}(TAo) = {(1 - c)/[1 - c + (ct)]}(TAo) > 0 if TA > 0.

Therefore, if taxes fall, that is, if TA < 0, the budget surplus decreases.
15. Assume a simple model of the expenditure sector with a positive income tax rate (t). Show mathematically
how a decrease in autonomous investment (Io ) would affect the budget surplus.

122
A decrease in autonomous investment (Io) will have a multiplier effect and will therefore decrease national
income and tax revenue. The budget surplus will decrease as shown below:

BS = t(Y) = t(Io) < 0

16. "An increase in government purchases will always pay for itself, as it raises national income and hence the
government's tax revenues." Comment on this statement.

An increase in government purchases will increase the budget deficit. If we assume a model of the expenditure
sector with income taxes, then the multiplier equals [1/(1 - c')] with c' = c (1- t). The change in the budget
surplus that arises from a change in government purchases can be calculated as

BS = t(Y) - G = t[1/(1 - c')](G) - G = {[t - 1 + c - (ct)]/[1 - c + (ct)]}(G)

= - {[(1 - c)(1 - t)]/(1 - c + (ct)]}(G) < 0, sine G > 0.

Therefore, if government purchases are increased, the budget surplus will decrease.

17. Is the size of the actual budget surplus always a good measure for determining fiscal policy? What about
the size of the full-employment budget surplus?

The actual budget surplus has a cyclical and a structural component. The cyclical component of the budget
surplus changes with changes in the level of income whether or not any fiscal policy measure has been
implemented. This implies that the actual budget surplus also changes with changes in income and is therefore
not a very good measure for assessing fiscal policy. The structural (full-employment) budget surplus is
calculated under the assumption that the economy is at full-employment. It therefore changes only with a
change in fiscal policy and is a much better measure for fiscal policy than the actual budget surplus. One
should keep in mind, however, that the balanced budget theorem implies that the government can stimulate
national income by an equivalent and simultaneous increase in taxes and government purchases, thereby
affecting the actual or the full-employment budget surplus.

18. Assume a model of the expenditure sector with income taxes, in which people who pay taxes, have a higher
marginal propensity to consume than people who receive government transfers, and the consumption function
is of the following form: C = Co + c(Y - TA) + dTR, with c < d.
(a) What will happen to the equilibrium level of income and the budget surplus if government purchases are
reduced by the same lump sum amount as taxes?
(b) What will happen to the equilibrium level of income and the budget surplus if government transfers
are reduced by the same lump sum amount as taxes?

a. Assume that TAo = G = - 100 ==>

Y = [(-c)/(1 - c')(TAo) + [1/(1 - c')](G) = [(1 - c)/(1 - c')](-100) < 0 c' = c(1 - t)
National income would decrease.

BS = t(Y) + TAo - G = t(Y) < 0

The budget surplus would decrease by the loss in income tax revenue.

123
b. Assume that TAo = TRo = - 100 ==>

Y = [(-c)/(1 - c')](TAo) + [d/(1 - c')](TRo) = [(d - c)/(1 - c')](-100) < 0 c' = c(1 - t)

National income would increase.


BS = t(Y) + TAo - TRo = t(Y) < 0

The budget surplus would decrease.

19. True or false? Why?


"The higher the marginal propensity to import, the lower the size of the multiplier."

True. Imports represent a leakage out of the income flow. An increase in autonomous spending will raise
income and we will see the usual multiplier effect. However, if imports are positively related to income, this
effect is reduced since higher imports reduce the level of domestic demand.

Closed Economy Model Open Economy Model

Sp =C+I+G Sp = C + I + G + NX
C = Co + cY C = Co + cY
G = Go G = Go
I = Io I = Io
NX = NXo - mY with m > 0

From Y = Sp ==>

Y = (Co + Io + Go) + cY Y = (Co + Io + Go + NXo) + (c - m)Y

Y = Ao + cY Y = Ao + (c - m)Y

Y = [1/(1 - c)]Ao Y = [1/(1 - c + m)]Ao

Therefore the multiplier is defined as

[1/(1 - c)] [1/(1 - c + m)]

Clearly the open economy multiplier falls short of the closed economy multiplier. This is because leakages
reduce demand. If income taxes were included in these models, they too would reduce the multipliers, as
income taxes represent another leakage from the income flow.

CHAPTER 10

MONEY, INTEREST, AND INCOME

124
Answers to Problems in the Textbook:

Conceptual Problems:

1. The model in Chapter 9 assumed that both the price level and the interest rate were fixed. But the IS-LM
model lets the interest rate fluctuate and determines the combination of output demanded and the interest rate
for a fixed price level. It should be noted that while the upward-sloping AD-curve in Chapter 9 (the
[C+I+G+NX]-line in the Keynesian cross diagram) assumed that interest rates and prices were fixed, the
downward-sloping AD-curve that is derived at the end of Chapter 10 from the IS-LM model lets the price level
fluctuate and describes all combinations of the price level and the level of output demanded at which the goods
and money sector simultaneously are in equilibrium.

2.a. If the expenditure multiplier () becomes larger, the increase in equilibrium income caused by a unit
change in intended spending also becomes larger. Assume investment spending increases due to a change in
the interest rate. If the multiplier  becomes larger, any increase in spending will cause a larger increase in
equilibrium income. This means that the IS-curve will become flatter as the size of the expenditure multiplier
becomes larger.
If aggregate demand becomes more sensitive to interest rates, any change in the interest rate causes the
[C+I+G+NX]-line to shift up by a larger amount and, given a certain size of the expenditure multiplier , this
will increase equilibrium income by a larger amount. As a result, the IS-curve will become flatter.

2.b. Monetary policy changes affect interest rates and this leads to a change in intended spending, which is
reflected in a change in income. In 2.a. it was explained that a steep IS-curve means either that the multiplier 
is small or that desired spending is not very interest sensitive. Therefore, an increase in money supply will
reduce interest rates. However, this does not result in a large increase in aggregate demand if spending is very
interest insensitive. Similarly, if the multiplier is small, then any change in spending will not affect output
significantly. Therefore, the steeper the IS-curve, the weaker the effect of monetary policy changes on
equilibrium output.

3. Assume that money supply is fixed. Any increase in income will increase money demand and the resulting
excess demand for money will drive the interest rate up. This, in turn, will reduce the quantity of money
balances demanded to bring the money sector back to equilibrium. But if money demand is very interest
insensitive, then a larger increase in the interest rate is needed to reach a new equilibrium in the money sector.
As a result, the LM-curve becomes steeper.
Along the LM-curve, an increase in the interest rate is always associated with an increase in income. This
means that an increase in money demand (due to an increase in income) has to be offset by a decrease in the
quantity of money demanded (due to an increase in the interest rate) to keep the money sector in equilibrium.
But if money demand becomes more income sensitive, a smaller change in income is required for any specific
change in the interest rate to keep the money sector in equilibrium. Therefore, the LM-curve becomes steeper
as money demand becomes more income sensitive.

4.a. A horizontal LM-curve implies that the public is willing to hold whatever money is supplied at any given
interest rate. Therefore, changes in income will not affect the equilibrium interest rate in the money sector. But
if the interest rate is fixed, we are back to the analysis of the simple Keynesian model used in Chapter 9. In
other words, there is no offsetting effect (or crowding-out effect) to fiscal policy.

125
4.b. A horizontal LM-curve implies that changes in income do not affect interest rates in the money sector.
Therefore, if expansionary fiscal policy is implemented, the IS-curve shifts to the right, but the level of
investment spending is no longer negatively affected by rising interest rates, that is, there is no crowding-out
effect. In terms of Figure 10-3, the interest rate not longer serves as the link between the goods and assets
markets.

4.c. A horizontal LM-curve results if the public is willing to hold whatever money balances are supplied at a
given interest rate. This situation is called the liquidity trap. Similarly, if the Fed is prepared to peg the interest
rate at a certain level, then any change in income will be accompanied by an appropriate change in money
supply. This will lead to continuous shifts in the LM-curve, which is equivalent to having a horizontal LM-
curve, since the interest rate will never change.

5. From the material presented in the text we know that when intended spending becomes more interest
sensitive, then the IS-curve becomes flatter. Now assume that an increase in the interest rate stimulates saving
and therefore reduces the level of consumption. This means that now not only investment spending but also
consumption is negatively affected by an increase in the interest rate. In other words, the [C+I+G+NX]-line in
the Keynesian cross diagram will now shift down further than previously and the level of equilibrium income
will decrease more than before. In other words, the IS-curve has become flatter.
This can also be shown algebraically, since we can now write the consumption function as follows:

C = C* + cYD - gi

In a simple model of the expenditure sector without income taxes, the equation for aggregate demand will
now be

AD = Ao + cY - (b + g)i.

From Y = AD ==> Y = [1/(1 - c)][Ao - (b + g)i] ==>

i = [1/(b + g)]Ao - [(1 - c)/(b + g)]Y

Therefore, the slope of the IS-curve has been reduced from (1 - c)/b to (1 - c)/(b + g).

6. In the IS-LM model, a simultaneous decline in interest rates and income can only be caused by a shift of
the IS-curve to the left. This shift in the IS-curve could have been caused by a decrease in private spending due
to negative business expectations or a decline in consumer confidence. In 1991, the economy was in a
recession and firms did not want to invest in new machinery and, since consumer confidence was very low,
people were not expected to increase their level of spending. In the IS-LM diagram the adjustment process can
be described as follows:

Io  ==> Y  (the IS-curve shifts left) ==> md  ==> i  ==> I  ==> Y . Effect: Y  and i  .

i ISo LM

IS1
i1

126
i2

0
Y2 Y1 Y

Technical Problems:

1.a. Each point on the IS-curve represents an equilibrium in the expenditure sector. Therefore the IS-curve can
be derived by setting

Y = C + I + G = (0.8)[1 - (0.25)]Y + 900 - 50i + 800 = 1,700 + (0.6)Y - 50i ==>


(0.4)Y = 1,700 - 50i ==> Y = (2.5)(1,700 - 50i) ==> Y = 4,250 - 125i.

1.b. The IS-curve shows all combinations of the interest rate and the level of output such that the expenditure
sector (the goods market) is in equilibrium, that is, intended spending is equal to actual output. A decrease in
the interest rate stimulates investment spending, making intended spending greater than actual output. The
resulting unintended inventory decrease leads firms to increase their production to the point where actual
output is again equal to intended spending. This means that the IS-curve is downward sloping.

1.c. Each point on the LM-curve represents an equilibrium in the money sector. Therefore the LM-curve can be
derived by setting real money supply equal to real money demand, that is,

M/P = L ==> 500 = (0.25)Y - 62.5i ==> Y = 4(500 + 62.5i) ==> Y = 2,000 + 250i.

1.d. The LM-curve shows all combinations of the interest rate and level of output such that the money sector is
in equilibrium, that is, the demand for real money balances is equal to the supply of real money balances. An
increase in income will increase the demand for real money balances. Given a fixed real money supply, this
will lead to an increase in interest rates, which will then reduce the quantity of real money balances demanded
until the money market clears. In other words, the LM-curve is upward sloping.

1.e. The level of income (Y) and the interest rate (i) at the equilibrium are determined by the intersection of the
IS-curve with the LM-curve. At this point, the expenditure sector and the money sector are both in equilibrium
simultaneously.

From IS = LM ==> 4,250 - 125i = 2,000 + 250i ==> 2,250 = 375I ==> i = 6

==> Y = 4,250 - 125*6 = 4,250 - 750 ==> Y = 3,500

Check: Y = 2,000 + 250*6 = 2,000 + 1,500 = 3,500

125 IS

LM

127
6

0
2,000 3,500 4,250 Y

2.a. As we have seen in 1.a., the value of the expenditure multiplier is  = 2.5. This multiplier  is derived in
the same way as in Chapter 9. But now intended spending also depends on the interest rate, so we no longer
have Y = Ao, but rather

Y = (Ao - bi) = (1/[1 - c + ct])(Ao - bi) ==> Y = (2.5)(1,700 - 50i) = 4,250 - 125i.

2.b.This can be answered most easily with a numerical example. Assume that government purchases increase
by G = 300. The IS-curve shifts parallel to the right by

==> IS = (2.5)(300) = 750.

Therefore IS': Y = 5,000 - 125i

From IS' = LM ==> 5,000 - 125i = 2,000 + 250i ==> 375i = 3,000 ==> i = 8

==> Y = 2,000 + 250*8 ==> Y = 4,000 ==> Y = 500

When interest rates are assumed to be constant, the size of the multiplier is equal to  = 2.5, that is, (Y)/(G)
= 750/300 = 2.5. But when interest rates are allowed to vary, the size of the multiplier is reduced to 1 = (Y)/
(G) = 500/300 = 1.67.

2.c. Since an increase in government purchases by G = 300 causes a change in the interest rate of 2
percentage points, government spending has to change by G = 150 to increase the interest rate by 1
percentage point.

2.d. The simple multiplier  in 2.a. shows the magnitude of the horizontal shift in the IS-curve, given a change
in autonomous spending by one unit. But an increase in income increases money demand and the interest rate.
The increase in the interest rate crowds out some investment spending and this has a dampening effect on
income. The multiplier effect in 2.b. is therefore smaller than the multiplier effect in 2.a.

3.a. An increase in the income tax rate (t) will reduce the size of the expenditure multiplier ( ). But as the
multiplier becomes smaller, the IS-curve becomes steeper. As we can see from the equation for the IS-curve,
this is not a parallel shift but rather a rotation around the vertical intercept.

Y = (Ao - bi) = [1/(1 - c + ct)](Ao - bi) ==> i = (1/b)Ao - (/b)Y = (1/b)Ao - (1/b)[1 - c + ct]Y

3.b. If the IS-curve shifts to the left and becomes steeper, the equilibrium income level will decrease. A higher
tax rate reduces private spending and this will lower national income.

3.c. When the income tax rate is increased, the equilibrium interest rate will also decrease. The adjustment to

128
the new equilibrium can be expressed as follows (see graph on the next page):

t up ==> C down ==> Y down ==> md down ==> i down ==> I up ==> Y up. Effect: Y  and i 

IS1
i ISo LM

i1

i2

0
Y2 Y1 Y

4.a. If money demand is less interest sensitive, then the LM-curve is steeper and monetary policy changes
affect equilibrium income to a larger degree. If money supply is assumed to be fixed, the adjustment to a new
equilibrium in the money sector has to come solely through changes in money demand. If money demand is
less interest sensitive, any increase in money supply requires a larger increase in income and a larger decrease
in the interest rate in order to bring the money sector into a new equilibrium.

i i

IS LM1 LM2 IS
LM1

i1 i1 LM2
i2
i2

0 Y1 Y2 Y 0 Y1 Y2 Y

The adjustment process in each of the two diagrams is the same; however, in the case of a more interest-
sensitive money demand (a flatter LM-curve), the change in Y and i will be smaller.

(M/P) up ==> i down ==> I up ==> Y up ==> md up ==> i up Effect: Y and i 

Section 10-5 derives the equation for the LM-curve and the equation for the monetary policy multiplier as

129
i = (1/h)[kY - (M/P)] and (Y)/(M/P) = (b/h)

respectively. If money demand becomes more interest sensitive, the value of h becomes larger and the slope of
the LM-curve becomes flatter, while the size of the monetary policy multiplier becomes smaller.

4.b. An increase in money supply drives interest rates down. This decrease in interest rates will stimulate
intended spending and thus income. If money demand becomes less interest sensitive, a larger increase in
income is required to bring the money sector into equilibrium. But this implies that the overall decrease in the
interest rate has to be larger, given that the interest sensitivity of spending has not changed.

5. The price adjustment, that is, the movement along the AD-curve, can be explained in the following way:
With nominal money supply (M) fixed, real money balances (M/P) will decrease as the price level (P)
increases. There is an excess demand for money and interest rates will rise. This will lead to a decrease in
investment spending and thus the level of output demanded will decrease. In other words, the LM-curve will
shift to the left as real money balances decrease.

6. In the classical case, the AS-curve is vertical. Therefore, any increase in aggregate demand due to
expansionary monetary policy will, in the long run, not lead to any increase in output but simply lead to an
increase in the price level. An increase in money supply will first shift the LM-curve to the right. This implies
a shift of the AD-curve to the right. Therefore we have excess demand for goods and services and prices will
begin to rise. But as the price level rises, real money balances will begin to fall again, eventually returning to
their original level. Therefore, the shift of the LM-curve to the right due to the expansionary monetary policy
and the resulting shift of the AD-curve will be exactly offset by a shift of the LM-curve to the left and a
movement along the AD-curve to the new long-run equilibrium due to the price adjustment. At this new long-
run equilibrium, the level of output and interest rates will not have changed while the price level will have
changed proportionally to the nominal money supply, leaving real money balances unchanged. In other words,
money is neutral in the long run (the classical case).

7.a. An increase in the demand for money will shift the LM-curve to the left, raising the interest rate and
lowering the level of output demanded. As a result, the AD-curve will also shift to the left. In the Keynesian
case, the price level is assumed to be fixed, that is, the AS-curve is horizontal. In this case, the decrease in
income in the AD-AS diagram is equivalent to the decrease in income in the IS-LM diagram, since there is no
price adjustment, that is, the real balance effect does not come into play.

7.b. An increase in the demand for money will shift the LM-curve to the left, raising the interest rate and
lowering the level of output demanded. As a result, the AD-curve will also shift to the left. In the classical case,
the level of output will not change, since the AS-curve is vertical. In this case, the shift in the AD-curve will
simply be reflected in a price decrease, but the level of output will remain unchanged. The real balance effect
causes the LM-curve to shift back to its original level, since the price decrease causes an increase in real
money balances.

Additional Problems:

130
1. True or false? Explain your answer.
“A decrease in the marginal propensity to save implies that the IS-curve will become steeper.”

False A decrease in the marginal propensity to save (s = 1 - c) is equivalent to an increase in the marginal
propensity to consume (c), which, in turn, implies an increase in the expenditure multiplier (). But with a
larger expenditure multiplier, any increase in investment spending due to a decrease in the interest rate will
lead to a larger increase in income. Therefore the IS-curve will become flatter and not steeper.

2. True or false? Explain your answer.


“If the central bank keeps the supply of money constant, then the money supply curve is vertical, which
implies a vertical LM-curve.”

False. Equilibrium in the money sector implies that real money supply is equal to real money demand, that is,

ms = M/P = md(i,Y).

This implies that any increase in income (Y) will increase the demand for money. To bring the money sector
back into equilibrium, interest rates (i) have to rise simultaneously to bring the quantity of money demanded
back to the original level (equal to the fixed supply of money). Therefore, to keep the money sector in
equilibrium, an increase in income must always be associated with an increase in the interest rate and the LM-
curve must be upward sloping.

3. "Restrictive monetary policy reduces consumption and investment." Comment on this statement.

A reduction in money supply raises interest rates, which will, in turn, have a negative effect on the level of
investment spending. The level of consumption may also decrease as it becomes more costly to finance
expenditures by borrowing money. But even if it is assumed that consumption is not affected by changes in the
interest rate, consumption will still decrease since restrictive monetary policy will reduce national income and
therefore private spending.

4. "If government spending is increased, money demand will increase." Comment.

A change in government spending directly affects the expenditure sector and therefore the IS-curve. But in an
IS-LM framework, the money sector is also affected indirectly. An increase in the level of government
spending will shift the IS-curve to the right, leading to an increase in income. But the increase in income will
lead to an increase in money demand, so the interest rate will have to increase in order to lower the quantity of
money demanded and to bring the money sector back into equilibrium. Overall no change in money demand
can occur, since equilibrium in the money sector requires that ms = M/P = md, that is, money supply has to be
equal to money demand, and money supply is assumed to be fixed.
5. "An increase in autonomous investment reduces the interest rate and therefore the money sector will no
longer be in equilibrium." Comment on this statement.

An increase in autonomous investment shifts the IS-curve to the right. The increase in income leads to an
increase in the demand for money, which means that interest rates increase. The increase in interest rates then
reduces the quantity of money demanded again to bring the money market back to equilibrium.

131
6. "A monetary expansion leaves the budget surplus unaffected." Comment on this statement.

Expansionary monetary policy, that is, an increase in money supply, will lower interest rates (the LM-curve
will shift to the right). Lower interest rates will lead to an increase in investment spending and the economy
will therefore be stimulated. But a higher level of national income increases the government’s tax revenues and
therefore the budget surplus will increase.

7. "Restrictive monetary policy implies lower tax revenues and therefore to an increase in the budget
deficit." Comment on this statement.

A decrease in money supply will shift the LM-curve to the left. This will lead to an increase in the interest rate,
which will lead to a reduction in spending and thus national income. But as income decreases, so does income
tax revenue. Therefore, the budget deficit will increase because of the change in its cyclical component.

8. “If the demand for money becomes more sensitive to changes in income, then the LM-curve becomes
flatter.” Comment on this statement.

Along the LM-curve, an increase in the interest rate is always associated with an increase in income. This
means that an increase in money demand (due to an increase in income) has to be offset by a decrease in the
quantity of money demanded (due to an increase in the interest rate) to keep the money sector in equilibrium.
But if money demand becomes more income sensitive, a smaller change in income is required for any specific
change in the interest rate to keep the money sector in equilibrium. Therefore, the LM-curve becomes steeper
(and not flatter) as money demand becomes more sensitive to changes in income.

9. “A decrease in the income tax rate will increase the demand for money, shifting the LM-curve to the
right.” Comment on this statement.

A decrease in the income tax rate (t) will increase the expenditure multiplier (). But with a larger expenditure
multiplier, any increase in investment spending due to a decrease in the interest rate will lead to a larger
increase in income. Since fiscal policy affects the expenditure sector, the IS-curve (not the LM-curve) will
shift. The IS-curve will become flatter and shift to the right. This will lead to a new equilibrium at a higher
level of income (Y) and a higher interest rate (i). But money supply is fixed and the LM-curve remains
unaffected by fiscal policy. Therefore, at the new equilibrium (the intersection of the new IS-curve with the old
LM-curve) the demand for money will not have changed, since the money sector has to be in an equilibrium at
ms = md(i,Y).
10. “If the demand for money becomes more insensitive to changes in the interest rate, equilibrium in the
money sector will have to be restored mostly through changes in income. This implies a flat LM-curve.”
Comment on this statement.

Any increase in income will increase money demand and this will drive the interest rate up. Therefore, the
quantity of money balances demanded will decline again until the money sector is back in equilibrium. But if
money demand is very interest insensitive, then a larger increase in the interest rate is needed to reach a new
equilibrium in the money sector. This means that the LM-curve is steep and not flat.

11. Assume the following IS-LM model:


Expenditure Sector Money Sector

132
Sp = C + I + G + NX M = 700
C = 100 + (4/5)YD P =2
YD = Y - TA md = (1/3)Y + 200 - 10i
TA = (1/4)Y
I = 300 - 20i
G = 120
NX = -20

(a) Derive the equilibrium values of consumption (C) and money demand (md).
(b) How much of investment (I) will be crowded out if the government increases its purchases by G =
160 and nominal money supply (M) remains unchanged?
(c) By how much will the equilibrium level of income (Y) and the interest rate (i) change, if nominal
money supply is also increased to M' = 1,100?

a. Sp = 100 + (4/5)[Y - (1/4)Y] + 300 - 20i + 120 - 20 = 500 + (4/5)(3/4)Y – 20i = 500 + (3/5)Y - 20i

From Y = Sp ==> Y = 500 + (3/5)Y - 20i ==> (2/5)Y = 500 - 20i

==> Y = (2.5)(500 - 20i) ==> Y = 1,250 - 50i IS-curve

From M/P = md ==> 700/2 = (1/3)Y + 200 - 10i ==> (1/3)Y = 150 + 10i

==> Y = 3(150 + 10i) ==> Y = 450 + 30i LM-curve

IS = LM ==> 1,250 - 50i = 450 + 30i ==> 800 = 80i ==> i = 10

==> Y = 1,250 - 50*10 ==> Y = 750

C = 100 + (4/5)(3/4)750 = 100 + (3/5)750 ==> C = 550

ms = M/P = 700/2 = 350 = md

Check: md = (1/3)750 + 200 - 10*10 = 350

25 ISo LMo

10

0
450 750 1,250 Y

133
b. IS = (2.5)160 = 400 ==> IS' = 1,650 - 50i

IS' = LM ==> 1,650 - 50i = 450 + 30i ==> 1,200 = 80i ==> i = 15

==> Y = 1,650 - 50*15 ==> Y = 900

Since i = + 5 ==> I = - 20*5 ==> I = - 100

Check: Sp = G + I = 160 – 100 = 60 ==> Y = 1(Sp) = 2.5*60 =150

i IS1
33
25 LMo

15
10

450 750 900 1,250 1,650 Y

c. From M'/P = md ==> 1,100/2 = (1/3)Y + 200 - 20i

==> (1/3)Y = 350 - 20i ==> Y = 3(350 - 20i) ==> Y = 1,050 + 30i

IS1 = LM1 ==> 1,650 - 50i = 1,050 + 30i ==> 600 = 80i ==> i = 7.5

==> Y = 1,650 - 50(7.5) = 1,275.

==> i = - 7.5 and Y = 375 as compared to (b).


i

25 IS1 LMo
LM1

10
7.5
0

450 900 1,050 1,275 1,650 Y

12. Assume the money sector can be described by these equations: M/P = 400 and md = (1/4)Y - 10i. In the
expenditure sector only investment spending (I) is affected by the interest rate (i), and the equation of the IS-
curve is: Y = 2,000 - 40i.
(a) If the size of the expenditure multiplier is  = 2, show the effect of an increase in government
purchases by G = 200 on income and the interest rate.

134
(b) Can you determine how much of investment is crowded out as a result of this increase in government
spending?
If the money demand equation were changed to md = (1/4)Y, how would your answers in (a) and (b) change?

a. From M/P = md ==> 400 = (1/4)Y - 10i ==> Y = 1,600 + 40i LM-curve

From IS = LM ==> 2,000 - 40i = 1,600 + 40i ==> 80i = 400 ==> i = 5

==> Y = 2,000 - 40*5 ==> Y = 1,800

IS = 2*200 = 400 ==> IS' = 2,400 - 40i

IS' = LM ==> 2,400 - 40i = 1,600 + 40i ==> 80i = 800 ==> i = 10

==> Y = 1,600 + 40*10 ==> Y = 2,000

Therefore i = + 5 and Y = + 200

Since the size of the expenditure multiplier is  = 2 but income only goes up by Y = 200, the fiscal policy
multiplier in the IS-LM model is 1= 1. But this means that the level of investment has been reduced by 100,
that is, I = -100. This can be seen by restating the IS-curve as follows:

Y = 2,000 - 40i = Y = 2(1,000 - 20i)

Since government purchases are changed by G = 200 ==> Y = 2(1,200 - 20i), which means that the IS-curve
shifts by IS = 2*200 = 400. But the increase in income is actually only Y = 200. This implies that
investment changes by I = -100. Investment is of the form I = Io – 20i; however, since the interest rate went
up by i = 5, investment changes by I = - 20*5 = - 100.

From Y = (Sp) ==> 200 = 2(Sp) ==> Sp = 100

But since Sp = G + I ==> 100 = 200 + I ==> I = - 100

c. If md = (1/4)Y, then we have the classical case, that is, a vertical LM-curve. In this case, fiscal expansion
will not change income at all. This occurs since the increase in G will be offset by a decrease in I of equal
magnitude due to an increase in the interest rate.

(M/P) = md ==> 400 = (1/4)Y ==> Y = 1,600 LM-curve

IS = LM ==> 2,000 - 40i = 1,600 ==> 40i = 400 ==> i = 10 ==> Y = 1,600

IS' = LM ==> 2,400 - 40i = 1,600 ==> 40i = 800

==> i = 20 ==> Y = 1,600 ==> I = - 200

13. Assume money demand (md) and money supply (ms) are defined as: md = (1/4)Y + 400 - 15i and ms =
600, and intended spending is of the form: Sp = C + I + G + NX = 400 + (3/4)Y - 10i.

135
Calculate the equilibrium levels of Y and i, and indicate by how much the Fed would have to change
money supply to keep interest rates constant if the government increased its spending by G = 50. Show your
solutions graphically and mathematically.

ms = md ==> 600 = (1/4)Y + 400 - 15i ==> (1/4)Y = 200 + 15i

==> Y = 4(200 + 15i) ==> Y = 800 + 60i LM-curve

Y = C + I + G + NX ==> Y = 400 + (3/4)Y - 10i ==>

(1/4)Y = 400 - 10i ==> Y = 4(400 - 10i) ==> Y = 1,600 - 40i IS-curve

From IS = LM ==> 1,600 - 40i = 800 + 60i ==> 100i = 800 ==> i = 8 ==> Y = 1,280

If government spending is increased by G = 50, the IS-curve will shift to the right) by (IS) = 4*50 = 200. If
the Fed wants to keep the interest rate constant, money supply has to be increased in a way that shifts the
LM-curve to the right by exactly the same amount as the IS-curve, that is, (LM) = 200.

From Y = 2(200 + 15i) ==> (Y) = 2(ms) ==> 200 = 2(ms)

==> (ms) = 100, so money supply has to be increased by 100.

Check: IS' = LM": 1,800 - 40i = 1,000 + 60i ==> 800 = 100i

==> i = 8 Y = 1,480

i
45 IS1

40
ISo LMo LM1

0
800 1000 1280 1480 1600 1800 Y

14. Assume the equation for the IS-curve is Y = 1,200 – 40i, and the equation for the LM-curve is
Y = 400 + 40i.
(a) Determine the equilibrium value of Y and i.
(b) If this is a simple model without income taxes, by how much will these values change if the
government increases its expenditures by G = 400, financed by an equal increase in lump sum taxes (TAo =
400)?

a. From IS = LM ==> 1,200 - 40i = 400 + 40i ==>

800 = 80i ==> i = 10 ==> Y = 400 + 40*10 ==> Y = 800

136
b. According to the balanced budget theorem, the IS-curve will shift horizontally by the increase in
government purchases, that is, IS = G = TAo = 400.

Thus the new IS-curve is of the form: Y = 1,600 - 40i.

From IS' = LM ==> 1,600 - 40i = 400 + 40i ==>

1,200 = 80i ==> i = 15 ==> Y = 400 + 40*15 ==> Y = 1,000

15. Assume you have the following information about a macro model:
Expenditure sector: Money sector:
S = - 200 + (1/5)YD ms = 400
TA = (1/8)Y - 40 md = (1/4)Y + 100 - 5i
TR = 60
I = 300 – 10i
G = 70
NX = 150 - (1/5)Y

Calculate the equilibrium values of investment (I), money demand (md), and net exports (NX).

C = YD - S = YD – [-200 + (1/5)Y] = 200 + (4/5)YD

Sp = C + I + G + NX = 200 + (4/5)[Y- (1/8)Y + 40 + 60] + 300 - 10i + 70 + 150 - (1/5)Y

= 720 + (4/5)(7/8)Y + (4/5)100 - 10i - (1/5)Y = 800 + (1/2)Y - 10i

Y = Sp ==> Y = 800 + (1/2)Y - 10i ==> (1/2)Y = 800 - 10i

Y = 2(800 - 10i) ==> Y = 1,600 - 20i IS-curve

ms = md ==> 400 = (1/4)Y + 100 - 5i ==> (1/4)Y = 300 + 5i

==> Y = 1,200 + 20i LM-curve

IS = LM ==> 1,600 - 20i = 1,200 + 20i ==> 40i = 400 ==> i = 10 Y = 1,400

==> I = 300 - 10*10 = 200 NX = 150 - (1/5)1,400 = - 130 md = ms = 300

16. Assume the following IS-LM model:


expenditure sector: money sector:
Sp = C + I + G + NX M = 500
C = 110 + (2/3)YD P =1
YD = Y - TA + TR md = (1/2)Y + 400 - 20i
TA = (1/4)Y + 20
TR = 80
I = 250 - 5i
G = 130

137
NX = -30
Calculate the equilibrium values of investment (I), real money demand (md), and tax
revenues (TA).
How much of investment (I) will be crowded out if the government increases spending
by G = 100?

a. Sp = C + I + G + NX = 110 + (2/3)(Y - TA + TR) + 250 - 5i + 130 - 30

= 460 + (2/3)[Y - (1/4)Y - 20 + 80] - 5i = 460 + (2/3)(3/4)Y + (2/3)60 - 5i = 500 + (1/2)Y - 5i

From Y = Sp ==> Y = 500 + (1/2)Y - 5i ==>Y = 2(500 - 5i) ==> Y = 1,000 - 10i IS-curve

From (M/P) = md ==> 500/1 = (1/2)Y + 400 - 20i ==> (1/2)Y = 100 + 20i ==>

Y = 200 + 40i LM-curve

From IS = LM ==> 1,000 - 10i = 200 + 40i ==> 800 = 50i ==> i = 16 ==> Y = 840

==> I = 250 - 5*16 = 170 TA = (1/4)840 - 20 = 190

Since (M/P) = md ==> md = 500

Check: md = (1/2)840 + 400 - 20*16 = 500

b. If government expenditures are increased by G = 100, then the IS-curve will shift by this change times
the multiplier, that is, IS = 2*100 = 200. Therefore, from

IS' = LM ==> 1,200 - 10i = 200 + 40i ==>1,000 = 50i ==> i = 20 ==> Y = 1,000

Since i = 4 ==> I = - 4*5 = - 20

Check: I' = 250 - 5*20 = 150

CHAPTER 11

MONETARY AND FISCAL POLICY

Solutions to the Problems in the Textbook:

Conceptual Problems:

1.a. An open market operation is an exchange of bonds for money or vice versa by the Fed. In an open market
purchase, the Fed buys bonds from the public (generally via government bond dealers) in exchange for money.
This action increases the monetary base and therefore the supply of money. In an open market sale, the Fed
sells bonds in exchange for money, decreasing the monetary base and therefore the supply of money.

138
1.b. When the Fed undertakes open market sales, it exchanges bonds for money. This decreases the monetary
base and the resulting decrease in money supply creates a portfolio disequilibrium. The public adjusts by
selling other assets, so asset prices decrease and yields (interest rates) increase. This increase in interest rates
has a negative effect on aggregate demand (investment spending) and output contracts. A lower level of
national income reduces money demand and therefore interest rates decline again. But if the price level is
assumed to be fixed (as in the IS-LM model), then interest rates still settle at a level higher than the original
one. Overall, in an IS-LM diagram, the LM-curve shifts to the left, leading to a higher level of interest rates
and a lower level of income.

2. The IS-curve is vertical, if investment spending is totally interest insensitive. This is called investment
insufficiency; in this case the monetary multiplier is zero. Since the parameter b in the investment equation
equals zero, the equation changes from

I = Io - bi to I = Io.

A horizontal LM-curve will also render monetary policy ineffective. This is called the liquidity trap. In this
case, money demand is totally interest elastic, and the parameter h in the money demand equation is assumed
to be infinitely large.
The fiscal policy multiplier is zero if the LM-curve is vertical. This case is called the classical case, and
money demand (and money supply) is assumed to be totally interest insensitive. Since the parameter h in the
money demand equation equals zero, the equation changes from

L = kY - hi to L = kY.

None of these three cases is very likely to occur. However, some economists assert that Japan in the late 1990’s
and the U.S. in the Great Depression were in, or close to, the liquidity trap.

3. A liquidity trap is a situation in which the public is willing to hold, at a given interest rate, however much
money the Fed is willing to supply. In this case, the LM-curve is horizontal and monetary policy is totally
ineffective. Fiscal policy (which will shift the IS-curve) is clearly the better choice to stimulate the economy in
such a situation, since no crowding out will occur. This means that fiscal policy will have its maximum effect.

4. Crowding out occurs when an increase in government spending raises interest rates, which reduces private
spending (especially investment). For example, an increase in government purchases (G) will increase income
(Y) and therefore consumption (C); but because the interest rate (i) will increase, the level of investment
spending (I), and most likely also net exports (NX), will decrease, changing the composition of GDP. Some
degree of crowding out will always occur as long as the LM-curve is upward sloping, that is, in all cases
except the liquidity trap. The steeper the LM-curve is, the greater the degree of crowding out. This implies that
if the LM-curve is steep monetary policy will be more effective than fiscal policy in stimulating national
income.

5. In the classical case, the LM-curve is vertical at the full-employment level of output. In this case, money
demand (and money supply) would be completely interest inelastic. After any type of disturbance, a return to
an equilibrium in the money sector could only be accomplished through changes in the level of output. In this
situation, fiscal policy would be completely ineffective, since it would be totally crowded out. On the other
hand, monetary policy would achieve its maximum effect.

139
Assume the government finances an increase in government spending by borrowing from the public (the
Treasury sells government bonds to finance the increase in the budget deficit). The increase in the demand for
credit by the government will lead to an increase in interest rates. If the Fed is worried about high interest
rates, it may monetize the budget deficit, that is, buy the government bonds that the public now holds. This will
inject money into the economy, and interest rates will drop again, so no crowding out of private spending may
occur, at least in the short run.
In an IS-LM model, the expansionary fiscal policy will shift the IS-curve to the right, while the Fed’s
action will shift the LM-curve to the right. This means that the AD-curve will shift further to the right than
would have been the case if the Fed had not accommodated the expansionary fiscal policy. But this causes
more upward pressure on the price level. In a recession, when there is little inflationary pressure, such a
fiscal/monetary policy mix may be beneficial and cause only a small increase in the price level. However, if
the economy is close to full employment, then we can expect a significant increase in the price level. In the
long run, when the AS-curve is vertical, there will be total crowding out, whether the Fed monetizes the
increase in the budget deficit or not.

7. A combination of restrictive fiscal policy and expansionary monetary policy will not significantly affect
aggregate demand or income, and neither will expansionary fiscal policy combined with restrictive monetary
policy. However, the first policy mix will decrease interest rates, while the latter will increase interest rates.
Therefore the composition of output will be different in each case.
The first combination will shift the IS-curve to the left and the LM-curve to the right, in which case
income will remain roughly the same while interest rates will be reduced. A tax increase will lower
consumption while increasing investment spending due to lower interest rates. The second combination will
shift the IS-curve to the right and the LM-curve to the left, leaving income roughly the same, while increasing
interest rates. This will decrease the level of investment spending, while either government spending or
consumption (through a tax cut) will increase.
Other considerations may involve the effect of a given policy mix on the budget surplus and the value of
the dollar (and therefore net exports). The first policy mix will increase the budget surplus. Lower interest rates
may also lead to an outflow of funds, which will lower the value of the dollar, leading to an increase in net
exports. The second policy mix will decrease the budget surplus. Higher interest rates may lead to an inflow of
funds, which will increase the value of the dollar, leading to a decrease in net exports.

Technical Problems:

1. If the government wants to change the composition of GDP towards investment and away from
consumption without changing the level of aggregate demand, it needs to implement a combination of
restrictive fiscal policy and expansionary monetary policy. An increase in personal income taxes or a decrease
in transfer payments will reduce consumption and thus aggregate demand. The IS-curve will shift to the left,
leading to a decrease in the level of output and the interest rate. To increase output to its original level, the Fed
can undertake expansionary monetary policy. This will shift the LM-curve to the right, leading to a further
decrease in the interest rate, thus stimulating investment, and, in turn, aggregate demand. If the intersection of
the new IS- and LM-curves is at the same income level as previously, then the decrease in the interest rate will
have stimulated investment spending sufficiently to exactly offset the decrease in consumption. (Note: The tax
increase can be combined with an investment subsidy. In this case, the IS-curve will not shift as far to the left
as before.)
The following diagram shows the effect of a decrease in transfer payments (TR) that is combined with an
increase in money supply (M/P). The adjustment process is as follows:

140
1-->2: TR ==> C  ==> Y == md  ==> i  ==> I  ==> Y . Effect: Y  and i .

2-->3: (M/P) up ==> i  ==> I  ==> Y  ==> md  ==> i  Effect: Y  and i .

Combined effect: Y about the same and i .

i IS1 LM1

IS2 LM2
1
i1
2
i2
3
i3

0
Y2 Y1 Y

A cut in the income tax rate will flatten the IS-curve and shift it to the right. Both the level of income and the
interest rate will increase. If the Fed increases money supply to keep the interest rate constant, then the LM-
curve will also shift to the right, maximizing the multiplier effect, since no crowding out will take place.
However, if money supply is held constant, then the LM-curve will not shift and the overall effect of this fiscal
expansion on income will be weakened, since the increase in the interest rate will crowd out investment.

i IS2
IS1 LM1
LM2
2
i2
1 3
i1

0
Y1 Y2 Y3 Y

The adjustment process is as follows:

1-->2: t  ==> C  ==> Y  == md  ==> i up ==> I  ==> Y down. Effect: Y  and i .

2-->3: (M/P)  ==> i  ==> I  ==> Y  ==> md  ==> i  Effect: Y  and i .

Combined effect: Y  and i about the same.

141
Either a removal of an investment subsidy or an increase in the income tax rate will shift the IS-curve to the
left. This will lead to a decrease in the level of income and the interest rate. A rise in the income tax rate will
reduce consumption, but investment will increase due to the decrease in the interest rate. Removing an
investment subsidy will reduce investment, but the effect will be partially offset by the decrease in the interest
rate. The decrease in income will lead to a decrease in consumption.

i IS1 i I1

IS2 LM I2

i1 i1

i2 i2

0 0
Y2 Y1 Y I2 I1 I

Note: An increase in the income tax rate will decrease the multiplier. The IS-curve will not only shift to the left
but will also become steeper. The removal of an investment subsidy, as shown in the textbook, will lead to a
parallel shift of the IS-curve to the left. Here, only a parallel shift is shown.

4. Monetary expansion will lead to lower interest rates, which will stimulate investment and thus output. The
LM-curve will shift to the right, and a new equilibrium will be reached at point E2 in Figure 11-8.
Fiscal expansion will lead to a higher level of output and higher interest rates. The IS-curve will shift to
the right and a new equilibrium can be reached at point E1 in Figure 11-8.
Fiscal expansion through an increase in government purchases will allow public spending as a share of
GDP to increase, while private spending (especially investment) as a share of GDP will decline. A reduction in
income taxes will increase the level of consumption, while decreasing the level of investment because of
higher interest rates. Monetary expansion will increase the level of investment spending (due to lower interest
rates) and consumption (due to higher income).
A more balanced growth can be achieved through an investment subsidy. This will shift the IS-curve to
the right and a new equilibrium will be reached at E1. But even though interest rates will rise, the impact of the
investment subsidy will not be totally lost. Here we have an example in which both consumption, induced by
higher income, and investment, induced by the subsidy, will rise.
Full employment can also be achieved through a combination of expansionary monetary and
expansionary fiscal policy. This will shift both the LM- and IS-curves to the right and we will end up
somewhere between points E1 and E2 in Figure 11-8.

Additional Problems:

1. True or false? Why?


"Fiscal policy is more effective when the interest sensitivity of money demand is lower."

False. If money demand is totally independent of interest rates, the LM-curve is vertical. This is the classical

142
case. A change in government spending has no effect on output, since there is complete crowding out. Clearly,
in the case of a normal (upward-sloping) LM-curve, less crowding out will occur and income will go up as
government spending increases. But the more interest insensitive money demand is, that is, the steeper the LM-
curve is, the smaller the increase in income will be, due to a larger crowding out effect.

2. Comment on the following statement:


"Crowding out is complete when money demand is perfectly interest inelastic."

Crowding out refers to the fact that an increase in public spending may lead to a decrease in private spending,
thus dampening the output expansion. An increase in government spending raises interest rates, which leads to
a reduction in investment spending. When money demand is perfectly interest inelastic, the LM-curve is
vertical at the level of real output that clears the money market. An increase in government spending will
stimulate income and encourage people to hold more money balances. The excess demand for money will
cause interest rates to rise to the level at which equilibrium in the money market is restored. If money demand
is perfectly interest inelastic, the rise in interest rates will not lower the quantity of money demanded. Instead,
income will have to go back to its original level before the money market is back in equilibrium. This means
that interest rates will have to increase until the level of investment spending has been reduced by the same
amount as government spending has been increased. Therefore the level of output demanded is unchanged and
crowding out is complete.

3. True or false? Why?


"Expansionary monetary policies reduce bond prices in the liquidity trap."

False. Expansionary monetary policies generally reduce interest rates and thus increase bond prices. In the
liquidity trap, however, interest rates do not change, since the LM-curve is horizontal. If the Fed increases the
money supply through an open market purchase, the public is willing to hold all the money the Fed supplies at
the prevailing interest rate. Nobody wants to shift into bonds and thus bond prices do not change.

4. True or false? Explain your answer.


"Expansionary fiscal policy is more effective when it is financed by borrowing from the public than when
it is monetized."

False. If the government finances an increase in its spending by selling bonds to the public, no change in the
supply of money will occur, and the IS-curve will shift without a corresponding shift in the LM-curve. On the
other hand, if an increase in the budget deficit is monetized, then money supply will increase, as the central
bank buys government bonds from the public. Therefore, the LM-curve will also shift to the right, leading to
lower interest rates than in the first case. Less crowding out will occur and the overall effect on income will be
greater--at least in the IS-LM model, that is, the short run. In the long run, when prices are allowed to be
flexible, then crowding out cannot be avoided by monetizing the debt, since an increase in the price level will
lead to lower real money balances and therefore higher interest rates.

5. True or false? Why?


"Crowding out is complete in the liquidity trap."

False. In the liquidity trap the public is prepared to hold whatever money is supplied at any given interest rate.
This implies that the LM-curve is horizontal. If government spending rises, the IS-curve will shift to the right.

143
Income will rise but interest rates will not increase. This means that there will be no crowding out.

i IS1 IS2

i1 LM

0
Y1 Y2 Y

6. Assume the government wants to increase output without changing interest rates. What kind of policy mix
would you recommend and how would your policy mix affect the composition of GDP? Explain your answer
and the adjustment processes that take place with the help of an IS-LM diagram.

A combination of expansionary fiscal and monetary policy will increase output without affecting interest rates.
Expansionary fiscal policy will shift the IS-curve to the right and income and interest rates will both increase.
Expansionary monetary policy will shift the LM-curve to the right and interest rates will decrease while
income increases. Thus we will have an increase in income without a change in interest rates.
Since income will increase, consumption will also increase, but since interest rates will not change,
induced investment will not be affected (Note: In a more advanced model, an increase in sales expectations
may actually increase the overall investment level. See Chapter 14). Therefore the level of investment as a
fraction of GDP will decrease, while consumption and government purchases will have a greater share. (A
more balanced growth can be achieved if investment subsidies are given.)

1-->2: G up ==> Y up ==> money demand up ==> i up ==> I down ==> Y up

Effect: Y up and i up

2-->3: (M/P) up ==> i down ==> I up ==> Y up ==> money demand up ==> i up.

Effect: i down and Y up Overall effect: Y up and i constant.

i IS2 LM1
IS1 LM2
2
i2

i1 1 3

0
Y1 Y2 Y3 Y

144
7. Discuss the effect of an investment subsidy on consumption. In your answer, indicate whether the effect
on consumption would differ if money demand were more interest sensitive.

An investment subsidy will stimulate investment spending and therefore income, which will lead to an increase
in consumption. If money demand were more interest sensitive, then the LM-curve would be flatter and the
shift of the IS-curve to the right would have a larger effect on income (and thus consumption). As income
increases, so does money demand, but if money demand were more interest sensitive, then a smaller increase
in the interest rate would be required to bring the money sector back to equilibrium. Thus, less crowding out
would occur and the overall increase in income or consumption would be greater.

i IS1 IS2 LM1

i3 LM2
i2

i1

0
Y1 Y3 Y2 Y

8. "Monetary policy cannot change real output as long as investment is independent of interest rates."
Comment on this statement.

When investment spending is not affected by changes in the interest rate but is determined solely by changes in
business expectations, then monetary policy is ineffective. In this case, the transmission mechanism breaks
down and monetary policy will not bring about changes in real output. Expansionary monetary policy may
reduce interest rates, but this will not increase the level of investment spending and the economy will not be
stimulated. In the IS-LM framework, we would have a vertical IS-curve. Thus, when the LM-curve shifts, we
simply see a change in the interest rate, while the output level remains constant.

Assume investment is very interest inelastic and money demand is very interest elastic. With the help of an
IS-LM diagram, explain the effect of a cut in the income tax rate (t) on investment (I), money demand (md),
and the budget surplus (BuS) and briefly explain the adjustment process.

Investment is interest inelastic so the IS-curve is steep; money demand is interest elastic so the LM-curve is
flat. An income tax cut will shift the IS-curve to the right and make it flatter. Income and the interest rate will
increase. Since the LM-curve is flat, the interest rate will not increase by much, so investment will decrease
only a little. The budget deficit will increase due to the tax cut. Higher income will lead to more money
demand but a higher interest rate will lead to lower money demand. Overall, money demand will remain
constant, since money supply hasn't changed. The adjustment process can be described a follows:

t  C  Y md i I Y effect: Y and i

145
i

LM

i2
i1

IS1 IS2
0
Y1 Y2 Y

10. Assume money demand is very interest inelastic and investment is very interest elastic. Explain how the
level of savings (S), money demand (md) and investment (I) would be affected if the government increased
welfare spending.

If money demand is very interest inelastic, the LM-curve is very steep, and if investment is very interest
elastic, the IS-curve is very flat. With a steep LM-curve and a flat IS-curve, fiscal policy is not very effective,
since most of it is crowded out. An increase in government transfer payments (TR) shifts the IS-curve to the
right and income and the interest rate increase. Since income has increased, saving has increased and since the
interest rate has increased, investment has decreased. In the end, money demand cannot be affected since
money supply has not changed. The increase in income increases money demand, but the increase in the
interest rate brings it back to its original level, that is, in equilibrium with money supply. The adjustment
process that takes place is as follows:

TR  Y md  i  I Y  Effect: Y  i 

Since i increases a lot a lot, the effect on I is large, as is the offsetting effect (the crowding out effect) on output
(Y). This means that the overall effect on Y is small.

i LM

i1

io
IS1

ISo
0
Yo Y1 Y

11. Use the formal IS-LM model derived in Chapter 10 to show algebraically how the degree of crowding out
that is associated with an increase in government spending (G) is determined by the different parameters in the
fiscal policy multiplier (b, k, h and ).

The equilibrium interest rate for the IS-LM model was derived in Equation (9) of Chapter 10 as

146
i = (k/h)Ao - [1/(h + kb)](M/P).

If we substitute this equation into the equation for investment, we get

I = Io - bi = Io - b{(k/h)Ao - [1/(h + kb)](M/P)},

and therefore we get

I = - (bk/h)(Ao) = - (bk/h){/[1 + k(b/h)]}(Ao) = - [(bk)/(h + kb)](Go).

From this we can see the following:

If the interest sensitivity of investment (b) goes up, then we have more crowding out.
If the interest sensitivity of money demand (h) goes up, then we have less crowding out.
If the income elasticity of money demand (k) goes up, then we have more crowding out.
If the multiplier () goes up, then we have more crowding out.
Note: Since b, k, and  are in both the numerator and the denominator of the factor preceding (G) in the
equation above, some students may have difficulty deciding whether this factor goes up or down as these
parameters increase. An easy way to find out is to calculate the inverse of this factor, which is

(h + kb)/(kb) = h/(kb) + 1.

As either k, , or b increases, then this inverse decreases and the factor will increase.

12. If the government increased the income tax rate (t) and the Fed responded by increasing the money
supply, how would investment (I), savings (S) and money demand (md) be affected?

1.->2. A higher income tax rate decreases the expenditure multiplier. The IS-curve becomes steeper and shift to
the left, so both income and the interest rate increase.
2.->3. An increase in money supply shifts the LM-curve to the right. The interest rate decreases, leading to an
increase in investment and thus income.
Overall, the interest rate will decrease, but it is unclear what will happen to income. A lower interest rate
means an increase in investment. Since income has not changed much, savings hasn't been affected much,
although it will change in the same direction as income. Since the tax rate is lower, most likely savings will
increase. Money demand has increased, since money supply has been increased (the money sector has to be in
equilibrium).
The adjustment processes that take place can be described as follows:

1.->2. t  C  Y md  i I  Y  effect: Y  i 

2.->3. ms  i I  Y md  i  effect: Y  i 

Overall effect: Y ? i 

i IS1 LM1 LM2


IS2

147
1
i1
2
i2

i3 3

0
Y1 Yo Y

13. "Combining income tax cuts with restrictive monetary policy is counterproductive, since it will lead to a
higher budget deficit and higher interest rates. What we need instead is a tax increase in combination with
expansionary monetary policy, since the tax increase will lower the budget deficit while the money expansion
stimulates the economy." Comment on this statement.

Neither policy mix described above is likely to significantly affect the level of output. A combination of
expansionary fiscal policy and restrictive monetary policy will lead to an increase in interest rates but it will
not significantly affect output. The IS-curve will become flatter and shift to the right, and the LM-curve will
shift to the left. The budget deficit will increase due to the tax cut.
Restrictive fiscal policy that is combined with expansionary monetary policy will also not significantly
affect output, but it will reduce interest rates. (The IS-curve will become steeper and shift to the left, and the
LM-curve will shift to the right.) The tax increase will lead to a decrease in consumption, but the decrease in
interest rates will lead to an increase in investment and a higher potential for future economic growth. The
budget deficit will decrease due to the higher tax rates. Lower interest rates will also help to finance the
existing national debt and may stimulate net exports, since a capital outflow may occur that will reduce the
value of the dollar.

14. “If investment is very interest inelastic, then most of an income tax rate cut will be crowded out; therefore
the Fed should always supplement a tax cut with an increase in money supply.” Comment on this statement
with the help of an IS-LM diagram and explain the adjustment process.

If investment is very interest inelastic, then the IS-curve is very steep. An income tax cut will shift the IS-curve
to the right and make it flatter. Therefore income and the interest rate increase. The increase in the interest rate
will crowd out only a small part of investment, since investment is very inelastic. If the Fed increases money
supply, the LM-curve will shift to the right and income will increase, while interest rates will go down.
Overall, we have an increase in income, but interest rates will be largely unaffected. Therefore, we do not have
to worry about the crowding out of investment. The adjustment processes that take place can be described as
follows:

1.->2. t  C  Y md i I Y effect: Y and i

2.->3. ms i I  Y md i  effect: Y and i

Overall Effect: Y and i unchanged


i
LM1

i2 2 LM2
1

148
i1 3

IS1 IS2
0

Y1 Y2 Y3 Y

15. "A cut in the income tax rate is not an effective way to stimulate the economy if money demand is very
interest elastic, since most of the tax cut will be crowded out." Comment on this statement.

A situation in which money demand is extremely interest elastic comes very close to the so-called liquidity trap
(the LM-curve will be almost horizontal). A decrease in the income tax rate (t) will stimulate consumption and
increase national income. The size of the expenditure multiplier () will increase and the IS-curve will become
flatter and shift to the right. The increase in income will initially induce people to hold more money balances
and thus provide upward pressure on interest rates. But since money demand is extremely interest sensitive, it
will take only a very small increase in interest rates to bring the money sector back to equilibrium. Therefore,
the crowding out effect on investment will be minimal and the tax cut will prove to be very effective in
stimulating national income.

16. "Expansionary monetary policy becomes more effective as the interest sensitivity of investment
increases." Comment on this statement.

One of the ways monetary policy affects the level of output demanded is by changing interest rates and thereby
the level of investment spending. The adjustment can be described as follows: An increase in money supply
lowers the interest rate. If investment is very interest elastic, a large increase in investment spending will
follow. This means that, given a certain size of the expenditure multiplier, income will change by more than in
the case when investment does not respond much to a change in interest rates. In other words, if investment is
very interest sensitive, then we have a flat IS-curve. For the same change in money supply, the flatter the IS-
curve is, the larger the change in real output will be.
The formal analysis of Chapter 10 shows that, if investment becomes more interest sensitive, then the
value of b increases. This leads to an increase in the monetary policy multiplier, which is defined as

(Y)/(M/P) = (b/h).

Note: b is also in the denominator of the equation for , and therefore an increase in b will lower the value of .
But the change in  is proportionally less than the change in b and thus the value of the monetary policy
multiplier will actually increase as b gets larger.

17. Assume that the marginal propensity to save increases. If the Fed wants to keep the level of output from
fluctuating, should it undertake open market purchases or sales? In your answer discuss the combined effect of
these changes on the composition of GDP.

An increase in the marginal propensity to save (s = 1 - c) will decrease the size of the expenditure multiplier
() and therefore the IS-curve will shift to the left and become steeper. If people save more and spend less,
then firms will experience an increase in unintended inventories. Firms will respond to this by decreasing
production and national income will decrease. Therefore, the Fed will have to stimulate the economy by
increasing the supply of money via open market purchases (which will shift the LM-curve to the right). As a

149
result of these two shifts we will have lower interest rates. This means that investment as a fraction of GDP
will increase, while consumption's share of GDP will decrease. (Lower interest rates may also cause an outflow
of capital, which will lower the value of the domestic currency, leading to an increase in net exports.)

18. "In 1991, the transmission mechanism broke down, since banks were still suffering from having made bad
real estate loans and were unwilling to increase their lending in response to the Fed's expansionary monetary
policy." Comment on this statement.

It is true that many banks had made bad loans in the late 1980s and were therefore extremely cautious in their
lending in 1991. They preferred to buy virtually risk-free government bonds. Thus, even though the Fed's
money expansion led to lower interest rates, private firms had little access to bank loans and the economy was
not significantly stimulated. But it would be an exaggeration to say that the transmission mechanism had
broken down, since bank lending finally picked up in 1992 after the Fed increased its expansionary monetary
policy effort. One can argue that, given the economic situation at the time, the Fed's initial policy measure
simply was not sufficient to achieve the desired result.

19. "If investment is very interest elastic and money demand is very interest inelastic, then fiscal policy is less
effective than monetary policy." Comment on this statement.

The more interest elastic investment is, the flatter the IS-curve will be. Expansionary fiscal policy (a shift of
the IS-curve to the right) becomes less effective, since the crowding-out effect becomes larger. Expansionary
monetary policy (a shift of the LM-curve to the right) becomes more effective, since the decrease in the
interest rate will now stimulate investment to a larger degree.
The more interest inelastic money demand is, the steeper the LM-curve will be, since any increase in
money demand due to an increase in income now has to be offset by a larger increase in the interest rate.
Expansionary fiscal policy becomes less effective, since any increase in income will increase money demand
and this will have to be offset by a larger increase in the interest rate, leading to a larger crowding-out effect.
Expansionary monetary policy becomes more effective, since the increase in money demand needed to bring
the money sector back into equilibrium must be achieved primarily through an increase in income.
The formal analysis in Chapter 10 introduces the fiscal and monetary policy multipliers as

(Y)/(G) =  = /[1 + k(b/h)] and (Y)/(M/P) = (b/h)

respectively. Therefore, if investment becomes more interest elastic, the value of b increases and the value of
the fiscal policy multiplier decreases, while the value of the monetary policy multiplier increases. But if money
demand becomes less interest elastic, then the value of h and the fiscal policy multiplier become smaller, while
the monetary policy multiplier becomes larger.

CHAPTER 12

INTERNATIONAL LINKAGES

Solutions to the Problems in the Textbook:

150
Conceptual Problems:

1. A country with a balance-of-payments deficit has made more payments of currency to foreigners than it
receives and the country’s central bank generally provides the needed funds. If the central bank refuses to do
so, the country will experience a decrease in money supply, which will eventually lead to a recession. As
domestic income goes down, less will be spent on imports. Since the domestic price level is likely to decrease
in a recession, people from abroad will begin to demand more of the country’s goods and exports will increase,
ultimately leading to a new equilibrium in the external balance.

2.a. A decrease in exports leads to a decrease in income and to a trade deficit. This situation cannot be
remedied with standard demand-side stabilization policy. The most appropriate policy response would be
protectionist measures (tariffs) or export subsidies combined with expansionary fiscal policy.

2.b. A decrease in private domestic saving and a corresponding increase in the level of domestic consumption
will lead to an increase in national income. If only more domestic goods are demanded, then no trade deficit
will occur. But if consumers also demand more foreign goods, then a trade deficit will occur. In either case, a
cut in government spending is the most appropriate policy response.

2.c. An increase in government spending will increase national income and lead to a trade deficit. A subsequent
cut in spending is the most appropriate policy response.

2.d. A shift from imports to domestic goods will increase national income and lead to a trade surplus. This
situation cannot be remedied with standard demand-side stabilization policy. A cut in government spending
combined with lower tariffs is the most appropriate policy response.

2.e. A reduction in imports combined with a corresponding increase in saving will lead to a trade surplus but no
change in national income. A cut in income taxes combined with lower tariffs is the most appropriate policy
response.

3. Expansionary monetary policy lowers the interest rate, which stimulates the level of investment spending
and therefore aggregate demand. If there is perfect capital mobility, lower interest rates will also lead to a
capital outflow, which will result in a depreciation of the currency. Export goods will be cheaper for foreigners,
so more of them will be demanded. The increase in net exports will lead to an even higher level of aggregate
demand and therefore national income.

4.a. An increase in the dollar/pound exchange rate means that now more U.S. dollars are required to buy one
British pound. Therefore the U.S. dollar has depreciated in value.

4.b. A depreciation in the value of the U.S. dollar is equivalent to an appreciation of the British pound.

5. Under a system of fixed exchange rates, a devaluation of the currency takes place when the price at which
foreign currencies can be bought is increased by official government action. Under a system of floating
exchange rates, currency depreciation takes place when there is an excess supply of the domestic currency on
foreign exchange markets, therefore making it less expensive to buy domestic currency in terms of foreign
currencies.

151
6. The purchasing power parity theory suggests that exchange rate movements mainly reflect the
differentials in national inflation rates. This theory holds up reasonably well in the long run, especially when
inflation rates are high and caused by changes in monetary growth. However, in the short run, even a monetary
disturbance will affect foreign competitiveness, since exchange rates move faster than the price level.
Similarly, when a real disturbance occurs, the purchasing power parity relationship will no longer hold, since
the adjustment will affect the terms of trade. Examples of such real disturbances are changes in technology,
shifts in export demand, or shifts in potential output in different countries.

7. Economists work with simplified models that are designed to represent the real world as much as
possible. Any assumptions made should approximate reality. Economists do care whether the purchasing
power parity relationship holds, since many models in international economics assume that it does, and it
seems intuitively correct.

8. A country is in external balance when the balance of payments is neither in surplus nor in deficit. Under
these circumstances, central bank reserves remain stable. A country is in internal balance when output is at the
full-employment level. Optimally, a country wants to be in external and internal balance simultaneously, but
often there is a conflict between these two policy goals. In such a case, policy makers have to decide which
goal has priority at the time.

9. In the Mundell-Fleming model with fixed exchange rates and perfect capital mobility, fiscal policy is more
effective than monetary policy. As a matter of fact, monetary policy is totally ineffective in this case. Monetary
policy affects interest rates, which causes a flow of capital that affects exchange rates. In order to keep
exchange rates fixed, the monetary policy has to be reversed immediately.

10.a.Expansionary monetary policy lowers domestic interest rates. This not only stimulates the level of
domestic investment spending but also results in an outflow of funds that will result in a depreciation of the
currency. This makes domestic goods more competitive on world markets and therefore stimulates aggregate
demand even more by increasing net exports.

10.b. The reaction of other countries depends very much on the performance of their economies (their
position in the business cycle) at the time. Policy makers in a country that is in a recession will be unhappy
since the beggar-thy-neighbor policy will lead to more unemployment. Policy makers in a country that is in a
boom may actually welcome a appreciation of their currency. The reason is that the increase in their public’s
demand for imported goods may reduce some of the inflationary pressure that they may experience in a boom.

10.c. The policy described in this problem is always a beggar-thy-neighbor policy. However, as the answer to
10.b. points out, depending on their situations, other countries may not mind if they experience inflationary
pressure.

Technical Problems:

1. An increase in government purchases (G) will increase the level of output (Y) and interest rates (i). This
will cause an inflow of capital resulting in a higher value of the country's currency. The currency appreciation
will lead to a loss in competitiveness, and net exports (NX) will be crowded out to the point where the demand

152
for domestic goods is reduced to the original level. In the end, the level of output and the interest rate will
remain unchanged. In an IS-LM model, an increase in G will shift the IS-curve to the right, increasing both the
levels of output (Y) and the interest rate (i). But the decrease in NX will shift the IS-curve back to the left, until
interest rates are back to the world level.

2. A country that is faced with both a recession and a current account deficit should employ expansionary
monetary policy. Expansion in money supply will lower domestic interest rates. This will not only stimulate
domestic investment and output, but also result in a depreciation of the currency. A lower currency value will
increase competitiveness and lead to an increase in the demand for the country's exports, further stimulating
output. In other words, under perfect capital mobility and flexible exchange rates, monetary expansion will
lead to output expansion, currency depreciation, and a trade balance improvement.
At the root of Finland's problem was the collapse of the Soviet Union (an important trade partner) and the fall
in prices of pulp and paper (important export items). Therefore, expansionary monetary policy probably would
not have been sufficient to remedy the situation. Expansionary fiscal policy (preferably investment subsidies
that are designed to help exporting firms develop new markets) in combination with some protectionist
measures would have been a better policy option.

3. If the U.S. interest rate is i = 4% and you expect the British pound to depreciate by 6%, then the yield on
British government securities would have to be if =10% or more to make the purchase of British government
securities with U.S. dollars profitable. When capital is perfectly mobile, the domestic interest rate (i) is equal to
the foreign interest rate (if) adjusted for the expected percentage change in the exchange rate (e), that is,

i = if + (%e) ==> if = i - (%e) = 4% - (-6%) = 10%.

4. Expansionary fiscal policy (assume an increase in government purchases) will shift the IS-curve to the
right, leading to an increase in the level of output and the interest rate. With perfect capital mobility there will
be an inflow of capital that will result in a currency appreciation. Under a fixed exchange rate system, the
central bank will have to respond by increasing the domestic money supply to avoid currency appreciation.
This will shift the LM-curve to the right until the domestic interest rate is again in line with world interest
rates. In this case there is no crowding out and the fiscal policy will have the full multiplier effect. (Note: The
IS-LM framework is a short-run model in which the prices are assumed to be fixed. But in the longer run,
prices can change and then the analysis is somewhat more complicated.)

i IS1 IS2 LM1

LM2
i2

if

0
Y1 Y2 Y3 Y

153
5. Expansionary fiscal policy will have its maximum effect under a fixed exchange rate system with perfect
capital mobility. This is because fiscal expansion must always be combined with monetary expansion to bring
domestic interest rates back in line with foreign interest rates. Expansionary fiscal policy will increase the level
of output demanded and the interest rate. But with perfect capital mobility, the higher domestic interest rates
will attract funds from abroad, which will put upward pressure on the value of the domestic currency. To avoid
currency appreciation, the central bank will have to increase money supply to bring interest rates back in line
with world levels. Therefore, no crowding out will take place and the level of output will increase by the full
multiplier effect.

6.a. In a simple model of the expenditure sector that includes international trade, any change in autonomous
domestic spending by Ao will have the following effect on national income and imports (disregarding any
repercussion effects):

Y = [1/(1 - c + m)](Ao) and Q = [m/(1 - c + m)](Ao).

This can be derived as follows:

Assume that the balance of trade is defined as: NX = X - Q = Xo - Qo - mY.

From Sp = C + I + G + NX ==> Sp = Co + cY + Io - bio + Go + Xo - Qo - mY = Ao - bio + (c - m)Y

with Ao = Co + Io + Go + Xo - Qo, (and assuming that TA = TR = 0, for simplicity)

From Y = Sp ==> Y = Ao - bio + (c - m)Y ==>

(1 - c + m)Y = Ao - bio ==> Y = [1/(1 - c + m)](Ao - bio)

Therefore Y = [1/(1 - c + m)](Ao)

Since Q = Qo - mY ==> Q = m(Y) = [m/(1 - c + m)](Ao)

6.b. The increase in the level of output (Y*) in foreign countries due to an increase in their exports (X*) (our
imports) is

Q = Xo* = m*(Y) and therefore

Y* = [1/(s* + m*)](Xo*) = [m*/(s* + m*)](Y) with s* = 1 - c*.

6.c. From Y = Sp ==> Y = Ao + Xo + (c - m)Y ==> Y = [1/(s + m)](Ao + Xo) with s = 1 - c ==>

Y = [1/(s + m)][(Ao) + (Xo)] ==> Y* = [m/(s* + m*)][1/(s + m)](Ao)

since Xo = (m*)(Y*) and Y* = [m*/(s* + m*](Y)

6.d. Xo = (m*)(Y*) = [m*/(s* + m*)][m/(s + m)](Ao)

6.e. Since Y = [1/1 - c + m)][(Ao) + (Xo)] and 1 - c = s ==>

154
Y = [1/(s + m)]{(Ao) + [m*/(s* + m*)][m/(s + m)(Ao)]} ==>

Y = [1/(s + m)]{1 + [(m*)/(s* + m*)][m/(s + m)]}(Ao).

The multiplier with repercussion is larger than the multiplier without repercussion since part of the import
leakage is recovered through increased exports as a consequence of foreign expansion.

6.f. The change in the trade balance without the repercussion effect is

NX = - m(Y) = [(- m)/(s + m)](Ao).

With the repercussion effect, however, the change in the trade balance is

NX = Xo - m(Y).

But Xo = m*(Y*) = [m*/s* + m*)][m/(s + m)](Ao) (as derived in 6.d.), and

Y = [1/(s + m)]{1 + [m*/(s* + m*)][m/(s + m)]}(Ao) (as derived in 6.e.). Hence

NX = (s - m)/(s + m)(Ao) with  = [m*/(s* + m*)][m/(s + m)]


which is clearly less negative than [(- m)/(s + m)](Ao).

Because of the induced increase in foreign income, our exports increase, offsetting our increase in imports.
Therefore the adverse effect on the trade balance is dampened by the repercussion effect.
Additional Problems:

1. "Fiscal policy cannot change real output under fixed exchange rates and perfect capital mobility."
Comment on this statement.

Expansionary fiscal policy will shift the IS-curve to the right, leading to an increase in the level of output
demanded and a higher interest rate. This will cause an inflow of funds that will result in a currency
appreciation. To maintain a fixed exchange rate, the central bank will have to respond by increasing the money
supply, shifting the LM-curve to the right. Therefore, the level of output demanded will increase even further.
The central bank will continue to increase money supply until the domestic interest rate is again in line with
world interest rates. Thus there will be no crowding out and the level of output will increase by the full
multiplier effect. Therefore the statement is false.

2. "Under a fixed exchange rate system and perfect capital mobility an increase in foreign interest rates will
cause the level of domestic output to rise." Comment on this statement.

If foreign interest rates rise, a capital outflow will occur, leading to a depreciation of the currency. To maintain
a fixed exchange rate, the central bank will be forced to buy domestic currency by selling its holdings of
foreign currency. This reduction in money supply will lead to higher domestic interest rates and thus less
output demanded. This will cause a recession and the level of domestic output will fall, not rise.

True or false? Why?


“If the U.S. dollar price of a Canadian dollar is 0.80, then the Canadian dollar price of a U.S. dollar is 1.20?

155
False. The exchange rate e = 0.80 is defined as the domestic currency price of a unit of foreign currency. Thus
the foreign price of a domestic currency unit is 1/e = 1/0.8 = 1.25.

4. "In the early 1980s, U.S. fiscal policy was very expansionary; this helped to foster economic growth in
the rest of the world." Comment on this statement.

The expansionary fiscal policy of the U.S. government, financed by increased borrowing, led to high U.S.
interest rates, which attracted a capital inflow from abroad. The increased demand for the U.S. dollar caused
the dollar to appreciate and U.S. goods became relatively more expensive, leading to a deterioration in the U.S.
trade balance. As U.S. citizens bought more import goods, foreign economic growth was stimulated. The
increase in the national income of foreign countries eventually caused an increase in the foreign demand for
U.S. goods, a good example of the repercussion effect.

5. "A decrease in foreign interest rates will cause domestic output to fall under perfect capital mobility and
flexible exchange rates." Comment on this statement.

If interest rates fall abroad, there will be an inflow of capital here. This will cause an appreciation of the
domestic currency. The price of domestic goods will become relatively more expensive on world markets,
resulting in a decrease in net exports. But a decrease in net exports implies a decrease in aggregate demand for
domestic goods, and domestic income and interest rates will go down. (The IS-curve will shift to the left until a
new equilibrium at the new and lower world interest rate is reached.)

6. "The U.S. experience of the early 1980s clearly demonstrated that huge budget deficits do not necessarily
crowd out investment." Comment on this statement.

The huge U.S. budget deficits were financed to a large extent by borrowing from abroad, that is, high U.S.
interest rates attracted foreign funds. This led to an appreciation of the U.S. dollar and U.S. goods became
relatively more expensive on world markets. Large U.S. trade deficits developed as exports declined and
imports increased. Therefore, the increase in the budget deficit did not crowd out investment but instead
crowded out net exports.

7. The re-unification of Germany required a large increase in the German budget deficit since it became
necessary to restructure businesses, provide income support for the unemployed, and invest in the
infrastructure in the new Eastern states. Explain the effects of such massive fiscal expansion for Germany and
its trade partners in the European Union.

The large increase in government spending (which was not financed by an equivalent tax increase or
accommodated by sufficient monetary expansion) resulted in a substantial increase in the German budget
deficit. Since the increase in demand from eastern Germany fell mostly on western German goods, the Western
economy overheated. Interest rates rose sharply, since the Bundesbank did not accommodate the expansionary
fiscal policy. This caused a capital inflow that put upward pressure on the value of the D-mark versus the
currencies of non-European trade partners, and a current account deficit resulted.
Initially Germany's European trade partners opted to essentially maintain the exchange rate of their
currencies versus the D-mark. Hence they had to allow the interest rates in their countries to increase to match
those in Germany to avoid a massive outflow of funds. However, since they did not experience the same fiscal
expansion as Germany, these economies' growth rates slowed sharply. Eventually, some countries had to allow

156
their currencies to depreciate.

8. "By running large budget deficits, the government will attract funds from abroad, which can then be used
to stimulate private investment. This action would be unwise, however, since foreign countries respond
unkindly to such a beggar-thy-neighbor policy." Comment on this statement.

Expansionary fiscal policy will increase income and interest rates. Higher domestic interest rates will crowd
out investment but will also cause an inflow of funds, resulting in a higher value of the domestic currency. This
will lead to a loss in competitiveness and the crowding out of net exports, while less investment is crowded
out. Other countries are likely to have a mixed reaction to this policy since, on the one hand, they will
experience an outflow of funds, but, on the other hand, their economies will be stimulated by the increased
demand for their goods. This is definitely not a beggar-thy-neighbor policy. A beggar-thy-neighbor policy is
when a country tries to reduce domestic unemployment by depreciating its currency. This can be achieved by
using expansionary monetary policy, which will lower interest rates, resulting in a capital outflow, lowering the
value of the domestic currency. This will increase the demand for domestic goods and lower the demand for
foreign goods. As a result, domestic unemployment will decrease while foreign unemployment will increase

9. "A depreciation-induced change in the trade balance does not work well when different countries'
economic cycles are highly synchronized." Comment on this statement.

Exchange depreciation creates domestic employment at the expense of other countries (this is why it is called a
beggar-thy-neighbor policy). It shifts demand from one country to another but does not change the level of
world demand. When many countries experience a simultaneous economic downturn, exchange rate
movements do not significantly increase world demand, even though they may affect the allocation of demand
among countries. While an individual country may feel compelled to raise domestic output by attracting
demand from other countries, a better way to increase demand in each country would be to coordinate fiscal
and monetary policy.

10. "Under a fixed exchange rate system, expansionary monetary policy depletes foreign reserves at the
central bank." Comment on this statement.

Expansionary monetary policy shifts the LM-curve to the right. The domestic currency will begin to
depreciate. Under a fixed exchange rate system, however, the central bank cannot allow that to happen and will
have to trade foreign currencies for domestic currency, thereby reducing the supply of money. This will shift
the LM-curve back to the left, and the foreign reserve holdings of the central bank will fall.

i LM1
IS
LM2

i1

i2

0 Y1 Y2 Y

157
11. Explain how a recession in the United States can affect the economies of other industrial nations.

If the U.S. economy enters a recession, private spending in the U.S. will decline, lowering the demand for
imports. U.S. prices are likely to rise less than those of other countries, which will stimulate U.S. exports,
while discouraging imports. As a result, a recession in the U.S. may contribute to a downturn in other
economies around the world.

12. Explain why a cut in government spending has a larger effect under a fixed exchange rate system and
perfect capital mobility than in a closed economy model.

In a closed economy model, restrictive fiscal policy shifts the IS-curve to the left, causing a decrease in the
level of output demanded and the interest rate. The decrease in the interest rate will increase the level of
investment spending. Therefore the level of output demanded will go down by less than the full amount of the
shift in the IS-curve. In the graph below, we move from 1 to 2.

i IS2 IS1 LM2

LM1

i2 3 1

i2 2

0
Y3 Y2 Y1 Y

In an open economy it gets more complicated, since the lower interest rate will cause an outflow of funds. This
will lead to downward pressure on the value of the domestic currency. If the central bank wants to maintain the
exchange rate, it has to decrease money supply. This will shift the LM-curve to the left and the level of output
demanded will decrease even further. With perfect capital mobility, the central bank will have to decrease
money supply until the domestic interest rate is again at the level of the world interest rate. Therefore, in the
end, the level of investment spending will not be affected and we will have the full multiplier effect. In the
graph above, we will move from 1 to 3.

13. How does a protectionist measure, such as the levying of tariffs on foreign goods, affect the trade balance,
the exchange rate, and the level of domestic output if there is perfect capital mobility?

If a tariff is imposed, the relative price of imported goods will increase and, as a consequence, the demand for
domestic goods will increase. Domestic income and interest rates will increase, leading to capital inflows and
an appreciation of the currency. This will, in turn, lower the relative price of imported goods again. With
perfect capital mobility the currency appreciation will eventually progress to the point where the overall
change in net exports will be zero. Therefore the level of domestic output and the interest rate will also be
unaffected.

158
14. Explain how restrictive fiscal policy affects the level and composition of output under flexible exchange
rates and perfect capital mobility.

Restrictive fiscal policy under perfect capital mobility and flexible exchange rates will cause a depreciation of
the domestic currency that will induce a dollar for dollar increase in net exports such that the level of output
demanded will remain unchanged. When government spending is reduced, the IS-curve will shift to the left
and the domestic interest rate will decline below the level of the world interest rate. A capital outflow will
occur, leading to a depreciation of the domestic currency. Therefore, net exports will increase since the relative
price of domestic goods will now be lower. The decrease in net exports will shift the IS-curve back to its
original location. Therefore, the level of output will not change, although its composition will.

i IS1 LM

IS2

i1

i2

0
Y2 Y1 Y

15. With the help of an IS-LM diagram show the effect of restrictive monetary policy on output under flexible
exchange rates and with perfect capital mobility.

A decrease in money supply shifts the LM-curve to the left, so interest rates rise while the level of output
demanded decreases. The higher interest rates cause an inflow of capital, which causes the currency to
appreciate. This leads to a decline in exports and an increase in imports, since the relative price of domestic
goods on world markets has increased. The decline in net exports causes the IS-curve to shift to the left. A new
equilibrium will be established at the original interest rate (the world interest rate) but at a lower level of
domestic output.

i IS2 IS1 LM2

2 LM1
i2
3
i1 1

0
Y3 Y2 Y1 Y

16. "In a flexible exchange rate system with perfect capital mobility, expansionary fiscal policy will always

159
crowd out net exports." Comment on this statement.

Under perfect capital mobility, the BP-curve is horizontal and domestic interest rates are equal to those in the
rest of the world. Expansionary fiscal policy will shift the IS-curve to the right, increasing the level of output
and interest rates. When domestic interest rates are above those in the rest of the world, there will be a capital
inflow that will cause the currency to appreciate. This will lead to a fall in exports and an increase in imports,
since the relative price of domestic goods will increase. This reduction in net exports will shift the IS-curve
back to its original position. In the end, the expansionary fiscal policy will be fully crowded out by a decrease
in net exports.

i
IS1 IS2 LM

i2

i1 BP

0
Y1 Y2 Y

CHAPTER 13

CONSUMPTION AND SAVING

Solutions to the Problems in the Textbook:

Conceptual Problems:

1.a. According to the life-cycle theory of consumption, people try to maintain a fairly stable consumption path
over their lifetime. Individuals save during their working years so they can keep up the same consumption
stream after they retire. This implies that wealth increases steadily until retirement while consumption remains
stable. We should therefore expect the ratio of consumption to accumulated saving (wealth) to decrease over
time up to retirement.

1.b. After retirement, wealth is used up to finance consumption during the remaining years. Therefore the ratio
of consumption to accumulated saving (wealth) increases again after retirement, eventually approaching 1.

2.a. Suppose that you and your neighbor both work the same number of years until retirement and you both
have the same annual income. If your neighbor is in bad health and does not expect to live as long as you do,
she will expect to have fewer retirement years in which to use accumulated wealth to finance a steady
consumption stream. Your neighbor's goal for retirement saving will not be as high as yours, and compared to

160
you, she will have a higher level of consumption over her working years.
Since planned annual consumption (C) is determined by the number of working years (WL), the number of
years to live (NL), and income from labor (YL), we get the equation:

C = [(WL)/(NL)](YL).

WL and YL are the same for you and your neighbor, but NL is smaller for your neighbor. Therefore you will
have a lower level of consumption (C).
(Note: Students may come up with a variety of different answers. For one, your neighbor, who is in bad health,
currently has much larger medical bills than you do. Therefore she may not be able to save as much for
retirement, even if she might expect to live as long as you. On the other hand, she may not have large medical
bills now, but expects them later, as she gets older. This may induce her to save more now. While such
arguments are valid, instructors should point out that the answer should be related to the life-cycle theory.)

2.b. If we assume for simplicity that the rate of return on Social Security is the same as the rate of return on
private saving, then the introduction of a Social Security system based on a trust fund should not have any
effect on your level of consumption. Social Security may be considered a form of "forced saving," since you
are forced to pay Social Security taxes during your working years and will, in return, receive benefits during
your retirement years. However, most likely you would have voluntarily saved as much as the government is
now “forcing” you to save with levying a Social Security tax. Therefore your consumption behavior will not
change. Still, the levying of a Social Security tax reduces disposable income during your working years,
increasing the ratio of consumption to disposable income (the average propensity to consume). If private
saving were simply replaced with government saving, national saving would not be affected.
In reality, however, the Social Security system is not strictly financed through a trust fund, but largely on a
pay-as-you-go basis. The size of the Social Security trust fund was fairly insignificant until the system was
amended in 1983. Now the trust fund is increasing and, in effect, contributing to the federal budget surplus.
But because of our aging population, predictions are that the Social Security system will experience severe
financial difficulties within the next 20-30 years. If the credibility of the system becomes an issue, people may
intensify their saving efforts, since they no longer feel they can rely on the public system to provide for them
during retirement. In the past, most of the Social Security taxes were not "saved" but immediately used by the
government to finance the benefits of the current retirees. This is why most economists claim that the Social
Security system has led to a decrease in the national savings rate and a decrease in the rate of capital
accumulation. The magnitude of this decrease, however, has not been clearly established.

3.a. If you get a yearly Christmas bonus, you immediately treat it as part of your permanent income and spend
it accordingly, that is, C = c(Y). In other words, your current consumption will change significantly.

3.b. If you get a Christmas bonus for only this year, you will consider it as transitory income. Since your
permanent income is hardly affected, you will consume only a small fraction of it and save the rest. In other
words, your current consumption will not be significantly affected.

4. Gamblers (or thieves) seldom have a very stable income. However, their consumption is determined by
their permanent income, that is, their expected average lifetime income. Whether they have a large or small
income during any given period, their consumption pattern remains relatively stable, since their permanent
income is not significantly affected by temporary changes in earnings.

5. Both theories, in their own way, try to explain why the short-run mpc is smaller than the long-run mpc.

161
The life-cycle theory attributes the difference to the fact that people prefer a smooth consumption stream over
their lifetime. Therefore the average expected lifetime income is the true determinant of current consumption.
The permanent income theory suggests that the difference is due to measurement errors. Measured income has
two components, that is, permanent and transitory income. But only permanent income is a true determinant of
current consumption.

6.a. One possible explanation could be that the “baby boomers” were still in their dissaving phase. In other
words, if households of the baby boom generation still had to buy houses or pay for expenses related to
childcare in their late twenties, they may not have been able to save for retirement yet.

6.b. If the above explanation is correct, one can expect an increase in saving as these “baby boomers” age,
become more financially solvent, and begin to prepare for retirement.

7. The ranking from highest to lowest value should be first (a), then (d), and then (b). Clearly, (c) should be
lower than (a), but where exactly it ranks after that depends largely on the severeness of the liquidity
constraint.

8. A series follows a random walk when future changes cannot be predicted from past behavior. In other
words, it does not have a mean or clear long-run value. Any major change comes about because of random
shocks. Hall asserted that changes in current consumption largely come from unanticipated changes in income.
According to the life-cycle theory or permanent-income theory, people try to smoothen out their consumption
stream in such a way that its expected value is always the same in each period. Therefore, we can express
future consumption as the expected value plus some error term, that is, some random value that is
unpredictable. This error term is a shock to future income that is spread over the remaining lifetime. Hall
supported the permanent-income hypothesis by showing that lagged consumption is the most significant
determinant of future consumption.

9. The problem of excess sensitivity means that consumption responds more strongly to predictable changes
in current income than the life-cycle theory and permanent-income theories predict. The problem of excess
smoothness means that consumption does not respond as strongly to unpredictable changes in current income
as these theories predict. However, the existence of these problems does not invalidate the theories. It simply
means that the theories can explain consumption behavior only to a certain degree.

10. Precautionary (or buffer stock) saving can be explained by uncertainty. It could be uncertainty in regard to
one’s life expectancy or one’s time of retirement (affecting the accumulated saving needed to finance
retirement), or uncertainty about future spending needs (which may be caused by a change in family
composition or health). Clearly, if we account for such uncertainties, we bring the model much closer to reality.
For example, many elderly still continue to save after retirement in anticipation of predicted high medical costs
not covered by Medicare.

11.a.It is unclear whether an increase in the interest rate leads to an increase or a decrease in saving. On the
one hand, as the interest rate increases, the return on saving increases and people may therefore increase their
savings effort (due to the substitution effect). On the other hand, a higher return on saving implies that a given
future savings goal can now be reached with a smaller savings effort in each year (due to the income effect).

162
11.b.The income effect and the substitution effect generally tend to go in different directions, and the overall
outcome depends on the relative magnitude of these two effects. Until now, empirical evidence has not
established a significant sensitivity of saving to changes in the interest rate. This would imply that the income
and the substitution effects have about the same magnitude.

12.a.According to the Barro-Ricardo hypothesis, it does not matter whether an increase in government
spending is financed by taxation or by issuing debt.

12.b. The Barro-Ricardo hypothesis states that people realize that government debt financing by issuing
bonds simply postpones taxation. In other words, people know that the government will have to raise taxes in
the future to pay back what they have borrowed now. Therefore, expansionary fiscal policy that results in an
increase in the budget deficit will no stimulate the economy since it will lead to an increase in saving rather
than consumption. People want to be prepared to pay future taxes.

12.c.There are two main objections to the Barro-Ricardo hypothesis. One is based on liquidity constraints, that
is, people may want to consume more but may not be able to borrow as much as they like. Therefore, if there is
a tax cut, they will consume more, rather than save the tax cut. The other argument is that those people who
benefit from a tax cut or an increase in government spending are not the same as those who will have to pay
the higher taxes to pay off the debt. This argument assumes that people are not concerned about the welfare of
their descendants.

Technical Problems:

1.a. If income remains constant over time, permanent income equals current income. Your permanent income
this year is YP0 = (1/5)(5*20,000) = 20,000.

1.b. Your permanent income next year is YP1 = (1/5)(15,000 + 4*20,000) = 19,000.

1.c. Since C = 0.9YP, your consumption this year is C0 = 0.9*20,000 = 18,000.

Your consumption next year is C1 = 0.9*19,000 = 17,100.

1.d. In the short run, the mpc = (0.9)(1/5) = 0.18; but in the long run, the mpc = 0.9.

1.e. We have already calculated this and next year's permanent income. In each of the coming years you add
$30,000 and subtract $20,000, and therefore your permanent income (which is your average over a five year
period) will increase by $2,000 each year until it reaches $30,000 after 5 years.

YPo = (1/5)(5*20,000) = 20,000


YP1 = (1/5)(1*30,000 + 4*20,000) = 22,000
YP2 = (1/5)(2*30,000 + 3*20,000) = 24,000
YP3 = (1/5)(3*30,000 + 2*20,000) = 26,000
YP4 = (1/5)(4*30,000 + 1*20,000) = 28,000
YP5 = (1/5)(5*30,000) = 30,000

Y
30,000

163
28,000
26,000
24,000
22,000
20,000

0 1 2 3 4 5 time

2.a. The person lives for NL = 4 periods and earns a lifetime income of

YL = 30 + 60 + 90 + 0 = 180.

Therefore consumption in each period will be Ci = (1/4)180 = 45, i = 1, 2, 3, 4.

This implies that saving in each period is:

S1 = 30 - 45 = - 15; S2 = 60 - 45 = + 15; S3 = 90 - 45 = + 45; S4 = 0 - 45 = - 45.

2.b. If liquidity constraints exist and the person cannot borrow in the first period, then she will consume all of
her income, that is, Y1 = C1 = 30.
For the remaining three periods the person wants a stable consumption stream. Thus she will consume C(i) =
(1/3)(60 + 90 + 0) = 50 in each of the remaining three periods i = 2, 3, 4.

2.c. An increase in wealth of only $13 is not enough to offset the difference in consumption patterns between
period 1 and the other periods. Therefore all of the increase in wealth will be consumed in period 1, such that
C1 = 43. In the remaining three periods, consumption will be the same as in 2.b.
An increase in wealth of $23 will be enough to offset the difference in consumption patterns. Lifetime
consumption in each period will now be Ci = (1/4)(180 + 23) = 50.75. This means that 20.75 (or almost all of
the additional wealth) will be used up in the first period; the remaining 2.25 will be distributed over the next
three years.

3.a. According to the life-cycle theory and permanent income hypothesis (LC-PIH), the change in
consumption equals the surprise element, that is, CLC-PIH = . According to the traditional theory, the
change in consumption equals Ctr = c(YD). Therefore if a fraction  of the population behaves according to
the traditional theory and the other fraction behaves according to LC-PIH, then the total change in
consumption is

C = (Ctr) + (1 - )(CLC-PIH) = c(YD)+ (1 - )c = (.7)(.8)10 + (.3) = 5.6 + (.3)

3.b. C = (.3)(.8)10 + (.7) = 2.4 + (.7)

3.c. C = (0)(.8)10 + 1 = 

4.a. If the real interest rate increases, the opportunity cost of consuming should increase. Therefore, the

164
average propensity to save, that is, the fraction of total income that is saved, should increase.

4.b. If you only save for retirement and your savings goal is fixed, then you actually will save less. With a
higher interest rate it will take less saving each year to achieve your goal.

4.c. The first case (4.a.) describes the substitution effect, whereas the second case (4.b.) describes the income
effect. Unless the magnitude of each of these effects is known, we cannot predict the overall effect of this
interest rate increase on saving.

5. One way to increase saving would be to either privatize or eliminate the Social Security system, so people
would have to save for retirement on their own. (Eliminating Social Security is not a very popular measure, but
the privatization of Social Security is often discussed.) This would do away with the negative effect on saving
that comes from the pay-as-you-go nature of financing Social Security. Another way might be to make it more
difficult to borrow. The U.S. tax system encourages people (and firms) to borrow rather than save.

Additional Problems:

1. As a share of GDP, how large is consumption compared to the other three main components. Would you
expect consumption's share to increase or decrease in a recession?

Consumption expenditures are roughly two thirds of total GDP, which is higher than the other three
components (investment, government purchases, and net exports) taken together. The ratio of consumption to
GDP, however, does not always remain constant. In a recession, for example, when income is below trend, we
should expect the consumption-to-GDP ratio to increase, while in a boom, when income is above trend, we
should expect the ratio to decrease. The reason is that current consumption is based on permanent rather than
current income and when current income is greater than permanent income, the ratio of consumption to income
(the apc) goes down. This argument is reinforced by the concept of automatic stability. When GDP falls,
personal disposable income falls by less and thus consumption does not fall dramatically.

2. True or false? Why?


"The marginal propensity to consume out of transitory income is greater than the marginal propensity to
consume out of permanent income."

False. The permanent-income hypothesis argues that consumption is related to permanent disposable income.
Individuals will only revise their consumption behavior significantly if they perceive a change in income as
permanent. Very often people are uncertain as to whether a rise in income is permanent or transitory, so they do
not significantly revise their consumption patterns immediately. This suggests a lower marginal propensity to
consume out of transitory income than out of permanent income.

3. Do you think that the marginal propensity to consume out of current income would differ between tenured
professors who have a high degree of job security and professional gamblers who never know when luck will
strike?

Tenured professors have a high degree of job security and their income does not vary a great deal. They can
therefore relatively accurately estimate their permanent income. This means that their current consumption is
largely based on current income, implying that their short-run mpc is fairly high. Gamblers, on the other hand,

165
never know what their income in any given year is going to be. Therefore, they base their consumption
decisions on their average expected lifetime income (permanent income) rather than on current income. This
implies that their short-run mpc is fairly low.

4. Is the short-run marginal propensity to save different between farmers and government employees? Why
or why not?

Government employees generally have very stable incomes and high job security. Therefore they base their
consumption decision to a large extent on current income so their short-run mpc is high, while their short-run
mps is low. Farmers, on the other hand, have highly variable incomes, depending on weather conditions.
Therefore they tend to base their consumption decisions on their permanent income. Their short-run mpc is
low, while their short-run mps is high.

5. "If most people base their consumption decisions on their current rather than their permanent income,
then the short-run multiplier is greater than the long-run multiplier." Comment on this statement.

If most people follow the traditional theory and base their consumption decisions mostly on current income,
then their mpc out of current income is high, making the value of the short-run multiplier high. But if most
people follow the permanent-income theory and base their consumption decisions primarily on permanent
income, then the short-run mpc is low, making the value of the short-run multiplier low. In either case, as long
as some people follow the permanent-income theory, then the short-run multiplier should always be smaller
than the long-run multiplier.

6. Assume you define your permanent income as the average of this and the past four years’ incomes and
you always consume 4/5 of your permanent income. Your earnings record over these years has been: Yt =
40,000, Yt-1 = 38,000, Yt-2 = 34,000, Yt-3 = 32,000, Yt-4 = 31,000.
If next year your income increases to Yt+1 = 46,000, by how much will your consumption change between
year t and year t+1?

YPt = (1/5)(40,000 + 38,000 + 34,000 + 32,000 + 31,000) = (1/5)175,000 = 35,000

Ct = (4/5)YPt = (4/5)35,000 = 28,000

YPt+1 = (1/5)(46,000 + 40,000 + 38,000 + 34,000 + 32,000) = (1/5)190,000 = 38,000

Ct+1 = (4/5)YPt+1 = (4/5)38,000 = 30,400

Therefore your consumption will change by C = 2,400.

7. Assume a distant aunt gives you several thousand dollars and you use the money to pay back part of your
student loan. Does your behavior correspond to the prediction of the permanent-income theory? Why or why
not?

Paying back your debts actually can be seen as an act of "saving." Therefore, since you use some unexpected
income to save (rather than consume), your behavior fits the permanent income theory nicely.

166
8. "Early retirement raises aggregate consumption." Comment on this statement.

Early retirement reduces lifetime income and increases the length of retirement. The life-cycle model states
that individuals consume on the basis of their average lifetime income to maintain a stable consumption path
throughout their lives. In an economy with a constant population and no technological progress, aggregate
consumption will fall if retirement age drops because people who retire earlier have to accumulate funds for
more retirement years over fewer working years. As this can only be accomplished with greater saving,
consumption has to be reduced.
However, if the population is growing and retirement benefits are financed through taxes levied on workers
currently employed, then aggregate consumption may actually rise. In this case, the working population will be
paying for the reduction in lifetime earnings experienced by those who have retired early, and there is less need
for retirement saving.

9. The simple life-cycle hypothesis predicts that people save over their working years but dissave during
their retirement years. Do we actually observe such behavior? If not, can you explain why not?

Most elderly actually do not dissave, but they do save less than they did during their working years. One of the
reasons that the elderly still save may be the fact that they anticipate large medical bills as they grow older and
therefore prefer to keep a certain buffer stock of saving. The elderly may also hope to leave some of their
savings as bequests to their children or grandchildren.

10. On October 19, 1987, the Dow Jones industrial average dropped about 500 points, or a little more than
23%. What effect should a decline in stock values of this magnitude have had on aggregate demand according
to the life-cycle theory of consumption?

According to the life-cycle theory, any change in wealth should affect consumption behavior. The decline in
stock values constituted roughly a $500 billion decline in wealth. However, we did not see a huge decrease in
consumption in 1987, since the wealth effect tends to be fairly small. In addition, the Fed reacted promptly,
announcing that liquidity would be provided if needed.

11. Does the random walk model of consumption disprove the permanent income hypothesis? Why or why
not?

Robert Hall tried to disprove the permanent income theory by applying the concept of rational expectations to
the theory of consumption. He asserted that consumption patterns may follow a random walk, that is, changes
in consumption may come from unanticipated changes in income. However, by concluding that lagged
consumption is the most significant determinant of future consumption, Hall actually supported the predictions
of the permanent-income hypothesis.

12. How is Hall’s random walk model of consumption related to the permanent-income hypothesis and what
are the implications of these theories for fiscal policy?

Hall asserted that changes in current consumption largely come from unanticipated changes in income. Any
major change in consumption comes about because of random shocks. According to the permanent-income
theory, people try to smoothen out their consumption stream in such a way that its expected value is always the

167
same in each period. Therefore, we can express future consumption as the expected value plus some error term,
that is, some random value that is unpredictable. This error term is a shock to future income that is spread over
the remaining lifetime. Hall supported the permanent-income hypothesis by showing that lagged consumption
is the most significant determinant of future consumption. The implication for fiscal policy is that a temporary
tax change will not significantly affect current consumption, unless there are liquidity constraints.

13. True or false? Why?


"A temporary tax surcharge never has a significant effect on current consumption."

False. If individuals know that the tax surcharge is temporary they will not alter their spending patterns as the
tax change has little impact on their permanent income. However, when liquidity constraints exist, individuals
may be forced to adjust their consumption behavior immediately. If individuals barely earn enough to finance
their current consumption, for example, they may be forced to cut their current consumption if a temporary tax
surcharge is levied.

14. "As a response to a temporary increase in personal and corporate income taxes consumers will reduce
their spending and firms will cut production and increase prices. Therefore all we will get is stagflation, that is,
an increase in both unemployment and inflation, and tax revenues won't increase." Comment on this statement.

The life-cycle/permanent-income theory of consumption predicts that temporary changes in income will not
significantly affect the level of consumption. Thus a temporary tax surcharge should not significantly affect
aggregate demand. A similar argument can be made about firms, since changes in production are often costly
and therefore a temporary surcharge on corporate income taxes should not affect the level of output and prices.
The levels of national income and prices should not be affected significantly but we should see a (temporary)
increase in tax revenues due to the surcharge. (Note, however, that if consumers and firms face liquidity
constraints, they may react to a temporary surcharge in the way described in the statement.)

15. "Any tax cut that results in an increase in the budget deficit will fail to stimulate aggregate demand."
Comment on this statement. In your answer explain the effect of such a tax cut on interest rates, money supply,
and private domestic saving.

The Barro-Ricardo proposition states that a tax cut that results in a budget deficit increase leads to higher
saving. Since people will anticipate a future tax increase to finance the higher deficit, permanent income will
not be affected. Thus consumption will not be affected; instead people will save the tax cut. Since this is purely
a fiscal policy measure, money supply is not affected. The increase in the budget deficit will lead to higher
interest rates due to the increased demand for credit. (Note that evidence from the 1980s does not support this
hypothesis. The Reagan tax cuts in 1981 resulted in a large increase in the budget deficit but there was no
subsequent increase in saving.)

16. Assume the government announces plans for fiscal expansion that are likely to result in increased
government borrowing. What effect should this have on aggregate consumption, money supply, the income
velocity of money, the trade deficit, and savings?

The Barro-Ricardo proposition states that if fiscal expansion results in a budget deficit, the public will
anticipate a future tax increase to finance the deficit. They believe that their permanent income will not be
affected and choose to save rather than consume more. Therefore, we should expect an increase in private

168
saving but no significant change in consumption. Thus there is no significant change in national income and,
since this is solely a fiscal policy, money supply is also not affected. Therefore there is no change in the
income velocity. The trade deficit may also not be significantly affected, since domestic saving supports the
budget deficit. However, evidence from the 1980s does not lend support for this hypothesis. Saving did not
increase after the Reagan tax cuts that resulted in a huge increase in the budget deficit. Instead, we saw an
increase in consumption and the trade deficit, since higher interest rates caused an inflow of funds, leading to
an appreciation of the U.S. dollar. The income velocity also increased, due to the increase in economic activity.

CHAPTER 14

INVESTMENT SPENDING

Solutions to the Problems in the Textbook:

Conceptual Problems:

1. Even if the economy has achieved the desired capital stock some (gross) investment still must take place
to keep the capital stock at this level. The level of investment has to be sufficient to cover depreciation (due to
wear and tear or because capital becomes obsolete).

2. High-tech capital (such as computers) becomes obsolete at a very fast rate and therefore needs to be
replaced much earlier if firms want to stay competitive. Therefore the rate of depreciation will increase if more
is invested in high-tech machines.
Human capital also depreciates since knowledge tends to become outdated (new theories are advanced
and new discoveries are made continuously). Thus knowledge needs to be updated. Who for example, wants to
be treated by a physician who hasn't kept up with new advances in medical technology? Similarly, since one
can also think of health as human capital, we can see that, as we grow older, our stock of health tends to
depreciate. But the more we invest in health, that is, the healthier we live and the more preventive measures we
take, the slower this stock of human capital will depreciate.

3. The interest rate cannot simply be considered as the rental cost of capital but is also an opportunity cost.
Retained earnings can be used to invest in new machinery but also to make a loan to someone else. In other
words, at any time retained earnings can be "financially invested," that is, given to someone in need of funds
(the government, for example), in which case these funds would earn interest. For example, if the yield on a
government bond or a commercial paper is much higher than the expected rate of return on an investment in
real capital, a firm may not want to undertake this investment and "invest" in a government bond.

4. The price of a share of stock in a company should, in an efficient stock market, be equal to the price of a
claim on the capital in the company. Tobin’s q is an estimate of the value the stock market places on a firm’s
assets relative to the cost of producing those assets. In other words, it can be thought of as the ratio of the
market value of a firm to the replacement cost of capital. The replacement cost of capital is a measure for the
marginal cost of capital. If q is greater than 1, then a firm should add physical capital, since for each dollar’s
worth of new machinery the firm can sell stock for q > 1 dollars. But this means that the marginal product of
capital exceeds its marginal costs.

169
5. A sudden increase in the demand for a firm's product will increase expectations of future sales and induce
a firm to increase its desired capital stock. This will require an increase in net investment. The speed with
which the capital stock is increased to its new desired level depends on whether the firm believes that the
increase in sales is permanent or temporary and on the cost to the firm of adjusting the capital stock to its new
desired level quickly.

6. Large firms have easier access to credit than small firms, since large firms tend to have an established and
good credit rating. Therefore, small firms are often limited in their investment opportunities by their retained
earnings. But if we have many more small firms than large firms, we may observe larger output fluctuations
over the business cycle. This is due to the fact that small firms will invest less in a recession than they
otherwise might have because credit is not available to them and profits are down. On the other hand, in a
boom these small firms are likely to invest more than they would otherwise, since profits are high and they
want to compensate for the lack of investment during the past downturn.

7.a. When profits are high, more internal funds are available for firms. Firms generally will use these funds for
financing new capital investments, even at times when outside funding is not easily obtainable. Higher profits
may come from increased sales. This may make entrepreneurs more optimistic about future sales and
encourage them to increase investment spending. Higher profits also mean higher dividends and thus higher
stock values. But if stock values are high, firms are more inclined to raise new funds for additional investment.
It should be noted, however, that higher profits and the availability of internal funds should not distract from
the fact that the cost of capital still is an important factor in investment decisions. Any interest that could be
earned on such funds has to be seen as an opportunity cost.

7.b. Credit rationing by banks occurs since banks realize that while a cautious entrepreneur may be deterred
from investing when interest rates are high, a more reckless entrepreneur may still invest in spite of high rates.
Entrepreneurs who are more reckless are also more likely to fail and default on their loans. Therefore in times
of tight funds, rationing credit may be a more effective way for banks to ensure profits than increasing the
interest rate on loans. Credit rationing may also occur for other reasons. For example, the Federal Reserve can
impose credit limits on financial institutions.

8.a. Mortgage interest payments are an important consideration in buying a house. Even a small change in
mortgage interest rates can significantly affect homebuyers' monthly mortgage payments. Therefore most
people will wait to buy a home until interest rates are fairly low. The purchase of a home is different from the
purchase of a consumption good, since most people can delay purchasing a home for some time if market
conditions are unfavorable. If interest rates are very low, many more prospective homebuyers will qualify for a
mortgage. On the supply side, housing developers with large financing needs are more likely to undertake new
construction if the cost of credit declines. But as the supply of new housing increases, the price for houses may
drop, inducing more people to buy.

8.b. A state usury law prohibits banks or thrifts from charging mortgage rates above a certain maximum. This
in effect provides a price ceiling on mortgage rates. But when market interest rates go above this interest rate
ceiling, the mortgage market cannot adjust to an equilibrium and there will be excess demand for mortgage
funds. At such high interest rates banks may channel their funds away from mortgages and into other, higher
yielding ventures. Thus the supply of mortgage credit may decrease. If the inflation rate is high, the real
interest rate that homebuyers actually pay may be quite low. Therefore homebuyers may demand more

170
mortgage funds, creating excess demand for mortgages while there is no opportunity for the market to clear
due to the price ceiling. This explains why a low level of housing investment may exist even at low real
interest rates.

9. The flexible accelerator model tries to explain the speed at which firms adjust their capital stock over
time. The larger the gap between the desired and actual capital stock is, the higher the level of the firm’s
investment spending on machinery. Much, but not all, inventory investment can be explained with the help of
this accelerator model. The level of inventory investment is based on changes in output and therefore sales
expectations.

10. Unanticipated inventory accumulation occurs when the demand for a product is lower than was expected.
Firms generally respond by decreasing their level of production and a recession may be imminent. However, a
planned inventory increase generally occurs when firms expect an economic upturn and want to be ready for
the anticipated increase in the demand for their product.

11. The inventory-to-sales ratio did not increase in the 1990-91 recession, probably because of new and better
methods of management and firms keeping much tighter control over their inventories. Advanced computer
technology and the synchronization of shipments of material allow firms to operate with leaner, less costly
inventories. Therefore the inventory-to-sales ratio is not only less likely to increase due to an undesired
inventory increase at the beginning of a recession, but it is also less likely to increase due to a desired
inventory increase at the beginning of a new boom. It appears that the role of inventory spending in a business
cycle may have changed.

12. The level of net investment is the addition to the capital stock, that is, IN = K. A low level of net
investment implies slow growth in the capital stock and hence future productive capacity. A low rate of capital
accumulation generally implies lower future living standards.

In the long run (when the AS-curve is vertical), monetary policy will not affect the real interest rate. However,
it will affect inflation and therefore the nominal interest rate. The nominal interest rate (in) is determined by
the real interest rate (ir) plus the inflation rate (), that is,

in = ir + 

But if the nominal interest rate increases, the nominal cost of borrowing funds for investments rises and
some borrowers may no longer qualify for loans because of perceived cash flow problems. Banks may actually
limit credit when nominal interest rates increase, because they feel that borrowers still interested in loans may
be high credit risks. This is particularly true for residential investment, since housing is very sensitive to real
and nominal interest rates. One reason is that in the U.S. tax system nominal interest payments are tax-
deductible, while nominal capital gains due to inflation remain untaxed. A higher rate of inflation may also
have a negative impact on stock market values due to increased uncertainty about the future. But a decrease in
stock values may make it more difficult for firms that want to raise funds for investment projects through
issuing new stocks. Expectations about inflation also may affect the timing of investments. As a result, the
level of investment may be affected by monetary policy despite the fact that real interest rates have not
changed.

171
Technical Problems:

1. The rental cost of capital (rc) is equal to the real interest rate (r) plus the rate of depreciation (d), with the
real interest rate defined as the nominal interest rate (i) minus the expected inflation rate (e):

rc = = r + d = i - e + d.

A car rental firm would want to know how fast its cars will depreciate (or what is needed to keep the stock
of cars at the original level) and what the costs are of having funds tied up in owning the cars. In order to make
a profit, a car rental firm would charge more per car than the interest it could get (or has to pay) on the funds
tied up plus the depreciation rate times the value of the car. Since the rental firm charges a nominal price and is
charged a nominal interest rate by a bank for any funds that are borrowed, the charge would be

P > (i + d)V, with V = the value of the car.

For example, if the current market interest rate is 10%, the rate of depreciation is 20%, and the value of
the car is $10,000, then the car rental company would charge at least

(0.1 + 0.2)(10,000) = (0.3)(10,000) = 3,000

per year or roughly $8.22 dollars per day. If we further assume that the car is rented only half of the time,
then the costs would go up to $16.44 per day. Since the $16.44 charge does not include any other costs to the
firm or any profits, we should expect the actual price for a car rental to be higher.

2.a. If the net present discounted value (NPV) of a project is positive, the project is profitable. Assuming that
Year 1 is the present year, the net present value of the project can be calculated in the following way:

NPV = R1 + R2/(1 + r) + R3/(1 + r)2

At an interest rate of r = 5% we get

NPV = - 200 + 100/(1.05) + 120/(1.1025) = - 200 + 95.24 + 108.84 = 4.08 > 0.

This means that the project is profitable and should be undertaken.

2.b. Using the same equation as in 2.a. but with an interest rate of r = 10%, we get
NPV = - 200 + 100/(1.1) + 120/(1.21) = - 200 + 90.91 + 99.17 = - 9.92 < 0.

This means that the project is unprofitable and should not be undertaken.

3.a. A temporary investment tax credit should have a positive effect on investment for the period to which it
applies. In the long run, however, we should not see a significant effect on investment arising from a temporary
investment tax credit. The desired capital stock depends mainly on a firm's estimate of future or permanent
output and a temporary investment tax credit should not affect the firm's desired long-run capital stock.

3.b. A temporary investment tax credit will increase the number of current projects, since firms will invest in
projects that they otherwise might have delayed. Firms may also initiate some marginal projects that were

172
unprofitable under previous conditions. Therefore the level of investment in the year that the tax credit is
imposed will increase. Since so many projects are undertaken during the current year, fewer projects will be
undertaken the following year, when the tax credit is no longer in effect. Thus the level of investment in the
second year will be lower than it would have been otherwise.

3.c. While a permanent tax credit does not induce firms to accelerate investment projects, it may encourage
them to undertake marginal investment projects that might not previously have been profitable. We should
therefore see an overall increase in the level of investment, both in the first year of the tax credit and beyond.
The effect of a permanent tax credit on investment in the long run will be greater than that of a temporary tax
credit.

4.a. The difference between output and final sales is a result of inventory adjustments. Before a recession,
actual sales may go down, while output (which is based on expected sales) is slow to respond. Therefore we
have an unanticipated inventory accumulation. This will cause firms to cut back their production, lowering
output even more. On the other hand, if firms expect an increase in sales they will raise production to increase
their inventories. This can increase output before sales pick up.

4.b. In 1981, GDP exceeded final sales, which meant that inventories increased. The initial inventory increase
may have been intentional as industries prepared for an upswing after the recession of 1980. However, later in
the year, as the economy headed into the recession of 1981/82, sales and GDP both decreased. Since GDP
exceeded sales, an undesired inventory accumulation occurred. While output declined sharply, firms cut
inventories. In the first quarter of 1982 there was a desired inventory decrease as sales exceeded output. In
mid-1982, sales and GDP were almost equal, but by the fourth quarter sales started to increase faster than GDP.
This meant that aggregate demand had picked up, a clear sign that the economy was recovering. This led to an
undesired inventory decrease.

4.c. In a period of slow and steady growth, firms anticipate an increase in sales and want to be prepared by
holding a large stock of inventory. However, since the growth is slow and steady, we should expect output to
exceed sales only slightly and both should grow at about the same rate. Therefore we may see a fairly constant
inventory-to-sales ratio.

output
$ sales

0 time

5.a. The total value of the outstanding shares is $25 million. Therefore Tobin’s q has the following value:

q = (value of all shares)/(replacement cost of capital) = 25/18 > 1.

173
But a value of q that is greater than 1 implies that the value of the marginal product of capital is greater
than its marginal cost. Therefore the firm should invest.

5.b. With a replacement cost of $25 million, q = 1. Therefore net investment should be zero. But there should
still be some gross investment to replace the capital stock that depreciates.
With a replacement cost of $28 million, q < 1. In this case, the firm should “disinvest,” that is, let its
capital stock depreciate but not replace it.

6.a. From K* = (Y)/rc ==> K* = [(0.3)(5)]/(0.12) = 12.5

==> the desired capital stock is valued at $12.5 trillion.

6.b. From K* = [(0.3)(6)]/(0.12) = 15

==> the desired capital stock is now valued at $15 trillion.

174
6.c. From I1 = (K* - K) ==> I1 = (0.4)(15 - 12.5) = (0.4)(2.5) = 1

==> in the first year, net investment will be I1 = $1 trillion, and the new capital stock will be

K1 = $13.5 trillion.

From I2 = (K* - K) ==> I2 = (0.4)(15 - 13.5) = 0.6

==> in the second year, net investment will be I2 =$0.6 trillion and the new capital stock will be

K2 = $14.1 trillion.

6.d. As indicated above, the answers in 6.c. refer to net investment, that is, an addition to the capital stock.
Gross investment would include replacement of worn out capital.
7. The q-theory of investment predicts that high stock values will induce corporate managers to invest more
in real capital. In an efficient stock market, the price of a share of stock in a company should be equal to the
price of a claim on the capital in the company. Tobin’s q can be thought of as the ratio of the market value of a
firm to the replacement cost of capital, since it is an estimate of the value the stock market places on a firm’s
assets relative to the cost of producing those assets. If the value of q is high, a firm wants to add physical
capital and the level of investment rises. But this means that in periods in which stock prices rise rapidly,
corporations will increase their investment spending.

Additional Problems:

1. Explain why the stock of existing housing would decline if no more new houses were built.

There will always be some depreciation, as existing houses burn or are torn down. If no new houses are built
(gross investment is zero), the housing stock will decline (net investment will be negative).

2. Low interest rates should encourage firms to invest. So why did the U.S. have a low level of investment
spending during the 1930s when interest rates were very low?

During the 1930s the economy was in a deep recession. Therefore, in spite of low interest rates, businesses
were reluctant to invest. The level of investment depends not only on the interest rate but also on changes in
income, that is, the sales expectations of businesses. In the 1930s, most businesses did not expect sales to
increase.

3. "Firms will undertake investments as long as the value of the marginal product of capital is below the
rental cost of capital." Comment on this statement.

If this were true, then the last unit of capital employed would cost more than the value of the increase in
output. Competitive firms minimize costs when the rental cost of capital is equal to the value of additional
output produced as one more unit of capital is added. Otherwise it would pay the firm to use more (if the value
of the marginal product of capital exceeded the rental rate) or less (if the value of the marginal product of
capital were less than the rental rate) capital in the production process.
4. "Investment is pro-cyclical in the flexible accelerator model." Comment on this statement.

Assuming away depreciation and the real time required to make adjustments to the capital stock, the desired
capital stock (K*) is proportional to real output, where a is the capital-output ratio, that is,

K*t = aYt .

If investment makes up part of the difference between the actual and the desired capital stock, then

It = [K*t - Kt-1] ==> It = a[Yt - Yt-1].

Therefore, we can see that if output is growing, the level of investment is positive. But if output is falling,
investment is negative. This means that investment in the accelerator model is pro-cyclical, that is, it follows
output. Note, however, that as long as output is increasing at an increasing rate, investment will increase. On
the other hand, if the increases in output become smaller, the level of investment will decline.

5. Why is the level of investment inversely related to the level of the interest rate? Can you think of any
consumer spending that may also depend on the interest rate? Explain your answer.

When calculating the present discounted value of an investment, the interest rate is in the denominator. Thus a
decrease in the interest rate increases the net present value of an investment. As more investment projects
become profitable, the level of investment rises. Also, the neoclassical theory predicts that the desired capital
stock increases as the interest rate, and therefore the rental cost of capital decreases, that is,

K* = Y/rc.

Durable consumption goods, such as cars and appliances, are often purchased on credit. Therefore, spending
on durable goods is influenced by interest rate changes, which affect the overall costs of purchasing these
goods. But, as we have seen in the previous chapter, one can interpret the purchase of a durable good as an
“investment.”

6. "Restrictive monetary policy has a negative effect on investment." Comment on this statement.

If inflationary expectations are held constant, monetary restriction will raise both real and nominal interest
rates, at least in the short run. Since the level of investment is inversely related to the real interest rate,
investment will fall. Business fixed investment and housing investment will fall as the returns generated by
projects have to compete with the much higher costs of funds. Inventory investment will also fall as firms find
it more expensive to warehouse raw materials and finished goods.
In the long run, restrictive monetary policy will not have any effect on real interest rates, but will lower
nominal interest rates due to a lower inflation rate. When nominal interest rates are lower, more people may
qualify for mortgage loans and therefore housing investment will increases. A lower inflation rate may also
have a positive impact on stock values. But higher stock values may enable more corporations to issue new
stocks to finance more investment projects. Thus monetary policy may affect investment even if it does not
affect the real interest rate.

7. Which of the following two policy measures would affect GDP more and why: a temporary investment
tax credit for businesses or a temporary income tax cut for individuals? Explain your answer.

A temporary investment tax credit should have a positive effect on investment for the period in which it
applies. Firms may decide to accelerate projects that they would have undertaken later. They also may initiate
some marginal projects that would otherwise not have been profitable. But in the long run, we should not see a
significant effect on investment from a temporary investment tax credit. On the other hand, if the government
temporarily decreases income taxes, consumption will not be significantly affected according to the permanent
income hypothesis (unless liquidity constraints exist). Therefore the economy will not be stimulated at all by a
temporary income tax cut.
8. Which would stimulate the economy more: a $50 billion tax cut financed by a $50 billion spending cut, or
a temporary tax credit of 15% on each productive investment undertaken next year? Explain your answer.

According to the balanced budget theorem, a decrease in taxes and government spending by $50 billion would
actually decrease national income. A temporary investment tax credit, which has a positive effect on the
economy, is therefore much more beneficial. A temporary tax credit would make marginal investment projects
profitable. Some firms would also undertake projects earlier to take advantage of the tax credit. Temporary
investment tax credits can be used successfully to stimulate economic activity.

9. “Firms are more likely to invest if current GDP increases.” Comment on this statement.

The level of desired capital stock generally depends on the level of expected future (or permanent) output. But
if the desired capital stock is above the actual capital stock, firms will invest. Entrepreneurs will invest more if
they feel optimistic about the future and expect their sales to increase. If they are pessimistic about the future
and have low sales expectations, they will invest relatively little. Current output affects capital demand only to
the extent that it affects expectations about future output.

10. For a given nominal interest rate, how does an increase in the expected rate of inflation affect the level of
investment?

The rental cost of capital is defined as rc = i - e + d. This means that an increase in the expected rate of
inflation (e) decreases the rental cost of capital (rc). But a decrease in the rental cost of capital increases the
level of investment, as more investment projects become profitable. A decrease in rc also increases the desired
capital stock K* = (Y)/rc and therefore encourages investment.

11. Comment on the following statement:


“The discounted cash flow analysis is inconsistent with the neoclassical theory of investment.”
The neoclassical theory of investment states that the level of desired capital stock increases with an increase in
expected output (sales) and a decrease in the rental cost of capital. In practice, firms base their investment
decisions on the discounted cash flow analysis. An investment project is evaluated by calculating its net
present discounted value. If a firm expects an increase in sales, the cash flow associated with any investment
project will be larger and the net present discounted value of the investment will rise. At the same time, if the
rental cost of capital decreases, then the net present discounted value of the project increases and the firm is
more likely to undertake the investment. The decision making of firms using this discounted cash flow analysis
is therefore consistent with the neoclassical theory of investment.

12. Assume your firm buys a new computer system at a cost of $12,600 that will save you $5,600 after one
year, $4,840 more after the second year and another $4,000 after the third year. Then the computer will become
outdated and no further saving will accrue. Is this a worthwhile investment if we assume that there is no
inflation and that the market rate of interest remains at i = 10% over these three years?

The net present discounted value of your investment is

NPV = - 12,600 + 6,600/(1+ 0.1) 1 + 4,840/(1+ 0.1)2 + 4,000/(1+ 0.1)3

= - 12,600 + 6,000 + 4,000 + 3,000 = + 400 > 0.

Since the net present discounted value of your investment is positive, it is profitable and you should undertake
this investment.

Evaluate the following project, assuming that the market interest rate remains at i = 10%.

Year 0 Year 1 Year 2 Year 3


costs $3,700 $605 $363 $0
revenues $0 $2,640 $2,420 $400

The investment should be undertaken, since the net present discounted value of this investment is positive. It
can be calculated as follows:

NPV = -3,700 + (2,640 – 605)/(1 + .1)1 + (2,420 - 363)/(1 + .1)2 + 400/(1 + .1)3

= - 3,700 + 1,850 + 1,700 + 300 = 150 > 0

Since NPV > 0, this investment is profitable and should be undertaken.

14. Would you undertake the following investment project? Why or why not?

Year 0 Year 1 Year 2 Year 3


costs $3,993 $4,840 $3,630 $0
revenues $0 $3,630 $4,840 $3,993

This problem does not list an interest rate so, under normal circumstances, we cannot solve the problem. But if
we simply add up the costs and the benefits, we can see that both sums are equal. Since the costs occur earlier
than the benefits, we can conclude that the project is unprofitable, since we could have invested the funds we

178
spent at any positive interest rate. For example, if the market interest rate was i = 10%, then the net present
value would be:

NPV = - 3,993 + (3,630 - 4,840)/1.1 + (4,840 - 3,630)/1.21 + 3,993/1.331

= - 3,993 - 1,100 + 1,000 + 3,000 = - 1,093

Since NPV < 0, this investment is not profitable and should not be undertaken.
The investment project is unprofitable at any positive real interest rate. But it is conceivable that the real
interest rate is actually negative, if the rate of inflation is greater than the nominal interest rate. In this case, the
project would be profitable. At high rates of inflation it makes sense to borrow money and pay it back later
with money that has lost some of its purchasing power.

15. "Credit rationing reinforces monetary policies." Comment on this statement.

Credit is rationed when lending institutions limit the amount that firms or consumers can borrow. Tight money
policy raises the cost of borrowing and banks may be concerned that irresponsible borrowers will continue to
borrow at the higher rates while more conservative borrowers will be deterred. To avoid the higher risks
incurred with irresponsible customers, banks often limit the amount of credit given to any one customer. As
fewer funds are made available, less investment takes place. Thus the rationing of credit reinforces restrictive
monetary policy.
However, in the early 1990s the Fed tried to stimulate the U.S. economy by repeatedly lowering the
discount rate. Banks nonetheless chose to increase their holdings of Treasury bills rather than to expand their
lending, since they were still recovering from huge losses encountered in the real estate market. In this case,
the banks, in effect, rationed credit and worked against the expansionary monetary policy of the Fed.

16. Is inventory investment pro-cyclical? Why or why not?

During periods of economic expansion, firms increase production to meet sales and add to inventories, since
the desired stock of inventories increases with output. But when the boom reaches its peak and economic
growth and sales slow, inventories rise involuntarily until firms cut back production. This may actually
contribute to the upcoming recession. Before growth resumes after the recession, firms may want to increase
their inventories to be ready for the increase in demand for their product. When demand picks up, inventories
serve to fill the demand until sales can be met through increased production. Inventory investment is usually
not pro-cyclical but more counter-cyclical, that is, during recessions it rises and during booms it falls.

17. Briefly how explain inventory cycles relate to the multiplier-accelerator model that states that the level of
investment is positively related to the change in income.

According to the multiplier-accelerator principle, any economic disturbance will lead to business cycles. A
change in investment will bring about a larger change in national income, but the level of investment is
proportional to the change in income. This can be expressed as follows:

It = [K*t - Kt-1] ==> It = a[Yt - Yt-1]. ==> I = k(Y), with k = a.

The multiplier principle states that the change in investment will cause a larger change in national income, or
Y = (I). (Note that  is the expenditure multiplier and different from the a mentioned above.) As long as

179
the economy is growing at an increasing rate, the level of investment will go up, stimulating growth even
further. As the economy peaks, however, a reduction in the growth rate of income will reduce the level of
investment, which will lead the economy into a recession. If, for example, firms expect an economic upswing,
they will increase inventories since they expect future sales to increase. They will step up production, which
will stimulate the economy. When the economy approaches its full potential, bottlenecks will occur, so prices
and interest rates will increase. While actual sales may still increase (since the economy is still growing), they
may fall below expected sales, so there may be an unwanted increase in inventories. Firms will then cut
production, which may eventually cause layoffs and possibly a recession.

18. "Even though the economy may still be growing, a decrease in the growth rate of income may in itself be
sufficient to lead the economy into a recession." Comment on this statement.

The multiplier-accelerator principle implies that any economic disturbance can lead to economic cycles. A
change in investment will bring about a larger change in national income, but the level of investment is
proportional to the change in income. As long as the economy grows at an increasing rate, the level of
investment will also increase which will further stimulate the economy. However, a reduction in the growth
rate of income will reduce the level of investment. If the level of investment decreases, the change of
investment becomes negative. This will be multiplied to produce an even larger negative change in output and
the economy will go into a recession. For example, we can explain inventory cycles in the following way: If
firms expect an economic upswing, they will let inventories grow since they expect future sales to increase
also. To increase inventories they will have to step up production, which will stimulate the economy. When the
economy approaches its full potential, then bottlenecks will occur, and prices and interest rates will increase.
While actual sales may still increase (since the economy is still growing), they may fall below expectations, so
there will be an unwanted increase in inventories. Firms will then cut their production level which may
eventually cause some layoffs and a reduction in spending, that is, the economy enters a recession.

19. Explain why a lending institution may prefer a variable mortgage rate, while the homebuyer may prefer a
fixed mortgage rate. Relate your answer to the situation of many S&Ls in the late 1970s and early 1980s.

A variable mortgage rate shifts the risk of a change in interest rates from the lending institution onto the
homebuyer. If interest rates go up, then mortgage interest payments go up as well. Many homebuyers,
especially those who barely qualify for mortgage loans, will not want to take this risk and will therefore prefer
a fixed mortgage rate that will guarantee a fixed payment each month.
A fixed mortgage rate leaves the risk of changing interest rates with the lending institution, which may
incur losses if interest rates go up substantially. As interest rates rose substantially in the late 1970s, S&Ls
were faced with low returns on assets and high costs of obtaining new funds. As a result, they incurred huge
losses. S&Ls held a large part of their assets in mortgages that had been negotiated in the 1960s when interest
rates were low. At the same time consumers shifted their funds from S&Ls into primary securities that paid
higher yields. The S&Ls therefore were faced with a liquidity problem.

20. Comment on the following statement:


"Continued high budget deficits will adversely affect the construction industry."

If the government finances an increase in spending by borrowing from the public, the budget deficit increases.
As the demand for credit increases, interest rates increase as well. This negatively affects the housing sector,
which is highly interest sensitive. Higher mortgage interest rates not only lead to a decrease in the demand for
new houses, but may also deter some housing developers from financing new projects.

180
CHAPTER 15

THE DEMAND FOR MONEY

Solutions to the Problems in the Textbook:

Conceptual Problems:

1. Money is any asset that is widely used as a medium of exchange. People want to hold money since it
simplifies their current or future transactions. Since the nominal value of money is always known with
certainty, it is a very safe and liquid asset, unless there is a very high rate of inflation. Therefore holding money
protects asset holders against potential capital losses on riskier assets.

2. In a society in which there was no money, all trade would be completely based on barter. But barter is
fairly inefficient, since suppliers of goods and services can't always find people who are both in need of the
goods and services being offered and willing to trade something of value to the suppliers. Therefore, sooner or
later, some form of money has to be invented to facilitate trade. A good example of a money substitute is U.S.
cigarettes, which were used as medium of exchange during the hyperinflation in Germany in 1922/23, in
P.O.W. camps after World War II, and in some Eastern European countries during the transition from command
economies to more market-based economies when people lost confidence in their own currencies.
It is easier to imagine a society in which there is no currency but everyone has a credit or debit card. The
cashless purchase of goods via credit or debit cards (or via e-commerce) is not only convenient for customers
but can also provide valuable information to sellers about the buying habits of consumers (which is one reason
grocery stores now willingly accept credit or debit cards). In a cashless society all transactions would require
electronic fund transfers through bank computers. At the end of the month, all accounts would have to be
settled with the company that issued the credit or debit card, again through electronic fund transfers. This
requires investment in computer technology, not only for banks but also for merchants. Such a system may
seem highly efficient, but it may not work well for very small purchases, such as a magazine at a newspaper
stand, a can of soda from a vending machine, or a local phone call. However, financial institutions have
already developed pre-paid cash cards or phone cards that allow the purchase of smaller items or phone calls,
with the amount of the purchase automatically deducted from the card.
It should be noted, however, that a cashless society still requires money, that is, funds that can be stored in
accounts, whether these are traditional bank accounts used to settle outstanding bills or accounts that exist
simply for e-commerce. However, if pre-paid cash cards, debit and credit cards, or e-commerce funds are used
increasingly, monetary aggregates may have to be redefined.

3. One could certainly make an argument for including credit card limits as part of money stock, since
people substitute credit cards for cash as a means of payment. They may also use unused credit lines for
precautionary purposes. But if we include this form of credit in the money stock, it becomes difficult to justify
excluding others. Therefore it may be better not to include credit card limits in the definition of money.

181
4. In deciding how many traveler's checks to take on a vacation, an individual has to weigh the cost of
obtaining the checks against the probability of cash being stolen and the possibility that personal checks or
credit cards might not be honored. The lower the cost of traveler's checks and the higher the probability that
cash may be stolen, the larger the sum that is likely to be carried in traveler's checks. On the other hand,
holding cash or traveler's checks involves an opportunity cost, since no interest can be earned on cash or
traveler's checks. Therefore, if credit cards are accepted at the vacation spot, vacationers may rely primarily on
credit cards as a means of payments, since they can earn interest on their money holdings until they have to
pay their credit card bill. Many travelers now actually prefer to rely on ATM cards to obtain cash in foreign
countries. This not only allows them to carry less cash or traveler's checks, but also eliminates the fee for
exchanging currency. Some ATM cards also serve as debit cards and can be used for the purchase of goods or
services. Possible drawbacks of relying on ATM cards include the possibility of a strike among bank
employees in the foreign country or the absence of ATM machines in remote areas.

5. The opportunity cost measures the benefit or revenue foregone from using a resource in one way rather
than another. Individuals who hold money give up the return they could have earned had the money been
invested in a bond (or any other interest-earning asset not included in the definition of money). To guarantee
lenders a positive rate of return, bonds pay nominal interest rates that include an inflation premium to cover the
expected loss in purchasing power over the period that the bond is held. Therefore it is the nominal and not the
real interest rate that is the opportunity cost of holding money. However, when the rate of inflation is very
high, nominal interest rates do not adequately reflect the rate of inflation. In this case, the inflation rate (if it is
higher than the interest rate) should be used as the appropriate opportunity cost.

6. The more inflation increases, the more individuals fear a loss of purchasing power from their money
holdings. People are therefore not as likely to hold much money in periods of high inflation. An increase in
inflation will increase the nominal interest rate, which is the opportunity cost of holding money. This will
reduce the demand for money. From the equilibrium condition in the money sector (md = L(i,Y) = M/P = ms),
we can see that if prices (P) increase more than nominal money supply (M), then real money balances (M/P)
will decrease and the demand for real money balances (md) will also decrease to get the money sector back
into equilibrium. In periods of high inflation, nominal money supply tends to increase rapidly, so nominal
money holdings become larger.

7. During periods of deflation, the cost of holding cash decreases and people tend to carry more cash.
Therefore muggers can expect a much higher return from their activity.

Technical Problems:

1.a. Cash machines that allow withdrawals from saving accounts reduce the need to hold precautionary funds
in cash or checking accounts. Therefore the demand for M1 would decrease, but the demand for M2 would not
be affected since M1 is included in M2.

1.b. If more tellers worked at your bank, then your waiting time would be reduced, so you might be willing to
go to the bank more often to make withdrawals. The demand for M1 would decrease, but the demand for M2
would not be affected since M1 is included in M2.

1.c. An increase in inflationary expectations would reduce the demand for money, since people would be afraid
of losing purchasing power. In times of low inflation, people may switch from low interest accounts to higher

182
interest accounts. The demand for M1 decreases, while the demand for M2 is not affected. However, if
inflation is very high, people may hold less money in any form and spend more on durable goods. In this case
the demand for both M1 and M2 might decrease.

1.d. Increased use of credit cards would reduce the need to hold money for transactions, even though people
would still need to settle their monthly credit card charges in a convenient way. They would most likely shift
funds from demand deposits into time deposits, that is, away from M1 and into M2. The demand for M1 would
decrease, but the demand for M2 would not be affected.

1.e. If the fear of a collapse of the government led to the fear that money would lose its value, people would try
to spend as much money as possible to buy durable or tradable goods before the government collapses. Thus
the demand for both M1 and M2 would decrease.

1.f. With an increase in the interest rate on time deposits, people would shift more funds from demand deposits
into time deposits. The demand for M1 would decrease while the demand for M2 would remain unaffected.
However, if interest rates on M2 components increased much more than the yield on other assets, the demand
for other assets could also decrease. In such a case, the demand for M2 would increase.

1.g. With the rise in e-commerce, more cashless transactions take place. Therefore people would most likely
shift funds from demand deposits into high-interest earning accounts until their accounts had to be settled.
Most e-commerce transactions involve the use of credit cards, so the answer here is the same as in 1.d. Funds
are shifted away from M1 and into M2, and the demand for M1 decreases, but the demand for M2 is not
affected. However, some banks have already established e-cash accounts exclusively for e-commerce;
increased use of e-cash requires a re-definition of monetary aggregates.

2.a. As the economy goes into a recession, income and interest rates are both likely to decline. Since velocity
decreases as income and interest rates decrease, we should expect the velocity to decline in a recession. Also,
since velocity is defined as nominal GDP over nominal money supply, it is easy to see that velocity should
decrease, since nominal income declines while nominal money supply is not affected.

2.b. The Fed can influence velocity by injecting or withdrawing money through its open market operations. If
money supply increases more than nominal GDP, velocity decreases.

3. This problem is just a reversal of the inventory approach used by Baumol and Tobin, since now only one
withdrawal has to be made at the end of the month, while many deposits can be made during the month. The
firm would have to calculate the benefits from interest earned and compare them to the cost of making the
deposits. It would therefore calculate its cash holdings in the same way as in the inventory approach.

4.a. The person tries to minimize the cost of managing the portfolio. This cost is: C = ntc + (iY/2n)

n = 1 ==> C = 1*1 + (0.005)(1,600)/(2*1) = 1 + 4 = 5

n = 2 ==> C = 2*1 + (0.005)(1,600)/(2*2) = 2 + 2 = 4

n = 3 ==> C = 3*1 + (0.005)(1,600)/(2*3) = 3 + 4/3 = 4.34

n = 4 ==> C = 4*1 + (0.005)(1,600)/(2*4) = 4 + 1 = 5

183
Therefore 2 transactions are optimal.

4.b. With two transactions, the optimal cash holding is: md = Y/(2n) = 1,600/(2*2) = $400.

4.c. n = 1 ==> C = 1*1 + (0.005)(1,800)/(2*1) = 1 + 4.5 = 5.5

n = 2 ==> C = 2*1 + (0.005)(1,800)/(2*2) = 2 + 2.25 = 4.25

n = 3 ==> C = 3*1 + (0.005)(1,800)/(2*3) = 3 + 1.5 = 4.5

n = 4 ==> C = 4*1 + (0.005)(1,800)/(2*4) = 4 + 1.125 = 5.125

Therefore it is still optimal to make two transactions, but the optimal cash holding is now

md = 1,800/(2*2) = $450.

This is a 12.5% increase, since (450 - 400)/400 = .125.

Additional Problems:

1. Do people use credit cards more often when interest rates are high or low?

Many credit card companies charge a high but fixed interest rate on all balances that are not immediately paid
off. Therefore, in periods of high interest rates, the interest charge becomes relatively smaller and thus we
should expect the use of credit cards to go up. Also, since it may take up to a month from the time of purchase
to get the credit card bill and people generally have an additional month to pay the balance, card holders get, in
effect, an interest-free loan for up to two months. In times of high interest rates people have a greater incentive
to keep their funds in an interest-earning account for these two months. Therefore they have a greater incentive
to use their credit cards in times of high interest rates.

2. “The opportunity cost of holding a bond is the return from holding money." Comment.

The opportunity cost is the benefit foregone when using a resource one way instead of another. People who
hold bonds receive a rate of return but they are giving up the utility that holding money provides to them
through its role as a medium of exchange. Since different individuals value such utility differently, the
opportunity cost of holding a bond varies from one individual to another. Money is also held to protect against
capital losses that may occur from a decrease in bond values arising from an increase in interest rates. The
avoidance of such a capital loss can be seen as part of the return of holding money.

3. "The real interest rate is the opportunity cost of holding money." Comment on this statement.

The opportunity cost measures the benefit or the revenue foregone when using a resource in one way rather
than another. If individuals hold money, they give up the return they would have earned had they used the
money to purchase a bond. Bonds pay nominal interest rates to guarantee the lender a real return plus a
premium to cover the expected loss in the purchasing power of the principal over the period of the "loan."

184
Therefore it is the nominal and not the real interest rate that is the opportunity cost of holding money. If the
rate of inflation is higher than the prevailing nominal interest rate, then the rate of inflation should be used as
the opportunity cost.

4. "Money market deposit accounts are offered by banks, are FDIC insured, and have check-writing
privileges. Therefore they should be included in M1 rather than M2." Comment on this statement.

Even though money market deposit accounts (MMDAs) limit the number of checks that can be written per
month, they do provide a very cheap and convenient way of making immediate payments. Therefore it can be
argued that they should be included as part of M1. However, once the check limit is reached, such accounts are
much less liquid than normal checking accounts. Therefore it makes sense to include MMDAs as part of M2
rather than M1.

5. How did the introduction of so-called NOW-accounts affect the money demand of M1 and M2? How did
it affect the income velocity of M1 and M2?

With the introduction of NOW-accounts, many people transferred funds from their saving accounts to these
interest-earning checking accounts. This meant that more money balances were held in M1 assets and the
income velocity for M1 decreased. Since M1 is part of M2, such transfers did not affect money balances held
in M2 or the income velocity of M2.

6. Derive the square-root formula of the Baumol-Tobin transactions demand model. Then explain how this
formula can be used to explain that high-income individuals on average hold a smaller proportion of their
income in money assets than low-income people.

The square-root formula can easily be derived by minimizing the total cost of holding money with respect to
the number of transactions n. This total cost is the sum of the transaction cost (ntc) and the interest cost imd,
with md being the average cash balance, that is,

md = Y/(2n).

Therefore from C = ntc + (iY)/(2n) ==>

C/n = tc - (iY)/(2n2) = 0 ==> tc = (iY)/(2n2) ==> (tcY)/(2i) = [Y/(2n)]2 = md2

==> md = [(tcY)/(2i)]1/2.

According to this formula, money holdings increase proportionally less than income. This implies that
individuals in higher income brackets can hold proportionally less money than people in lower income
brackets. Most likely this is due to the fact that high-income individuals tend to have better access to financial
markets than low-income individuals, which means that they have a lower probability of becoming illiquid.

7. Use the Baumol-Tobin transactions demand approach to calculate the interest elasticity of money demand
for transaction.

From the Baumol-Tobin approach we can derive the square-root formula, that is,

185
md* = [(tcY)/(2i)]1/2.

This equation shows that money demand (md) is positively related to income (Y) but inversely related to the
interest rate (i). From this square-root formula, the interest elasticity of money demand can be derived as
follows:

md/i = - [(1/2)[(tcY)/(2i)]-1/2(tcY/2i2) = - (1/2)(1/i)md

==> [(md/md)]/[(i/i)] = (md/i)(i/md) = - 1/2

8. "According to the classical quantity theory of money, any decrease in nominal money supply will lead to
a decrease in the unemployment rate." Comment on this statement.

The classical quantity theory of money suggests that velocity is constant and that output stays at its full-
employment level. Any change in nominal money supply will therefore lead to a proportional change in the
price level. According to the classical quantity theory, a decrease in nominal money supply will lead to a
decrease in the price level and not the unemployment rate.

9. Which is greater: the transaction velocity of money or the income velocity of money?

The income velocity of money only includes transactions involving final goods and services, whereas the
transaction velocity of money includes all transactions, including intermediate goods and wealth transfers.
Therefore the transaction velocity of money is greater than the income velocity of money.

10. "Financial innovation raises the velocity of money." Comment on this statement.

The money demand function can be expressed as a function of interest rates, income, and financial innovation
(e), where a positive value of e reduces the demand for money, that is,

md = L(i, Y, e).

Velocity is defined as V = (PY)/M = Y/(M/P).

If we assume that the money sector is in equilibrium, then money demand can be substituted for money supply,
and we get

md = L(i,Y,e)= M/P = ms ==> V = Y/L(i, Y, e) ==> V = f(Y,i,e),

that is, velocity is a function of income, the interest rate and financial innovation. Therefore, if there are
financial innovations (and the value of e goes up), the demand for money declines. This raises the number of
times a dollar bill circulates in the economy during a given period of time. In other words, it increases the
income velocity of money.

11. "An increase in the income tax rate will increase the income velocity of money." Comment on this
statement. In your answer consider only the short-run effects of such a policy change.

186
Restrictive fiscal policy, such as an income tax increase, will affect GDP negatively and lead to lower interest
rates, at least in the short run. Since the income velocity of money is positively affected by an increase in
income and the interest rate, velocity should decrease. Another way of getting to the same result is to look at
velocity as the ratio of nominal GDP to nominal money supply, that is,

V = (PY)/M.

Since restrictive fiscal policy leads to a (short-run) decrease in nominal GDP (PY), while nominal money
supply (M) remains unaffected (the Fed was not involved in this fiscal policy measure), velocity (V) should
decrease.

12. "Fiscal expansion will increase the income velocity of money." Comment on this statement.

Expansionary fiscal policy increases income and the interest rate. Since the income velocity of money is
positively affected by increases in income and the interest rate, the income velocity should go up. Similarly,
from the equation MV = PY, it follows that V = (PY)/M. Since expansionary fiscal policy raises nominal GDP
(PY) but does not affect nominal money supply (M), we should expect velocity (V) to go up.

13. Assume you know that the supply of money (M) has increased by 6%, real output (Y) has increased by
2.5% and the income velocity (V) of money has dropped by 3%. What is the rate of inflation?

From the quantity theory of money equation (MV = PY) ==> %M + %V = %P + %Y ==>

%P = %M - %Y + %V ==> %P = 6% - 2.5% + (-3%) = + 0.5%.

Therefore the rate of inflation (%P) should be 0.5%.


Note: The equation that shows the percentage changes in money supply, output, velocity, and prices generally
only applies for very small changes in these variables. Therefore, the result for this problem is just an
approximation of the actual change we should expect in the price level.

14. Assume that M2 has increased by 8%, inflation has increased by 4%, and real output has grown at a rate
of 1.5% over the same time period. What does this imply for V2, the income velocity of M2?

From the equation MV = PY, we get %M + %V = %P + %Y. It follows that

8% + %V = 4% + 1.5% ==> %V = - 2.5%,

that is, V2 must have decreased by 2.5%. (The note for question 14 applies here as well.)

CHAPTER 16

THE FED, MONEY, AND CREDIT

Solutions to the Problems in the Textbook:

187
Conceptual Problems:

1. The three tools the Fed has to conduct monetary policy are open market operations, discount rate changes,
and reserve requirement changes. If the Fed wants to increase the money supply, it has the following options:
first, the Fed can buy government bonds from the public (mostly banks), thereby increasing bank reserves.
These open market purchases will induce banks to extend their loans, which will create more money. Second,
it can lower the discount rate, so it becomes less costly for banks to borrow reserves from the Fed. This also
will induce banks to create more money by extending more loans. Finally, the Fed can lower the required-
reserve ratio, which again will allow banks to lend more.

2. The currency-deposit ratio is the ratio of currency outstanding to bank deposits. The Fed cannot directly
influence this ratio, since it is determined by the behavior of the public and influenced by the convenience of
obtaining cash and by seasonal patterns (increased Christmas shopping, for example). However, by changing
either bank regulations (that would affect the ease of obtaining cash) or interest rates (that would change the
opportunity cost of holding cash), the Fed may indirectly affect how much currency the public is willing to
hold.

3.a. 3.a. IS2 3.b.


i IS LM2 i IS1 LM
LM1
i2
i2
i1 i1

0 0
Y2 Y1 Y Y1 Y2 Y

If most disturbances come from the money sector (a shift in money demand), interest rate targets work better
than money targets. In the IS-LM diagram below we can see that as money demand increases due to changing
expectations, the LM-curve will shift to the left and the interest rate will increase. By increasing money supply
and shifting the LM-curve back to the right, the central bank can get the economy back to the original
equilibrium.

3.b. If most disturbances come from the expenditure sector, the central bank is better off targeting money
supply. If spending increases, the IS-curve shifts to the right and the interest rate increases. If the central bank
tried to get the interest rate back to its original level by increasing money supply, the disturbance would
intensify, since the LM-curve would also shift to the right. Thus, the central bank should keep money supply
(and thus the LM-curve) stable to keep the disturbance at a minimum.

4.a. A bank run occurs when depositors, worried about the safety of their assets, rush to withdraw their
deposits.

4.b. If a bank is in trouble because it has made some bad investment decisions, people may expect it to fail.

188
Thus they may want to withdraw their deposits before it is too late. Since other depositors are likely to behave
in the same way, a run on the bank can be anticipated. Even a fairly financially sound bank may not be able to
withstand a run, since most assets are tied up in loans. Almost all U.S. banks are FDIC insured and therefore a
run on a bank is very unlikely. With FDIC insurance, depositors know that they can get at least their principal
back from the government should a bank fail, and therefore they do not panic easily.

4.c. During the Great Depression, a large-scale run on banks lead to liquidity problems and bank failures. This
decreased the lending power of the whole banking system. In other words, depositors lost their confidence in
banks and withdraw their deposits. This increased the currency-deposit ratio, leading to a decrease in the
money multiplier and a contraction in money supply.

4.d. The existence of the FDIC increases the public's confidence in the banking system, so a run on banks is
highly unlikely. Therefore the currency-deposit ratio is low and the value of the money multiplier is high. The
money multiplier is also more stable since the public does not withdraw deposits any time a bank failure
occurs.

5.a. There are basically two reasons why the Fed does not adhere more closely to its monetary growth targets
in the short run. The first is technical: due to the variability of the money multiplier and the lag in collecting
data on money supply figures, the Fed is not always able to achieve its monetary growth target. The second
reason is that the Fed, in the short run, uses interest rate targets concurrently with monetary growth targets, and
it is impossible to succeed at both at the same time. Therefore, as the Fed responds to changes in the economy,
it may move away at least temporarily from its monetary growth target. The Fed's desire to have some short-
run flexibility while still maintaining long-run credibility, may cause a temporary deviation from the
announced monetary growth target.

5.b. The targeting of nominal interest rates can be self-defeating, especially in times of high inflation. If
(nominal) interest rates increase, the Fed has to increase money supply to reduce interest rates to their original
level. However, expansionary monetary policy will lead to more inflation and this will ultimately result in
higher nominal interest rates. The so-called Fisher-equation states that the nominal interest rate (in) is equal to
the real interest rate (ir) plus the rate of inflation (), that is,

in = ir + .
In the long run, the real interest rate will not be affected by expansionary monetary policy, but the
nominal interest rate will be higher due to increased inflation. Another attempt to further reduce the nominal
interest rate by expanding money supply even more will aggravate inflation even more and ultimately not
succeed in bringing interest rates down.

6.a. Nominal GDP is an ultimate target of monetary policy.

6.b. The discount rate is an instrument of monetary policy.

6.c. The monetary base is an immediate target of monetary policy.

6.d. M1 is an intermediate target of monetary policy.

6.e. The Treasury bill rate is an intermediate target of monetary policy.

6.f. The unemployment rate is an ultimate target of monetary policy.

189
7. When banks ration credit, interest rates are no longer a good indication of existing market conditions.
Credit is rationed when lending institutions limit the amount that their customers can borrow based on
concerns that such borrowing may not be financially prudent. In this situation, the Fed should not use interest
rate targets as a guide for its monetary policy, since interest rates no longer reflect true market conditions.

8. The Fed has much more control over intermediate targets (money supply or interest rates) than it does over
ultimate targets (GDP, unemployment, or inflation). Changes in these intermediate targets do not have an
immediate effect on the ultimate targets and therefore the Fed can easily reverse or re-enforce its policy
measure. Because of the long lags associated with monetary policy, the Fed uses these intermediate targets to
get feedback on the effects of a policy change and the likeliness that a policy measure will achieve its ultimate
goal. However, concentrating solely on intermediate targets does not guarantee that the ultimate objectives will
be achieved.

9. From the quantity theory of money equation MV = PY, we get

%M + %V = %P + %Y ==> %P = %M - %Y + %V.

If real GDP (Y) is assumed to grow at a rate of 3.5%, the Fed has to let money supply (M) grow at a rate of
3.5% to keep prices (P) stable, assuming that velocity (V) remains stable. The Fed can control nominal GDP
through changes in nominal money supply only as long as the behavior of money demand (and thus velocity)
is relatively predictable. The long-run GDP growth rate has been around 2.25%, far below the 3.5% mentioned
here, and expansionary monetary policy will not achieve such a high growth rate. But there is a very close
relationship between money supply changes and price changes in the long run, while real GDP growth is
primarily influenced by other factors. If the Fed overestimates the rate at which potential GDP grows, then it is
likely to stimulate the economy too much and induce high inflation. Therefore, nominal GDP targeting rather
than real GDP targeting may be a better approach, since the former creates a policy tradeoff between
unemployment and inflation. In other words, we will get less growth but also less inflation if potential GDP
growth is overestimated.

Technical Problems:

1. Assume the Fed sells Treasury bills valued at $10 million to a bank.

Fed Balance Sheet: Assets Liabilities


Govt. securities - $10 Currency 0
Other assets 0 Bank deposits - $10

Bank Balance Sheet: Assets Liabilities


Deposits at the Fed - $10 Deposits 0
Govt. securities + $10
Other assets 0

The bank has now lost $10 million in reserves (deposits at the Fed). If required reserves are no longer
sufficient, then the bank will have to acquire new reserves.

190
If a bank depositor buys the Treasury bills, then the balance sheet will be:

Bank Balance Sheet: Assets Liabilities


Reserves - $10 Deposits - $10
Other assets 0

Again, the bank may have to make up for the loss of reserves.

2. Assume the Fed buys $10 million worth of gold and then decides to sterilize the effect of this purchase on
the monetary base through open market operations.

Fed Balance Sheet: Assets Liabilities


Gold + $10 Currency 0
Other assets 0 Member bank deposit + $10

The purchase of gold increased the monetary base (bank reserves) by $10 million.

Fed Balance Sheet


After Sterilization: Assets Liabilities
Gold + $10 Bank deposits (+10 -10) = $0
Govt. securities - $10

The sale of government securities to banks again decreased the monetary base (bank reserves) by $10 million,
so there is no overall change in the monetary base.

3.a. If the reserve-deposit ratio is 100%, then banks cannot create any loans and the money multiplier is equal
to 1. This means that the Fed has total control over the money supply, since it has control over bank reserves.
However, this would significantly change the banking industry, since banks no longer would be able to extend
loans.

3.b. Since banks would not be able to issue any loans, the assets side would contain only reserves.

3.c. Banking could still remain profitable as long as banks were able to generate service charges to cover their
operating costs.

In deciding whether monetary base targeting or interest rate targeting is better for the Fed in its conduct of
monetary policy, it would be good to know whether the goods sector or the money sector is more prone to
disturbances. If most disturbances occur in the goods sector (assume the IS-curve shifts to the right), then
monetary base targeting is better, since interest rate targets would force the Fed to aggravate the disturbance.
Under interest rate targeting, the Fed would be forced to change money supply (shifting the LM-curve to the
right) and aggregate demand would be changed even more. If most disturbances occur in the money sector
(assume the LM-curve shifts to the left), then interest rate targeting is better, since the Fed can easily offset the
disturbance. Under interest rate targeting the Fed could change money supply (shifting the LM-curve to the
right again) without affecting aggregate demand.

IS2

191
i IS LM2 i IS1 LM
LM1
i2
i2
i1 i1

0 0
Y2 Y1 Y Y1 Y2 Y

Additional Problems:

1. How does an increase in the currency-deposit ratio affect the money multiplier? What is the effect of an
increase in the reserve-deposit ratio?

The money multiplier is defined as mm = (1 + cu)/(cu + re), where

cu = CU/D = currency-deposit ratio, and


re = R/D = reserve-deposit ratio.

An increase in the currency-deposit ratio means that people hold more currency and banks have fewer funds to
create deposits. Therefore the money multiplier decreases. An increase in the reserve-deposit ratio means that
banks now hold more reserves, so fewer deposits can be created. Again, the money multiplier decreases.

2. Assume that an increasing number of department and grocery stores accept credit and debit cards and
more consumers use these cards to do their shopping. How will the money multiplier and money supply be
affected?

If more consumers make purchases using credit or debit cards rather than cash, then less currency is held and
the currency-deposit ratio will be lower. This implies a larger money multiplier and, given a fixed stock of
high-powered money, an increase in money supply.

3. "The introduction of the FDIC after the Great Depression not only calmed the worries of the public but
also made monetary policy easier for the Fed." Comment on this statement.

The introduction of the FDIC lowered the public's fear of new bank failures. Consumer confidence in the
banking system increased and people held less currency. Banks also were able to reduce their excess reserves,
since they no longer feared a widespread bank run. The currency-deposit and the reserve-deposit ratios both
declined, and the size of the money multiplier increased. In addition, the money multiplier became more stable,
since consumers became less likely to panic after a bank failure occurred. The larger and the more stable the
money multiplier, the easier it is for the Fed to control money supply by changing the monetary base through
open market operations.

4. Assume money supply (M) is $1,200 billion, total bank deposits (D) are $800 billion and the required

192
reserve-deposit ratio is 10%. What would the Fed have to do to lower money supply by 5%? Explain your
answer.

We know that M = CU + D ==> CU = M - D = 1,200 - 800 = 400.

If we assume that banks do not hold excess reserves, then

R = (0.1)D = (0.1)800 = 80 and H = CU + R = 400 + 80 = 480.

Thus the money multiplier is M/H = mm = 1,200/480 = 2.5.

If the Fed wants to reduce money supply by 5% or $60 billion, it has to reduce high-powered money (H) by
$24 billion, by selling $24 billion worth of Treasury bills. In other words,

M = mm(H) == > - 60 = 2.5(H) ==> (H) = - 60/2.5 = - 24

5. Assume the currency-deposit ratio is 30%, the required reserve-deposit ratio is 8% and the excess reserve-
deposit ratio is 2%. How much would money supply change if the Fed made open market sales valued at $20
million?

The money multiplier is defined as: M/H = mm = (1 + cu)/(cu + re).

In this example the size of the money multiplier is equal to

mm = (1 + 0.3)/(0.3 + 0.08 + 0.02) = (1.3)/(0.4) = 3.25.


An open market sale valued at $20 million would decrease high-powered money (H) by $20 million.
Therefore, money supply (M) would decrease by $65 million, since

M = mm(H) = (3.25)(-20) = - 65.

6. Assume bank deposits are $3,200 billion, the required reserve-deposit ratio is 10%, and currency
outstanding is $400 billion. What should the Fed do to decrease money supply by $100 million?

Ms = Cu + D = 400 + 3,200 = 3,600 and H = Cu + R = Cu + (0.1)D = 400 + 320 = 720

==> money multiplier = Ms/H = mm = 3,600/720 = 5

==> Ms = mm(H) ==> - 100 = 5(H) ==> H = - 20

If the Fed wants to decrease money supply by $100 million, bank reserves have to be decreased by $20 million
through the open market sale of government securities. (Note: The assumption was that excess reserves are
zero, which may not be true.)

7. True or false? Why?


"An open market sale raises the monetary base and therefore money supply."

False. An open market sale occurs when the Fed sells government bonds to the private sector, primarily banks,

193
in return for currency. Reserves held in the form of deposits at the Fed decrease, and therefore the monetary
base (the stock of high-powered money) decreases as does money supply, since banks cannot loan out as much
as previously.

8. What problems would arise if the Fed tried to conduct open market operations via the stock market?

Theoretically, the Fed could change high-powered money and thus the supply of money by buying and selling
stocks. The problem, however, would be how to decide which stocks to buy and sell, since the Fed's actions
would affect the values of the stocks being bought or sold.

9. "Large open market sales may have a negative impact on the demand for money, the budget surplus, the
income velocity of money, and consumption." Comment on this statement.

Open market sales decrease bank reserves and therefore money supply. This increases interest rates, leading to
a lower level of investment and income. Since income tax revenues decrease in a recession, the budget surplus
will also decrease. Since interest rates are higher, the interest payments on the national debt will increase. A
lower level of income means a lower level of consumption. The income velocity of money generally declines
in a recession. However, the decline in money occurs before the decline in income. Thus we first see an
increase in velocity in the short run, followed by a decrease.

10. Which is the most useful tool for the Fed to conduct its monetary policy? In your answer discuss the
advantages and disadvantages of each of the tools that the Fed has at its disposal.

The Fed has three basic tools to conduct monetary policy are open market operations, discount rate changes,
and reserve requirement changes.
Open market operations are used most often by the Fed since it can be undertaken every business day, can
be undertaken to a large or small degree, and can be easily reversed. Bank reserves are immediately affected to
a desired degree with the initiative lying solely with the Fed.
The discount rate can be used as a signal for a change in monetary policy, but often a change in the
discount rate simply reflects an adjustment to existing money market conditions. The disadvantage of using the
discount rate is that it is up to banks to change the level of bank reserves. Bank reserves only change when
banks borrow more or less from the Fed. Since this behavior cannot be anticipated, bank reserve changes
cannot be accurately anticipated.
Reserve requirement changes are used only rarely, since this is an extremely blunt tool. A reserve
requirement change will affect the money multiplier and have a huge effect on money supply. Generally banks
are given ample time to adjust to changes.

11. Comment on the following statement:


"Changes in the discount rate are always a sign that the Fed has changed its monetary policy."

The discount rate is the rate at which banks can borrow from the Fed. The federal funds rate is the rate at
which banks can borrow from each other. Banks generally prefer to borrow at the lowest rate. They do not like
to borrow too often or too much from the Fed, however, since the Fed may then question their way of doing
business. But if the demand for bank reserves increases and the difference between the federal funds rate and
the discount rate gets too large, banks have an incentive to borrow from the Fed more often than usual. In this
case total bank reserves will increase more than the Fed would like. As a result, the Fed may adjust the

194
discount rate to bring it more in line with the federal funds rate. Therefore, while an increase in the discount
rate may signal a shift in the Fed's policy, it may also simply reflect the Fed's response to a change in money
market conditions.

12. In 1991-92, the Fed repeatedly lowered the discount rate, but failed to stimulate the economy. Explain this
fact. Subsequently, the Fed lowered the reserve requirements for banks. In your opinion, what was the Fed's
objective in doing this, and was the objective achieved?

Lowering the discount rate is not always successful in increasing money supply (and thus stimulating the
economy), since it requires that banks take the initiative to change bank reserves. In 1991-92, the U.S. was in a
recession and negative business expectations persisted. Many banks needed to recover from loan losses they
had incurred and did not want to extend credit even though they were encouraged to do so by the Fed.
The Fed finally lowered the reserve requirements for banks in a further effort to stimulate the economy
but also to increase the profitability of banks. Banks do not earn interest on the reserves they hold, so a
decrease in reserve requirements allowed them to increase their earnings and reduce their portfolio risk by
buying Treasury-bills. While the economy was not immediately stimulated by new loans, at least the
profitability of banks increased, creating more stability within the banking system.

13. "Open market sales are more effective than increasing the discount rate in changing money supply."
Comment. In your answer explain the short-run effects of restrictive monetary policy on velocity, the budget
surplus, and national saving.

With open market operations, the Fed has the initiative and bank reserves are immediately affected. Open
market operations can be undertaken to a small or large extent on every business day, the Fed can determine
the level of impact on bank reserves, and the Fed's actions can be easily reversed. Discount rate changes affect
banks' cost of borrowing from the Fed, but leave the initiative to react to the banks. Thus, the Fed cannot easily
predict the exact effect on bank reserves. For example, in 1991 the Fed changed the discount rate 15 times but
banks did not borrow more from the Fed or increase their lending due to unfavorable economic conditions. If
the Fed restricts money supply, interest rates will increase, leading to a decrease in economic activity. Initially,
the income velocity (V = PY/M) will increase due to the lower money supply (M), but it will take time to
affect income. But as national income (Y) decreases, income velocity will decline. Other results will include a
decrease in the budget surplus (due to lower tax revenues) and national saving (due to lower income and a
lower government surplus).

14. Assume the Fed lowered the discount rate. How would personal saving, the budget surplus and aggregate
money demand be affected?

A lower discount rate is intended to encourage banks to borrow more from the Fed. It is not always clear that
banks will respond as expected, but if they do, bank reserves will increase and so will money supply, as banks
increase their lending activity. This will lower interest rates, leading to an increase in investment and national
income. Personal saving will increase with a higher income level. Similarly, tax revenues will go up,
increasing the budget surplus. Lower interest rates and higher income will increase money demand. (We also
can see this from the fact that money supply has increased. Since the money sector has to move into a new
equilibrium, money demand has to go up if money supply is increased.)

15. Should you expect the federal funds rate to be above the discount rate or vice versa? Explain.

195
The Fed is the lender of last resort and banks can always borrow from the Fed if the need arises. When banks
borrow from the Fed, they are charged a rate called the discount rate. But banks also have the option to borrow
from each other at the federal funds rate. Banks generally prefer to borrow at the lowest rate possible.
However, they do not like to borrow too heavily from the Fed, since the Fed is a regulator of banks. Banks fear
that their behavior will be questioned if the Fed takes notice and thus prefer to borrow from each other. In
doing so, they drive the federal funds rate above the discount rate.

16. "Reserve requirements act as an unfair tax on banks." Comment on this statement.

Banks are forced to hold their reserves either as vault cash or as deposits at the Fed earning no interest in either
case. Since other financial institutions have no such reserve requirement, it could be argued that this unfairly
taxes banks. On the other hand, reserves guarantee a certain amount of liquidity for the banking system, which
may be necessary, should there be a run on banks. The reserves held as deposits at the Fed also serve to
facilitate the check clearing process. For these reasons, the tax can be viewed as necessary and therefore less
"unfair."

17. Does the Fed have control over the federal funds rate and over bank reserves? If so, can the Fed control
both simultaneously?

The Fed has indirect control over the federal funds rate, since it has control over the supply of total bank
reserves in the banking system through open market operations. However, the Fed cannot control the demand
for bank reserves. If the demand for bank reserves increases, the federal funds rate will rise. If the Fed chooses
to peg the federal funds rate, it has to create additional bank reserves via open market purchases. On the other
hand, if the Fed chooses to control the level of bank reserves, it has to let the federal funds rate fluctuate.
Therefore, the Fed cannot control the federal funds rate and the level of bank reserves simultaneously.

18. "By lowering the reserve requirements for banks, the Fed reduces the budget deficit, national saving, and
the income velocity of money." Comment on this statement.

If the Fed lowers the reserve requirement, banks have more money to lend out and can thus increase their
earnings by making more loans or buying T-bills. If banks extend their loans, then money supply will increase
and interest rates will decrease, stimulating investment and national income. Saving will increase with a higher
level of income. Similarly, tax revenues will go up, reducing the budget deficit. Interest payments on the
national debt will also decrease with lower interest rates, which will also help to lower the deficit. Velocity will
initially decrease, since money supply will increase before income. But as income increases, then velocity will
increase again. Ultimately, velocity may not change by much, since the income elasticity of money demand is
close to one in the long run.

19. "Restrictive monetary policy over a long time period will lead to lower interest rates." Comment on this
statement.

Long-run effects of monetary policy are different from short-run effects. Restrictive monetary policy leads to
higher interest rates in the short run due to less liquidity (liquidity effect). But higher interest rates will reduce
aggregated demand, which reduces prices and national income. Thus the level of interest rates will start to
decline again (price-income effect). Lower prices will eventually lead to lower inflationary expectations and

196
thus lower nominal interest rates (price-anticipation effect). In the end, real interest rates (ir) will return to their
original level and nominal interest rates (in) will be lower, since the inflation rate ( ) is lower. This is shown
in the so-called Fisher equation: in = ir + .

20. "The elimination of required reserves on bank deposits would decrease the Fed's control over money
supply. But if money supply increased uncontrollably, then high rates of inflation would result." Comment on
the following statement.

The Fed has a number of policy instruments at its disposal to control the level of bank reserves (and thus
money supply). The required-reserve ratio is only one such instrument. The Fed can always influence bank
reserves through the use of open market operations. Even if reserve requirements are abolished, the money
multiplier will always have a finite value, since banks will always hold some (excess) reserves to meet their
daily cash needs and emergency needs. If the reserve requirement were eliminated, the money multiplier would
become larger, since banks would not choose to voluntarily hold as many reserves as the Fed required.
However, large-scale open market operations would still enable the Fed to exercise great influence over bank
reserves and therefore money supply.

"In order to keep national income stable, the FOMC has to purchase government securities whenever interest
rates increase." Comment on the following statement with the help of an IS-LM diagram and explain the
adjustment process.

Interest rates can go up for two reasons: either an increase in spending (a shift of the IS-curve to the right) or
an increase in money demand (a shift of the LM-curve to the left). In the first case, to keep income stable the
Fed has to sell (not purchase) government securities, to induce a decrease in bank reserves (and thus money
supply) and shift the LM-curve to the left. But this will lead to even higher interest rates, lowering investment
and thus future economic growth. In the second case, the Fed can succeed by buying government securities,
shifting the LM-curve back to the right thus, in effect, negating the disturbance.

IS1 LM1
i ISo LMo 12 I  Y  md  i I  Y 
i2 3
i1 2 effect: Y  i

io 1 2 3 Ms  i  I  Y  md  i 

effect: Y i 

0 Overall effect: Y = const., as desired, i 


Yo Y1 Y

LM1
i ISo LMo 12 md  i  I  Y md i

i1 effect: Y i 

io 2 3 Ms  i I Y md  i 

Effect: Y i

197
0 Overall effect: Y = const., as desired, i ?
Y1 Yo Y
22. Comment on the following statement:
"The Fed should always conduct open market purchases whenever interest rates increase."

If the increase in the interest rate is caused by an increase in spending (the IS-curve has shifted to the right),
the Fed should not purchase government securities since this will shift the LM-curve to the right as well,
making the disturbance worse.

i IS2
IS1 LM1 LM2

i2

i1

Y1 Y2 Y3 Y

But when disturbances come from the money sector (assume an increase in money demand shifts the LM-
curve to the left, increasing interest rates), the Fed can restore the original equilibrium by purchasing
government securities, shifting the LM-curve back to the right.

23. True or false? Why?


"Money demand shocks will not affect the level of output as long as the Fed pegs the interest rate."

True. Assume money demand increases, shifting the LM-curve to the left and leading to an increase in the
interest rate. If the Fed pegs the interest rate, it will respond by increasing the money supply. This will shift the
LM-curve back to the right, moving the level of output demanded and the interest rate back to their original
levels. A decrease in money demand requires a reduction in the money supply to bring interest rates and output
back to their original levels.

i IS LM2
LM1

i2
i1

0
Y2 Y1 Y

198
CHAPTER 16

THE FED, MONEY, AND CREDIT

Solutions to the Problems in the Textbook:

Conceptual Problems:

1. The three tools the Fed has to conduct monetary policy are open market operations, discount rate changes,
and reserve requirement changes. If the Fed wants to increase the money supply, it has the following options:
first, the Fed can buy government bonds from the public (mostly banks), thereby increasing bank reserves.
These open market purchases will induce banks to extend their loans, which will create more money. Second,
it can lower the discount rate, so it becomes less costly for banks to borrow reserves from the Fed. This also
will induce banks to create more money by extending more loans. Finally, the Fed can lower the required-
reserve ratio, which again will allow banks to lend more.

2. The currency-deposit ratio is the ratio of currency outstanding to bank deposits. The Fed cannot directly
influence this ratio, since it is determined by the behavior of the public and influenced by the convenience of
obtaining cash and by seasonal patterns (increased Christmas shopping, for example). However, by changing
either bank regulations (that would affect the ease of obtaining cash) or interest rates (that would change the
opportunity cost of holding cash), the Fed may indirectly affect how much currency the public is willing to
hold.

3.a. 3.a. IS2 3.b.


i IS LM2 i IS1 LM
LM1
i2
i2
i1 i1

0 0
Y2 Y1 Y Y1 Y2 Y

If most disturbances come from the money sector (a shift in money demand), interest rate targets work better
than money targets. In the IS-LM diagram below we can see that as money demand increases due to changing
expectations, the LM-curve will shift to the left and the interest rate will increase. By increasing money supply
and shifting the LM-curve back to the right, the central bank can get the economy back to the original
equilibrium.

3.b. If most disturbances come from the expenditure sector, the central bank is better off targeting money
supply. If spending increases, the IS-curve shifts to the right and the interest rate increases. If the central bank

199
tried to get the interest rate back to its original level by increasing money supply, the disturbance would
intensify, since the LM-curve would also shift to the right. Thus, the central bank should keep money supply
(and thus the LM-curve) stable to keep the disturbance at a minimum.

4.a. A bank run occurs when depositors, worried about the safety of their assets, rush to withdraw their
deposits.

4.b. If a bank is in trouble because it has made some bad investment decisions, people may expect it to fail.
Thus they may want to withdraw their deposits before it is too late. Since other depositors are likely to behave
in the same way, a run on the bank can be anticipated. Even a fairly financially sound bank may not be able to
withstand a run, since most assets are tied up in loans. Almost all U.S. banks are FDIC insured and therefore a
run on a bank is very unlikely. With FDIC insurance, depositors know that they can get at least their principal
back from the government should a bank fail, and therefore they do not panic easily.

4.c. During the Great Depression, a large-scale run on banks lead to liquidity problems and bank failures. This
decreased the lending power of the whole banking system. In other words, depositors lost their confidence in
banks and withdraw their deposits. This increased the currency-deposit ratio, leading to a decrease in the
money multiplier and a contraction in money supply.

4.d. The existence of the FDIC increases the public's confidence in the banking system, so a run on banks is
highly unlikely. Therefore the currency-deposit ratio is low and the value of the money multiplier is high. The
money multiplier is also more stable since the public does not withdraw deposits any time a bank failure
occurs.

5.a. There are basically two reasons why the Fed does not adhere more closely to its monetary growth targets
in the short run. The first is technical: due to the variability of the money multiplier and the lag in collecting
data on money supply figures, the Fed is not always able to achieve its monetary growth target. The second
reason is that the Fed, in the short run, uses interest rate targets concurrently with monetary growth targets, and
it is impossible to succeed at both at the same time. Therefore, as the Fed responds to changes in the economy,
it may move away at least temporarily from its monetary growth target. The Fed's desire to have some short-
run flexibility while still maintaining long-run credibility, may cause a temporary deviation from the
announced monetary growth target.

5.b. The targeting of nominal interest rates can be self-defeating, especially in times of high inflation. If
(nominal) interest rates increase, the Fed has to increase money supply to reduce interest rates to their original
level. However, expansionary monetary policy will lead to more inflation and this will ultimately result in
higher nominal interest rates. The so-called Fisher-equation states that the nominal interest rate (in) is equal to
the real interest rate (ir) plus the rate of inflation (), that is,

in = ir + .
In the long run, the real interest rate will not be affected by expansionary monetary policy, but the
nominal interest rate will be higher due to increased inflation. Another attempt to further reduce the nominal
interest rate by expanding money supply even more will aggravate inflation even more and ultimately not
succeed in bringing interest rates down.

6.a. Nominal GDP is an ultimate target of monetary policy.

200
6.b. The discount rate is an instrument of monetary policy.

6.c. The monetary base is an immediate target of monetary policy.

6.d. M1 is an intermediate target of monetary policy.

6.e. The Treasury bill rate is an intermediate target of monetary policy.

6.f. The unemployment rate is an ultimate target of monetary policy.

7. When banks ration credit, interest rates are no longer a good indication of existing market conditions.
Credit is rationed when lending institutions limit the amount that their customers can borrow based on
concerns that such borrowing may not be financially prudent. In this situation, the Fed should not use interest
rate targets as a guide for its monetary policy, since interest rates no longer reflect true market conditions.

8. The Fed has much more control over intermediate targets (money supply or interest rates) than it does over
ultimate targets (GDP, unemployment, or inflation). Changes in these intermediate targets do not have an
immediate effect on the ultimate targets and therefore the Fed can easily reverse or re-enforce its policy
measure. Because of the long lags associated with monetary policy, the Fed uses these intermediate targets to
get feedback on the effects of a policy change and the likeliness that a policy measure will achieve its ultimate
goal. However, concentrating solely on intermediate targets does not guarantee that the ultimate objectives will
be achieved.

9. From the quantity theory of money equation MV = PY, we get

%M + %V = %P + %Y ==> %P = %M - %Y + %V.

If real GDP (Y) is assumed to grow at a rate of 3.5%, the Fed has to let money supply (M) grow at a rate of
3.5% to keep prices (P) stable, assuming that velocity (V) remains stable. The Fed can control nominal GDP
through changes in nominal money supply only as long as the behavior of money demand (and thus velocity)
is relatively predictable. The long-run GDP growth rate has been around 2.25%, far below the 3.5% mentioned
here, and expansionary monetary policy will not achieve such a high growth rate. But there is a very close
relationship between money supply changes and price changes in the long run, while real GDP growth is
primarily influenced by other factors. If the Fed overestimates the rate at which potential GDP grows, then it is
likely to stimulate the economy too much and induce high inflation. Therefore, nominal GDP targeting rather
than real GDP targeting may be a better approach, since the former creates a policy tradeoff between
unemployment and inflation. In other words, we will get less growth but also less inflation if potential GDP
growth is overestimated.

Technical Problems:

1. Assume the Fed sells Treasury bills valued at $10 million to a bank.

Fed Balance Sheet: Assets Liabilities


Govt. securities - $10 Currency 0
Other assets 0 Bank deposits - $10

201
Bank Balance Sheet: Assets Liabilities
Deposits at the Fed - $10 Deposits 0
Govt. securities + $10
Other assets 0

The bank has now lost $10 million in reserves (deposits at the Fed). If required reserves are no longer
sufficient, then the bank will have to acquire new reserves.

If a bank depositor buys the Treasury bills, then the balance sheet will be:

Bank Balance Sheet: Assets Liabilities


Reserves - $10 Deposits - $10
Other assets 0

Again, the bank may have to make up for the loss of reserves.

2. Assume the Fed buys $10 million worth of gold and then decides to sterilize the effect of this purchase on
the monetary base through open market operations.

Fed Balance Sheet: Assets Liabilities


Gold + $10 Currency 0
Other assets 0 Member bank deposit + $10

The purchase of gold increased the monetary base (bank reserves) by $10 million.

Fed Balance Sheet


After Sterilization: Assets Liabilities
Gold + $10 Bank deposits (+10 -10) = $0
Govt. securities - $10

The sale of government securities to banks again decreased the monetary base (bank reserves) by $10 million,
so there is no overall change in the monetary base.

3.a. If the reserve-deposit ratio is 100%, then banks cannot create any loans and the money multiplier is equal
to 1. This means that the Fed has total control over the money supply, since it has control over bank reserves.
However, this would significantly change the banking industry, since banks no longer would be able to extend
loans.

3.b. Since banks would not be able to issue any loans, the assets side would contain only reserves.

3.c. Banking could still remain profitable as long as banks were able to generate service charges to cover their
operating costs.

In deciding whether monetary base targeting or interest rate targeting is better for the Fed in its conduct of
monetary policy, it would be good to know whether the goods sector or the money sector is more prone to

202
disturbances. If most disturbances occur in the goods sector (assume the IS-curve shifts to the right), then
monetary base targeting is better, since interest rate targets would force the Fed to aggravate the disturbance.
Under interest rate targeting, the Fed would be forced to change money supply (shifting the LM-curve to the
right) and aggregate demand would be changed even more. If most disturbances occur in the money sector
(assume the LM-curve shifts to the left), then interest rate targeting is better, since the Fed can easily offset the
disturbance. Under interest rate targeting the Fed could change money supply (shifting the LM-curve to the
right again) without affecting aggregate demand.

IS2
i IS LM2 i IS1 LM
LM1
i2
i2
i1 i1

0 0
Y2 Y1 Y Y1 Y2 Y

Additional Problems:

1. How does an increase in the currency-deposit ratio affect the money multiplier? What is the effect of an
increase in the reserve-deposit ratio?

The money multiplier is defined as mm = (1 + cu)/(cu + re), where

cu = CU/D = currency-deposit ratio, and


re = R/D = reserve-deposit ratio.

An increase in the currency-deposit ratio means that people hold more currency and banks have fewer funds to
create deposits. Therefore the money multiplier decreases. An increase in the reserve-deposit ratio means that
banks now hold more reserves, so fewer deposits can be created. Again, the money multiplier decreases.

2. Assume that an increasing number of department and grocery stores accept credit and debit cards and
more consumers use these cards to do their shopping. How will the money multiplier and money supply be
affected?

If more consumers make purchases using credit or debit cards rather than cash, then less currency is held and
the currency-deposit ratio will be lower. This implies a larger money multiplier and, given a fixed stock of
high-powered money, an increase in money supply.

3. "The introduction of the FDIC after the Great Depression not only calmed the worries of the public but
also made monetary policy easier for the Fed." Comment on this statement.

The introduction of the FDIC lowered the public's fear of new bank failures. Consumer confidence in the

203
banking system increased and people held less currency. Banks also were able to reduce their excess reserves,
since they no longer feared a widespread bank run. The currency-deposit and the reserve-deposit ratios both
declined, and the size of the money multiplier increased. In addition, the money multiplier became more stable,
since consumers became less likely to panic after a bank failure occurred. The larger and the more stable the
money multiplier, the easier it is for the Fed to control money supply by changing the monetary base through
open market operations.

4. Assume money supply (M) is $1,200 billion, total bank deposits (D) are $800 billion and the required
reserve-deposit ratio is 10%. What would the Fed have to do to lower money supply by 5%? Explain your
answer.

We know that M = CU + D ==> CU = M - D = 1,200 - 800 = 400.

If we assume that banks do not hold excess reserves, then

R = (0.1)D = (0.1)800 = 80 and H = CU + R = 400 + 80 = 480.

Thus the money multiplier is M/H = mm = 1,200/480 = 2.5.

If the Fed wants to reduce money supply by 5% or $60 billion, it has to reduce high-powered money (H) by
$24 billion, by selling $24 billion worth of Treasury bills. In other words,

M = mm(H) == > - 60 = 2.5(H) ==> (H) = - 60/2.5 = - 24

5. Assume the currency-deposit ratio is 30%, the required reserve-deposit ratio is 8% and the excess reserve-
deposit ratio is 2%. How much would money supply change if the Fed made open market sales valued at $20
million?

The money multiplier is defined as: M/H = mm = (1 + cu)/(cu + re).

In this example the size of the money multiplier is equal to

mm = (1 + 0.3)/(0.3 + 0.08 + 0.02) = (1.3)/(0.4) = 3.25.


An open market sale valued at $20 million would decrease high-powered money (H) by $20 million.
Therefore, money supply (M) would decrease by $65 million, since

M = mm(H) = (3.25)(-20) = - 65.

6. Assume bank deposits are $3,200 billion, the required reserve-deposit ratio is 10%, and currency
outstanding is $400 billion. What should the Fed do to decrease money supply by $100 million?

Ms = Cu + D = 400 + 3,200 = 3,600 and H = Cu + R = Cu + (0.1)D = 400 + 320 = 720

==> money multiplier = Ms/H = mm = 3,600/720 = 5

==> Ms = mm(H) ==> - 100 = 5(H) ==> H = - 20

204
If the Fed wants to decrease money supply by $100 million, bank reserves have to be decreased by $20 million
through the open market sale of government securities. (Note: The assumption was that excess reserves are
zero, which may not be true.)

7. True or false? Why?


"An open market sale raises the monetary base and therefore money supply."

False. An open market sale occurs when the Fed sells government bonds to the private sector, primarily banks,
in return for currency. Reserves held in the form of deposits at the Fed decrease, and therefore the monetary
base (the stock of high-powered money) decreases as does money supply, since banks cannot loan out as much
as previously.

8. What problems would arise if the Fed tried to conduct open market operations via the stock market?

Theoretically, the Fed could change high-powered money and thus the supply of money by buying and selling
stocks. The problem, however, would be how to decide which stocks to buy and sell, since the Fed's actions
would affect the values of the stocks being bought or sold.

9. "Large open market sales may have a negative impact on the demand for money, the budget surplus, the
income velocity of money, and consumption." Comment on this statement.

Open market sales decrease bank reserves and therefore money supply. This increases interest rates, leading to
a lower level of investment and income. Since income tax revenues decrease in a recession, the budget surplus
will also decrease. Since interest rates are higher, the interest payments on the national debt will increase. A
lower level of income means a lower level of consumption. The income velocity of money generally declines
in a recession. However, the decline in money occurs before the decline in income. Thus we first see an
increase in velocity in the short run, followed by a decrease.

10. Which is the most useful tool for the Fed to conduct its monetary policy? In your answer discuss the
advantages and disadvantages of each of the tools that the Fed has at its disposal.

The Fed has three basic tools to conduct monetary policy are open market operations, discount rate changes,
and reserve requirement changes.
Open market operations are used most often by the Fed since it can be undertaken every business day, can
be undertaken to a large or small degree, and can be easily reversed. Bank reserves are immediately affected to
a desired degree with the initiative lying solely with the Fed.
The discount rate can be used as a signal for a change in monetary policy, but often a change in the
discount rate simply reflects an adjustment to existing money market conditions. The disadvantage of using the
discount rate is that it is up to banks to change the level of bank reserves. Bank reserves only change when
banks borrow more or less from the Fed. Since this behavior cannot be anticipated, bank reserve changes
cannot be accurately anticipated.
Reserve requirement changes are used only rarely, since this is an extremely blunt tool. A reserve
requirement change will affect the money multiplier and have a huge effect on money supply. Generally banks
are given ample time to adjust to changes.

205
11. Comment on the following statement:
"Changes in the discount rate are always a sign that the Fed has changed its monetary policy."

The discount rate is the rate at which banks can borrow from the Fed. The federal funds rate is the rate at
which banks can borrow from each other. Banks generally prefer to borrow at the lowest rate. They do not like
to borrow too often or too much from the Fed, however, since the Fed may then question their way of doing
business. But if the demand for bank reserves increases and the difference between the federal funds rate and
the discount rate gets too large, banks have an incentive to borrow from the Fed more often than usual. In this
case total bank reserves will increase more than the Fed would like. As a result, the Fed may adjust the
discount rate to bring it more in line with the federal funds rate. Therefore, while an increase in the discount
rate may signal a shift in the Fed's policy, it may also simply reflect the Fed's response to a change in money
market conditions.

12. In 1991-92, the Fed repeatedly lowered the discount rate, but failed to stimulate the economy. Explain this
fact. Subsequently, the Fed lowered the reserve requirements for banks. In your opinion, what was the Fed's
objective in doing this, and was the objective achieved?

Lowering the discount rate is not always successful in increasing money supply (and thus stimulating the
economy), since it requires that banks take the initiative to change bank reserves. In 1991-92, the U.S. was in a
recession and negative business expectations persisted. Many banks needed to recover from loan losses they
had incurred and did not want to extend credit even though they were encouraged to do so by the Fed.
The Fed finally lowered the reserve requirements for banks in a further effort to stimulate the economy
but also to increase the profitability of banks. Banks do not earn interest on the reserves they hold, so a
decrease in reserve requirements allowed them to increase their earnings and reduce their portfolio risk by
buying Treasury-bills. While the economy was not immediately stimulated by new loans, at least the
profitability of banks increased, creating more stability within the banking system.

13. "Open market sales are more effective than increasing the discount rate in changing money supply."
Comment. In your answer explain the short-run effects of restrictive monetary policy on velocity, the budget
surplus, and national saving.

With open market operations, the Fed has the initiative and bank reserves are immediately affected. Open
market operations can be undertaken to a small or large extent on every business day, the Fed can determine
the level of impact on bank reserves, and the Fed's actions can be easily reversed. Discount rate changes affect
banks' cost of borrowing from the Fed, but leave the initiative to react to the banks. Thus, the Fed cannot easily
predict the exact effect on bank reserves. For example, in 1991 the Fed changed the discount rate 15 times but
banks did not borrow more from the Fed or increase their lending due to unfavorable economic conditions. If
the Fed restricts money supply, interest rates will increase, leading to a decrease in economic activity. Initially,
the income velocity (V = PY/M) will increase due to the lower money supply (M), but it will take time to
affect income. But as national income (Y) decreases, income velocity will decline. Other results will include a
decrease in the budget surplus (due to lower tax revenues) and national saving (due to lower income and a
lower government surplus).

14. Assume the Fed lowered the discount rate. How would personal saving, the budget surplus and aggregate
money demand be affected?

A lower discount rate is intended to encourage banks to borrow more from the Fed. It is not always clear that

206
banks will respond as expected, but if they do, bank reserves will increase and so will money supply, as banks
increase their lending activity. This will lower interest rates, leading to an increase in investment and national
income. Personal saving will increase with a higher income level. Similarly, tax revenues will go up,
increasing the budget surplus. Lower interest rates and higher income will increase money demand. (We also
can see this from the fact that money supply has increased. Since the money sector has to move into a new
equilibrium, money demand has to go up if money supply is increased.)

15. Should you expect the federal funds rate to be above the discount rate or vice versa? Explain.

The Fed is the lender of last resort and banks can always borrow from the Fed if the need arises. When banks
borrow from the Fed, they are charged a rate called the discount rate. But banks also have the option to borrow
from each other at the federal funds rate. Banks generally prefer to borrow at the lowest rate possible.
However, they do not like to borrow too heavily from the Fed, since the Fed is a regulator of banks. Banks fear
that their behavior will be questioned if the Fed takes notice and thus prefer to borrow from each other. In
doing so, they drive the federal funds rate above the discount rate.

16. "Reserve requirements act as an unfair tax on banks." Comment on this statement.

Banks are forced to hold their reserves either as vault cash or as deposits at the Fed earning no interest in either
case. Since other financial institutions have no such reserve requirement, it could be argued that this unfairly
taxes banks. On the other hand, reserves guarantee a certain amount of liquidity for the banking system, which
may be necessary, should there be a run on banks. The reserves held as deposits at the Fed also serve to
facilitate the check clearing process. For these reasons, the tax can be viewed as necessary and therefore less
"unfair."

17. Does the Fed have control over the federal funds rate and over bank reserves? If so, can the Fed control
both simultaneously?

The Fed has indirect control over the federal funds rate, since it has control over the supply of total bank
reserves in the banking system through open market operations. However, the Fed cannot control the demand
for bank reserves. If the demand for bank reserves increases, the federal funds rate will rise. If the Fed chooses
to peg the federal funds rate, it has to create additional bank reserves via open market purchases. On the other
hand, if the Fed chooses to control the level of bank reserves, it has to let the federal funds rate fluctuate.
Therefore, the Fed cannot control the federal funds rate and the level of bank reserves simultaneously.

18. "By lowering the reserve requirements for banks, the Fed reduces the budget deficit, national saving, and
the income velocity of money." Comment on this statement.

If the Fed lowers the reserve requirement, banks have more money to lend out and can thus increase their
earnings by making more loans or buying T-bills. If banks extend their loans, then money supply will increase
and interest rates will decrease, stimulating investment and national income. Saving will increase with a higher
level of income. Similarly, tax revenues will go up, reducing the budget deficit. Interest payments on the
national debt will also decrease with lower interest rates, which will also help to lower the deficit. Velocity will
initially decrease, since money supply will increase before income. But as income increases, then velocity will
increase again. Ultimately, velocity may not change by much, since the income elasticity of money demand is
close to one in the long run.

207
19. "Restrictive monetary policy over a long time period will lead to lower interest rates." Comment on this
statement.

Long-run effects of monetary policy are different from short-run effects. Restrictive monetary policy leads to
higher interest rates in the short run due to less liquidity (liquidity effect). But higher interest rates will reduce
aggregated demand, which reduces prices and national income. Thus the level of interest rates will start to
decline again (price-income effect). Lower prices will eventually lead to lower inflationary expectations and
thus lower nominal interest rates (price-anticipation effect). In the end, real interest rates (ir) will return to their
original level and nominal interest rates (in) will be lower, since the inflation rate ( ) is lower. This is shown
in the so-called Fisher equation: in = ir + .

20. "The elimination of required reserves on bank deposits would decrease the Fed's control over money
supply. But if money supply increased uncontrollably, then high rates of inflation would result." Comment on
the following statement.

The Fed has a number of policy instruments at its disposal to control the level of bank reserves (and thus
money supply). The required-reserve ratio is only one such instrument. The Fed can always influence bank
reserves through the use of open market operations. Even if reserve requirements are abolished, the money
multiplier will always have a finite value, since banks will always hold some (excess) reserves to meet their
daily cash needs and emergency needs. If the reserve requirement were eliminated, the money multiplier would
become larger, since banks would not choose to voluntarily hold as many reserves as the Fed required.
However, large-scale open market operations would still enable the Fed to exercise great influence over bank
reserves and therefore money supply.

"In order to keep national income stable, the FOMC has to purchase government securities whenever interest
rates increase." Comment on the following statement with the help of an IS-LM diagram and explain the
adjustment process.

Interest rates can go up for two reasons: either an increase in spending (a shift of the IS-curve to the right) or
an increase in money demand (a shift of the LM-curve to the left). In the first case, to keep income stable the
Fed has to sell (not purchase) government securities, to induce a decrease in bank reserves (and thus money
supply) and shift the LM-curve to the left. But this will lead to even higher interest rates, lowering investment
and thus future economic growth. In the second case, the Fed can succeed by buying government securities,
shifting the LM-curve back to the right thus, in effect, negating the disturbance.

IS1 LM1
i ISo LMo 12 I  Y  md  i I  Y 
i2 3
i1 2 effect: Y  i

io 1 2 3 Ms  i  I  Y  md  i 

effect: Y i 

0 Overall effect: Y = const., as desired, i 


Yo Y1 Y

208
LM1
i ISo LMo 12 md  i  I  Y md i

i1 effect: Y i 

io 2 3 Ms  i I Y md  i 

Effect: Y i

0 Overall effect: Y = const., as desired, i ?


Y1 Yo Y
22. Comment on the following statement:
"The Fed should always conduct open market purchases whenever interest rates increase."

If the increase in the interest rate is caused by an increase in spending (the IS-curve has shifted to the right),
the Fed should not purchase government securities since this will shift the LM-curve to the right as well,
making the disturbance worse.

i IS2
IS1 LM1 LM2

i2

i1

Y1 Y2 Y3 Y

But when disturbances come from the money sector (assume an increase in money demand shifts the LM-
curve to the left, increasing interest rates), the Fed can restore the original equilibrium by purchasing
government securities, shifting the LM-curve back to the right.

23. True or false? Why?


"Money demand shocks will not affect the level of output as long as the Fed pegs the interest rate."

True. Assume money demand increases, shifting the LM-curve to the left and leading to an increase in the
interest rate. If the Fed pegs the interest rate, it will respond by increasing the money supply. This will shift the
LM-curve back to the right, moving the level of output demanded and the interest rate back to their original
levels. A decrease in money demand requires a reduction in the money supply to bring interest rates and output
back to their original levels.

i IS LM2
LM1

209
i2
i1

0
Y2 Y1 Y

CHAPTER 18
BIG EVENTS: THE ECONOMICS OF DEPRESSION, HYPERINFLATION, AND DEFICITS

Solutions to the Problems in the Textbook:

Conceptual Problems:

1.a. The Keynesian explanation for the Great Depression concentrates on the fall of investment spending and
reduction in aggregate consumption. The decrease in aggregate demand was reinforced by restrictive fiscal
policy as the government tried to balance the budget.

1.b. The monetarists' explanation for the Great Depression concentrates on the decline in money supply. The
Fed failed to prevent the large number of bank failures and consumers lost confidence in the banking system.
This led to an enormous increase in the currency-deposit ratio, causing a decrease in the money multiplier. The
resulting severe decline in the supply of money then lead to the economic downturn.

1.c. Both explanations given above fit the facts and there is no inherent conflict between them; in fact, they
complement one another quite nicely. The combination of inept fiscal and monetary policy may have turned
what could have been an average recession into a major depression.

1.d. The Great Depression is one of the most drastic economic events in recent history and any theory that can
adequately explain its causes (or find remedies to prevent similar occurrences in the future) will attract
attention. Some economists argue that the Great Depression proves that the economy is inherently unstable and
requires a long time to adjust back to a full-employment equilibrium. Others argue that it proves that
government policy is often misguided and that it may be better to rely on market forces to bring us back to an
equilibrium.

2. In the long run no major inflation can persist without rapid money growth, since the inflation rate is equal
to the growth rate of money supply adjusted for the trend in real output and changes in velocity. In the short
run, however, changes in output growth and velocity are quite unpredictable and affect the inflation rate. Such
short-run fluctuations can be caused by supply shocks or policy changes.

3.a. The key question for governments desiring to reduce inflation is how cheaply (in terms of lost output) they

210
want to achieve a desired inflation rate. A gradual strategy attempts a slow and steady return to a low inflation
rate by reducing monetary growth slowly in an attempt to avoid a significant increase in unemployment. This
approach takes a lot longer than the cold-turkey approach that attempts to reduce inflation quickly by
immediately and sharply reducing monetary growth. While inflation and inflationary expectations will be
reduced faster, a higher level of unemployment leading to a decrease in the level of output will result in the
short run until the economy has time to adjust back to the full-employment level of output. Which strategy will
be chosen by policy decision-makers also depends on how fast wages and prices are believed to adjust to their
equilibrium level.

3.b. If policy makers have a credibility problem, inflationary expectations will not adjust downwards quickly
and it will take a lot longer (and probably a much higher increase in the unemployment rate) to reduce
inflation. With a gradual approach, people will be very reluctant to change their inflationary expectations, since
they may not be convinced that the central bank will actually stick to its announced policy objective of
reducing inflation. A cold-turkey approach may be more attractive since it has a credibility bonus, that is, there
is an immediate confirmation that the central bank is committed to reducing inflation. Therefore a new full-
employment equilibrium can be reached more quickly.

4. People often worry about budget deficits, relating them to their own financial situations and inferring that
nobody, not even the government, can live on borrowed money for a long period of time. The real issues
surrounding budget deficits, however, are much more complicated, and most people do not fully comprehend
how large budget deficits actually affect the economy and their daily lives (through increased interest rates
and, possibly, increased inflation or increased future tax rates).

211
In the short run, budget deficits caused by expansionary fiscal policy will stimulate the economy. However, the
increased need of the government to borrow will drive up interest rates and crowd out private spending,
especially investment and net exports. The lower rate of capital accumulation means less future economic
growth. Part of the budget deficit may have been financed from abroad, so foreigners will have to be repaid
(with interest) and this will diminish domestic living standards. Finally, if the Fed is worried about higher
interest rates and decides to monetize the debt, a higher rate of inflation will result. To avoid inflation,
continued high budget deficits eventually require higher taxes or budget-cutting measures to counteract the
increased high interest payments on the debt.
As long as the national debt grows more slowly than GDP, financing the national debt does not create a major
problem. If the debt-income ratio increases, however, the financing of further budget deficits creates
difficulties. Therefore people should be worried less about the size of the deficit or the debt and more about the
debt-income ratio, the way in which government deficits are financed, and the possibility of lower future
economic growth due to a lower rate of capital accumulation.

5. The further the economy is from the full-employment level of output, the more the Fed should be willing
to monetize the deficit. For one, at a time when unemployment is high, inflationary pressure is probably low,
so increasing money supply will not cause rapid price increases. Secondly, a small increase in inflation may be
tolerable since a high level of unemployment is costly in terms of lost output. Therefore it might be desirable
to return to full employment fast by increasing money supply. However, when the economy is close to full
employment, bottlenecks develop more easily. In this situation, monetizing the deficit will ultimately fail to
keep interest rates down or stimulating the economy further. Instead a higher rate of monetary growth will
cause inflation to increase sharply.

6. The ability of the government to raise additional tax revenue through the creation of money (and therefore
inflation) is called seigniorage. Inflation tax revenue is defined as the product of the inflation rate times the real
money base. Inflation acts just like a tax since the government is able to spend more by printing money while
people are forced to spend less since part of their income is used to increase their nominal money holdings.

7.a. During the hyperinflation of 1922-23, the German government financed almost all of its spending through
the creation of money. Naturally, the excessive monetary expansion caused inflation to skyrocket, reaching an
average monthly inflation rate of 322 percent.

7.b. The tax revenue that can be gained through an increase in the inflation rate is defined as:

inflation tax revenue = (inflation rate)*(real money base).


Figure 18-3 shows that it is possible for the government to increase the inflation tax revenue temporarily as
long as money is printed faster than people expect. But excessive monetary growth causes inflation to increase
rapidly and people will start to reduce their money holdings in an attempt to avoid the inflation tax. Eventually,
the monetary base will decline, and the whole process will break down.

Russia was burdened with an huge budget deficit and a large external debt as it tried to transform its centrally
planned economy to a free market economy. Real output decreased substantially and all kinds of bottlenecks
occurred. Much of the economic activity was on the black market, so collecting tax revenues became difficult.
With government revenues far outpacing tax revenues, the government budget got totally out of balance and
hyperinflation resulted as the central bank created money to allow for more government spending. Inflation
peaked at 2,600% in 1993. The ruble totally collapsed in value and the Russian economy is now burdened with
a huge foreign debt that it is unable to service. To bring the budget back into balance, subsidies to loss-making
state enterprises must be cut and tax collection must be strictly enforced. Laws also must be passed to protect
private property and encourage private enterprises, and inflation must be brought under control. Boris Yeltsin,
Russia’s president at the time, introduced a new line of ruble notes in January, 1998, replacing the old 1,000-
ruble bills with the new one-ruble notes in an effort to curtail money growth.

9.a. Interest payments on the national debt can be divided into real payments and payments due to inflation. In
other words, we have to distinguish between real and nominal interest rates. During periods of inflation most
interest payments are offset by the decrease in the real value of the debt. What we should be concerned about,
however, is the interest payments in real terms.

9.b. The national debt is a burden on society primarily because of the negative effect it has on the rate of
capital accumulation. The increased borrowing needs of the government drive up real interest rates, which then
crowds out private spending, especially investment. As a result, future economic growth may be impaired.
High interest rates may also crowd out net exports, which may lead to a loss of competitiveness and a decline
in the manufacturing sector. This will also have a negative effect on future living standards. In addition, the
part of the debt that is held by foreigners will have to be repaid (with interest), creating a tax burden on future
generations. The part of the debt that is financed domestically does not create the same burden, since future tax
increases will be used to pay U.S. citizens. It will, however, have redistributive effects, away from (mostly
lower- and middle-income) taxpayers to (mostly high-income) bondholders.

10. If an amendment to require an annually balanced budget were implemented, the government would no
longer be able to use discretionary fiscal policy as a stabilization tool. In addition, if the amendment called for
annually balancing the actual budget, it could do more harm than good.
Assume that the economy is at full employment and the budget is balanced. If the economy enters a recession,
the cyclical component of the budget surplus will become negative and an actual budget deficit will develop.
To balance the budget, the government can either increase taxes or decrease spending to create a surplus in the
structural component. This fiscal restriction will cause a deeper recession, increasing the cyclical deficit even
more. It is doubtful that the government would actually succeed in balancing the budget. For this reason most
economists do not favor a balanced budget amendment.

11. It is more important to look at the debt-income ratio than at the absolute value of the national debt, since
the debt-income ratio tells us how much of our current income we would have to give up to pay back the debt.
Obviously a $5.6 trillion national debt is much harder to deal with in an economy with a GDP of $5 trillion
than in an economy with a GDP of $10 trillion. Similarly, if the economy grows at a higher rate than the debt,
we have less reason to worry about whether it is possible to service the debt. However, if the national debt
grows faster than GDP over a long time period, we may have to worry about our ability to service the debt.

12. The answer to this question is student specific. There was no easy way to finance the massive increase in
government expenditures that was required to build up the infrastructure in the new Eastern states and to
ensure transfer payments to the large numbers of unemployed and retired people. The German government
struggled with this question and the transition period has not been easy.
The huge amount of funds required to finance these expenditures should not be raised solely by increasing
taxes. Tax increases are not only highly unpopular but may also provide disincentives to work, save and invest
and may thus have a significant negative impact on the performance of the economy. Chancellor Kohl initially
promised that Germany’s re-unification would not be financed through a tax increase, but when the costs of the
re-unification became more apparent, he had to break that promise.
An increase in expenditures that is debt financed leads to higher interest rates, crowding out some private
spending (investment and net export). In addition, it leads to a capital inflow that strengthens the value of the
domestic currency. In Germany, this process created a problem since exchange rates among countries of the EC
were fixed within a relatively small range. The European currency crisis of 1992 was caused by the increase in
the German budget deficit that was largely debt financed. The subsequent inflow of funds from other EC-
countries required massive government intervention to keep exchange rates within their assigned range and
caused problems for other European countries. Eventually these countries had to devalue their currencies. In
addition, Germany experienced a current account deficit because of the high value of the D-Mark relative to
the currencies of non-EC countries. Thus, the situation in Germany in the early 1990s strongly resembled the
situation in the U.S. in the early 1980s, with its high interest rates due to large government borrowing (and
fairly tight monetary policy) and subsequent trade imbalance.
Financing large increases in government expenditures through money creation would serve to keep interest
rates low, which would not only reduce interest payments on the national debt but also help to keep the value
of the D-Mark from rising. However, money creation would also lead to increased inflation. The Bundesbank,
known for its independence and its aversion to inflation, chose to let money supply grow only to a small
degree. As a result, German interest rates rose substantially, leading to an inflow of funds and a weaker
performance of the economy.

13.a. The pay-as-you-go financing of the Social Security system results in a transfer of income from the young
to the old. This is possible for three reasons. A growing population will make financing public pension systems
easier. A high growth in income due to higher productivity will allow retirement benefits to be higher than past
contributions. Finally, since older people are more likely to vote than younger people they can enforce this
intergenerational transfer through the political system.

13.b. The pay-as-you-go financing of Social Security causes a savings replacement effect, that is, the
government does not save Social Security contributions but uses them immediately to pay for the benefits of
those currently retired. This decrease in national saving reduces the rate of capital accumulation and negatively
affects future living standards.

13.c. Current proposals for Social Security reform include the following suggestions: first, allow people to
invest at least part of their funds in private retirement accounts to ensure that they are productively invested;
second, increase payroll taxes, tax retirement benefits, and increase the retirement age to make financing the
system easier.

Technical Problems:
1.a. Period % change in real GNP
1929 - 1930 (285.2 - 314.7)/314.7 = - 9.37%
1930 - 1931 (263.3 - 285.2)/285.2 = - 7.68%
1929 - 1931 (263.3 - 314.7)/314.7 = - 16.33%

1.b. Period % change in real GDP


1990 - 1991 (6,079.0 - 6,138.7)/6,138.7 = - 0.97%

(The numbers of the real GDP in 1990 and 1991 are taken from the Economic Report of the President,
February, 1997. Table 18-1 lists output in terms of GNP, used at that time, and not GDP, which is used now as
a measure of total output.)

1.c. There is no official distinction between a depression and a severe recession, and it is obvious that if the
real GNP had stopped decreasing in 1931 rather than in 1933, the depression would have been much less
severe. From Table 18-1, we see that real GNP continued to decline to $222.1 billion in 1933, an overall
decline of 29.42% (almost double the rate calculated in 1.a.) and that it took until 1939 to return to the 1929
level of real GNP.
While it is always hard to guess "what would have happened if...," it seems accurate to say that if the
government had taken immediate steps to actively stimulate economic growth through expansionary fiscal and
monetary policies, the severity of the Great Depression could have been greatly reduced. In this respect, the
first sentence of Question 1 is correct. However, it should probably be revised to say: "The severity of the
Great Depression could have been greatly reduced if active expansionary government policies had been
employed much sooner."

The actual government surplus went from +1.0% of GNP in 1929 to -3.8% in 1931 and then to -3.1% in 1932
and -2.5% in 1933. However, the full-employment budget surplus (a concept that had not as yet been
developed at the time) went from - 0.8% of potential GNP in 1929 to -3.1% in 1931, -0.9% in 1932 and +1.6%
in 1933. The large size of the actual budget deficits may have led to the incorrect conclusion that fiscal policy
had been too expansionary, when in actuality the size of the budget deficits was due to the decline in GNP. The
existence of a full-employment budget surplus in 1933 indicates that fiscal policy was too restrictive, and this
greatly contributed to the severity of the depression. The full-employment budget surplus provides a much
better indication of whether fiscal policy is expansionary or restrictive than the actual budget surplus.

3. With real output remaining constant, the additional tax revenue gained from inflation is:

inflation tax revenue = (inflation rate)*(real money base).

If the real money base is 10% of GDP and the inflation rate increases from 0% to 10%, we should expect
an increase in government tax revenues of 1%, as long as the real money base remains constant. However, as
inflation increases, people reduce their money holdings and banks reduce their excess reserves since it
becomes more costly to hold money. In countries that have sophisticated banking systems, money holdings
(and therefore the inflation tax revenue) fall to a much larger extent than in countries where there are fewer
alternatives to cash holdings.

4. The inflation-adjusted budget deficit can be calculated in the following way:


inflation-adjusted deficit = total deficit - (inflation rate)*(national debt)

= 4% - (12%)(30%) = 4% - 3.6% = 0.4%,

that is, the inflation-adjusted deficit is 0.4% of GDP.

From the equation MV = PY ==> %M + %V = %P + %Y ==> %P = %M - %Y + %V

==>  = m - y + v.

In other words, inflation () is the difference between M2 growth (m) and economic growth (y) adjusted for
changes in the income velocity of M2 (v). If we subtract the second column (output growth) from the first
column (money growth) in Table 18-9, we get the inflation rate minus the change in the velocity. In other
words, Column 3 (the inflation rate) would only be equal to the difference of the first two columns if velocity
remained constant. The numbers in this table imply that the changes in velocity varied from - 6.3% in 1880-
1889 to + 3.1% in 1920 - 1929. But from 1960 to 1989 the changes in velocity were very close to 0%.
Therefore over the last three decades, the rate of inflation can be explained very well by the difference between
M2 growth and economic growth.

6.a. According to Table 18-8, total government outlays were 18.8% of GDP in the 1960s and 20.7% of GDP in
the 1990s. This is an increase of roughly 10%, which we can calculate in the following way:

(20.7 - 18.8)/18.8 = 0.101064 = 10.1%

6.b. National defense decreased from 8.6% of GDP in the 1960s to 4.1% of GDP in the 1990s. However,
entitlements and other mandatory spending increased from 6.3% to 11.2% of GDP. These are now the largest
component of government outlays.

6.c. Besides the increase in entitlements, which almost doubled over the last three decades, interest payments
on the national debt more than doubled from 1.3% in the 1960s to 3.0% in the 1990s.

7.a. According to Table 18-9, total federal revenues were 17.9% of GDP in the 1960s and 18.6% of GDP in the
1990s. This constitutes an increase of almost 4%, which we can calculate in the following way:

(18.6 - 17.9)/17.9 = 0.03911 = 3.9%

7.b. In the 1960s the corporate income tax was the second largest component of federal revenues with 3.8%,
but it decreased to 1.9% in the 1990s. Social insurance taxes and contributions increased from 3.5% to 6.6%
and are now the second largest component of federal revenues, only slightly behind the 8.4% coming from the
individual income tax.

7.c. Since the corporate income tax and other revenues decreased and the individual income tax increased only
slightly from 7.8% to 8.4%, the increase in tax revenues from the 1960s to the 1990s came primarily from
social insurance taxes and contributions.

8. From the numbers in Tables 18-8 and 18-9 we can calculate the budget deficit (defined as government
outlays minus government revenues) as a percentage of GDP.

1960s: 18.8% - 17.9% = 0.9%


1970s: 20.1% - 18.0% = 2.1%
1980s: 22.3% - 18.4% = 3.9%
1990s: 20.7% - 18.6% = 2.1%

As we can see, federal budget deficits as a percentage of GDP more than quadrupled from the 1960s to the
1980s, but then started to decline again in the 1990s to a bit more than double the percentage in the 1960s.

9. The debt-income ratio is defined as (national debt)/GDP. Therefore if the national debt grows by 5% a year
but GDP grows by only 4% the debt-income ratio will increase.

Additional Problems:

1. In the early 1930s, interest rates were extremely low and the supply of money fell sharply. Does this mean
that the Fed used expansionary or restrictive monetary policy? Explain your answer.

The low interest rates in the 1930s resulted from a sharp decline in the demand for credit due to the sharp
decline in aggregate demand, rather than from expansionary monetary policy. The decrease in money supply
was the result of a sharp decline in the size of the money multiplier (as a result of the many bank failures) and
not the result of restrictive policy by the Fed. The money multiplier fell sharply, since consumers lost
confidence in the banking system and started to hold much more currency, sharply increasing the currency-
deposit ratio. Surviving banks became much more careful and increased their excess reserve holdings,
decreasing the money multiplier even more.

2. "The Great Depression was a direct result of the Fed's inept monetary policy." Comment on this statement
and indicate whether you think that the Fed has learned from past mistakes.

The Keynesian explanation of what caused the Great Depression concentrates on the collapse of investment,
the reduction in consumption, and the decrease in aggregate demand that was further exacerbated by poor
fiscal policy. Monetarists concentrate on the behavior of money, asserting that the Fed failed to prevent the
collapse of the banking system, which led to a sharp decline in the money multiplier. They see the resulting
decline in money supply as the primary cause of the Great Depression. Both explanations seem to fit the facts
and there is no inherent conflict between them. While the Great Depression was not caused by government
actions, its severity could have been greatly reduced if active expansionary policies had been employed by the
government much sooner.
It can be assumed that the Fed has learned from its mistake. For example, after the stock market crash of
October, 1987 (when stock values dropped by more than 24%) the Fed, conscious of what had happened in
1929, immediately assured financial markets that it would provide the liquidity needed to prevent a financial
collapse. The Fed also undertook open market purchases in an effort to drive interest rates down.
The recession in 1981/82 serves as another good example. Although the downturn seems to have been
caused by the Fed's overly restrictive monetary policy, the Fed reversed its course and implemented
expansionary policies as soon as the size of the downturn became clear.

3. Do you think that a downturn in the economy as large as the one we experienced during the Great
Depression could happen again?

The answer to this question is student specific, even though most economists would agree that it could not. The
introduction of the FDIC and other regulatory agencies, and programs such as the Social Security system and
unemployment insurance have brought institutional changes that have increased automatic stability and limited
the impact of even large disturbances. It also can be assumed that policy makers have learned from past
mistakes. In other words, recent history has shown that the government does not sit still if the economy
experiences a large disturbance. The recession of 1981/82 serves a good example. With an unemployment rate
of almost 11%, it was the most severe recession since the Great Depression. It lasted a relatively short time,
however, since expansionary policies were implemented almost immediately after the magnitude of the
downturn became clear.

4. What are the factors that would make an anti-inflation policy less costly in terms of increased
unemployment and subsequent loss of output?

A more gradual approach to reducing inflation is generally less costly than other options because it causes less
unemployment. Such an approach can only be successful, however, if it has a high degree of credibility. It must
be announced in advance by policy makers whose past records indicate that they are willing to adhere to an
announced policy. The greater the relative importance of the expected future rate of inflation in determining
the current rate of inflation is, and the more wage and price flexibility there is, the more successful the anti-
inflation policy would be.
5. “Higher monetary growth will be followed by higher wage demands.” Comment.

The equation %P = %M - %Y + %V implies that in the long run the rate of inflation (%P) is
determined by the growth rate of money supply (%M) adjusted for the growth rate of income (%Y) and
changes in velocity (%V). In addition, from the equation w = W/P, that is, real wages equal nominal wages
divided by the price level, we can calculate that (%w) = (%W) - (%P). This can be reformulated into the
equation (%P) = (%W) - (%MPN), assuming that w = MPN in a competitive labor market. Thus we can
see that inflation increases if nominal wages increase more than labor productivity. But workers base their
wage demands on their inflationary expectations and these are largely based on the growth rate of money
supply. Therefore an increase in money supply tends to be followed by higher nominal wages.

"An increase in the growth rate of money supply by 2% will lead to an increase in interest rates of 2%."
Comment on this statement.

From the equation MV = PY, it follows that %P = %M - %Y + %V, or  = m - y + v. Thus we can
conclude that, in the long run, the rate of inflation (%P = ) will go up by 2% if the rate of money growth
(%M = m) is increased by 2%. This assumes that economic growth (%Y = y) and the growth of velocity
(%V = v) do not change. According to the Fisher equation, in = r + , the nominal interest rate (in) is equal to
the real interest rate (r) plus the rate of inflation (). If the rate of inflation goes up by 2% nominal interest
rates will do so as well in the long run. However, in the short run, the increase in inflation and interest rates
will be less than 2%, since the short-run AS-curve is upward sloping and remains fixed.

7. "Hyperinflation requires a cold-turkey approach." Comment on this statement.


An economy experiencing hyperinflation needs to implement drastic measures to reduce inflationary
expectations enough to stabilize at a lower rate of inflation. In the 1980s the governments of Israel and Bolivia
used the cold-turkey approach and succeeded in ending periods of hyperinflation. Israel used wage and price
controls to avoid a large increase in the unemployment rate, but supplemented them with sharp budget cuts and
credit rationing. Bolivia sharply reduced its budget deficit, curtailed monetary growth, stabilized exchange
rates, and stopped external debt service. As a result, the rate of inflation in these countries declined rapidly and
sharply.

8. "A cold-turkey approach is a better policy for fighting inflation than a gradualist approach, since it
requires a shorter time to establish a long-run equilibrium at a desired lower inflation rate." Comment on this
statement.

The gradualist approach lowers money growth over a long period of time to minimize the severity of the
resulting recession. The cold-turkey approach achieves the reduction in inflation at the cost of a significant
increase in short-run unemployment. Normative considerations determine the relative merits of the two
approaches. The cold-turkey approach may benefit from a credibility bonus, since workers and firms do not
have to guess whether the government is really committed to lowering the rate of inflation. The public may
therefore reduce their inflationary expectations faster and the economy may adjust back to full-employment
much faster. However, if long-term contracts exist, wages and prices cannot adjust quickly and a rapid return to
a low-inflation equilibrium at full-employment is fairly unlikely.

9. "The most important factor in the Fed's fight against inflation is credibility." Comment on this statement
and explain the concept of time inconsistency.

If a policy measure designed to reduce inflation has credibility, then labor unions are more likely to adjust their
inflationary expectations and their wage demands downwards in contract negotiations. The likely outcome will
be low inflation and low unemployment. Restrictive monetary policy will shift the AD-curve to the left,
lowering inflation but increasing unemployment. If inflationary expectations, wages, and prices adjust quickly,
then the short-run AS-curve will shift quickly to the right, and the economy will return rapidly to the full-
employment level of output at a lower inflation rate. The Fed's credibility is an important factor in this process,
but so are long-term contracts. If long-term wage contracts exist, then wages cannot adjust quickly to their
market-clearing level, and it will take a longer time to return to the full-employment level of output.
While the credibility of policy makers is very important in the fight against inflation, it is not easily
earned. If the government has not consistently adhered to its announced policies in the past, it will encounter
the problem of time inconsistency. For example, if labor unions settle for lower wage increases, the Fed may
be tempted to create additional jobs by increasing monetary growth, knowing that wages are locked in for the
time being. The outcome is lower unemployment but a higher than expected rate of inflation, and workers' real
wages will be lower than expected. If policy makers have shown a tendency to abandon their announced anti-
inflation policy after wages have been settled, any future policy announcement will suffer from time
inconsistency. In that case, labor unions will always expect faster monetary growth and demand higher wages
in upcoming wage negotiations regardless of what the Fed announces. As a result, inflation will not be
reduced.

10. "There is a limit to how much additional tax revenue the government can create through inflation."
Comment on this statement.

The financing of government spending through the creation of high-powered money is an alternative to explicit
taxation. The inflation tax revenue is defined as follows:

inflation tax revenue = (inflation rate)*(real money base).

To maintain the purchasing power of their real balances when prices are rising, people must hold increasing
amounts of nominal money balances. Inflation acts just like a tax since the government is able to spend more
by printing money while people are forced to spend less and pay more to the government in exchange for extra
money. But with increased inflation, holding money becomes more costly, and there is a limit to how much
revenue the government can raise through this inflation tax. People reduce their real currency holdings and
banks hold as little in excess reserves as possible. Eventually the real money base falls so much that the
government's inflation tax revenue begins to decrease again.

11. “The rational expectations approach and the monetarist approach both assert that expansionary monetary
policy simply increases inflation without any significant effect on output or the unemployment rate.” Comment
on this statement.

The rational expectations approach asserts that all individuals and firms have access to all the necessary
information and consistently make optimal decisions based on rationally formed expectations. Wages and
prices are assumed to be flexible, so markets always clear rapidly. When a policy change is anticipated, people
will always try to adjust to the long-run outcome of that policy. This implies that any announced expansionary
monetary policy will immediately be reflected in a higher inflation rate without any significant change in the
rate of unemployment. Only unanticipated monetary policy changes will affect output and unemployment in
the short run.
Monetarists believe that monetary policy will have a short-run effect on output and the rate of
unemployment, even if it is anticipated. Monetary restriction will shift the AD-curve to the left and it will take
some unemployment to create downwards pressure on wages before the economy can adjust back to the full-
employment level of output (by shifting the short-run AS-curve to the right).

12. Monetarists and the rational expectations school share the belief that government intervention usually
makes things worse and that the Fed should adopt a monetary growth rule. But they disagree on how the
economy reacts to monetary policy changes. In what way?

Both monetarists and the rational expectations school believe that credibility is very important in the fight
against inflation and that credibility can best be established through policy rules. Thus they prefer a monetary
growth rule to discretionary policies. However, they disagree on how the economy adjusts to a change in
money supply growth. Monetarists believe that the economy reacts slowly to any policy change, with long and
variable lags. Since markets do not clear rapidly, an increase in unemployment will follow a significant
reduction in money supply growth. The rational expectations school, however, believes that markets do clear
rapidly, and that inflation can therefore be reduced fairly rapidly and without great costs in terms of higher
unemployment.

13. "Inflation taxes money holders." Comment on this statement.

The financing of government spending through the creation of money (called seigniorage) is an alternative to
taxation. The government can obtain additional resources by printing more money. This, however, will cause
inflation to rise. To maintain the purchasing power of their real balances when prices are rising, people must
hold increasing amounts of nominal money balances. The public is forced to spend less of its income and pays
the difference to the government in exchange for extra money. In this situation the government is said to be
financing itself through the inflation tax, that is, it can spend more while the public spends less, just as if the
government had raised taxes.

14. “Higher budget deficits cause more money growth and therefore an increase in the inflation rate.”
Comment on this statement. In your answer briefly discuss the dilemma that the Fed faces when the
government runs continuous large budget deficits.

First we have to ask whether the increase in the budget deficit is cyclical or structural. An increase in the
cyclical deficit occurs if the economy goes into a recession. In this case we should not expect any increase in
inflation. An increase in the structural deficit occurs due to expansionary fiscal policy. In this case, there may
be some temporary inflationary pressure. However, fiscal policy is not inflationary in the long run, since there
is no change in money supply. Since interest rates increase, private spending (investment and/or net exports) is
crowded out. The Fed may decide to intervene and money finance the deficit, that is, decrease interest rates
through open market purchases. In this case, we have an increase in money supply and inflation. Some people
claim that debt financing may actually be more inflationary than money financing in the long run, since higher
interest rates lead to higher interest payments on the national debt. This adds to the budget deficit, and
eventually a higher national debt needs to be money financed and then we can expect more inflation.
When the government consistently runs large budget deficits, the Fed faces a problem. It can opt to
accommodate the expansionary fiscal policy by lowering interest rates by monetizing the debt, and as a result
be blamed for the resulting inflation. It can also opt not to accommodate the expansionary fiscal policy, but
then be blamed for the resulting high interest rates, which are likely to crowd out investment and net exports.

15. “An increase in government spending financed by borrowing from the public will increase the supply of
money.” Comment on this statement.

When the government borrows from the public, the public gives the government money in exchange for bonds.
If the government spends this money right away on programs, money supply is not affected at all. This is
simply a fiscal policy measure and does not involve the Fed. However, if the Fed follows a policy of pegging
nominal interest rates to a certain fixed rate, then open market purchases will be undertaken any time the
government increases its borrowing from the public. The government's increased borrowing needs will put
upward pressure on interest rates, but the Fed's open market purchases will increase bank reserves and keep
interest rates from rising. In the U.S., the Treasury does not directly borrow from the Fed and thus fiscal and
monetary policy are largely independent. But there are some countries where the government borrows heavily
directly from the central bank through the printing of new money. Under these circumstances, monetary and
fiscal policy are much more closely linked.

16. If the federal government runs a budget surplus rather than a deficit, how will the public’s bond holdings
and the supply of money be affected?

Any time the federal government has an excess of tax revenues over outlays, a budget surplus results. If the
government uses the surplus to retire some of the existing national debt by repaying government securities that
are coming due, bond holdings by the public will decrease. But money supply will not change, unless the Fed
wants high-powered money to change in accordance with the budget surplus. Assume that the budget surplus
accumulates on the Tax and Loan Account (the receiving account of the Treasury). The Treasury can transfer
the funds from the Tax and Loan Account to its account at the Fed (the payment account), which will cause
bank reserves to decline. If these funds are immediately used to retire government securities, bank reserves
will increase back to their original level, and high-powered money will not be affected. However, the Fed can
always decide on its own to undertake open market operations to change bank reserves and thus high-powered
money.

17. Discuss the relative merits of tax financing versus debt financing of government programs as far as
efficiency and equity is concerned.

If government spending is financed by a tax increase, the increased income tax rate may provide a disincentive
to work. If any other form of taxation is used, some other misallocation of resources will occur. If spending
increases are financed through borrowing, the increased demand for funds will lead to upward pressure on
interest rates, which will affect interest sensitive sectors in the economy. Expansionary fiscal policy will lead to
the crowding out of some private spending. The most interest sensitive sectors in the economy, such as
agriculture, the construction industry, or banking are affected to a larger degree than the less interest sensitive
sectors. Thus deficit financing also leads to a misallocation of resources. Since investment is negatively
affected by the increase in the cost of capital, the rate of capital accumulation will decrease and future
economic growth will be slower.
There is also the question of equity, since tax financing affects current taxpayers, whereas debt
financing affects future taxpayers. In addition, it is important to consider who will benefit from the programs
that are financed.

18. "The best way to cut the size of the U.S. budget is to sharply reduce defense spending, since the U.S.
already is spending too much on its military." Comment on this statement.

The answer to this question is student specific. Many students believe that the U.S. currently spends too much
on the military. However, this is a value judgment and a case can be made for cuts in other spending as well.
Few students realize that defense spending as a fraction of GDP has decreased over the last four decades while
spending for social programs and interest payments on the debt has increased. Therefore it is unlikely that a
reduction in the size of the budget can be achieved successfully without cuts in entitlement programs.

19. Assume the government increases spending financed by issuing bonds. Does this create a burden on
future generations? Why or why not? Would it make a difference whether the spending increase was used to
finance the building of a new highway or to send troops abroad to avoid social unrest in another country?

If an increase in government spending is financed through the sale of government bonds, the national debt
increases. The increased borrowing needs of the government may drive interest rates up and this may crowd
out private spending (investment and/or net exports). If the rate of capital accumulation is negatively affected,
it will lower future living standards and create a burden on future generations. If the funds are used to send
troops abroad, future generations would most likely suffer more than if the same funds are used to build a
highway. Building a highway creates a long-lasting government asset for future generations. This can be
considered “government investment.’ However, if the funds are used to send troops abroad, then no real capital
will be created and this can be considered “government consumption.”

20. “The debt-income ratio has been steadily declining since World War II, when it was at an all-time high of
over 100%.” Comment on this statement.

The debt-income ratio is the size of the national debt divided by the size of nominal GDP. It is true that the
debt-income ratio was at an all-time high of over 100% at the end of World War II and significantly decreased
afterwards, since the economy was growing faster than the debt. However, as Figure 18-5 shows, the debt-
income ratio increased again in the 1980s as the size of the national debt increased at a faster pace than GDP.
In the 1990s this trend reversed as the economy grew stronger and budget surpluses developed. The (gross)
debt-income ratio was about 58% in 1999.

21. "The federal budget needs to be balanced every year so the national debt does not grow any larger."
Comment on this statement.

Balancing the federal budget annually does not make much sense, since the government would have to give up
its ability to use fiscal stabilization policy effectively. If the economy were to go into a recession, a budget
deficit would develop. To balance the budget, a cut in government spending or a tax increase would have to be
implemented. But such measures would lead to a deepening of the recession, making balancing the budget
even more difficult.

22. "Budget deficits cause money growth and thus inflation." Comment on this statement.

There is no automatic link between budget deficits and the monetary base, since the Fed determines
independently how much high-powered money to create. In other words, the Fed is not required to
accommodate expansionary fiscal policy. However, if the Fed is committed to maintaining constant nominal
interest rates on government bonds, an increase in the deficit will result in open market purchases and an
increase in the monetary base. If the Fed instead emphasizes monetary growth targets, there is no automatic
link between increases in the deficit and increases in the monetary base. However, should high deficits persist,
the debt burden and interest payments may become unsustainable and the government may no longer be able to
finance the debt by borrowing from the public. In such a case, the Fed is forced to create money, which will
increase the inflation rate.

23. "The Fed can lower the federal budget deficit through open market purchases." Comment on this
statement. In your answer discuss whether money or debt financing is more inflationary.

Open market purchases increase bank reserves and therefore money supply. This lowers interest rates, leading
to a higher level of investment and income. Since income tax revenues increase in a boom, the budget deficit
will decrease. Since interest rates are lower, the interest payments on the national debt are lower. But if the
debt is money financed, inflation will increase due to higher money growth and this may lead the economy into
a wage-price spiral. If the deficit is debt financed, money growth and inflation will not change. But some
people argue that increased interest rates increase the interest payments on the national debt, which increase
annual deficits. Eventually, the national debt has to be money financed anyway, but since the debt will now be
much larger, debt financing may actually be more inflationary than money financing in the long run.

24. What are the implications for national saving, interest rates, and future living standards of moving from
federal budget deficits to budget surpluses?

Moving from a federal budget deficit to a budget surplus means an increase in national saving. This will lead
to lower interest rates, which will stimulate investment. A higher level of investment will lead to more future
economic growth and a higher future living standard. Lower interest rates will also cause an outflow of some
funds and this will decrease the value of the dollar, making U.S. goods more competitive abroad. The budget
surpluses can be used to pay off some of the national debt, leading to lower interest payments. Highly interest
sensitive sectors will be affected more by the decrease in interest rates than other sectors. The assumption
made here is that budget surpluses will not decrease the current level of economic activity. If restrictive fiscal
policy caused the budget surpluses and the fiscal restriction had a negative effect on the economy, then saving
and investment would decrease and future living standards might not increase.

25. Briefly state the advantages and disadvantages of the Social Security system as it is currently structured.
Would you revise the system if you could? If so, how?

The answer to this question is student specific. The advantages of the system include that it provides a
guaranteed retirement income for everyone and helps reduce poverty among the elderly. However, there is
evidence that the system has reduced national saving and therefore the rate of capital accumulation, since it is
financed on a pay-as-you-go basis. The system also may induce people to retire earlier than they would
otherwise. Since there is an increasing segment of elderly in the population the tax burden on the young to
finance retirement benefits for the old is increasing. Suggestions for revisions might include: Increasing the
retirement age, taxing Social Security benefits above a certain income level, providing more incentives for
private retirement saving, allowing part of the Social Security trust fund to be invested in corporate bonds and
stocks, and allowing people to take care of their own private retirement savings accounts.

“Social Security benefits for the elderly should not be based on past earnings. Instead everyone who retires
should get the same benefits.” Comment on this statement.

The system described above would be considered unfair by those who feel that people should get from the
system what they paid in. For many the rate of return on the taxes paid would change drastically, and this could
change their savings behavior. However, since the system would be better understood by people, financial
planning for retirement would become easier. In addition, the system would change from a social insurance
program to an income redistribution program.

27. Do you believe that the Social Security system lowers the U.S. savings rate? Why or why not? In your
answer, give your opinion whether the government should change the Social Security system. How would you
address some of the concerns you may have about the system?

Social Security is financed on a pay-as-you-go basis, that is, payroll tax revenues collected from current
workers are immediately used to pay for benefits for retirees. This means that private saving isn’t replaced by
government saving. This savings replacement effect reduces national saving. However, the system may induce
people to retire earlier, and the life-cycle hypothesis suggests that people will save more during fewer working
years to provide adequate funds for a longer retirement period (the induced retirement effect). The recognition
effect argues that the pure existence of the system makes people realize that they need funds for retirement.
The goal feasibility effect states that people realize that they can achieve a certain goal (funds for retirement or
bequest) more easily if they supplement their Social Security benefits with additional savings. The desire to
bequest funds to children and the uncertainties about the future of the system may induce people to save more
(the bequest effect). Thus the system may actually encourage saving. Some economists argue that the system
simply replaced the private inter-generational income transfers that took place previously, so there should not
be any overall effect on savings at all. The issue has to be settled by empirical studies. Most studies show a
negative the effect of Social Security on national saving, yet no conclusive result about the magnitude of the
effect on has been established.
The need to maintain Social Security as it exists can be defended with the argument that the
government should provide a safety net for the elderly. Some people may not have the foresight to save enough
for retirement or they may not be able to do so because of financial constraints during their working years.
They also may misjudge their life expectancy. Therefore there always will be some elderly in need of
government support. The Social Security system forces people to save for retirement.
Many people believe that Social Security causes a reduced rate of capital accumulation, since
the system is financed on a pay-as-you-go basis. Thus the following changes to the system are often suggested:

Increase the retirement age and abolish the earnings test.


Impose eligibility requirements that take into account earnings from assets.
Increase incentives for private savings to supplement retirement savings.
Build up the Social Security Trust Fund by increasing payroll taxes without an equivalent increase in benefits.
Privatize the Social Security system (either totally or partially) and allow funds to be invested in the stock
market.

CHAPTER 19

INTERNATIONAL ADJUSTMENT AND INTERDEPENDENCE

Solutions to the Problems in the Textbook:

Conceptual Problems:

1.a. A loss of export markets leads not only to an external deficit but also to a decrease in the level of output.
Since a policy dilemma between an external and an internal balance exists, a combination of expenditure-
switching and expenditure-raising policies is required. The levying of tariffs helps to achieve an external
balance, while an increase in government spending helps to achieve an internal balance.

1.b. A reduction in saving and an increase in the demand for domestic goods will lead to an increase in national
income. As a result, imports will increase, leading to a balance of payments deficit. This is not a dilemma
situation and an expenditure-reducing policy (a cut in spending) will remedy the situation.

1.c. An increase in government spending will lead to an increase in national income and the level of imports.
This will cause a balance of payments deficit. This is not a dilemma situation and an expenditure-reducing
policy (a cut in spending) will suffice to reestablish an internal and external balance.

1.d. A shift in the demand from imports to domestic goods will increase output and lead to a trade surplus. This
is a dilemma situation and a combination of expenditure-switching and expenditure-reducing policies should
be employed. Lowering tariffs combined with a cut in government spending will help to achieve an external
and internal balance.

1.e. A reduction in imports with a corresponding increase in saving will not affect the level of national income
but it will lead to a trade surplus. An appropriate response is a reduction in income taxes combined with lower
tariffs. This will help to reduce the trade surplus without affecting national income.
2. Temporary trade imbalances can be easily financed, since the central bank is not likely to run out of foreign
currency reserves rapidly. A country that has a temporary payments imbalance may even borrow foreign
currency from abroad as long as the level of domestic investment spending is sufficient to increase the level of
output, some of which will eventually be exported. A temporary disturbance in trade patterns can come from a
change in exchange rates caused by unstable exchange rate expectations.
If a country suffers a permanent imbalance, however, the central bank will eventually run out of foreign
currency reserves. A permanent imbalance may be caused by a loss in foreign market share. The adjustment
can happen through the automatic adjustment process, but that could involve long periods of high
unemployment. Therefore domestic policy changes are required to move the economy more rapidly toward
balance.

3. The answer to this question is student specific. Government intervention can help smooth out temporary
fluctuations in exchange rates. However, this is often problematic since at the time a currency appreciation
occurs, the government can never be sure whether a disturbance is transitory and soon likely to reverse itself or
more persistent and fundamental in nature. If unstable exchange rate expectations lead to capital flows that
cause exchange rate movements and undesired changes in domestic output, central bank intervention seems
justified. But central banks also often intervene to try to reduce inflation by preventing import prices from
rising. The cost of reducing inflation in this way is a steady loss in competitiveness, which may even result in a
foreign exchange crisis. Ultimately, inflation has to be stopped by restrictive monetary policy. Central banks
may also intervene to affect trade flows and to bring about desired changes in domestic output. In the long run,
however, governments are better off not opposing strong market forces by intervening in the foreign exchange
market. If a government really wants to bring about change, it should employ domestic policies. It may
supplement its policy by intervening in the foreign exchange market as well to emphasize its intent.
Government intervention is more effective if it is backed up by credible policies. There is evidence that non-
sterilized intervention affects exchange rates more successfully, since there is a change in money supply equal
to the amount of the intervention. But if the intervention is sterilized, domestic money supply will remain
unchanged and even a massive intervention may not affect the exchange rate to the desired degree.

4. A wage-price spiral occurs when price increases feed back into higher wage demands and the higher cost of
production (due to higher wages) is then passed on to consumers in the form of still higher prices. For
example, devaluation of the domestic currency will increase import prices and therefore the general price level.
To make up for the loss in purchasing power, labor unions may negotiate higher nominal wages. Firms then
pass the cost on to consumers in the form of higher prices. If the government is committed to full-employment
and the central bank continues to increase money supply, the wage-price spiral will continue. In this case, we
will encounter high inflation, but relative prices will not change, and the nominal devaluation ultimately will
have no effect on the real exchange rate. To avoid inflation, the central bank has to stop increasing money
supply. In this case, higher wages will cause layoffs and thus unemployment, which puts downward pressure
on wages and prices to end the spiral.

5. In a target-zone arrangement central banks agree to limit the range of fluctuation of exchange rates. Such
an arrangement requires coordination of fiscal and monetary policies among the governments of different
countries. This can be a problem, since some countries may be unwilling to give up their ability to conduct
domestic economic policies to achieve other goals. On the other hand, target-zone arrangements are attractive
since they assure businesses that prices quoted in foreign currency will not fluctuate significantly, which
simplifies cross-border trade.
6. In determining exchange rates we assume that capital is perfectly mobile, that is, that capital markets are
sufficiently integrated. This implies that interest rates across countries have a tendency to adjust to the same
level, since otherwise international capital flows would take place in response to interest rate differentials. This
assumption is incorrect, however, since capital flows do affect exchange rates, and therefore we must integrate
expectations about exchange rate changes into the model. We must also take into consideration tax differences
among countries, the restrictions some countries put on the transfer of assets, and the possibility of political
instability in some countries. These factors may explain the interest differentials that we observe and the
reasons capital markets are not be sufficiently integrated.

Equation (7) states that

inflation differential  interest rate differential  depreciation rate.

This is only an approximate, long-term relation since exchange rates can move independently of prices, and
interest rate differentials may exist due to obstacles to the international flow of capital.

8. In a system of freely floating exchange rates, domestic monetary policy can be used effectively, whereas
in a system of fixed exchange rates, monetary policy is completely ineffective. A freely floating exchange rate
system is less crisis prone since exchange rates are determined solely by market forces. It therefore provides a
better framework for macroeconomic stability. However, spillover effects make macroeconomic stabilization
policy more difficult in a system of freely floating exchange rates. These spillover (or interdependence) effects
can lead to a loss of competitiveness among other problems. There is therefore good reason for countries to
coordinate their policies.

9. Central banks often intervene to affect exchange rates in the belief that they can affect trade flows and
prevent import prices from rising. They may also believe that exchange rate fluctuations have occurred simply
due to erratic changes in expectations. However, it is often unclear whether exchange rate movements are
transitory or the beginning of a new trend with strong market forces at work. This makes intervention
problematic. While non-sterilized intervention will affect exchange rates because it will affect the domestic
money supply, it is not clear that sterilized intervention will have the desired effects.
10. It appears that spillover (or interdependence) effects are much stronger under a system of floating exchange
rates. Therefore it can be argued that macroeconomic stabilization policy can be just as difficult under a system
of freely floating exchange rates as under a system of fixed exchange rates.

Technical Problems:

1. The imposition of a tariff raises the relative price of imports and increases the demand for domestic goods.
This increases the level of domestic output, causing interest rates to rise. Higher domestic interest rates will
lead to an inflow of funds and the domestic currency will begin to appreciate. The higher value of the domestic
currency will lower the relative price of imports again. With perfect capital mobility, the currency appreciation
will progress to the point where the overall change in net exports is zero. In the end, the levels of output and
the interest rate will be back at their original levels.

2. Assume a $10 billion balance of payments deficit occurs:


The central bank's balance sheet before sterilization:

Assets Liabilities
foreign exchange - 10 member bank deposits - 10
other reserves 0 currency 0

monetary base - 10 monetary base - 10

The central bank's balance sheet after sterilization:

Assets Liabilities
foreign exchange - 10 member bank deposits 0
govt. securities + 10 currency 0

monetary base 0 monetary base 0

3.a.
i
ISo LMo

i1 BB' = 0

io BB = 0

0
Y* Y
If the home country is initially in an internal and external balance, the IS-, LM- and BB-curves all
intersect at the full-employment level of national income Y* (see the graph below). If the foreign interest rate
increases, funds will flow out, leading to a balance of payments deficit. To restore external balance, interest
rates must increase or the level of national income must decrease. In other words, the BB-curve must shift up.

3.b. A reduction in money supply combined with an increase in government spending will increase the interest
rate and help to restore an external and internal balance. This policy mix will shift the LM-curve to the left and
the IS-curve to the right. A new equilibrium can be reached at the full-employment level of income but at a
higher interest rate. In other words the IS'-, LM'- and BB'-curves will all intersect again at Y*.

i IS' LM'
ISo LMo

i1 BB' = 0

io BB = 0
0
Y* Y

3.c. If the government takes no action, then an automatic adjustment process will take place through changes in
money supply and prices. The balance of payments deficit will lead to a reduction in money supply (a shift in
the LM-curve to the left). Assuming that prices are flexible, the price level will decrease due to the reduction in
aggregate demand as a result of the decrease in money supply. The lower domestic price level will increase
competitiveness in global markets, and net exports will increase (a shift of the IS-curve to the right). The
decrease in the price level will also increase real money balances so the shift of the LM-curve to the left will be
partially reversed. A new long-run equilibrium will eventually be established at the full-employment level of
output Y*.

4. A country experiencing a permanent increase in its exports will develop a trade surplus. Under the
assumption that the central bank does not undertake sterilization operations, the increase in net exports
combined with the increase in money supply will lead to an increase in aggregate demand. This will result in
an increase in domestic prices, making import goods relatively less expensive and export goods relatively more
expensive. Eventually, this will lead to a decrease in net exports. As long as the surplus exists, domestic prices
will continue to rise until an internal and external balance is reestablished. In the long run, over-employment
will also cause upward pressure on wages and prices, causing the upward-sloping aggregate supply curve to
shift to the left.

5. After a currency depreciation, exports should rise and imports should decline, both in the short run and in
the long run. However, empirical evidence suggests that in the short run the volume effects are too small to
overcome the price effect. Only in the long run, when consumers and producers have had time to adjust to the
change in relative prices, will the volume effects be large enough to overcome the price effect.

6. An increase in money supply will shift the LM-curve to the right, driving the domestic interest rate below
the foreign interest rate. A capital outflow will occur and the domestic currency will start to depreciate. As a
result, exports will rise, imports will decline, and the IS-curve will shift to the right due to the increase in net
exports (NX). The level of output demanded and the domestic interest rate will rise until they reach the level of
the foreign interest rate. However, the new level of output demanded will be above the full-employment level
and this will cause upward pressure on prices. The rising price level will reduce real money balances and the
LM-curve will start moving back to the left. Since the domestic interest rate will be above the foreign interest
rate, funds will flow in and the domestic currency will appreciate. As a result, imports will rise and exports will
decline. The IS-curve will now shift back to the left, due to this decrease in net exports (NX). The level of
output demanded will decline again, and the domestic interest rate will decrease until it reaches the level of the
foreign interest rate. In the end, the level of output demanded will again be at the full-employment level.

i IS1 LMo(po) = LM1(p1)


ISo
LM1(po)
4
i1 1=5 3
io

0
Yo Y2 Y4 Y3 Y

7. Newspapers such as the Wall Street Journal publish foreign exchange rates daily. Current editions may
provide entries similar to the ones below:

Britain Canada Japan


(Pound) (Dollar) (Yen)

Spot 1.5172 .6759 .009432


Forward
30 days 1.5181 .6764 .009489
90 days 1.5160 .6774 .009590
180 days 1.5230 .6791 .009752

Note that these are the costs of one unit of foreign currency in U.S. dollars. Newspapers also generally list how
much of the foreign currency can be purchased for one U.S. dollar. For example, the spot price for one British
pound is $1.5172. This is equivalent to saying that one U.S. dollar will purchase 0.6591 pounds< that is,
1/1.5172.
According to the interest rate parity theorem, the difference between the forward and spot exchange rates (in
percentage terms) should be roughly equal to the difference between domestic and foreign interest rates for
assets that have the same maturity as indicated in the forward contract. The forward exchange rates can be
thought of as the market's best prediction of future spot rates on the dates when the future contracts mature.

Additional Problems:

1. "A trade imbalance can persist as long as the central bank wants it to." Comment.

Under a system of fixed exchange rates, the central bank of a country with a balance of payments deficit has to
sell foreign currency. This will reduce high-powered money and therefore domestic money supply, which will
result in a recession, a reduction in domestic prices, and an improvement in the trade balance. This adjustment
process can only be suspended through sterilization (the central bank offsets the decrease in high-powered
money through simultaneous open market purchases). A persistent balance of payments deficit can only exist if
the central bank actively keeps money supply above the level that would ensure an external balance. Under a
system of flexible exchange rates, a central bank can maintain an imbalance through intervention in the foreign
exchange market.

2. "A balance of payments surplus may lead to inflation, while a balance of payments deficit may lead to
unemployment." Comment on this statement.

A country that has a large balance of payments deficit (surplus) may intervene in the foreign exchange market
by selling (buying) foreign currency. Unless the intervention is sterilized, this will lead to a reduction
(increase) in the monetary base and thus money supply. But a reduction (increase) in money supply may
ultimately cause a recession (increase in inflation).

3. Should a country ever borrow from abroad to finance a balance of trade deficit? Explain.

When deciding whether to finance a balance of trade deficit by borrowing from abroad, it is important to know
if the imbalance appears to be temporary or permanent. A temporary imbalance can be financed by borrowing
from abroad, if a country feels confident that it will be able to repay its debt. As long as there is sufficient
domestic investment to produce and export additional output, there should be relatively little concern about
borrowing from abroad.
However, a permanent imbalance cannot be financed by borrowing from abroad, since a country cannot
indefinitely spend more than its income. When a country runs a balance of payments deficit, the demand for
foreign exchange is by definition larger than the amount being supplied, and the central bank has to sell the
difference. Unless the central bank sterilizes its foreign exchange market intervention, this will result in a
reduction in high-powered money and thus money supply. This may ultimately lead the domestic economy into
a recession with high unemployment. To avoid this situation, the government may instead decide on a
combination of expenditure-reducing and expenditure-switching policies.
4. In the early 1980s the U.S. trade deficit increased sharply, yet at the same time the value of the U.S. dollar
increased steadily. Explain this fact.

One should expect that an increase in the U.S. trade deficit will lead to a depreciation in the value of the dollar.
A trade deficit arises when Americans spend more on foreign goods and services than foreigners spend on
American goods and services. As a result, there is an excess supply of U.S. dollars on the foreign exchange
market, which should lead to a fall in the value of the U.S. dollar.
However, in the early 1980s, the U.S. had large federal budget deficits and the increased borrowing needs
of the government put upward pressure on U.S. interest rates. A large inflow of foreign funds resulted, so the
demand for dollars on the foreign exchange market increased sharply. This caused the U.S. dollar to appreciate.
Since the upward pressure on the value of the dollar was greater than the downward pressure on the dollar
caused by the increase in the trade deficit, the dollar appreciated significantly from 1981 to 1985.

5. True or False? Why?


"If a country has a balance of payments deficit and its central bank wants to manipulate exchange rates, it can
sterilize the intervention by undertaking open market sales of bonds."

False. The central bank of a country with a balance of payments deficit may intervene in foreign exchange
markets by selling some of its foreign currency holdings. The resulting reduction in the amount of high-
powered money can only be offset by an open market purchase of bonds (not sales). Such sterilized
intervention serves to maintain a constant level of domestic money supply.

6. True or False? Why?


"A country that has high unemployment and a balance of payments surplus needs a combination of
expenditure-reducing and expenditure-switching policies."

False. There is no policy conflict when the economy is faced with unemployment and a surplus in the balance
of payments. Expansionary fiscal policy will reduce unemployment and increase the interest rate. Higher
domestic interest rates will attract funds from abroad, which will result in an appreciation of the home currency
and a reduction in the payments surplus. A country that has high unemployment and a balance of payments
surplus need only employ an expenditure-increasing policy to bring the economy back to a situation of internal
and external balance.

7. Under a fixed exchange rate system there is a direct link between the balance of payments and the
domestic money supply. Does such a direct link also exist under a system of freely floating exchange rates?

Under a system of freely floating exchange rates, there is no direct link between the balance of payments and
domestic money supply, since there are no changes in international reserves to affect the monetary base (high-
powered money). Only if the central bank decides to manipulate exchange rates by actively intervening in
foreign exchange markets will the monetary base and money supply be affected. But a central bank always has
the option to sterilize its intervention in foreign exchange markets through appropriate open market operations,
leaving money supply unchanged.

8. "The increases in U.S. trade deficits in the early 1980s were a result of expansionary fiscal policy by the
Reagan administration and restrictive monetary policy by the Fed under Chairman Volcker." Comment on this
statement.

Large federal budget deficits caused by the Reagan tax cuts in the early 1980s increased the borrowing needs
of the U.S. government and provided upward pressure on U.S. interest rates. The restrictive monetary policy of
the Fed made interest rates go even higher, resulting in a large capital inflow from abroad. The increased
demand for U.S. dollars on the foreign exchange market lead to an appreciation of the U.S. dollar. As U.S.
goods became less price competitive on world markets, a trade imbalance developed. Starting in 1986, the
dollar began to depreciate again as U.S. interest rates declined significantly and the U.S. trade deficit increased
sharply.

9. "Germany experienced problems in the early 1990s similar to those in the United States in the early
1980s." Comment on this statement.

In the early 1990s Germany needed a large increase in government spending to build up the infrastructure in
the new Eastern states and the budget deficit increased substantially. Since the German Bundesbank tried to
keep money supply fairly constant, interest rates rose sharply, leading to an inflow of capital. This put upward
pressure on the value of the D-Mark, making German goods less competitive in world markets. As a result,
Germany's trade surplus with other (non-EC) countries decreased substantially. This was very similar to the
scenario in the U.S. in the early 1980s, when a mix of expansionary fiscal and restrictive monetary policy led
to high U.S. interest rates, an inflow of funds from abroad, and a huge U.S. trade imbalance.

10. "If the rate of inflation in the United States increases, then the value of the U.S. dollar will decline."
Comment on this statement.

As domestic inflation increases, U.S. goods become more expensive compared to foreign goods. Foreigners
buy fewer U.S. goods, decreasing the demand for dollars in foreign exchange markets and the value of the U.S.
dollar. Americans buy more foreign goods, increasing the supply of dollars in foreign exchange markets and
depreciating the value of the dollar further.

11. "The trade imbalance between the U.S. and Japan can only be improved by imposing tariffs on Japanese
imports." Comment on this statement, noting who would benefit and who would lose under tariffs.
The arguments used in support of trade restrictions against Japan are that the Japanese make it difficult for the
U.S. to sell goods in their country and the Japanese government unfairly subsidizes its export industries,
enabling them to sell their goods more cheaply on the world market. Some people argue that trade restrictions
may be necessary to avoid the loss of jobs resulting from the trade imbalance. But any kind of trade restriction
means interference with the free market system, which may lead to inefficiencies. Trade restrictions imposed
by one country will almost certainly lead to a reaction from other countries and in the end everybody will be
worse off.
If trade restrictions are imposed, tariffs are generally preferable to quotas, since tariffs provide additional
tax revenues for the government. A tariff is like an excise tax on the foreign import good and raises the price of
the import good. Under a tariff the beneficiary is the domestic government (since it gets increased tax
revenues), domestic firms, since they will sell more products, and domestic workers (whose jobs are
protected). However, the domestic consumer is the loser, since foreign imports become more expensive.
Foreign manufacturers lose out as well, since they receive lower revenues and some foreign workers may lose
their jobs.

12. Discuss the possibility that an exchange rate depreciation may actually worsen the trade balance.

After an exchange rate depreciation, the relative price of imports rises. Initially the trade balance is likely to
worsen since the volume of imports will not change rapidly, while the value of those imports measured in
domestic currency will increase due to the change in relative price. But the increased relative price of imports
increases competitiveness (exports rise as domestic goods become cheaper for foreigners to buy and imports
decline as imports become more expensive for domestic consumers to buy). The short-term volume effect
tends to be fairly small and does not outweigh the price effect. Therefore the depreciation will temporarily
worsen the trade balance. But after a period of adjustment, the volume effect will become substantial enough to
improve the trade balance. This slow response of trade flows to exchange rate movements is called the "J-
curve effect."

13. "If the Fed used expansionary monetary policy to lower the value of the U.S. dollar, the U.S. trade
imbalance would immediately improve." Comment on this statement.

When the Fed uses expansionary monetary policy, domestic interest rates decrease and, eventually, the rate of
inflation increases. Both events lead to a depreciation of the U.S. dollar. After an exchange rate depreciation,
the relative price of imports rises, making U.S. goods more competitive. Consequently, the trade balance
slowly improves, although it may initially actually worsen the trade balance, since the volume of imports
changes gradually, while the value measured in domestic currency increases more quickly. Eventually, the
increased relative price of imports will increase competitiveness, so exports will rise (as domestic goods are
now cheaper for foreigners to buy) and imports will decline (as foreign goods are more expensive for domestic
consumers to buy). The short-term volume effect will be fairly small and will be outweighed by the price effect
in the long run. Therefore a currency depreciation will worsen the trade balance at first but improve it in the
long run as the volume effect becomes more substantial. This slow response of trade flows to exchange rate
movements is called the "J-curve effect."

14. Comment on the following statement:


"The purchasing power parity theory does a good job of explaining exchange rate movements."

The purchasing power parity theory suggests that exchange rate movements mainly reflect the differentials in
national inflation rates. In the long run, the purchasing power parity theory holds up reasonably well,
especially when inflation rates are high and caused by monetary changes. However, in the short run, even a
monetary disturbance will affect competitiveness, since exchange rates move much more rapidly than prices.
Similarly, when a real disturbance occurs, the purchasing power parity relationship will no longer hold, since
the adjustment will affect the equilibrium terms of trade. Examples of such real disturbances are changes in
technology, shifts in export demand, and shifts in potential output in different countries.
15. Explain why restrictive monetary policy to control inflation may be unsuccessful if the central bank pays
too much attention to stabilizing the value of the domestic currency.

If the central bank employs restrictive monetary policy, interest rates will increase which will lower the level
of investment, leading to a decline in economic activity and eventually a lower the rate of inflation. High
interest rates will lead to an inflow of funds and appreciate the value of the currency. A higher currency value
will make domestic goods less competitive on world markets, leading to a decrease in net exports. But if the
central bank pays too much attention to the value of the domestic currency, monetary policy will not be very
effective, since the policy will have to be at least partially reversed as soon as the value of the currency is
affected by the inflow of funds in response to higher interest rates.

16. Comment on the following statement:


"An income tax cut would simply stimulate the economies of Asia and Europe. We need a spending cut
instead, since this would help to reduce the U.S. trade deficits."

If the government undertakes expansionary fiscal policy, interest rates will increase, crowding out private
spending, especially investment and net exports. The higher interest rates will attract funds from abroad and
this will increase the value of the U.S. dollar, making U.S. goods less competitive on world markets. As a
result, domestic investment may be crowded out or jobs may be lost as more goods are imported and fewer
exported. Therefore, the economies of foreign countries will be stimulated at the expense of domestic industry.
On the other hand, while spending cuts may help to reduce the trade deficit, they may have a negative
effect on the economy, since aggregate demand will be lowered. As the economy goes into a recession, imports
are likely to decline. In a recession domestic prices are likely to decline and this will increase competitiveness
in global markets. However, in terms of unemployment and loss of output, the cost of a recession may be too
high.

17. Comment on the following statement:


"Some foreign countries artificially create a comparative advantage by paying exploitative wages to their
workers; this is why we need to protect domestic industry through tariffs."

One argument used in support of trade restrictions against foreign countries is that other countries either
unfairly subsidize their export industries or allow low wage levels, which enable their industries to sell their
goods cheaply on the world market. According to some people, the U.S. should impose trade restrictions, such
as tariffs, to avoid the loss of jobs resulting from trade imbalances. But trade restrictions by one country will
almost certainly cause a reaction from other countries and in the end everybody will be worse off. Any kind of
trade restriction also interferes with the free market system, leading to inefficiencies.

18. Assume interest rates in the U.S. are currently 5% and you expect the Japanese yen to appreciate by 2%.
How much interest would the Japanese government have to pay on its government bonds for Americans to buy
them?
If you expect the Japanese yen to appreciate by 2% and U.S. interest rates are currently 5%, then the yield on
Japanese government bonds would have to be more than 3% for Americans to profitably purchase Japanese
bonds with U.S. dollars. When capital is completely mobile, the domestic interest rate is equal to the foreign
interest rate adjusted for the expected percentage change in the exchange rate:

i = if + (e/e) ==> if = i - (e/e) = 5% - (2%) = 3%.

19. "Either expansionary fiscal policy or a currency depreciation will increase domestic national income by
decreasing the level of foreign output demanded." Comment on this statement.

The central bank can intervene in exchange rate markets by buying foreign currency. If it does so, domestic
money supply and national income increase, while the currency depreciates. The currency depreciation
increases competitiveness and net exports, while income in other countries decreases. Expansionary fiscal
policy, on the other hand, leads to an increase in national income and domestic interest rates. The increase in
interest rates leads to a capital inflow, appreciation of the domestic currency, and a loss in competitiveness. A
trade deficit develops as other countries' exports increase. The income in other countries therefore increases.

20. "A combination of expansionary fiscal and restrictive monetary policy will lower inflation, increase
investment, and lower the trade deficit." Comment on this statement.

A combination of expansionary fiscal and restrictive monetary policy will lead to an increase in real interest
rates with little or no change in aggregate demand or national income. High interest rates will either crowd out
private investment or lead to an inflow of funds from abroad, which will appreciate the value of the domestic
currency. This will make domestic goods less competitive on world markets, so trade deficits will develop. The
restrictive monetary policy will eventually decrease the rate of inflation.

CHAPTER 20

ADVANCED TOPICS

Solutions to the Problems in the Textbook:

Conceptual Problems:

1. Some of the aspects of the theories discussed in this chapter complement each other, but there are also
major disagreements. The real business cycle theory is an outgrowth of the rational expectations theory and the
random walk of GDP theory, while the new Keynesian models respond to the rational expectations theory by
sho CHAPTER 19

INTERNATIONAL ADJUSTMENT AND INTERDEPENDENCE

Solutions to the Problems in the Textbook:


Conceptual Problems:

1.a. A loss of export markets leads not only to an external deficit but also to a decrease in the level of output.
Since a policy dilemma between an external and an internal balance exists, a combination of expenditure-
switching and expenditure-raising policies is required. The levying of tariffs helps to achieve an external
balance, while an increase in government spending helps to achieve an internal balance.

1.b. A reduction in saving and an increase in the demand for domestic goods will lead to an increase in national
income. As a result, imports will increase, leading to a balance of payments deficit. This is not a dilemma
situation and an expenditure-reducing policy (a cut in spending) will remedy the situation.

1.c. An increase in government spending will lead to an increase in national income and the level of imports.
This will cause a balance of payments deficit. This is not a dilemma situation and an expenditure-reducing
policy (a cut in spending) will suffice to reestablish an internal and external balance.

1.d. A shift in the demand from imports to domestic goods will increase output and lead to a trade surplus. This
is a dilemma situation and a combination of expenditure-switching and expenditure-reducing policies should
be employed. Lowering tariffs combined with a cut in government spending will help to achieve an external
and internal balance.

1.e. A reduction in imports with a corresponding increase in saving will not affect the level of national income
but it will lead to a trade surplus. An appropriate response is a reduction in income taxes combined with lower
tariffs. This will help to reduce the trade surplus without affecting national income.

2. Temporary trade imbalances can be easily financed, since the central bank is not likely to run out of foreign
currency reserves rapidly. A country that has a temporary payments imbalance may even borrow foreign
currency from abroad as long as the level of domestic investment spending is sufficient to increase the level of
output, some of which will eventually be exported. A temporary disturbance in trade patterns can come from a
change in exchange rates caused by unstable exchange rate expectations.
If a country suffers a permanent imbalance, however, the central bank will eventually run out of foreign
currency reserves. A permanent imbalance may be caused by a loss in foreign market share. The adjustment
can happen through the automatic adjustment process, but that could involve long periods of high
unemployment. Therefore domestic policy changes are required to move the economy more rapidly toward
balance.

3. The answer to this question is student specific. Government intervention can help smooth out temporary
fluctuations in exchange rates. However, this is often problematic since at the time a currency appreciation
occurs, the government can never be sure whether a disturbance is transitory and soon likely to reverse itself or
more persistent and fundamental in nature. If unstable exchange rate expectations lead to capital flows that
cause exchange rate movements and undesired changes in domestic output, central bank intervention seems
justified. But central banks also often intervene to try to reduce inflation by preventing import prices from
rising. The cost of reducing inflation in this way is a steady loss in competitiveness, which may even result in a
foreign exchange crisis. Ultimately, inflation has to be stopped by restrictive monetary policy. Central banks
may also intervene to affect trade flows and to bring about desired changes in domestic output. In the long run,
however, governments are better off not opposing strong market forces by intervening in the foreign exchange
market. If a government really wants to bring about change, it should employ domestic policies. It may
supplement its policy by intervening in the foreign exchange market as well to emphasize its intent.
Government intervention is more effective if it is backed up by credible policies. There is evidence that non-
sterilized intervention affects exchange rates more successfully, since there is a change in money supply equal
to the amount of the intervention. But if the intervention is sterilized, domestic money supply will remain
unchanged and even a massive intervention may not affect the exchange rate to the desired degree.

4. A wage-price spiral occurs when price increases feed back into higher wage demands and the higher cost of
production (due to higher wages) is then passed on to consumers in the form of still higher prices. For
example, devaluation of the domestic currency will increase import prices and therefore the general price level.
To make up for the loss in purchasing power, labor unions may negotiate higher nominal wages. Firms then
pass the cost on to consumers in the form of higher prices. If the government is committed to full-employment
and the central bank continues to increase money supply, the wage-price spiral will continue. In this case, we
will encounter high inflation, but relative prices will not change, and the nominal devaluation ultimately will
have no effect on the real exchange rate. To avoid inflation, the central bank has to stop increasing money
supply. In this case, higher wages will cause layoffs and thus unemployment, which puts downward pressure
on wages and prices to end the spiral.

5. In a target-zone arrangement central banks agree to limit the range of fluctuation of exchange rates. Such
an arrangement requires coordination of fiscal and monetary policies among the governments of different
countries. This can be a problem, since some countries may be unwilling to give up their ability to conduct
domestic economic policies to achieve other goals. On the other hand, target-zone arrangements are attractive
since they assure businesses that prices quoted in foreign currency will not fluctuate significantly, which
simplifies cross-border trade.

6. In determining exchange rates we assume that capital is perfectly mobile, that is, that capital markets are
sufficiently integrated. This implies that interest rates across countries have a tendency to adjust to the same
level, since otherwise international capital flows would take place in response to interest rate differentials. This
assumption is incorrect, however, since capital flows do affect exchange rates, and therefore we must integrate
expectations about exchange rate changes into the model. We must also take into consideration tax differences
among countries, the restrictions some countries put on the transfer of assets, and the possibility of political
instability in some countries. These factors may explain the interest differentials that we observe and the
reasons capital markets are not be sufficiently integrated.

Equation (7) states that

inflation differential  interest rate differential  depreciation rate.

This is only an approximate, long-term relation since exchange rates can move independently of prices, and
interest rate differentials may exist due to obstacles to the international flow of capital.

8. In a system of freely floating exchange rates, domestic monetary policy can be used effectively, whereas
in a system of fixed exchange rates, monetary policy is completely ineffective. A freely floating exchange rate
system is less crisis prone since exchange rates are determined solely by market forces. It therefore provides a
better framework for macroeconomic stability. However, spillover effects make macroeconomic stabilization
policy more difficult in a system of freely floating exchange rates. These spillover (or interdependence) effects
can lead to a loss of competitiveness among other problems. There is therefore good reason for countries to
coordinate their policies.

9. Central banks often intervene to affect exchange rates in the belief that they can affect trade flows and
prevent import prices from rising. They may also believe that exchange rate fluctuations have occurred simply
due to erratic changes in expectations. However, it is often unclear whether exchange rate movements are
transitory or the beginning of a new trend with strong market forces at work. This makes intervention
problematic. While non-sterilized intervention will affect exchange rates because it will affect the domestic
money supply, it is not clear that sterilized intervention will have the desired effects.
10. It appears that spillover (or interdependence) effects are much stronger under a system of floating exchange
rates. Therefore it can be argued that macroeconomic stabilization policy can be just as difficult under a system
of freely floating exchange rates as under a system of fixed exchange rates.

Technical Problems:

1. The imposition of a tariff raises the relative price of imports and increases the demand for domestic goods.
This increases the level of domestic output, causing interest rates to rise. Higher domestic interest rates will
lead to an inflow of funds and the domestic currency will begin to appreciate. The higher value of the domestic
currency will lower the relative price of imports again. With perfect capital mobility, the currency appreciation
will progress to the point where the overall change in net exports is zero. In the end, the levels of output and
the interest rate will be back at their original levels.

2. Assume a $10 billion balance of payments deficit occurs:

The central bank's balance sheet before sterilization:

Assets Liabilities
foreign exchange - 10 member bank deposits - 10
other reserves 0 currency 0

monetary base - 10 monetary base - 10

The central bank's balance sheet after sterilization:

Assets Liabilities
foreign exchange - 10 member bank deposits 0
govt. securities + 10 currency 0

monetary base 0 monetary base 0

3.a.
i
ISo LMo

i1 BB' = 0
io BB = 0

0
Y* Y
If the home country is initially in an internal and external balance, the IS-, LM- and BB-curves all
intersect at the full-employment level of national income Y* (see the graph below). If the foreign interest rate
increases, funds will flow out, leading to a balance of payments deficit. To restore external balance, interest
rates must increase or the level of national income must decrease. In other words, the BB-curve must shift up.

3.b. A reduction in money supply combined with an increase in government spending will increase the interest
rate and help to restore an external and internal balance. This policy mix will shift the LM-curve to the left and
the IS-curve to the right. A new equilibrium can be reached at the full-employment level of income but at a
higher interest rate. In other words the IS'-, LM'- and BB'-curves will all intersect again at Y*.

i IS' LM'
ISo LMo

i1 BB' = 0

io BB = 0

0
Y* Y

3.c. If the government takes no action, then an automatic adjustment process will take place through changes in
money supply and prices. The balance of payments deficit will lead to a reduction in money supply (a shift in
the LM-curve to the left). Assuming that prices are flexible, the price level will decrease due to the reduction in
aggregate demand as a result of the decrease in money supply. The lower domestic price level will increase
competitiveness in global markets, and net exports will increase (a shift of the IS-curve to the right). The
decrease in the price level will also increase real money balances so the shift of the LM-curve to the left will be
partially reversed. A new long-run equilibrium will eventually be established at the full-employment level of
output Y*.

4. A country experiencing a permanent increase in its exports will develop a trade surplus. Under the
assumption that the central bank does not undertake sterilization operations, the increase in net exports
combined with the increase in money supply will lead to an increase in aggregate demand. This will result in
an increase in domestic prices, making import goods relatively less expensive and export goods relatively more
expensive. Eventually, this will lead to a decrease in net exports. As long as the surplus exists, domestic prices
will continue to rise until an internal and external balance is reestablished. In the long run, over-employment
will also cause upward pressure on wages and prices, causing the upward-sloping aggregate supply curve to
shift to the left.

5. After a currency depreciation, exports should rise and imports should decline, both in the short run and in
the long run. However, empirical evidence suggests that in the short run the volume effects are too small to
overcome the price effect. Only in the long run, when consumers and producers have had time to adjust to the
change in relative prices, will the volume effects be large enough to overcome the price effect.

6. An increase in money supply will shift the LM-curve to the right, driving the domestic interest rate below
the foreign interest rate. A capital outflow will occur and the domestic currency will start to depreciate. As a
result, exports will rise, imports will decline, and the IS-curve will shift to the right due to the increase in net
exports (NX). The level of output demanded and the domestic interest rate will rise until they reach the level of
the foreign interest rate. However, the new level of output demanded will be above the full-employment level
and this will cause upward pressure on prices. The rising price level will reduce real money balances and the
LM-curve will start moving back to the left. Since the domestic interest rate will be above the foreign interest
rate, funds will flow in and the domestic currency will appreciate. As a result, imports will rise and exports will
decline. The IS-curve will now shift back to the left, due to this decrease in net exports (NX). The level of
output demanded will decline again, and the domestic interest rate will decrease until it reaches the level of the
foreign interest rate. In the end, the level of output demanded will again be at the full-employment level.

i IS1 LMo(po) = LM1(p1)


ISo
LM1(po)
4
i1 1=5 3

io

0
Yo Y2 Y4 Y3 Y

7. Newspapers such as the Wall Street Journal publish foreign exchange rates daily. Current editions may
provide entries similar to the ones below:

Britain Canada Japan


(Pound) (Dollar) (Yen)

Spot 1.5172 .6759 .009432


Forward
30 days 1.5181 .6764 .009489
90 days 1.5160 .6774 .009590
180 days 1.5230 .6791 .009752

Note that these are the costs of one unit of foreign currency in U.S. dollars. Newspapers also generally list how
much of the foreign currency can be purchased for one U.S. dollar. For example, the spot price for one British
pound is $1.5172. This is equivalent to saying that one U.S. dollar will purchase 0.6591 pounds< that is,
1/1.5172.
According to the interest rate parity theorem, the difference between the forward and spot exchange rates (in
percentage terms) should be roughly equal to the difference between domestic and foreign interest rates for
assets that have the same maturity as indicated in the forward contract. The forward exchange rates can be
thought of as the market's best prediction of future spot rates on the dates when the future contracts mature.

Additional Problems:

1. "A trade imbalance can persist as long as the central bank wants it to." Comment.

Under a system of fixed exchange rates, the central bank of a country with a balance of payments deficit has to
sell foreign currency. This will reduce high-powered money and therefore domestic money supply, which will
result in a recession, a reduction in domestic prices, and an improvement in the trade balance. This adjustment
process can only be suspended through sterilization (the central bank offsets the decrease in high-powered
money through simultaneous open market purchases). A persistent balance of payments deficit can only exist if
the central bank actively keeps money supply above the level that would ensure an external balance. Under a
system of flexible exchange rates, a central bank can maintain an imbalance through intervention in the foreign
exchange market.

2. "A balance of payments surplus may lead to inflation, while a balance of payments deficit may lead to
unemployment." Comment on this statement.

A country that has a large balance of payments deficit (surplus) may intervene in the foreign exchange market
by selling (buying) foreign currency. Unless the intervention is sterilized, this will lead to a reduction
(increase) in the monetary base and thus money supply. But a reduction (increase) in money supply may
ultimately cause a recession (increase in inflation).

3. Should a country ever borrow from abroad to finance a balance of trade deficit? Explain.

When deciding whether to finance a balance of trade deficit by borrowing from abroad, it is important to know
if the imbalance appears to be temporary or permanent. A temporary imbalance can be financed by borrowing
from abroad, if a country feels confident that it will be able to repay its debt. As long as there is sufficient
domestic investment to produce and export additional output, there should be relatively little concern about
borrowing from abroad.
However, a permanent imbalance cannot be financed by borrowing from abroad, since a country cannot
indefinitely spend more than its income. When a country runs a balance of payments deficit, the demand for
foreign exchange is by definition larger than the amount being supplied, and the central bank has to sell the
difference. Unless the central bank sterilizes its foreign exchange market intervention, this will result in a
reduction in high-powered money and thus money supply. This may ultimately lead the domestic economy into
a recession with high unemployment. To avoid this situation, the government may instead decide on a
combination of expenditure-reducing and expenditure-switching policies.
4. In the early 1980s the U.S. trade deficit increased sharply, yet at the same time the value of the U.S. dollar
increased steadily. Explain this fact.

One should expect that an increase in the U.S. trade deficit will lead to a depreciation in the value of the dollar.
A trade deficit arises when Americans spend more on foreign goods and services than foreigners spend on
American goods and services. As a result, there is an excess supply of U.S. dollars on the foreign exchange
market, which should lead to a fall in the value of the U.S. dollar.
However, in the early 1980s, the U.S. had large federal budget deficits and the increased borrowing needs
of the government put upward pressure on U.S. interest rates. A large inflow of foreign funds resulted, so the
demand for dollars on the foreign exchange market increased sharply. This caused the U.S. dollar to appreciate.
Since the upward pressure on the value of the dollar was greater than the downward pressure on the dollar
caused by the increase in the trade deficit, the dollar appreciated significantly from 1981 to 1985.

5. True or False? Why?


"If a country has a balance of payments deficit and its central bank wants to manipulate exchange rates, it can
sterilize the intervention by undertaking open market sales of bonds."

False. The central bank of a country with a balance of payments deficit may intervene in foreign exchange
markets by selling some of its foreign currency holdings. The resulting reduction in the amount of high-
powered money can only be offset by an open market purchase of bonds (not sales). Such sterilized
intervention serves to maintain a constant level of domestic money supply.

6. True or False? Why?


"A country that has high unemployment and a balance of payments surplus needs a combination of
expenditure-reducing and expenditure-switching policies."

False. There is no policy conflict when the economy is faced with unemployment and a surplus in the balance
of payments. Expansionary fiscal policy will reduce unemployment and increase the interest rate. Higher
domestic interest rates will attract funds from abroad, which will result in an appreciation of the home currency
and a reduction in the payments surplus. A country that has high unemployment and a balance of payments
surplus need only employ an expenditure-increasing policy to bring the economy back to a situation of internal
and external balance.

7. Under a fixed exchange rate system there is a direct link between the balance of payments and the
domestic money supply. Does such a direct link also exist under a system of freely floating exchange rates?

Under a system of freely floating exchange rates, there is no direct link between the balance of payments and
domestic money supply, since there are no changes in international reserves to affect the monetary base (high-
powered money). Only if the central bank decides to manipulate exchange rates by actively intervening in
foreign exchange markets will the monetary base and money supply be affected. But a central bank always has
the option to sterilize its intervention in foreign exchange markets through appropriate open market operations,
leaving money supply unchanged.

8. "The increases in U.S. trade deficits in the early 1980s were a result of expansionary fiscal policy by the
Reagan administration and restrictive monetary policy by the Fed under Chairman Volcker." Comment on this
statement.

Large federal budget deficits caused by the Reagan tax cuts in the early 1980s increased the borrowing needs
of the U.S. government and provided upward pressure on U.S. interest rates. The restrictive monetary policy of
the Fed made interest rates go even higher, resulting in a large capital inflow from abroad. The increased
demand for U.S. dollars on the foreign exchange market lead to an appreciation of the U.S. dollar. As U.S.
goods became less price competitive on world markets, a trade imbalance developed. Starting in 1986, the
dollar began to depreciate again as U.S. interest rates declined significantly and the U.S. trade deficit increased
sharply.

9. "Germany experienced problems in the early 1990s similar to those in the United States in the early
1980s." Comment on this statement.

In the early 1990s Germany needed a large increase in government spending to build up the infrastructure in
the new Eastern states and the budget deficit increased substantially. Since the German Bundesbank tried to
keep money supply fairly constant, interest rates rose sharply, leading to an inflow of capital. This put upward
pressure on the value of the D-Mark, making German goods less competitive in world markets. As a result,
Germany's trade surplus with other (non-EC) countries decreased substantially. This was very similar to the
scenario in the U.S. in the early 1980s, when a mix of expansionary fiscal and restrictive monetary policy led
to high U.S. interest rates, an inflow of funds from abroad, and a huge U.S. trade imbalance.

10. "If the rate of inflation in the United States increases, then the value of the U.S. dollar will decline."
Comment on this statement.

As domestic inflation increases, U.S. goods become more expensive compared to foreign goods. Foreigners
buy fewer U.S. goods, decreasing the demand for dollars in foreign exchange markets and the value of the U.S.
dollar. Americans buy more foreign goods, increasing the supply of dollars in foreign exchange markets and
depreciating the value of the dollar further.

11. "The trade imbalance between the U.S. and Japan can only be improved by imposing tariffs on Japanese
imports." Comment on this statement, noting who would benefit and who would lose under tariffs.

The arguments used in support of trade restrictions against Japan are that the Japanese make it difficult for the
U.S. to sell goods in their country and the Japanese government unfairly subsidizes its export industries,
enabling them to sell their goods more cheaply on the world market. Some people argue that trade restrictions
may be necessary to avoid the loss of jobs resulting from the trade imbalance. But any kind of trade restriction
means interference with the free market system, which may lead to inefficiencies. Trade restrictions imposed
by one country will almost certainly lead to a reaction from other countries and in the end everybody will be
worse off.
If trade restrictions are imposed, tariffs are generally preferable to quotas, since tariffs provide additional
tax revenues for the government. A tariff is like an excise tax on the foreign import good and raises the price of
the import good. Under a tariff the beneficiary is the domestic government (since it gets increased tax
revenues), domestic firms, since they will sell more products, and domestic workers (whose jobs are
protected). However, the domestic consumer is the loser, since foreign imports become more expensive.
Foreign manufacturers lose out as well, since they receive lower revenues and some foreign workers may lose
their jobs.

12. Discuss the possibility that an exchange rate depreciation may actually worsen the trade balance.

After an exchange rate depreciation, the relative price of imports rises. Initially the trade balance is likely to
worsen since the volume of imports will not change rapidly, while the value of those imports measured in
domestic currency will increase due to the change in relative price. But the increased relative price of imports
increases competitiveness (exports rise as domestic goods become cheaper for foreigners to buy and imports
decline as imports become more expensive for domestic consumers to buy). The short-term volume effect
tends to be fairly small and does not outweigh the price effect. Therefore the depreciation will temporarily
worsen the trade balance. But after a period of adjustment, the volume effect will become substantial enough to
improve the trade balance. This slow response of trade flows to exchange rate movements is called the "J-
curve effect."

13. "If the Fed used expansionary monetary policy to lower the value of the U.S. dollar, the U.S. trade
imbalance would immediately improve." Comment on this statement.

When the Fed uses expansionary monetary policy, domestic interest rates decrease and, eventually, the rate of
inflation increases. Both events lead to a depreciation of the U.S. dollar. After an exchange rate depreciation,
the relative price of imports rises, making U.S. goods more competitive. Consequently, the trade balance
slowly improves, although it may initially actually worsen the trade balance, since the volume of imports
changes gradually, while the value measured in domestic currency increases more quickly. Eventually, the
increased relative price of imports will increase competitiveness, so exports will rise (as domestic goods are
now cheaper for foreigners to buy) and imports will decline (as foreign goods are more expensive for domestic
consumers to buy). The short-term volume effect will be fairly small and will be outweighed by the price effect
in the long run. Therefore a currency depreciation will worsen the trade balance at first but improve it in the
long run as the volume effect becomes more substantial. This slow response of trade flows to exchange rate
movements is called the "J-curve effect."

14. Comment on the following statement:


"The purchasing power parity theory does a good job of explaining exchange rate movements."

The purchasing power parity theory suggests that exchange rate movements mainly reflect the differentials in
national inflation rates. In the long run, the purchasing power parity theory holds up reasonably well,
especially when inflation rates are high and caused by monetary changes. However, in the short run, even a
monetary disturbance will affect competitiveness, since exchange rates move much more rapidly than prices.
Similarly, when a real disturbance occurs, the purchasing power parity relationship will no longer hold, since
the adjustment will affect the equilibrium terms of trade. Examples of such real disturbances are changes in
technology, shifts in export demand, and shifts in potential output in different countries.
15. Explain why restrictive monetary policy to control inflation may be unsuccessful if the central bank pays
too much attention to stabilizing the value of the domestic currency.

If the central bank employs restrictive monetary policy, interest rates will increase which will lower the level
of investment, leading to a decline in economic activity and eventually a lower the rate of inflation. High
interest rates will lead to an inflow of funds and appreciate the value of the currency. A higher currency value
will make domestic goods less competitive on world markets, leading to a decrease in net exports. But if the
central bank pays too much attention to the value of the domestic currency, monetary policy will not be very
effective, since the policy will have to be at least partially reversed as soon as the value of the currency is
affected by the inflow of funds in response to higher interest rates.

16. Comment on the following statement:


"An income tax cut would simply stimulate the economies of Asia and Europe. We need a spending cut
instead, since this would help to reduce the U.S. trade deficits."

If the government undertakes expansionary fiscal policy, interest rates will increase, crowding out private
spending, especially investment and net exports. The higher interest rates will attract funds from abroad and
this will increase the value of the U.S. dollar, making U.S. goods less competitive on world markets. As a
result, domestic investment may be crowded out or jobs may be lost as more goods are imported and fewer
exported. Therefore, the economies of foreign countries will be stimulated at the expense of domestic industry.
On the other hand, while spending cuts may help to reduce the trade deficit, they may have a negative
effect on the economy, since aggregate demand will be lowered. As the economy goes into a recession, imports
are likely to decline. In a recession domestic prices are likely to decline and this will increase competitiveness
in global markets. However, in terms of unemployment and loss of output, the cost of a recession may be too
high.

17. Comment on the following statement:


"Some foreign countries artificially create a comparative advantage by paying exploitative wages to their
workers; this is why we need to protect domestic industry through tariffs."

One argument used in support of trade restrictions against foreign countries is that other countries either
unfairly subsidize their export industries or allow low wage levels, which enable their industries to sell their
goods cheaply on the world market. According to some people, the U.S. should impose trade restrictions, such
as tariffs, to avoid the loss of jobs resulting from trade imbalances. But trade restrictions by one country will
almost certainly cause a reaction from other countries and in the end everybody will be worse off. Any kind of
trade restriction also interferes with the free market system, leading to inefficiencies.

18. Assume interest rates in the U.S. are currently 5% and you expect the Japanese yen to appreciate by 2%.
How much interest would the Japanese government have to pay on its government bonds for Americans to buy
them?

If you expect the Japanese yen to appreciate by 2% and U.S. interest rates are currently 5%, then the yield on
Japanese government bonds would have to be more than 3% for Americans to profitably purchase Japanese
bonds with U.S. dollars. When capital is completely mobile, the domestic interest rate is equal to the foreign
interest rate adjusted for the expected percentage change in the exchange rate:

i = if + (e/e) ==> if = i - (e/e) = 5% - (2%) = 3%.

19. "Either expansionary fiscal policy or a currency depreciation will increase domestic national income by
decreasing the level of foreign output demanded." Comment on this statement.

The central bank can intervene in exchange rate markets by buying foreign currency. If it does so, domestic
money supply and national income increase, while the currency depreciates. The currency depreciation
increases competitiveness and net exports, while income in other countries decreases. Expansionary fiscal
policy, on the other hand, leads to an increase in national income and domestic interest rates. The increase in
interest rates leads to a capital inflow, appreciation of the domestic currency, and a loss in competitiveness. A
trade deficit develops as other countries' exports increase. The income in other countries therefore increases.

20. "A combination of expansionary fiscal and restrictive monetary policy will lower inflation, increase
investment, and lower the trade deficit." Comment on this statement.

A combination of expansionary fiscal and restrictive monetary policy will lead to an increase in real interest
rates with little or no change in aggregate demand or national income. High interest rates will either crowd out
private investment or lead to an inflow of funds from abroad, which will appreciate the value of the domestic
currency. This will make domestic goods less competitive on world markets, so trade deficits will develop. The
restrictive monetary policy will eventually decrease the rate of inflation.

CHAPTER 21
Conceptual Problems

wing why prices may still be sticky even under the assumption of rational expectations. In both the real
business cycle theory and the new Keynesian models, expectations are formed rationally, but only the real
business cycle theory assumes that markets clear very rapidly. These theories therefore disagree on the speed
of the price adjustment. They also disagree on the sources of shocks to the economy. The question of whether
output follows a random walk has also been an important point of disagreement. If output shocks are important
they must come from the supply side and they must have permanent effects.

2. Rational expectations differ from perfect foresight in that expectations are imperfect. While rational people
will make efficient use of all information available, they may still make incorrect forecasts. However, when
expectations are rational, no systematic errors are made. The best guess for the forecast error term is zero.
Under perfect foresight, monetary policy is always neutral, that is, the effect of monetary policy on real output
is zero even in the short run. Under rational expectations, monetary policy has a short-run effect on output if
the policy change is unanticipated, but it is neutral in the long run.

3. The propagation mechanism is the process by which a disturbance spreads through the economy. The
concept of intertemporal substitution of leisure is most often used to describe why people work more in some
periods than in others. Assume, for example, that there is a negative shock to technology such that the marginal
product of labor (and therefore the real wage rate) drops temporarily until the market clears. Workers realize
that, for the near future the real wage rate is lower than usual. Therefore they may choose to take some time off
or work fewer hours than usual, intending to make up for it later when the wage rate has increased again. In
this way, a small change in wages (resulting from a shock in technology) can create a large change in output,
even though labor supply is fairly insensitive to wage rate changes in the long run.

4. The models by Mankiw and Lucas both assume rational expectations. However, in Mankiw's menu-cost
model of aggregate supply, firms have enough market power to set their own prices. Prices are sticky, since
firms are reluctant to change them due to the perceived loss of market share or profit or the costs of doing so
(the menu cost). In Lucas' imperfect-information model of aggregate supply, firms are assumed to be price
takers and prices adjust immediately to clear markets. Mankiw's model is a new Keynesian model that argues
that fiscal and monetary policies have short-run effects on output, even if they are anticipated. Lucas' model is
a new classical model that argues that monetary and fiscal policy have only short-run effects on output if they
are unanticipated. However, both models permit a demand-based explanation of the business cycle.

5. The key assumption in Mankiw's model is that firms have the market power to set their own prices and
therefore face a downward-sloping demand curve for their product. Since the private benefits from changing
prices are smaller than the social benefits and are often outweighed by the menu costs, rational decision-
making under imperfect competition may still lead to socially undesirable outcomes.

6. Deep parameters are parameters that are an integral part of microeconomic theory. These fundamental
microeconomic relations describe the preferences of consumers and workers or firms' production functions,
including the marginal product of labor and capital.

7. In Lucas' imperfect-information model, shocks to aggregate supply will have their greatest effect when
they come unexpectedly and rarely, and when they affect the economy as a whole while most previous shocks
were only region or market specific. Most firms observe prices only in their own market. Therefore if one of
these rare economy-wide shocks affects all prices, it is interpreted simply as a movement in prices of these
region (or market) specific goods. As a response, firms will change their production and we will therefore see a
larger change in output.

8.a. A difference-stationary process is dominated by permanent shocks. A trend-stationary process is


dominated by transitory shocks.

8.b. If output is trend-stationary, most changes are transitory, suggesting that they were caused by fluctuations
in aggregate demand. Shocks to the AD-curve (changes in fiscal or monetary policies) are more likely than
supply shocks to cause temporary deviations from the full-employment level of output. If output is difference-
stationary, then the opposite is true, that is, disturbances in aggregate demand are not very important and most
shocks occur on the supply side, causing permanent changes in the full-employment level of output.

8.c. If output were trend-stationary with breaks, then supply shocks would be fairly rare and in the interim
between them, disturbances in aggregate demand would play a greater role.

Technical Problems:

1.a. Equation (3) can be used to solve for output and can be written as

y = [1/(1 + )]m + [1/(1 + )](v - pe) + [/(1 + )]y*

= [1/(1 + 2/3)]9 + [1/(1 + 2/3)](8 - 5) + [(2/3)/(1 + 2/3)]7 = (3/5)*9 + (3/5)*3 + (2/5)*7

= 50/5 = 10.

Equation (4) can be used to solve for price and can be written as

p = [/(1 + )](m + v - y*) + [1/(1 + )]pe = [(2/3)/(1 + 2/3)](9 + 8 - 7) + [1/(1 + 2/3)]5

= (2/5)*10 + (3/5)*5 = 35/5 = 7.

1.b. The model forecasts a price level of p = 7, although the expected price level that was used is pe = 5. This
implies that the predictions for the economy are inconsistent with the predictions of the model. However, truly
rational forecasters should make predictions that are consistent with the way that the economy actually
operates (or is assumed to operate in the model).

1.c. A perfect foresight model would predict that output does not deviate from its full-employment level and
that changes in money supply simply affect the price level, that is,

pe = p = m + v - y* = 9 + 8 - 7 = 10 and y = y* = 7.
1.d. The perfect foresight model has two shortcomings: first, it cannot explain any deviation from the full-
employment level of output. Second, it assumes that people always have all relevant information and always
guess the price level correctly. This, however, seems unrealistic.

2. Equation (11) states that the equilibrium solution for the level of output is

y = y*e + [1/(1 + )]m + [/(1 + )]y.

Equation (12) states that the equilibrium solution for the price level is

p = me + v - y*e + [/(1 + )](m - y*).

From these equations we can see that anticipated changes in potential output (y*e) enter both equations with a
coefficient of 1, positively in Equation (11) and negatively in Equation (12). This means that output will rise
and prices will fall if we have a supply shock, just as the perfect foresight model predicts.
Unanticipated changes in potential output enter both equations with a coefficient of /(1 + ), again positively
in Equation (11) and negatively in Equation (12). This means that output will rise and prices will fall if we
have a supply shock, but by less than the AD-AS model predicts.

3. While early studies seemed to support the view that only unanticipated monetary changes affect output,
later studies questioned this view. Evidence was found that shifts in monetary policy that were intended to
lower inflation tended to be followed by recessions.

4.a. Equation (35) shows the total change in output caused by a change in the marginal product of labor. This
can be expressed in the following way:

%Y = {1 + 3[(1 - )/(1 -  - )]}(%a) = {1 + 3[(1 - 0.35)/(1 - 0.35 - 0.05)]}(0.10)

= [1 + 3(0.65)/(0.6)](0.10) = 0.425.

4.b. Since neither  or  is close to one, there should not be a strong intertemporal substitution of leisure.

5.a. Empirical evidence suggests that intertemporal substitution of leisure is not very strong over the long run.

5.b. Since intertemporal substitution of leisure is not very strong over the long run, the propagation mechanism
indicates that small shocks to technology do not generate large output changes.

6.a. According to equation (16), price expectations would change in the following way:

[E(p|pi)] = (1/)( pi) = (1/1) (0.75)4 = 3,

that is, if local prices rose to 4 times their original level, people would expect an overall price rise of three
times as much.
6.b. According to equation (15), output will increase by

yi = [pi - E(p|pi)] = (1/2)(4 - 3) = 1/2,

that is, output in the region will increase by half as much as a result of these price increases.

6.c. If  = 0.25, then

[E(p|pi)] = (1/)(pi) = (1/1) (0.25)4 = 1, and

yi = [pi - E(p|pi)] = (1/2)(4 - 1) = 3/2,

that is, output will now change more than previously.

6.d. If  = 1, then

[E(p|pi)] = (1/) (pi) = (1/1) (1)4 = 4, and

yi = [pi - E(p|pi)] = (1/1)(4 - 4) = 0,

that is, output will now not change at all.

Additional Problems:

1. "If people have rational expectations, then they will never make forecasting errors and the economy will
always remain at the full-employment level of output." Comment on this statement.

If people have rational expectations they will make the best use of all information available to them before
making an economic decision. The expectations formed are consistent with the way the economy actually
operates. People are still prone to making errors, but not systematic ones. Therefore, if there is an anticipated
change in policy, people will adjust accordingly, markets will clear, and the economy will stay at full-
employment. But if the policy change is unanticipated, people will have forecast incorrectly and the economy
will deviate from the full-employment level of output in the short run. As soon as people learn of their mistake,
however, they will take action to correct it.

2. "A fully anticipated change in policy should not have any effect on real output in a rational expectations
model." Comment on this statement.

According to the simple rational expectations approach, markets always clear immediately and stabilization
policy is therefore ineffective. Any change in the AD-curve will be fully anticipated and bring about a
simultaneous and equivalent shift in the upward-sloping AS-curve. As a result, nominal wages and prices will
rise proportionately, but real output will remain the same. While transitory deviations from full-employment
are possible, they are simply the result of expectations errors and do not last long. If forecasting errors are
made, the government needs only to provide accurate new statistics and markets will correct themselves,
returning the economy to full employment.
Not everyone agrees with this approach and empirical evidence tends to support the idea that even
anticipated policy changes have real effects on the economy. For example, if wages are set by contracts over a
period of time, a price change resulting from a policy change will have an effect on real wages, and thus
employment and output. We will have an upward-sloping AS-curve that shifts over time as wages are
renegotiated. Since wage contracts tend to be valid for more than one year, and since not all wage contracts are
renegotiated at the same time, it is possible that even an anticipated policy change can have a fairly long-
lasting effect on the economy.

3. "A perfect-foresight model predicts that expansionary monetary policy has no effect on the level of
output." Comment on this statement.

Expansionary monetary policy will shift the AD-curve to the right. If inflationary expectations are fixed, this
will lead to an increase in both the price level and the level of output. However, under perfect foresight price
expectations are formed endogenously and are consistent with the price prediction of the model. In other
words, if a price increase is anticipated, workers will know that their real wage rate will decrease unless their
nominal wage rate is increased. Therefore they will demand a higher nominal wage rate. But an increase in the
nominal wage rate in proportion to the price increase will leave the real wage rate unchanged. In this case, the
upward-sloping AS-curve will shift left by the same proportion as the AD-curve shifts right and the full-
employment level of output and employment will remain unchanged while prices increase. In other words,
under perfect foresight, an increase in money supply of 1% leads to a price increase of 1% but no change in the
level of output. Thus, monetary policy is neutral in both the short and long run. This assumes, however, that
economic agents are always right in their predictions and that output always stays at its full-employment level,
which is quite unrealistic.

P AD2 AS2
AD1
AS1
P2

P1

0
Y* Y

4. "In a Lucas imperfect-information model of aggregate supply, an unanticipated fiscal policy change will
be ineffective in changing output." Comment on this statement.

In the Lucas imperfect-information model of aggregate supply, the effects of fiscal expansion are very similar
to those of monetary expansion. An unexpected increase in government spending raises aggregate demand and
leads to an increase in the price level. This unanticipated increase in the overall price level will be interpreted
as a partial increase in the anticipated price level, which will increase the level of output to some degree. In
other words, the short-run effect of expansionary fiscal policy is a shift in the AD-curve to the right, without an
equivalent shift to the left of the AS-curve (which is upward sloping, with its position determined by the
expected price level). Therefore we will have an increase in the level of prices and output in the short run. In
the long run, the economy will adjust back to the full-employment level of output.

5. "The Lucas supply curve will shift to the left whenever the expected price level is below the actual price
level." Comment on this statement.
The expected price level determines the position of the Lucas supply curve. There is an equilibrium at the full-
employment level of output only if price predictions are correct. If the actual price level is above the expected
price level, workers perceive their real wage rates to be higher than they actually are. Thus, employment and
output are above the full-employment level. However, workers will soon learn of their prediction errors and
revise their forecasts. Realizing that real wages are actually not as high as they thought, they will reduce their
work effort and the economy will adjust back to the full-employment level of output. The upward-sloping AS-
curve will shift to the left and a full-employment equilibrium will be reached at the actual (and now also
predicted) price level.

6. Assume the Fed announces a 5% reduction in money supply, but then actually reduces money supply by
10%. Explain the effects of such an unanticipated policy change in the short run and long run according to the
Lucas supply curve model.

If money supply is decreased by 10% rather then the expected 5%, the resulting price decrease will not be fully
anticipated by workers. The AD-curve will shift left and the upward-sloping AS-curve will shift right, but the
AD-curve shift will be of a larger magnitude. The levels of output and prices will both decrease (the price
decrease would be somewhere between 5% and 10%). However, as soon as workers recognize their forecasting
mistakes, they will adjust their expectations downward and the upward-sloping AS-curve will shift further to
the right. In the long run, the full-employment level of output will be reached again and prices, nominal wages,
and nominal money supply will all have decreased by 10%.

AS1
P AD1 AS2
AD2
AS3
P2

P1

0
Y1 Y* Y

7. Comment on this statement:


"The Lucas model predicts policy irrelevance, that is, it suggests that it makes little difference whether fiscal or
monetary policy is used, since neither one will affect output."

The Lucas model predicts that neither fiscal nor monetary policy will affect the level of output in the long run.
However, it allows for transitory deviations from the full-employment level of output, which are caused by
forecasting errors. These deviations do not last long since people ultimately recognize that they have made
errors and correct their behavior.

8. "According to the Lucas imperfect-information model of aggregate supply, government policies should
not affect the unemployment rate." Comment on this statement.

The Lucas model assumes that the expected price level determines the position of the upward-sloping AS-
curve. When price expectations are correct, we will be at full employment. In an ideal case, any anticipated
change in policy that shifts the AD-curve will cause a simultaneous shift in the upward-sloping AS-curve, and
markets will clear rapidly. Nominal wages and prices change proportionally so we will stay at the full-
employment level of output. However, if policy changes are unanticipated, price expectations will adjust only
partially and we will see a transitory change in the level of output and therefore the unemployment rate. It
should be noted that empirical evidence tends to support the notion that even anticipated changes in
government policies have real effects in the short run (people's expectations are not always correct). In
addition, because of wage contracts, wages tend to be rigid in the short run. Prices also adjust only slowly in an
imperfectly competitive environment. Thus even anticipated policy changes tend to have a real effect on the
economy.
9. "The random walk of GDP theory argues that permanent output fluctuations arise primarily from
unanticipated fiscal or monetary policy changes." Comment on this statement.

Actually, the random walk of GDP theory argues that most aggregate demand disturbances (due to fiscal or
monetary policy changes) are transitory in nature, whereas supply shocks tend to have permanent effects on
output.

10. "According to the random walk of GDP theory, output fluctuations are always temporary and mostly the
result of changes in technology." Comment on this statement

The random walk of GDP theory argues that most aggregate demand disturbances (due to fiscal or monetary
policy changes) are transitory and generally not very severe in nature. On the other hand, supply shocks (due to
changes in material prices, technology, or productivity) tend to have permanent effects on output. Therefore
most long-lasting changes in output come from random shocks on the supply-side and cannot be accurately
predicted.

11. "The intertemporal substitution of leisure propagation mechanism argues that the elasticity of labor supply
in response to a permanent change in the wage rate is very high." Comment on this statement.

A propagation mechanism is a mechanism through which disturbances are spread throughout the economy.
Intertemporal substitution of leisure assumes that people prefer to work more in periods when the wage rate is
higher and less in periods when the wage rate is lower. Thus there is a very high elasticity of labor supply in
response to temporary changes in the wage rate. This, however, does not imply that labor supply is very
sensitive to permanent changes in wages. As a matter of fact, since there is nothing to be gained from working
more now and less later if wages increase permanently, the response of labor supply to a permanent wage rate
change may actually be quite small.

12. Explain the differences in the effect of intertemporal substitution of leisure between a temporary and
permanent wage increase.

The view that workers are willing to substitute leisure for work intertemporarily implies a high elasticity of
labor supply in response to a temporary wage rate change. When the wage rate increases temporarily, workers
may work more, but only in those periods when the wage rate is high. When the wage rate declines again,
workers will substitute leisure for work, that is, they will work less. A permanent wage increase would not
have the same effect, since it would not be to a worker's advantage to work more in one period and less in the
other.

13. "The real business cycle theory asserts that changes in the level of output can be largely explained by
unanticipated changes in money supply." Comment on this statement.

Most economists believe that an increase in money supply causes lower interest rates and more investment,
resulting in a higher level of national income. However, the real business cycle theory asserts that changes in
money supply do not cause major changes in output. Instead changes in money supply are a reflection of
changes in output and of the fact that the banking system expands the supply of money to accommodate an
increase in spending. In other words, the real business cycle theory does not give money a central role in
explaining fluctuations in output. Instead it argues that major fluctuations in output and employment are the
result of real shocks to the economy (changes in productivity, technology, or government spending).

14. The fact that an upswing in economic activity always seems to be preceded by an increase in money
supply can be explained in two ways. What are the two explanations, which do you agree with most, and why?

The answer to this question is student specific. Most economists believe that an increase in money supply
causes lower interest rates and more investment, resulting in a higher level of national income. However, the
real business cycle theory asserts that changes in money supply do not cause major changes in output, but are
instead a reflection of changes in output and of the fact that the banking system expands the supply of money
to accommodate an increase in spending. The real business cycle theory does not give money a central role in
explaining fluctuations in output. Instead it argues that major fluctuations in output and employment are the
result of real shocks to the economy (changes in productivity, technology, or government spending).

15. Comment on the following statement:


"The new Keynesian models assert that people do not have rational expectations, since the assumption of
rational expectations would imply that markets always clear immediately."

The new Keynesian models state that even under the assumption of rational expectations, socially undesirable
outcomes may still occur due to imperfect competition. Firms in imperfectly competitive markets are reluctant
to change prices due to the menu costs involved and therefore markets will not clear rapidly.

16. "If all economic agents have rational expectations, then all wages and prices must be perfectly flexible and
unemployment cannot exist." Comment on this statement.

The new Keynesian models argue that even if people have rational expectations, socially undesirable outcomes
may still occur due to imperfect competition and the existence of wage contracts. Prices may not change freely,
since firms in an imperfectly competitive market are reluctant to change them, due to the menu costs involved.
Nominal wages are set by contracts over a period of time, so the economy may adjust only slowly to a decrease
in aggregate demand. Thus a rate of unemployment higher than the natural rate can exist over an extended
period of time.

17. “Lucas’ imperfect-information model and Mankiw’s menu-cost model are actually quite similar and they
both have the same policy implications.” Comment on this statement.
Mankiw’s and Lucas’ models both assume rational expectations. However, in Mankiw’s menu-cost model of
aggregate supply, firms have enough market power to set their own prices. Prices are sticky, since firms are
reluctant to change them due to the perceived costs of doing so, namely the menu costs or the loss of market
share or profit. In Lucas’ imperfect-information model of aggregate supply, firms are assumed to be price
takers and prices adjust immediately to clear markets. Mankiw’s model is a new Keynesian model that argues
that fiscal and monetary policies have short-run effects on output, even if they are anticipated. Lucas’ model is
a new classical model that argues that monetary and fiscal policy have only short-run effects on output if they
are unanticipated. However, both models permit a demand-based explanation of the business cycle.

18. "The actual impact of fiscal and monetary policy is not so much determined by whether expectations are
rational but by whether wages and prices are flexible." Comment on this statement.

The rational expectations hypothesis says that people consistently make optimal choices to maximize their
utility using all available information. Since people know the outcome of an announced policy measure, they
always try to adjust immediately and therefore markets clear very rapidly. Only unannounced policy changes
have a temporary effect on the level of output, unemployment, and inflation. However, even if expectations are
rational, there are always wage contracts, costs of price adjustment (menu costs), imperfect information, and
imperfect competition. Wages and prices do not adjust immediately and markets cannot clear immediately. The
more rigid wages and prices are, the larger the effect of a disturbance. Similarly, the longer it takes for
inflationary expectations to adjust, the longer it takes for the economy to adjust back to full employment after a
disturbance or policy change.

You might also like